Wikipedia:Reference desk/Archives/Humanities/March 2006

From Wikipedia, the free encyclopedia

March 1[edit]

How many students graduated in the year 2001?[edit]

Students of what? The School of Hard Knocks? Dismas|(talk) 04:55, 1 March 2006 (UTC)[reply]
42 - Adrian Pingstone 06:36, 1 March 2006 (UTC)[reply]
Clearly not suitly emphazi. --BluePlatypus 06:49, 1 March 2006 (UTC)[reply]
You'll need to give us more information for us to be able to answer that question. Click the "edit" link on the right hand side of the heading to add more info. - Akamad 11:16, 1 March 2006 (UTC)[reply]
Where, and in what level (high school or collage)? – b_jonas 13:08, 3 March 2006 (UTC)[reply]

Straits of Malacca[edit]

There are piracy and terrorism in the Straits of Malacca. It is important sea lane to the world economy. The Air Force Maritime operation and coastguard is one of the means to prevent or combat those activities. Is there any other ways or means to curb it?

Thank you.

  • ---*
There have always been three ways to approach piracy. The first is to encourage or coerce those nations that hold the pirates to disallow/kill them. The second is to ensure that no one will fence stolen goods. Finally, the European and American naval powers in the 19th century signed a concord that any nation might attack any safe harbor of pirates without its being an act of war against the nation that had that port. That's why English and French navies would patrol up and down and simply open fire on pirate havens. There was talk, after 9/11, of modern nations doing something similar with terrorism, but then it fell apart in the heat of unilateralism. Geogre 12:38, 1 March 2006 (UTC)[reply]
I think the anti-piracy ieda goes back further than the 19th century. Cicero coined the phrase hostis humanis generis, describing pirates as enemies of all the world back in the first century BC. Lisiate 00:57, 3 March 2006 (UTC)[reply]
You're right, of course. The idea does go back that far, and the Classically educated 16th-19th century Europeans knew the concept and embraced it, in theory, but an international agreement had to wait for the day when none of them were unofficially sponsoring pirates. Even then, the various nations had places in their own territories where petty piracy occurred, but anyone caught would be punished. In Europe and the US, all sides lamented pirates with one side of their mouths and commissioned them with the other side, and even the Romans made deals with them. Geogre 11:00, 3 March 2006 (UTC)[reply]

Another way is mercenaries - escort for hire. There are several companies in that business in Singapore though they seem to have problem with Malaysian and Indonesian coast guard/military/police. __earth (Talk) 11:12, 7 March 2006 (UTC)[reply]

No meat. Why?[edit]

I go to a catholic school here in australia. it is ash wednesday today, so we are asked to fast, and abstain from meat.

now, i understand the belief behind why catholics fast. but i do not understand what significance or meaning is held in the abstinence from meat.

could someone please clear this up for me.

thanks! gelo 11:11, 1 March 2006 (UTC)[reply]

It is a memorial of the fasting that Jesus did before his ministry. Friday is specifically a memorial of Good Friday, and therefore it is the day of greatest privation. No matter what else you give up during Lent, red meat on Friday is a sacrifice in the Western Catholic tradition. (Note that the Archbishop of Norway represented coastal farmers who only had puffins and small amounts of fish in their diets, so he got a Pope to rule that Puffins are, from the point of view of the church, fish.) Generally, the idea is that red meat is a treat and a luxury, and you give that up in memorial of the passion. Geogre 12:41, 1 March 2006 (UTC)[reply]
See unclean animals for further. Marskell 12:43, 1 March 2006 (UTC)[reply]

Jack Vettriano[edit]

I am doing research on the artist Jack Vettriano and i cant find any information on the name of his ex wife, or when he was married. I have tried to access the marriages register but as a student i have not got enough money!!!

Please can anyone help?

Thank you

Natasha Miller

A google search on "Jack Vettriano" wife turned up over 30,000 hits. Good luck! --Halcatalyst 14:46, 1 March 2006 (UTC)[reply]
Tried googling "Jack Vettriano" divorced and found: Jack Vettriano, Married: 1980 Gail Cormack, divorced '88. [1] --Kainaw (talk) 20:42, 1 March 2006 (UTC)[reply]
Gosh, you know a lot about divorce, Kainaw :P --Halcatalyst 21:27, 1 March 2006 (UTC)[reply]

Naomi Rebekah lodges[edit]

i assume the the naomi rebekah lodges are a female version of the moose, elks, etc. i know that naomi and rebekah are famous bible mothers, but why are the names linked together and not just one as i don't think they are linked in the bible, although i could be wrong? 24.2.112.218 14:00, 1 March 2006 (UTC)[reply]

Isn't it connected with Freemasonry? But I couldn't find any info there. --Halcatalyst 21:23, 1 March 2006 (UTC)[reply]
Rebekah lodges are the women's part of the Independent Order of Odd Fellows, some individual lodges were named Naomi. Rmhermen 03:55, 2 March 2006 (UTC)[reply]

Xylophone[edit]

What is the range of a western xylophone? (Not one for children, one for real adult musicians.) The russian article says 3.5 or 4 octaves, but from where to where is that in absolute pitch? Thanks, David Da Vit 16:36, 1 March 2006 (UTC)[reply]

Take a look here, where a "typical playing range" is given as "F3 to C8", or here, where several instruments are listed with their ranges. --LarryMac 17:17, 1 March 2006 (UTC)[reply]
Which are C4 to C8 for 4 octave and F4 to C8 for 3.5 octave models. Rmhermen 03:57, 2 March 2006 (UTC)[reply]

Thanks. David Da Vit 15:06, 2 March 2006 (UTC)[reply]

Salary of people in CNBC India[edit]

How much salary do people like Udayan Mukherjee and senthil chengalvarayan of CNBC India get?

I very much doubt their salary is publicly disclosed, but TV anchors for popular programs are well paid. However, the proportion of journalists who end up as TV anchors is miniscule. The vast majority of journalists in Western countries are only moderately paid, though the situation might be a little different in India. --Robert Merkel 04:06, 2 March 2006 (UTC)[reply]

The last time I checked in 2003, Udayan a colleague of mine was getting a take home salary of INR 59,000 per month. His salary must have more than doubled since then. Most Indian journalists are payed real low...like me. They usually appoint interns and trainees for a few months at a time and save on paying any salary !

In 2012 Udayan Mukherjee's Salary may be more than Rs.6,00,000 lacs per month.

19th c. Atlantic crossings[edit]

How long did it take merchandise (not necessarily passengers) to cross the Atlantic in the 1800's? Specifically, I want to know how long it would take a woodworking tool made in Sheffield, England to reach a New England shipbuilding port such as Boston or New Bedford. Thanks everyone!

The precise answer you're looking for is not possible to give. There were slow boats, and slower boats, and slower boats still. Vessels did not specialize in freight or passengers: most commercial ships carried both. A specific item like the one you mentioned might have been carried over as part of a woodworker's personal kit, if not transported commercially. If it's a tool used in shipbuilding, unless it's very specialized, which is unlikely, it may have been found on most ships. That said, you might want to have a look at Shipbuilding. --Halcatalyst 02:47, 2 March 2006 (UTC)[reply]
Also, given the pace of technological development through the 19th century, you really need to give a more specific date. --Anonymous, 03:47 UTC, March 2.
The answer will vary greatly if you are discussing the early 1800 with only sailing ship or the late 1800s with increaingly fast steamers. Rmhermen 03:51, 2 March 2006 (UTC)[reply]
It also depends on the route taken, ships leaving from Glasgow in Scotland could make the journey in half the time it took ships leaving from England (sailing ships), hence Scottish dominance of the British Tobacco trade in the 18th century. Therefore depending on the state of the roads between sheffield & Glasgow it may well have been quicker to go by road to Glasgow then sail over, rather than sailing from an English port. AllanHainey 09:20, 2 March 2006 (UTC)[reply]

The Blue Riband was awarded for the fastest transatlantic crossing. The record time fell rapidly during the century, from 18 days in 1833 to 5 days in 1902. The Scottish clipper James Baines gained the record for a sailing vessel (12 days, 6 hours) in the mid-1850s. The U.S. clipper Flying Cloud held several records, though not for the Atlantic crossing. Gdr 21:32, 3 March 2006 (UTC)[reply]

BP trade with India's companies[edit]

Is there anywhere I can obtain a list of companies BP plc trades petroleum, petrochemicals, solar panels, natural gas, etc. to. I have had a really hard time obtaining specifics on this. The answers to this question are very vague. I'm interested in finding out which companies they trade with and what kind of trade relationship they have. Please help me on this as much as possible-thank you!

I doubt such a list exists - and where do you draw the line? Do you care about the companies that fill their cars up at BP fuel outlets? --Robert Merkel 04:08, 2 March 2006 (UTC)[reply]
This is not a well-known topic, and you want quite specific information, so it's unlikely many people here would be able to help you. The best place to get the information is from BP India itself, who maybe able to provide you with the information you need. Contact details and some information are here. --Canley 04:14, 2 March 2006 (UTC)[reply]

Line from poetry[edit]

I'm looking for the source of the following line, which if I recall correctly is the last line from a classical song, or possibly a poem... "A moment of midnight is worth the whole day". I tried google but that didn't help. Thx!

Here's one that's close (based on A moment of midnight): (see "blossom"). Probably not the real thing, though. --Halcatalyst 21:18, 1 March 2006 (UTC)[reply]
Nope, that's not it.

Frogger[edit]

Why do Americans refer to the French as Frogs? User:206.15.98.65

Because they eat frog legs, I assume. -- Mwalcoff 23:46, 1 March 2006 (UTC)[reply]
Because they're xenophobic? See Frog legs.
Slumgum 23:47, 1 March 2006 (UTC)[reply]

Because they eat frog leg and because FRog and FRench both start with FR. StuRat 00:07, 2 March 2006 (UTC)[reply]

  • "Did you know that French people everywhere in the world are named frogs? It's true! You must have heard it, of course; frogs. And if you put yourselves this question, I also am called a frog. It is not an insult, actually, it is a compliment. I always wanted to know why and where this whole thing began, which encouraged me to telephone the Center franco-ontarien of folklore of Sudbury, Ontario, Canada. They were very nice and gave me some anecdotes on this subject." Or so says the host of the bi-lingual Archived French Frog Stories. --Halcatalyst 02:55, 2 March 2006 (UTC)[reply]
  • PS The state of Maine is about 30% Franco-American and (at least in the 70s) about half of these people were francophone. The University of Maine had, and maybe still has, a unit in Student Affairs called the Franco-American Resource Opportunities Group -- F.A.R.O.G. Ironic, in view of the fact that many Anglos used "frog" and "nigger" with the same intent. --Halcatalyst 02:55, 2 March 2006 (UTC)[reply]

Americans are not the only ones to issue pejoratives based on diet. Americans referred to British as "limeys" because British sailors ate limes to avoid rickets, and they referred to the French as "frogs" because of frog's legs, and they referred to Germans as "krauts" because of sauerkraut, but one suggested German language pejorative of Americans is "peanut butter eaters." As insults go, these are not terribly bad. The original of "canuck" is worse, and the various vilifications of oriental peoples are all much more insulting ("chink" may derive from "chinese" in other languages," but it caught on because of other, nastier associations, and "slope" and "gook" are meant to insult and harm). Those insults that derive from the military always have more hatred in them than those that derive from traders. Geogre 14:59, 2 March 2006 (UTC)[reply]

Canuck doesn't discuss an offensive origin. What is it? Can you add it to the article? moink 00:01, 3 March 2006 (UTC)[reply]
Interesting. As I had heard it discussed by ethnologists, they believed that its origin was in an insult related to being a savage. Therefore, unlike a dietary insult, it was character based. I don't mind hitting the books to find some serious documentation. (Am. Heritage is usually pretty good with etymology.) Geogre 11:04, 3 March 2006 (UTC)[reply]

March 2[edit]

Early Man[edit]

What did the early men [neanderthals,cro-magnon,ealy homo sapiens sapiens] wear,eat,live in?

...all explained to some extent in our articles on and neanderthal and the many articles which link to it. Did you look there first, like it says at the top of the page? Grutness...wha? 01:03, 2 March 2006 (UTC)[reply]
Did you check Neanderthal, Cro-magnon, and Homo sapiens sapiens? All were omnivores and lived in shelters constructed with tools. The Neanderthal has no evidence of woven clothing, but Cro-magnon (and obviously Homo Sapiens Sapiens) do. --Kainaw (talk) 01:08, 2 March 2006 (UTC)[reply]


bumm rooomate[edit]

In had a roomate who rang up a 750 dollar phone bill and now refuses to pay i have property belonging to him but in my possesion and want to know if i can leagally sell it tom regain my money?

You need advice from a lawyer, not some random person on the reference desk. --Anonymous, 07:00 UTC, March 2, 2006.
It would not be legal, provided he still legally owns them. Willzyx 07:08, 2 March 2006 (UTC)[reply]
Does he have a receipt to prove it is his property? --Kainaw (talk) 19:12, 2 March 2006 (UTC)[reply]
Don't sell his stuff. You could potentially get in a lot of trouble. Get a lawyer or an arbitrater. The outcome may end up being the same, but at least if it's done right it'll be done and over. -LambaJan 21:34, 2 March 2006 (UTC)[reply]
Do you have a Small Claims Tribunal? They deal with these sorts of cases every day of the week. JackofOz 21:55, 2 March 2006 (UTC)[reply]

Help with a costume: Chinese courtesan in 1500[edit]

I have been cast in a live action roleplaying game set in a world loosely based on the island which is now called Taiwan in approximately 1500. I say loosely, because history is different, magic works, and, well, generally, it's a fantasy world. I have been cast as a "bailian," which may be a made up word or may exist. The description says: "A Bailian is a woman paid to be charming and entertaining and perhaps to have sex as well. They are respected, unlike whores, as a legitimate and graceful profession. Bailians generally operate either from a professional establishment or else are sold into concubinage."

I'm not terribly concerned about precise historical accuracy or anything, but I would like some idea of what such a person would wear. And I'm poor... I will be shopping at Goodwill and The Garment District, as well as making use of things already in my closet if they are appropriate. I have for example, two corsets, one black, one white.

Oh, and did I mention the game is 10 days long? And involves running, sometimes up and down stairs, sometimes shooting at people with toy guns or avoiding being shot at. If you're terribly interested, you can look at links to the pdf documents about the game.

I'd appreciate any advice anyone could give me on a costume. Really I'm just trying to evoke the period/place/role, rather than recreate it. moink 06:53, 2 March 2006 (UTC)[reply]

Although there is a wealth of difference between Taiwanese and Japanese history, I'd suggest looking up geisha costuming, since it sounds like a similar sort of role - primarily hostesses and entertainers, on very rare occasions also sexual partners. If you can find anything on early 16th century geisha clothing, and any comparison of early 16th century Taiwanese and Japanese clothing in general, it might give you a few pointers. Grutness...wha? 09:21, 2 March 2006 (UTC)[reply]
Sounds a lot like a Geisha AllanHainey 09:22, 2 March 2006 (UTC)[reply]
I thought about geishas too. But from looking at, for example sing-song girls, there doesn't seem to be much of a tradition (at least not one that survived to the 20th century) that such girls in Chinese culture (which I'm guessing is much closer to Taiwanese than Japanese is) have a formalized dress code. If I knew what women wore in that region/time period in general, that would be a good start. moink 21:28, 2 March 2006 (UTC)[reply]
I just found Han Chinese clothing. It looks like a good start, though the article needs a lot of work and is confusing. Also, I'm not sure how I can "fake" this without spending lots of money. moink 23:03, 2 March 2006 (UTC)[reply]
The dresses worn by the girls in the picture in the article look like they'd be really easy to sew. The cloth was probably not too expensive. One looked like she was wearing silk but the others looked like it was probably some form of cotton, which would be nice and cheap. -LambaJan 22:04, 4 March 2006 (UTC)[reply]
Thank you LambaJan. You're probably right, but unfortunately I'm a terrible seamstress. I failed the sewing half of home economics in junior high. I only passed the class because I aced the cooking half. But it looks like if I get a robe or something and combine it with a long skirt and a sash I might be able to evoke the look. moink 23:31, 4 March 2006 (UTC)[reply]
Yeah, it looks like the skirt matches the trim of the blouse. So a lightweight-fabric robe from a thrift shop would be a good base. You could then get some fabric for the skirt and cut some of it and sew on a good three inch trim, and use the rest for a skirt, which failing home-ec qualifies you to do well. It's EZ. If you want a three inch wide trim you just cut it 7 inches wide, which gives you three and a half for each side. Fold under the half inch on each side and pin and iron to keep it in place, then sew. You can even use scraps to make the sash, but you'd have to get at least a D in sewing to do that. :-) -LambaJan 20:43, 5 March 2006 (UTC)[reply]

Translation of 'Nessun Dorma' on Spicks and Specks[edit]

Does anybody remember the translation of the aria from Turandot that was on Spicks and Specks (ABC, Australia) on Wednesday? It was told by Richard Gill. --Alexs letterbox 08:20, 2 March 2006 (UTC)[reply]

I missed it (dammit!), but if it was "Nessun Dorma", it means "None shall sleep". JackofOz 10:02, 2 March 2006 (UTC)[reply]
Looks more like 'Nobody is sleeping' to me, or 'Nobody is to sleep', though I'm no expert on Italian. Maid Marion 10:28, 2 March 2006 (UTC)[reply]
Actually I've just found a complete translation via Google. Try this link. http://ingeb.org/songs/nessundo.html Maid Marion 10:31, 2 March 2006 (UTC)[reply]
This translation was quite funny, done using the Australian venacular. --203.129.37.213 22:47, 3 March 2006 (UTC)[reply]

I can't get to sleep now.
I can't get to sleep now.
All because of you, you rotten old princess,
I can't get to sleep now.
...
So come on princess, stop your rot,
I'd like to show you what I've got.
Oh yeah, what I've got!
...
So princess be a sport and try your luck,
I'd like to show you I'm a terrific fellow
Geoff/Gsl 04:04, 5 March 2006 (UTC)[reply]

Hitler's army[edit]

While sitting in psychology class we went into dicussing about the possibility of amphedemies used in the military foreine and domestic. I remember watching the history channel and I was looking for the information on how hitlers troops advanced so quickly with the help of drugs.

The British & American forces used amphetamines too (I believe the U.S.A. still uses them, & downers too, for pilots) in WWII & in more recent conflicts. AllanHainey 09:25, 2 March 2006 (UTC)[reply]
I suppose you mean amphetamines. That would make sense for soldiers, because it's an 'upper', giving you more energy (or rather the illusion of more energy). Then again, it also wears you out, so it might as well have been a cause for the ultimate defeat of the Germans. Interresting theory? DirkvdM 07:18, 3 March 2006 (UTC)[reply]

George Strait, Crystal Gayle, Eddie Rabbitt - Houston Livestock Show & Rodeo[edit]

Hi, I attended the first show that George Strait played at the Houston Livestock Show & Rodeo in the Astrodome. I believe it was 1983 (I was 12 at that time). My recollection, and my mother's, is that the performers were to be Eddie Rabbitt and Crystal Gayle. Eddie Rabbitt was sick and unable to perform, and they flew George Strait in from his ranch to perform. At the time, George Strait was relatively unknown, at least to such a broad audience.

I have seen George Strait at the rodeo many times since (I'm a big fan), most recently at the show earlier this week. Out of nostalgia, and to verify my recollection, I've tried to search the web about that first appearance. The only website that talks in detail about it is www.houstonhistory.com, but that site refers to an appearance by Eddie Rabbitt and Roseanne Cash in which Rabbitt got sick and Strait replaced him. I have no recollection of ever seeing Roseanne Cash in concert, nor does my mother (and we both do recall seeing Gayle at a rodeo). Cash is a very good artist, and I think we would've remembered seeing her in concert.

Could you please look into this and see if you can verify with whom George Strait performed in that first appearance at the Houston Rodeo? Thank you very much in advance for your assistance.

-- Laikaprincess

Generally, with this kind of question, you'd be better off asking in a forum devoted to the artist concerned. His official website has a fan club with a member's forum; that might be a good place to ask. --Robert Merkel 03:37, 3 March 2006 (UTC)[reply]

Security Deposit Hijack/Stopped Check Affect??[edit]

The homeowner from whom my 20yr. old daughter and her friend rented accused them, the day before they were to be out of the house, of damaging property in the house. The homeowner lives in the house, too and never mentioned this before hand.

They did not cause the damage. My daughter was able to locate the previous homeowners who confirmed that the damage existed even before they, themselves, had purchased the house.

The homeowners from whom the girls rented says - that they don't care what the previous owner said, the security deposit would not be returned and they expect the girls to pay any outstanding difference to repair the damage - which the homeowner said will be completed by their friend!! No other damage or concerns are at issue but the homeowners are just really being difficult, rude and un-cooperative.

Both the girls are just learning about living out on ther own. I have tried not to interfere but I feel they are being taken advantage of and intimidated by the much older homeowner who seems to see the young girls as easy targets.

The girls asked for a disclosure statement that might show whether or not there was damage when this homeowner purchased the property last fall. There was no official "walk-through" when the girls moved-in in Oct.'05.

They refused to present the disclosure statement or even talk about it. The homeowners even made the girls write additional checks for utility bills that should have already been covered by the first and last. They had given their 30 days notice and actually moved out several days before the 30 days were up. The homeowner had another person move into the house before the 30 days were up but refused to prorate or refund any of the prepaid rent to the girls.

I suggested the girls stop-payment on those last checks they'd written for utilities before they could be cashed. I really felt this was totally unfair and at least this was one issue they might be able to have control of. Though half the check amount went to the fee to stop payment I preferred it went to the bank than to these unscrupulous homeowners.

The previous homeowners are willing to write a statement to the affect that the damage was done some years ago and so should not be attributed to my daughter and friend.

We've discussed getting an attorney involved but the cost of taking these landlords to court would be much more than the security deposit and certainly more then the girls can afford. I've suggested that the homeowners never intended to give back the security deposit and the girls are going to have to accept the loss and consider it a lesson learned about living in society with people who lack civil honor, perceptions of fairness, or acceptable behavior. Sorry I'm ranting - it's been stressful.

Could any of this, the accusation of property damage, or the stopped payment on the check affect the girls' credit rating?? ----209.206.253.157Ever-The-Mom.

Wikipedia should not be used for legal advice. Beyond that, security deposits are commonly kept for various reasons. It is not abnormal to have it kept for random repairs. As for the credit rating, you have to pay money to put bad things on a person's credit report. Do you think that someone so cheap that they won't return a security deposit will pay to mess with someone's credit? The best option is to take it to small claims court. Absolutely anything else you do will quickly skirt with illegal activity and cause you more harm than the person you are fighting. --Kainaw (talk) 21:11, 2 March 2006 (UTC)[reply]
You can also find a wealth of information online and at your library or bookstore. Search for landlord tenant relations and look for advocacy organizations in your state. Landlord tenant is one of the most common civil disputes, so there is a lot of information available. Get ahold of the rules and the full relevant laws in your state and follow them very carefully. There are a number of deadlines, that if not followed can invalidate either party's claims to the money. Nolo press publishes some guides to small claims court and civil matters. The laws are typically biased towards the tenant because they typically do not know their rights, but you still have to look them up to find out. So do your research, and to get actual legal advice, consult an attorney. A couple hours of their time even if just for information may be worth your money. I of course am not an attorney and this is not legal advice. - Taxman Talk 21:27, 2 March 2006 (UTC)[reply]
What you are asking about will vary wildly depending upon state, and sometimes city. In the United States, some places are pro-renter in their laws, some pro-landlord. In general, the burden of proof is on the person trying to take the money, but the landlord is usually playing a game of chicken -- daring you to go to small claims court (in the US, that would be the venue). Again just in general private landlords (some dude in his house) do this because they haven't kept the security deposit in an escrow account, as they are required to do (in the US). Geogre 11:09, 3 March 2006 (UTC)[reply]

March 3[edit]

What J-Pop groups, other than SMAP are most popular in Japan and the US?[edit]

Many of SMAP's songs are hits and I enjoy them personally, but unfortunately, they haven't made enough songs. I cannot think of other J-Pop groups that perform as well at the level that SMAP is at. Therefore, would you care to tell me what other J-Pop singers & groups have many hit songs and are as well-received as SMAP is? Thank you. --Shultz III 00:44, 3 March 2006 (UTC)[reply]

So anyway, what J-Pop groups are as well-renowned as SMAP for their music? Thanks. --Shultz III 11:33, 3 March 2006 (UTC)[reply]

Where? Do you mean "What Japanese Pop groups are popular in the United States?" Do you mean "What Japanese Pop groups are popular in Merca?" If you mean the U.S., Hi Hi Puffy Ami Yumi is on Cartoon Network, so they have been getting decent album sales. --Kainaw (talk) 14:44, 3 March 2006 (UTC)[reply]
Why didn't your answer ring the right bells in me, Kainaw? HHPAY is for the too-young set. I'd like something that teens or young adults enjoy. Why did you even suggest if I meant what J-Pop groups are popular in a town I've never even heard of? Well anyways, I meant what are the most popular J-Pop groups in either Japan, the US, or both. --Shultz III 15:29, 3 March 2006 (UTC)[reply]
Music is a listener's preference. There are many groups. You'll have to find ones you like. Wikipedia, being an encylopedia, has a large list of them here. --Kainaw (talk) 15:58, 3 March 2006 (UTC)[reply]
I already looked. Weren't J-Pop artists rated on the likability of their songs somehow? If so, who had the highest ratings? What I'm kinda looking for are J-Pop artists/groups that are liked the most by a common audience. --Shultz III 16:09, 3 March 2006 (UTC)[reply]
check out B'z --Vsion 22:36, 3 March 2006 (UTC)[reply]
As an American music fan, I have to say the only Japanese pop artist I'm familiar with is Hikaru Utada, who recorded with Timbaland a while ago. -- Наташа ( UserTalk ) 01:12, 4 March 2006 (UTC)[reply]

sorry..but i must not agree with Utada Hikaru XP as a she's only popular because of Kingdom Hearts..but other than that..her music fairly is horrible XP. A few really great J POP groups i'd have to say are Mika Nakashima, Suiteshi-shojo (spelling), Yosuke Sakanoue and there's many more. It's all about tastes, so you might like Utada Hikaru. O and one really famous J POP artist would be T.M. Revolution. Personally i'm more into my J Rock, and Harajuku music. But Definately T.M. Revolution is a big hit in America, and Utada Hikaru only has reputation because of simple and clean..her other music isn't as great.

Other Jpop Artists: Every Little thing, Ayumi Hamasaki, L'arc~en~ciel (more j rock but really famous) sorry if this message is really confuzing

67.150.6.212Aijuko

Difference Btw[edit]

Are there any difference between a hospital and a medical center? I've read what the hospital article said by the way. and also about the size, which is larger? In the wiki hospital article it says it "usually conduct operations in virtually every field of modern medicine". Does this mean that general hospitals do not conduct operations in virtually every field of modern medicine? thanks

No. Purely a marketing choice. alteripse 01:54, 3 March 2006 (UTC)[reply]

A medical center may contain any medical practice - such as strictly outpatient care. A hospital is expected to include in-patient care as well as have some sort of emergency room. So, a hospital is a type of medical center, but not all medical centers are hospitals. --Kainaw (talk) 03:48, 3 March 2006 (UTC)[reply]
Hospitals all have in-patient care but some do not have emergency rooms. But some medical centers are hospitals and more. Rmhermen 16:15, 3 March 2006 (UTC)[reply]
In Britain hospital means hospital and medical centre/health centre means a building shared by a number of GPs (family doctors). Jameswilson 23:50, 3 March 2006 (UTC)[reply]
I'd agree with alterprise, at least as far as the US is concerned. In real estate it's location, location, location; in most of the rest of business, it's marketing, marketing, marketing. Ça me plait pas. On the other hand, I think there should be a lot more articles on business in Wikipedia. Surely we're not all pointy-headed intellectuals. Or are we? --Halcatalyst 04:28, 4 March 2006 (UTC)[reply]

Big pants in The Tempest?[edit]

In Act 2, Scene 2 of The Tempest, what the heck is going on? Is Trinculo physically inside Caliban's pants with him? What kind of garment is a "gaberdine"? (The article says it's a kind of fabric.) —Keenan Pepper 15:01, 3 March 2006 (UTC)[reply]

It is a kind of cloth, but that's not really important. Caliban lies down. Trinculo wonders if it's a fish or man, makes comments about Londoners being stupid, and thinks it's a dead man. Caliban thinks Trinculo is another of Prospero's airy spirits. As the rain comes, Trinculo gets under Caliban's cloak (a gaberdine overcoat/cloak). This is to shelter from the rain. Stephano comes along, drunk and drinking, and Caliban thinks it's another spirit, and a very bad one (because of the singing). He stands up, and Trinculo, who knows that he's under the cloak of a live man, rises with him. Stephano thinks Caliban is a man with four legs (Caliban's and Trinculo's) (hilarity for the audience). He talks to Caliban, and Trinculo recognizes his voice. He speaks (probably peaking out of the cloak at the audience, but unable to see Stephano), and Stephano thinks it's a two headed, four legged man, with a good and bad voice. Trinculo eventually calls out to Stephano and emerges from the cloak. Geogre 15:14, 3 March 2006 (UTC)[reply]
Ah, okay. I had this edition that glossed "gaberdines" as "pants", which confused me. Makes more sense if it's a big overcoat. Heh, some of these jokes are pretty funny: "How cam'st thou to be the siege of this moon-calf? Can he vent Trinculos?" —Keenan Pepper 15:44, 3 March 2006 (UTC)[reply]
She said the man in the gaberdine suit was a spy. --BluePlatypus 16:25, 3 March 2006 (UTC)[reply]
Be careful, his bowtie is really a camera.
As for the play, I have always found The Tempest less funny -- maybe because of all the cosmic portent in it and maybe because it's one of Shakespeare's plays where we see the differences between his values and our own very clearly -- but you can imagine that scene as being a great one for slapstick humor. (Two characters that bug us most, Malvolio and Caliban, with Shylock coming in, I think, third.) Geogre 18:35, 3 March 2006 (UTC)[reply]

Attorney General Ashcroft[edit]

We need a current office address for the retired Attorney General so that we can send an invitation to some upcoming events this year. Does anyone know the address, number or fax or email of how to currently reach his office? thank you.

I suggest you contact the Department of Justice --Halcatalyst 21:46, 3 March 2006 (UTC)[reply]

the article of Jack the Ripper disappear[edit]

It shocks me!Can someone help restore it?Alltonight 16:52, 3 March 2006 (UTC)[reply]

Looks fine to me now. —Keenan Pepper 17:22, 3 March 2006 (UTC)[reply]
I quite agree with you Keenan Pepper.
You probably suffered a database glitch. I've had a few of these where I get the "Wikipedia does not have an article with this title" page on something that does exist. The problem usually corrects itself after a couple of minutes. DJ Clayworth 19:38, 3 March 2006 (UTC)[reply]

Roman festivals[edit]

How many days out of the year did the Roman empire celebrate festivals?

A lot. You could see Fasti. AnonMoos 21:22, 3 March 2006 (UTC)[reply]
I believe that people in Rome worked only something like 100 days per year. (Or is this an exaggeration?) DirkvdM 08:55, 4 March 2006 (UTC)[reply]
Since slaves were people in Rome, it's clearly not true for all... - Nunh-huh 13:17, 4 March 2006 (UTC)[reply]
Right, I think I should have said 'Romans in Rome', which sounds a bit odd. Not all Romans lived in Rome (I think the term refers to 'free men' throughout the empire) and not all those who lived in Rome were called Romans. DirkvdM 09:04, 5 March 2006 (UTC)[reply]

Argentina's official names[edit]

I heard that the Argentine constitution recognizes 4 official names for the country: Argentine Republic, Argentine Nation, Argentine Confederation, and United Provinces. So, is Argentina the only state in the world with 4 denominations?

Well, I live in Argentina and here AFAIK the only official name is "Argentine Republic"; as stated in the article, "Argentine Nation" is only used for legislation purposes. The denominations of "Argentine Confederation" and "United Provinces" are almost extinct. --OneEuropeanHeart 20:25, 3 March 2006 (UTC)[reply]
On the other hand, countries with several official languages often have different names in each language. For example, Switzerland, with four languages, is officially the Schweizerische Eidgenossenschaft, Confédération suisse, Confederazione Svizzera, Confederaziun svizra, and for good measure, Confœderatio (or Confoederatio) Helvetica. --Anonymous, 02:22 UTC, March 4, 2006.
Switzerland certainly uses their names more too, the Swiss franc has four names on it. The Argentinian peso only uses "Argentine Republic". --BluePlatypus 12:32, 4 March 2006 (UTC)[reply]
  • Actually the Swiss franc only has the Latin name; there isn't room for four names on a coin (same with stamps, using the even shorter form "Helvetia"). It's their larger denominations, the paper money, where the four names are used. --Anon, 11:11 UTC, March 5, 2006.
What about the US? United States of America, United States, America, States, USA, U.S.A, US, U.S., US of A, U.S. of A. and Columbia (!?). That's 11! Not very official, though. But what is the official name? The article is called just United States, which doesn't seem right to me. However, someone has once warned me not to go down that road because "there be dragons there". :) DirkvdM 09:15, 5 March 2006 (UTC)[reply]
Well, the US Constitution uses "the United States". Superm401 - Talk 03:48, 6 March 2006 (UTC)[reply]
That's very odd indeed. The full name is 'United States of America', from which the official name uses the first bit and common usage the last bit. DirkvdM 06:00, 6 March 2006 (UTC)[reply]
The constitution seems to be having a bet each way. The full first paragraph is: "We the People of the United States, in Order to form a more perfect Union, establish Justice, insure domestic Tranquility, provide for the common defence[1], promote the general Welfare, and secure the Blessings of Liberty to ourselves and our Posterity, do ordain and establish this Constitution for the United States of America." JackofOz 06:05, 6 March 2006 (UTC)[reply]

March 4[edit]

Author[edit]

What Author/Novelist Can be a combination of Jorge luis borges and Stephen King,Also What Author can be a Combination of Jorge Luis Borges And H.P. Lovecraft?

You can. Try it.
Slumgum 01:04, 4 March 2006 (UTC)[reply]
What do you get if you cross a gorilla with a porcupine? I don't know, but I bet it gets a seat on the bus. JamesMLane t c 07:50, 4 March 2006 (UTC)[reply]
I'd say Jorge King and H.L. Luis. GeeJo (t) (c)  11:15, 4 March 2006 (UTC)[reply]
I think this is the third time this chap has asked this question. Persistent, if nothing else. Markyour words 13:04, 4 March 2006 (UTC)[reply]
Fourth at least. (S)he obviously didn't like my earlier answer of Ray Bradbury. Um... Edgar Allan Poe? Gabriel Garcia Marquez? Jane Yolen? Pugh? Pugh? Barney McGrew? Grutness...wha? 13:35, 4 March 2006 (UTC)[reply]
I think it's a koan, and the questioner is trying to help us achieve enlightenmnet. Geogre 13:58, 4 March 2006 (UTC)[reply]
Buddhism? Fugeddaboudit. Achieve enlightenmnet® wid Wikipediduh. --Halcatalyst 15:27, 4 March 2006 (UTC)[reply]

Law question[edit]

Currently i have a major question that i need to answer it within this week. Question: what are 'the effect of the Human Rights Act 1998' on statutory interpretation' (UK)?

i am very thankfull to you if you can help me to answer this question ...

Read the top of this page - don't post your e-mail address.
Slumgum 01:52, 4 March 2006 (UTC)[reply]

You'll have to be more specific. I don't even know what country you're talking about. If you want to provide more information, click on the <edit> button you see at the right of the screen, scroll to the bottom of the window that pops up, and write there. We're willing to help but can't write your paper for you. --Halcatalyst 02:37, 4 March 2006 (UTC)[reply]

I think it's for the UK. (It says it at the end of the question in brackets). - Akamad 13:29, 4 March 2006 (UTC)[reply]

the marriage tax[edit]

In the US, some politicians talk about "the marriage tax," claiming tax law discriminates against married people (and by implication favors living together). Yet married people filing federal income taxes jointly usually save money over filing separately. What evidence is brought forward to support the idea that there is a "marriage tax"? --Halcatalyst 05:05, 4 March 2006 (UTC)[reply]

Thank you. An informative article. --Halcatalyst 15:22, 4 March 2006 (UTC)[reply]

Persia VS Greece[edit]

Hello everyone, i was just wondering if you could help or provide me with some information regarding the significance of the battles of Thermopylae and Artemisium in 500-440BC involing archaeological and written sources. i have tried searching but have failed if you could help it would a great deal thank you.

I remembered this question being asked before, so I checked the archives and saw that you were pointed at the articles for the battles, which weren't sufficient. You'll find that the more specific the question you ask here, the better your response. GeeJo (t) (c)  10:12, 4 March 2006 (UTC)[reply]
  • i would look up "persopolis" on some Iranian websites - maybe... Amirman 11:02, 4 March 2006 (UTC)[reply]

If you think about it, an archeological source doesn't help much explain the significance of a battle in any way except confirming that it took place as described. Significance is always a matter of historical interpretation and construction-- armchair speculation at its best. Written sources may be better at providing an interpretive construction, but we often reject the interpretive constructions of ancient writers as inaccurate from our POVs. Our principal written source for both battles is Herodotus, who is wonderfully readable, but does not provide a discussion of "significance" in our modern academic sense. alteripse 13:08, 4 March 2006 (UTC)[reply]

  • When you say, "historical interpretation and construction-- armchair speculation at its best," do you mean that historical interpretation and construction are no more than armchair speculation, or rather that armchair speculation is a species of historical interpretation and construction?!? Does armchair speculation have any other important usages? --Halcatalyst 19:07, 4 March 2006 (UTC)[reply]
    • When one is assessing significance of an event, it is a matter of thinking about alternatives and what-ifs, weighing factors and results. It is an armchair exercise, rather than field detection or scientific research. It isn't bad, and can be quite entertaining. It is rarely a "useful" exercise, but rather an example of entertainment for an intellectual elite. That is why it pays so much less than mass entertainment. Do you disagree? alteripse 04:01, 5 March 2006 (UTC)[reply]

Info on portrait of Countess Potocka[edit]

My family in England posesses a portrait (25x21 inches) in an elaborate gold and black frame, of Countess Potocka (or Potoska). I have read all the information about the Potocki family in the Wikipedia encyclopedia and have not come across a similar picture.

I would like to find out which Countess Potocka is represented in our portrait and whether or not she is an ancester of our family.

My mother was told that she was the wife of Count Stanislaw Kostka Potocki (whose name was Aleksandra Lubomirska), but I do not know if this is correct. My mother was also told that she was somebody's mistress (a courtesan). I know that there was a Countess Potcka (née Hélène Massalka) who was first the Count Vincent Potocki's mistress (he was already married and then got a divorce) and then his wife. This Countess Potocka lived partly in Paris and had another famous lover. However, I haven't found out anything about this Count Vincent Potocki.

Can you advise me how I could find out more about the Countess Potocka in our picture?

Thank you in advance for any suggestions you can make. Yours sincerely Charlotte de Montalivet

This website seems to have a detailed family lineage of the Potockis: [3], [4], [5], [6], [7],

[8], [9]. David Sneek 14:45, 4 March 2006 (UTC)[reply]

Thanks for this - when I obtain more info on my Countess Potocka this will be the next step!

Without the Soviet Union, is it now safer to invade Cuba?[edit]

A hardliner is expected to replace Fidel Castro. This hardliner may distance Cuba further from the United States politically.

Is it safer to invade Cuba now than when the Soviet Union was here? If we invade Cuba, how strong would the army resist, how soon would the entire country fall, and would we soon make it our 51st State? --Shultz III 17:18, 4 March 2006 (UTC)[reply]

You'll have to take a number. Mr. Bush has his hands full right now. --Halcatalyst 18:57, 4 March 2006 (UTC)[reply]

It's certainly safer; Russia is far less likely to respond with nuclear weapons now than the Soviet Union would have been. As for resistance, that's a military issue and well beyond the scope of an amateur answer. Perhaps you should write the Joint Chiefs of Staff and ask? As for making it a 51st state, the chances are slim to none. The US government hasn't pushed Puerto Rico to become a state over our century-plus of control there, nor are we attempting to formally annex Iraq, Kuwait, Bosnia, Somalia, or any of the other sites of our recent military incursions. — Lomn Talk 20:02, 4 March 2006 (UTC)[reply]
About making Cuba the 51st state, it is closer to the continental US than Alaska. This is in great contrast to Iraq and the other places you mentioned. --Shultz III 21:38, 4 March 2006 (UTC)[reply]
I don't like my next-door neighbour, so I'm going to steal his house. Is it safer to invade now that he hasn't got a guard dog?
Slumgum 21:50, 4 March 2006 (UTC)[reply]
More like, "That competitor down the next block isn't affiliated with (name of major corporation) anymore, so is it safer to buy them out? --Shultz III 22:12, 4 March 2006 (UTC)[reply]
I can happily trade with them. They're not a competitor. We are in different lines of business.
p.s. It's wrong to edit other people's comments.
Slumgum 22:24, 4 March 2006 (UTC)[reply]
In general, an external threat is by far the fastest way to unite a country behind its political leadershÌip, even if that government is not particularly well-liked. It's fairly safe to say, though, that most countries the size of Cuba would have no hope of resisting the full force of the US military were it to be applied, particularly as the logistical problems facing the US in places like Iraq would not apply.
But, frankly, such an invasion is unlikely (absent Castro doing something crazy like attempting to acquire a nuclear weapon, or his death leading to political chaos). Without the Russians, Cuba is strategically near-irrelevant. If I may be blunt, if it wasn't for the frothing at the mouth of Cuban exiles who happen to be in a swing state, nobody in Washington would care much about the place any more. It may be a tinpot dictatorship with scant regard for human rights, but Cuba is hardly in the worst rank of those. --Robert Merkel 02:28, 5 March 2006 (UTC)[reply]
Conspiracy theorists would say that the main reason the Castro regime survived the end of the Cold War was because the US government feared that a freer Cuba would lead to an influx of Cuban immigrants to the US. So the US didnt apply as much pressure on Cuba as might have been expected. Jameswilson 03:10, 5 March 2006 (UTC)[reply]
After the Cuban missile crisis the US promised not to attack Cuba anymore. But did they make that promise to the USSR only and is it therefore no longer valid now that the USSR no longer exists?
The entire population is trained at guerrilla warfare. From early childhood Cubans are taught how to use guns. There is even an annual 'defence day' when the whole country enacts an invasion (or rather the defence against it). Then again, I've experienced one such day and it looks more like a show than an actual training. But I'm sure the people will resist massively. Quite a lot of people complain about (or even hate) the government, but the hatred for the US is rooted deeper. People haven't forgotten about the past just yet (and of course they are regularly reminded of it). And I don't just mean the Bay of Pigs, but more the situation before the 50's revolution, when the US were effectively in control and there was severe poverty (which caused the revolution to succeed because Castro had popular support - actual physical support). And of course how the US cheated the Cubans out of their revolution in 1898. So the US would have to fight for every street. That is the sort of war one cannot win, irrespective of the military force. The US may take over officially, but the country would be impossible to handle.
Jameswilson, the US were already doing all they could (short of military intervention) to bring down Cuba, so they could not step up their measures after the fall of the USSR. DirkvdM 09:42, 5 March 2006 (UTC)[reply]
Btw, funny, yesterday I recorded a movie called '51st state'. Haven't seen it yet, but I doubt it'll be about Cuba. :) DirkvdM 10:47, 5 March 2006 (UTC)[reply]

And why would you think the US would want Cuba? Maybe we could transfer Miami to them and both countries would be better off. Dalembert 13:06, 5 March 2006 (UTC)[reply]

Addition: In the 1950's that handful of untrained revolutionaries managed to beat a regular army in about two years' time thanks to the popular support. Now, however, all those people are also trained soldiers. That's an army of 11 million! And an interresting effect of this is that the government have to keep them relatively happy. If these people would have nothing to lose, like in the 50's, they would easily overthrow the government. I had thought about an alternative to the army for years, and those thoughts came together when I was in Cuba. There, they have the solution (at least in principle); make it an unpleasantr prospect for anouther country to invade you. Organise the underground resistance before the war breaks out. Big dif is, in Cuba they still have a regular army. Perfection this system and they could do away with it altogether. That should save them a fair bit of money. DirkvdM 18:52, 5 March 2006 (UTC)[reply]

Why would the US want to invade Cuba? They're not an enemy country in any way. All they have is a different political philosophy. Why can't you just get along? There is no logical reason why the US would be interested in it. Grutness...wha? 05:35, 6 March 2006 (UTC)[reply]

Maybe the US got tired of crude oil and want sugar instead? *take cover* __earth (Talk) 11:09, 7 March 2006 (UTC)[reply]
No, if we invade, it will be for the cigars and baseball players. --Kevin Myers | (complaint dept.) 04:57, 8 March 2006 (UTC)[reply]

South Carolina's Metros[edit]

S.C. has only about 4 true metros.Now I'm wondering if its possible in the future,will these metros be good contributors to South Carolina in the future?

Columbia, South Carolina Charleston, South Carolina Greenville, South Carolina Spartanburg, South Carolina

Andre' Ward

Sure, it's possible, OTOH, they might end up losing money and cost the taxpayers plenty (assuming they're publicly financed, which is probably a pretty good bet -- I'd put $0.25 on it). --Halcatalyst 19:00, 4 March 2006 (UTC)[reply]
I just answered with the assumption that metros meant metroplitan areas. Do you mean public transportation? If so, you have left off many cities in SC with good public transportation: Myrtle Beach, Orangeburg, Florence, Beaufort - just to name a few. Are you asking if the public transportation system will ever become a profit system for the state? Why limit this to SC? Why not ask if public transportation in *any* city will ever become profitable? Even the national bus, train, and air systems can't find a way to turn a profit. --Kainaw (talk) 00:56, 5 March 2006 (UTC)[reply]

March 5[edit]

Need help- why am I having trouble getting along with Australians?[edit]

Online, the only Australians I remember interacting with were corrosive and difficult to get along with. I'll not go into too many details because I think they're too personal to give out. There were two on Wikipedia, and another two on Wookieepedia, a Star Wars wiki.

Here, one berated me for making fun of Kim Il-sung and Kim Jong-il, saying that "this is an encyclopedia, not a Kindergarten." He might as well berate someone for making fun of Hitler, bin Laden, Hussein, or Stalin.

However, I seem to get along just fine with the vast majority of non-Australian users.

I know non-verbal gestures that are benevolent/benign in American culture may prove offensive in others (like rubbing the bottom of your chin is very offensive to Italians), but online, we never even see each other so that isn't relevant.

Therefore, what must I watch out for while speaking/posting at or around Australians that I usually don't need to watch out for while doing the same at or around people of other nationalities? --Shultz III 03:16, 5 March 2006 (UTC)[reply]

Firstly, sorry if my comments in regards to Cuban exiles offended you.
But in general, I don't think there's anything specific about Australians that you need to be careful of when conversing online; most Australians who regularly use global internet fora are well-versed in dealing with Americans. I suspect your issues may be to do with a small sample size; in other words, maybe you have managed to find two Aussies with whom you have a personality clash. It happens. Back when I was in high school, two American exchange students were in my class in the same year. One was a blond Californian cheerleader who, while not unintelligent, had clearly had any capacity for irony or negativity removed at birth, the other a redhead from Rhode Island with a dry, rather cynical wit. They were chalk and cheese; a perfect illustration that assuming that people born in the same nation will share similar personal characteristics is inappropriate. --Robert Merkel 09:11, 5 March 2006 (UTC)[reply]
Have you ever had any clashes with Dutch people? I'm one and we're generally pretty much to the point, not beating about the bush, calling things by their name and such, which can be rather offensive with 'politically correct' US'ers (especially me, I should add). So there's usually no offence meant, you just have to interpret it the right way. With Aussies it may be something similar. You don't give any explicit examples, so that makes it hard to judge this, but my experience with Aussies is that they like to take the mickey, but that's generally tongue in cheek. Aussies are also known for their 'rude' language, but that's jus the way they talk, so you have to interpret that the right way. The lingo (strine) is quite 'colourful' as exemplified by Kevin Bloody Wilson. Strikingly, though, on the internet they usually don't say 'fuck' twice per sentence, but maybe that's because there may be ladies present. And they like tall stories told with straight faces, but that doesn't seem relevant here.
That said, if you start comparing people to Hitler you're asking for it. :) DirkvdM 19:13, 5 March 2006 (UTC)[reply]

I've only ever met two Australians, through business, and went out to dinner with them. They stole literally everything they could: knives, forks, salt and pepper pots, beer glasses and a plate. I thought this was probably the reason that their ancestors were sent to Australia in the first place. Otherwise I got along fine with them, and I can forgive Australia for a lot of things, because it's given us Neighbours!
Slumgum 19:33, 5 March 2006 (UTC)[reply]

Mr. --Shultz III,
I have seen several of your postings, like the one about invading Cuba. Your problem is not with Australians, I believe that you do not get along with anybody that is not from your church in Kansas. Perhaps in the future you can think more thoroughly your questions. LG, Greensboro, NC

telephone amusement[edit]

i own a telephone, and live in canada. what 1-800 or toll free places can i call to amuse myself? are tHERE ANY PLACES WHERE I CAN CALL TO LISTEN TO MUSic on the telephone? or just whatever?--152.163.100.72 04:16, 5 March 2006 (UTC)[reply]

If there were, they'd be busy all the time. 1-800 numbers are mostly run by people who want your business, whereas you are expected to pay premium rates for entertainment lines. Try owning a radio instead. --Shantavira 10:20, 5 March 2006 (UTC)[reply]
:-) He could also try calling customer help numbers for credit card companies, cable companies, power companies, and telephone companies (since he owns one of those). He'll get to hear music for free for quite some time while he's on hold. Geogre 13:52, 5 March 2006 (UTC)[reply]
You could call companies that make things that you buy and ask questions about them, like: "How do you get all of that ketchup into your ketchup chips?" You can find the number on the package. -LambaJan 20:16, 5 March 2006 (UTC)[reply]

Okay, thanks. I'll try to call credit card companies to listen to music and then hang up when they answer. I already own a radio, btw. --152.163.100.72

  • That's really going to put them out. But if they're not fuzzed enough, send them a bill for your time waiting. --Halcatalyst 01:43, 6 March 2006 (UTC)[reply]

In Vancouver you used to be able to call a bunch of numbers to play trivia games, get various bits of info, etc. It was listed in the front of the phone book and operated by BC Tel, which I guess has since been taken over by Telus. I have no idea if it still exists. I used to play a telephone music trivia game as a lonely teenager. moink 01:04, 6 March 2006 (UTC)[reply]

  • Many services like that exist for cell phones, and a number still exist for regular lines. These contain ads or are ads. Several of the free cell phone calls are, for example, ads for a website. Obviously, Moviefone in the US is a very thriving business, but there are a number of others. In some municipalities, the city or region pays to have free medical advice or "learning annex" stuff available from a tape library. Those were interesting to me, at a certain age, as I could merely listen to the medical tape on sexual intercourse to get happy. (Ahhhh, the days before the Internet brought hardcore pornography and every expression of fetish to every person in the world, regardless of age or maturity.) Geogre 13:34, 6 March 2006 (UTC)[reply]

Dreaming quotes[edit]

My teacher has given me an excercise: to create a creative piece based around a particular quote on dreaming (not sleeping dreams, but dreaming -- hoping)

What would be a good one, whether it be from literature or spoken? gelo 08:35, 5 March 2006 (UTC)[reply]

"I have a dream" (Martin Luther King) is the best one I know of. JackofOz 09:01, 5 March 2006 (UTC)[reply]
There are many quotations on dreams on wikiquote. My favorite is by Marcel Proust: "If a little dreaming is dangerous, the cure for it is not to dream less but to dream more, to dream all the time." David Sneek 10:19, 5 March 2006 (UTC)[reply]
Which is why Proust spent his whole life moping in his bedroom and trying to remember all the time he had lost... alteripse 12:58, 5 March 2006 (UTC)[reply]
"What happens to a dream deferred? Does it shrivel like a raisin in the sun?" When dreams are deferred, additional dividend income must be claimed by the shareholder to the taxation department. This income is not subject to dividend taxation rate reductions and is taxable at the full marginal rate. If this deferral can be countered with a dream supplement for low dreaming rate individuals, then the primary income derived from this dream, when exercised in the current tax year, may be used to reduce the taxpayer's marginal rate through tax credits. If however, no dream supplement can be claimed, then the full deferral time must be considered a capital gain and be taxed accordingly. AnonMoos 23:25, 5 March 2006 (UTC)[reply]
LOL! -LambaJan 22:55, 6 March 2006 (UTC)[reply]
We can all fly as high as the highest dreams we can dream. Except penguins. Markyour words 19:44, 6 March 2006 (UTC)[reply]
I'm dreaming of a white Christmas Black Carrot 00:03, 7 March 2006 (UTC)[reply]

birth record[edit]

hello, my name is valerie campbell and i am reasearching my family tree and have come to a full stop and dont know how to find out anymore,please could you help me? i have a relative catherine MALLON born in Mexico in 1850ish, she is on the 1871 cencus as being a british subject, i have tried all the records of consular births on the 1837 web site, and have tried to contact the mexican consulate with no success, i wonder can you tell me if there are any records anywhere of people who become british citsens, as i am now assuming she was not a british subject at birth, any help you can offer will be gratefully accepted, thank you valerie

The Saperia family apparently did it this way]. But Mallon is an Irish name, not Spanish, so maybe she was always of British nationality and her parents just happened to be in Mexico when she was born. Jameswilson 00:20, 6 March 2006 (UTC)[reply]
You could try the British records office, if she did become, rather than be born as, a british citizen they'd probably have a record of it. I doubt if these records are on the internet though so you might have to pay them a visit. AllanHainey 14:01, 6 March 2006 (UTC)[reply]
Having had another wee think on this if she was born in Mexico in the 1850's it is highly likely that she was the daughter of an Irish or Scottish soldier fighting in the Mexican War. During that war 47% of Zachary Taylor's army was foreign born (about 24% Irish) & a large number of them turned their coats & went to fight for the Mexicans (due to shared Catholic religion, money & attraction of Mexican senioritas). Also I believe the Mexicans generally relied a lot on mercenary forces. Infact the most effective military group on the Mexican side was the Saint Patricks Batallion made up of Irish (most of them), English & Scottish born soldiers who deserted from the Union armies. It is possible that in the 1871 census she was recorded as a British subject because she/her parents wanted to keep British nationallity (handy thing to have in the 19th centuary) or couldn't get Mexican citizenship, or possibly she acquired/re-acquired British citizenship between the 1850s & '70s. AllanHainey 13:15, 7 March 2006 (UTC)[reply]

British/Indian Geography[edit]

In which Presidencies were Itarsi and Harda situated during British rule in India?

Thank you

None, I would guess. Though they are both in Madhya Pradesh, so must be close to Bombay Presidency. http://harda.nic.in/ has some history, though it's pathetically written. deeptrivia (talk) 14:43, 5 March 2006 (UTC)[reply]

Baby birth[edit]

Is there any particular period of the year when maximum babies are born or the births are evenly distributed throughout the year?

Yes, there is. I will give you an example relating to teen births in Texas here: [10] - scroll down to page 8. It is a beautiful application of a particular type of time series analysis, but unfortunately, we have no articles on nonextensive entropy yet. --HappyCamper 17:24, 5 March 2006 (UTC)[reply]
This pdf, on and around p 45, has statistics for the entire U.S. in the year 2000, broken-down both by month and day of the week. The variation between the minimum birth rate in April and the maximum in August & September is less than 10%, though. I'd expect things could be quite different in other countries with different cultures and climates. ×Meegs 17:50, 5 March 2006 (UTC)[reply]
I had been told before that the Christmas/New Year holidays caused a spike in British/American births around August/September. The reasoning is that there is a lot of drinking and celebration, which translates to a lot of sex, which translates to a lot of births. But, when I researched this, I checked all countries that celebrate Christmas on Dec 25 and New Years on Jan 1. It turns out that only countries in the northern hemisphere see that spike. In the southern hemisphere, the spike is 6 months away: Feb/Mar. Then, I read a sociology study on cold weather and sex which claimed people have sex more in the winter than in the summer. I always meant to look for research to find if the birth spike is greater in countries furthest from the equator - since they would have the most correlation between cold weather and sex. But, I never got around to it. One more nugget I found was a study on birth control. Before birth control's popularity in the 60s, the end-of-summer spike was more prominent. Since birth control, birth rates are rather similar all year around. --Kainaw (talk) 18:08, 5 March 2006 (UTC)[reply]
The variation is so small that planning, perhaps in anticipation of the academic calendar, may play a significant part too. I too have often heard the hypothesis that seasonal cold temperature causes (not simply correlates with) high conception rates, but I'm not sure how well it's been studied. I've also heard speculation tying births to the distribution of marriages through the year (something like summer wedding – winter conception – fall birth). ×Meegs 18:26, 5 March 2006 (UTC)[reply]
The paper I linked to above addresses this. Actually, it is a follow up to another related paper. To be brief, conceptions are highly correlated with the school year. --HappyCamper 18:36, 5 March 2006 (UTC)[reply]
All this is about the time of year. I don't know how precisely one can pinpoint the day of conception (based on birth statistics?), but I think there's a good chance that those take place more in the weekends. DirkvdM 19:23, 5 March 2006 (UTC)[reply]
This depends on the quality of the data set being analyzed. It is possible to detect weekly periodicities, and effects of holidays. For example, see page 16 of this paper by the same group of researchers. "...First of all, we notice that the number of births corresponding to Saturdays and Sundays are much smaller than those of the week days. In addition to Saturdays and Sundays, there are other days with very small number of births that are identified with holidays..." - I cannot find the paper I am looking for, but there is another related one published which illustrates how they were able to extract these periodicities from the data set. I don't recall the researchers going so far as pinpointing the day though, as they seemed to focused on something more important that came out of the data set - the discrepancies between the unmarried and married teens. --HappyCamper 20:30, 5 March 2006 (UTC)[reply]
I now wonder if social circumstances, which may be caused by the day of the week, can determine whether a birth will take place. Can a woman (consciously or not) 'hold back' a birth because circumstances aren't right? That would mess up this whole research (bloody women :) ). DirkvdM 06:07, 6 March 2006 (UTC)[reply]
Not really. But doctors can induce women early in the week, or schedule Caesarians, at their convenience and the parents'. moink 06:31, 6 March 2006 (UTC)[reply]
Exactly. "Births" in the developed world are somewhat artificial in their timing. Doctors can not only induce when it has "been too long" (with "too long" having some relationship to the weekend), but Caesarians are going to happen when the surgical staff is present and ready, as there is a good chance of delaying a day or two to miss the weekend. For that matter, the normovariant in term among humans is vast. "9 months gestation" is far more approximate than you might think, so concluding anything precise about conception is really stretching. The bulge in the curve might be close, but that's only because 9 months is the bulge of the curve in gestational period. If one really wanted to know about conception, one would get better data with surveys than with trying to work backward from births. Geogre 13:41, 6 March 2006 (UTC)[reply]
Precisely, but that would involve asking people about their sex life and that is notoriously unreliable. DirkvdM 08:46, 7 March 2006 (UTC)[reply]
It actually introduces an interesting subject. When a woman gets pregnant, she and the father both begin thinking about the night of conception. Most couples will be able to answer the question. However, they are likely to romanticize. If they're an active couple, they'll pick the night they wish the conception was. If they're sporadically active, they might know the actual time. If they're rarely active, they'll know. So a young/newly married couple will pick that luxurious night after dinner and dancing, even if the culprit was the morning quickie two days before or the argued-into-it-when-she-was-sore-and-bored session two nights later. The unreliability will be due not so much to embarassment but to self-delusion. Geogre 13:02, 7 March 2006 (UTC)[reply]

Gallipoli[edit]

Could anyone direct me to a good source describing Gallipoli's impact on future political events? I realise what it did for Turkey (Kemal Ataturk) and for the Anzacs but I need a good source for my history project and I've trolling the net for hours!

Thanks.

You do realize that not all sources are online? On this topic, you should probably read some actual books. Secondly, define "future political events". For instance, you could do an interesting, if rather provocative project on the current mood of Anzacophilia and government-encouraged enthusiasm for the Australian military in the context of the Gallipoli myth (not that Gallipoli didn't happen, but the story is told with emphases and omissions that ensure it resembles more a morality tale than a factual account of the campaign and its context), but that may well not be what you have in mind. Finally, realize that the percevied impact of Gallipoli (and more generally, WWI military history) on future political events depends rather strongly on who's doing the telling. The conservative side of politics may prefer, for instance, to airbrush the New Guard, our very own bunch of mainly WWI-veteran fascists, out of our history.
That said, you could try the Australian War Memorial website for some good, if somewhat tepid, interpretations of events. Relevant Wikipedia articles include Conscription in Australia, and ANZAC Day, amongst others. --Robert Merkel 00:19, 6 March 2006 (UTC)[reply]


Thanks. I've read a few books on this topic (1915 by Lyn Macdonald) but there doesn't seem to be much specialisation in what I am hoping to write about. Do you know anything else of use?

Denis.

If I recall correctly, this disaster was masterminded in part by Winston Churchill. The result didn't help his career any...suppose he had had the opportunity to be PM earlier, during the 20s say, and later viewed by the public as 'already having had his chance' and therefore unavailable in the 30s and 40s. That would certainly have an effect on history. It is possible to argue that Gallipoli helped delay Churchill's rise, which was ultimately of great benefit to England at least. JK

Interesting theory, but who knows who might have led Britain through WWII if Churchill hadn't have been there, and who's to say whether the alternative leadership might have been equally effective? Churchill's government made some very good decisions, but probably some dumb ones as well (it's not at all clear that the strategic bombing campaign was worth the enormous resources put into it, for instance). --Robert Merkel 05:24, 10 March 2006 (UTC)[reply]

Anthony Eden?? JK

Merv Griffin Show theme?[edit]

Where can I find a copy of the Merv Griffin Show theme? WashingtonWillie 03:56, 6 March 2006 (UTC)[reply]

There is a DVD set "The Merv Griffin Show: 40 of the Most Interesting People of All Time" set to be released in the US on April 11th. That conceivably might have the theme song on it. --LarryMac 22:22, 6 March 2006 (UTC)[reply]

Tonight's Oscars[edit]

I can't remember now...after which joke did host Jon Stewart say, "I am a loser"? --138.88.122.208 05:46, 6 March 2006 (UTC)[reply]

After the one he didn't tell because he lost it? DirkvdM 06:08, 6 March 2006 (UTC)[reply]
You mean the "some people say Hollywood is a cesspool...I don't have a punchline. I just thought you should know that" bit?

House of Reps.2005 Australia[edit]

11 members of which party were suspended from the House of Representatives in 2005? also, where to search using keywords to access info. Thanx! 203.59.81.50 05:22, 6 March 2006 (UTC)[reply]

You could try searching by typing your keywords in the box with the word "Search" next to it -or- go to http://www.google.com and typing your keywords in the search box that appears there. --Kainaw (talk) 18:03, 6 March 2006 (UTC)[reply]

Canadian Money[edit]

I found some Canadian money from the 1950s, 1960s, 1970s and 1980s, $42.50 to be exact. Is all of it still valid as currency? i.e. if I take it to the bank I can exchange it for actual American money. WashingtonWillie 17:36, 6 March 2006 (UTC)[reply]

Yes, it's still valid currency. Based on the info at Canadian dollar, it would appear everything post-1871 is still valid currency, though collectors' prices would likely eclipse face value that far back. — Lomn Talk 18:57, 6 March 2006 (UTC)[reply]


The Universal Currency Converter says that 42.40 in Canadian dollars is worth US$37.2953. Obviously if there is anything collectible in your selection this could increase, and exchange fees would surely decrease that amount. Still, it's a couple of cases of Molson and some Tim Horton's donuts, eh? --LarryMac 22:17, 6 March 2006 (UTC)[reply]
As to the coins, your bank is very not at all likely to be willing to convert them; it's not worth the trouble. Coins from 10 cents upward that are old enough to contain silver (i.e. they aren't attracted by a magnet -- the changeover was in the 1960s sometime) may carry a small premium value at a coin dealer for that reason, but again, for small amounts it won't be worth the trouble. As to paper money, it's certainly still valid but that doesn't mean your bank will accept it; they may not know what the old designs of Canadian paper money looked like, and not think it worth the trouble to find out. No harm in asking them, though. --Anonymous, 23:23 UTC, March 6.
ISTR the changeover to cupro-nickel in most Commonwealth countries was about 1947. Grutness...wha? 01:12, 7 March 2006 (UTC)[reply]
Please don't confuse things by bringing in other countries with different coinage histories. The only Canadian coin that has ever been cupro-nickel is the 5 cent coin (the "nickel") and then only from 1982 to 2001. Silver coins stayed silver until the 1960s. --Anonymous, 05:48 UTC, March 7, 2006.
My apologies - I had assumed that all commonwealth countries changed over at te same time. I'd forgotten that much of the world's nickel is from Canada and that it would therefore make more sense for them to have changed to that. Grutness...wha? 12:47, 7 March 2006 (UTC)[reply]

Bermondsey Town Hall, London, England[edit]

Would like information (and perhaps some pictures please) of the Bermondsey Town Hall built in late 1800's. We understand it was designed by one George(?) Green who was related (Grandfather?) to my husband. The Town Hall was destroyed during WWII. Thank you for your help.

You might get better luck trying to contact Southwark Borough Council, or perhaps trying this website. Grutness...wha? 01:07, 7 March 2006 (UTC)[reply]
 THIS IS FROM FAIRMONTHOTELS .
                                                        THE MANAGEMENT OF FAIRMONT HOTEL WISHES TO INFORM  INTERESTED APPLICANT OF THE VACANCIES IN THE HOTEL.WE NEED SOME CAPABLE MEN AND WOMEN TO COME AND WORK AND STAY IN THE HOTEL NEWLY OPEN BRANCH IN USA AND CANADA,DEPENDING ON YOUR CHOICE.THE HOTEL WILL TAKE CARE OF YOUR RESERVATION, TICKET AND VISA. IF YOU ARE INTERESTED IN WORKING WITH US YOU CAN CONTACT THE EMPLOYMENT DEPARTMENT ON THE BELOW EMAIL ADDRESS:
 [email protected] 
FOR MORE INFORMATION AND PROCEDURE. 

THANKS,LINDA

PERSONNEL MANAGER
   (FARMONTHOTEL)

Briefing Paper[edit]

I am so frustrated. I cannot find anywhere how to write a political brief to the Royal Commission or Inquiry.

Unfortunately, we're frustrated by a lack of information. What Royal Commission or Inquiry are you attempting to contact? Is it at a national or subnational level (and please specify exactly which national or subnational entity it is)? — Lomn Talk 23:30, 6 March 2006 (UTC)[reply]
Alternately, your question could be read as "how do I write a political brief generally?", so it would help if you'd clarify that as well. — Lomn Talk 23:32, 6 March 2006 (UTC)[reply]

March 7[edit]

Question #1[edit]

June 6th, 1944?

See June 6, 1944. Superm401 - Talk 01:43, 7 March 2006 (UTC)[reply]

Superman[edit]

Ok, follow me on this. Superman's first appearance was in 1938. Assuming he's 29 in that appearance he would have been born in 1909. What kind of transportation would Ma and Pa Kent be taking to find a crashed rocketship in Smallville, Kansas?

The Model T was introduced in 1908. I don't know if they would be wealthy enough to be early adopters though. moink 02:14, 7 March 2006 (UTC)[reply]
Afaik he didn't land as a baby but had grown into a kid by the time he landed. (I am hiding my head under the table for shame for knowing this). Anyway, who says he grew at the same speed that humans do? DirkvdM 08:54, 7 March 2006 (UTC)[reply]
I remember a flashback showing him as a baby when his earthly parents first got him. I'm not ashamed of liking comics and being 150 years old :-). --Halcatalyst 15:49, 7 March 2006 (UTC)[reply]
Why not check the nicely-written Wikipedia article on Superman? The "Golden Age" origin has Kal-El being found by the Kents circa WW I. I too love my comics, although I'm only 147 ;-) --LarryMac 16:36, 7 March 2006 (UTC)[reply]
Actually, I was not referring to comics in general but to Superman specifically. But that's probably a cultural thing - I wouldn't be ashamed of any knowledge of Asterix because being a Dutchman (European?) I naturally grew up with that. DirkvdM 08:37, 9 March 2006 (UTC)[reply]

Seven Different Films for the 7 Major Oscars[edit]

The 7 Oscars for Best Picture, Best Director, Best Actor, Best Actress, Best Supporting Actor, Best Supporting Actress, and Best Cinematography are sometimes referred to as the "major Oscars". This year, the 7 majors went to 7 different films. Has this ever happened before? JackofOz 02:28, 7 March 2006 (UTC)[reply]

Never if you count cinematography as major. If you only count the other 6, then it's happened twice. In the order listed above, for movies made in 1952: The Greatest Show on Earth, The Quiet Man, High Noon, Come Back Little Sheba, Viva Zapata!, and The Bad and the Beautiful; but The Quiet Man and The Bad and the Beautiful both won for cinematography (there were separate awards for color and B&W at that time). And for movies made in 1956: Around the World in 80 Days, Giant, The King and I, Anastasia, Lust for Life, and Written on the Wind; but Around the World in 80 Days won the color cinematography Oscar. (Somebody Up There Likes Me won for B&W.)
Other people have other ideas of what is a major Oscar -- some exclude the supporting parts, some exclude directing, some count the two screenplay awards -- and obviously these choices will produce different answers.
--Anonymous, 06:08 UTC, March 7, 2006.
George C Scott and Marlon Brando notwithstanding, I'm sure most people who won any Oscar at all would regard it as a major achievement in their life. Thanks for that answer. JackofOz 07:58, 7 March 2006 (UTC)[reply]

A related question - Rachel Weisz won best supporting actress with the leading female role in The Constant Gardener. Have there been other times when the leading female in a movie won Best Supporting Actress (or the leading male won Best Supporting Actor, for that matter)? Grutness...wha? 12:49, 7 March 2006 (UTC)[reply]

Lots of 'em for the ladies. I did a quick scan, there may be some mistakes: Jennifer Connelly (A Beautiful Mind), Marcia Gay Harden (Pollock), Kim Basinger (LA Confidential), Marisa Tomei (My Cousin Vinnie), Jessica Lange (Tootsie -- well maybe Dustin was the leading female there), Tatum O'Neal (Paper Moon -- won as supporting actress, though she's the main character), Josephine Hull (Harvey). Less so with the guys. Jack Nicholson in Terms of Endearment (or was Jeff Daniels the main guy, I don't remember). Jim Broadbent in Iris, perhaps, although the guy who played the younger version of the character may share the lead. Jason Robards (Julia) is the only clear cut example I can think of. --Kevin Myers | (complaint dept.) 16:34, 7 March 2006 (UTC)[reply]
Timothy Hutton in Ordinary People. Does Joel Grey in Cabaret count? User:Zoe|(talk) 21:10, 7 March 2006 (UTC)[reply]
If Jake Gyllenhaal had won for Brokeback Mountain, that would have qualified. I never understood why Heath Ledger had a BA nomination for his role as Ennis, but Jake (only) got a BSA for Jack Twist. In what sense was Jake's role subordinate or supporting of the role of Heath's? JackofOz 22:02, 7 March 2006 (UTC)[reply]
Note first that the supporting awards are actually called "Best Performance by an Actor/Actress in a Supporting Role"; it's entirely possible that all of the leading roles in a film are male and the largest female part would then be characterized as supporting. This is certainly true for L.A. Confidential, and I would say it also applies to some of the other movies listed above. Of course the reverse situation is also possible, though obviously less common.
The second point is that there is no official definition for what is a supporting role. Personally, I always like to see the award go to someone who was onscreen for only a short time and yet made a memorable contribution to the movie, like Helen Hayes in Airport and Judi Dench in Shakespeare in Love. But others prefer it to go to meatier parts that are still not lead roles. In fact the studios themselves decide whether to campaign for the nomination of a particular acting job as a lead or supporting role, and sometimes they choose the supporting category because they think they have a better chance of winning that way.
--Anonymous, 00:56 UTC, March 8, 2006.
Joel Grey in Cabaret doesn't count, since Michael York was the male lead.
Given the oddities of the nominating criteria, and the existence of many good ensemble films, an even stranger category are those actors who won the "Best Actor in a Leading Role" for a part in which they were arguably not the primary male lead in the film. All of these are debatable, but F. Murry Abraham in "Amadeus", Dustin Hoffman in "Rain Man", and Marlon Brando in "The Godfather" come to mind. The last one was somewhat controversial, because Al Pacino was nominated for best supporting actor, even though he's arguably the main character for much of the film, or at least the co-lead. It was said that the studio nominated Pacino in the supporting category so as not to "split the vote", and so ensure that Brando would win, which he did. But what happened in the supporting category is that the Godfather supporting actor vote was split between Pacino, James Caan, and Duvall, and so the supporting Oscar went to Joel Grey, who definitely did not deserve the Oscar over Pacino that year.
My guess is that for Brokeback Mountain, the studios put the guys into different categories so as not to split the vote, giving them both a good shot at winning. Sometimes when they put two leads into the same category, neither end up winning, like Sarandon and Davis in "Thelma and Louise". Though not always: Debra Winger and Shirely MacLaine were both nominated as leading actress in "Terms of Endearment, and MacLaine won. (The wrong one of the two, IMO.)
Another recent minor controversy was Anthony Hopkins in "Silence of the Lambs" -- nominated and winning for Best Actor, though he's off screen for much of the film. His characterization was so powerful though, that it felt like he was in every scene. --Kevin Myers | (complaint dept.) 02:15, 8 March 2006 (UTC)[reply]
When Cabaret is performed on Broadway, the actor in the emcee role is always nominated for the Tony Awards for Best Actor, not Best Supporting Actor (I can't remember the terms they use on the Tonys). User:Zoe|(talk) 02:57, 8 March 2006 (UTC)[reply]
I'm guessing there's a big difference between the film version and the stage version (which I've never seen). In the film version, the emcee is definitely not the lead role: if I remember correctly (and I don't always), it's practically a cameo role, with no character development and no involvement in any of the major plot points. --Kevin Myers | (complaint dept.) 04:25, 8 March 2006 (UTC)[reply]

Amendments[edit]

Which amendment gives the state of Virginia the power to make laws?

See United States Bill of Rights. The question to your homework question lies somewhere within that page. Dismas|(talk) 05:04, 7 March 2006 (UTC)[reply]
Better to look at the 1624 charter of the Virginia colony. Rmhermen 02:06, 8 March 2006 (UTC)[reply]

Was the Bible a Wikipedia?[edit]

It's a collection of writings evolving over time, collaboratively produced by various people of uncertain identity. It's at once authoritative and unreliable. It's controversial. It comes with concordances - the equivalent of a search box (or am I pushing the analogy too far here?) Adambrowne666 06:48, 7 March 2006 (UTC)[reply]

You might try writing an essay on it, but you'll have to do it yourself. Here's an interesting tidbit: the wikipedia is ranked number 20 on Alexa's global top sites, higher than any site whose primary focus is religion. For that matter, so is Adult Friend Finder ;) --Robert Merkel 07:03, 7 March 2006 (UTC)[reply]
Good answer, thanks - though disappointed you're not offering to write the essay.Adambrowne666 07:43, 7 March 2006 (UTC)[reply]
Being a 666, maybe you can start the ball rolling with the chapter on Revelations. JackofOz 07:47, 7 March 2006 (UTC)[reply]
Or 1 Kings, for that matter - the only other place in the Bible where that number is mentioned (there, IIRC, it refers to the amount of gold given in tribute at Solomon's Temple. many an interesting theory could be written about that!) Grutness...wha? 12:54, 7 March 2006 (UTC)[reply]
  • Seeing as the Bible was a collaboration of whoever wrote the old testament and the people who wrote the Gospels (and all the people who did translations after it), you could consider it to he a collaborative project although most people didn't work on it concurrently. The bible was not a Wikipedia thought. Wikipedia is an encyclopedia, the Bible is not. The word you're looking for is a wiki (which technically it isn't either, because it's not an online collaboration). - Mgm|(talk) 09:07, 7 March 2006 (UTC)[reply]

was the bible a collaboration? No. The authors worked independently and largely oblivious of each other (although there is of course much quoting of the earlier by the later ones). It was only compiled into a single "work" much later, so if you want, the Bible is a "Project Gutenberg" with a very paranoid board of editors (there was a lot of bickering about who should be in and who shouldn't). Can the bible be a wiki? yes. Is that a good idea ("from the holy scripture anyone can edit"; "can a thousand monkeys write a holy book")? possibly, but you'd need even more faith in the community than with Wikipedia... dab () 09:38, 7 March 2006 (UTC)[reply]

Torah redactor Biblical canon involve editing and deciding what stays and goes. Also Bible translations, and whatever edited for special-audience Bibles are called - children's versions and so on. Шизомби 10:20, 7 March 2006 (UTC)[reply]
There was some collaboration - in a very Wikipedia-like sense. There's one book in particular (I forget which - Ecclesiastes, maybe? Or one of the prophets?) which starts in one style, then about halfway through changes to a completely different writing style. It's clearly the work of two authors, one building on the "stub" started by the other. Grutness...wha? 12:54, 7 March 2006 (UTC)[reply]
Well, if you're referring to a book like the Book of Proverbs, it's a collection of sayings, and therefore it's an editorial compilation to start with (see the Proverbs of Alfred for what a literate clerical tradition mindful of historical accuracy can do to mess stuff up). Therefore, things by Solomon and things like things said by Solomon would get in. It's an agglutinative oral tradition. Or are you thinking of the two Isaiahs? In general, though, the books were not intentionally put together where anyone could edit. In fact, oral traditions tend to be remarkably, surprisingly accurate when we can compare "originals" to their late versions, so it's more like "the works anyone can copy." The copyists tried their best (and their best was quite something when you got to the scribes of Alexandria) to be totally accurate. Geogre 14:51, 7 March 2006 (UTC)[reply]
You might be thinking of the book of Isaiah, which scholars commonly divide into three parts due to evidence of different authors and time periods. --Halcatalyst 15:47, 7 March 2006 (UTC)[reply]
That's the one! thanks. Grutness...wha? 00:55, 8 March 2006 (UTC)[reply]
Don't forget Genesis, in which one part says Noah took two of each type of animal into the Ark and another part says he took seven of each type. User:Zoe|(talk) 03:00, 8 March 2006 (UTC)[reply]
Yeah, well Genesis is so riddled with inconsistencies that anyone who says it is the absolute truth clearly can't have read it - let's face it, man and woman were created on the sixth day (Genesis 1) but hadn't been created by the seventh day (Genesis 2), and what to make of Genesis 6:1-4? But this isn't the place to discuss that. Grutness...wha? 06:44, 8 March 2006 (UTC)[reply]
Well, the article on Noah's Ark, which Zoe was referring to, does a great job of presenting the fact that the Christian church had a thousand years or more of people who were quite content to see Genesis as absolutely true but not necessarily literally true. That story and the story of Jonah and the whale (leviathan) were problematic on a literal level and yet quite true. Let's put it this way: when Jesus spoke of a man who had two sons, and one of them left, was the story literal? Was it true? Literalism is like The Life of Brian, where Brian begins telling a story, and he says, "There was a man..." and the listeners object, saying, "What was his name?" Early church fathers applied a four-fold exegesis to the Bible: there were four simultaneous levels of true, and any one of them could be priviledged over another. Geogre 14:15, 8 March 2006 (UTC)[reply]
Is such an exigesis discussed anywhere on Wikipedia? --Halcatalyst 21:35, 9 March 2006 (UTC)[reply]
It wasn't the last time I looked, but it's also a tough one to write about. Do you put it into exegesis? Do you call it patristic exegesis? Do you call it Augustinian exegesis, when it came from several Fathers and not just him? I learned it as "four-fold patristic exegesis," but I doubt that's the most common way. The exegesis is covered by Augustine and, interestingly, by Bede. However, I'm not expert enough to write the article, as I keep getting confused and generally remember three levels. There is the literal, the allegorical, the anagogical, and the... the... I forget the fourth. One is the literal narrative. One is the soul's journey. One is Christ and the Church. One is the apocalypse. Each story in the Bible can be exposed to all four levels, but, according to Augustine and Bede, not each story has as much to tell you on all four levels, and that's one reason that a story can be fully true and yet have a less important or more important literal meaning. Geogre 22:16, 9 March 2006 (UTC)[reply]
Well, even Wikipedia has natural limitations. The topic will certainly be found in books in a theological library; maybe in an encyclopedia of religion; and some similar articles can be found on Wikipedia. But since there is no central plan, it has to just happen. [Given time enough, it will, just like monkeys working for a long, long time will reproduce the works of Shakespeare. Haw.] --Halcatalyst 04:18, 12 March 2006 (UTC)[reply]

Harris County[edit]

How much percent of Harris County, Texas is covered by built-up urban area?Bowei 07:51, 7 March 2006 (UTC)[reply]

One way to get a very rough estimate is to consult a satellite photo, e.g. from Google maps. Better yet, such an image in false colors (where red usually shows the most densely populated areas, and there might even be multiple colors). You could proably find such things in any good university library, the closer to Houston the better, of course. --Halcatalyst 02:15, 8 March 2006 (UTC)[reply]
Also, the article itself lists a large number of cities (including Houston) and only a few unincorporated areas, so this may be an indicator. Did you try following the external links at the bottom of the page?
Finally, I've been there a few times and it looks mostly urban to me. You can drive many, many miles and you're still in the metropolitan area. --Halcatalyst 02:22, 8 March 2006 (UTC)[reply]

Who are the owners of the ECB?[edit]

Dear Wikepedia,

  • Who are the owners of the European Central Bank?
  • Are the Council of Ministers elected as their counterparts in the European Parliament?
  • If no, then who appoints them? --Harsha Sankar question reformatted and email removed by --Robert Merkel 14:26, 7 March 2006 (UTC)[reply]
See European Central Bank. It's not really owned by anybody; it's a body established by the EU treaties between the EU nations. The Council of the European Union ministers are the ministers of the national governments. --Robert Merkel 14:26, 7 March 2006 (UTC)[reply]

slavery[edit]

What would you say are the differences between classical slavery (practised in Greece and Rome and in many other cultures) and the European slavery of the 17th-18th centuries? In the one case, people were made slaves as a result of military defeats; in the other they were, in a sense, harvested and sold for money. --Halcatalyst 16:06, 7 March 2006 (UTC)[reply]

That's a very broad question. You could easily write a book or two exploring the similarities and differences. Since slavery is almost universally immoral (basic empathy dictates that), there's always been some kind of justification for it. In ancient Greece, it was "part of the rules of the game". I defeat you, you become my slave. You defeat me, I become your slave. But even then there were differences. The Spartans had no qualms over using Greeks as slaves, which Athens and the rest felt was immoral. African slavery worked on the same principles (and there were variations in the attitudes there too). The difference was that foreign traders showed up, prepared to pay well for slaves. So tribal war degenerated into pure slave-hunting. The Europeans (and Arabs) who bought slaves, on the other hand, justified it through racism (Africans just weren't people) and religious reasons (Africans weren't Christians, so the morals of slavery didn't apply to them). But the Greeks had similar justifications; non-Greeks just weren't people (or non-Spartans or non-Romans). It's more or less impossible to make a moral distinction here. The Greeks and others sold slaves too, the difference was that there wasn't as large a market for them. So one main difference is the economic incentive. A second difference is that the moral hypocrisy was stronger in the case of the 18th century slave trade, since none of the European countries involved allowed slaves at home. The attitudes in Europe varied as well; some countries banned their citizens from participating in slave trade. So from the start, the later slavery was much more controversial. Probably because the justifications for it were weaker. (Few white people were being held as slaves in Africa). --BluePlatypus 16:50, 7 March 2006 (UTC)[reply]
Last claim is nonsense; learn some history. Muslims from Morocco to the Ottoman empire raided, captured and held European slaves until the beginning of the 19th century, up to 25,000 in Algiers alone. As late as the 16th century, Arab corsairs raided Iceland, Ireland, and the English Channel for slaves. Into the 17th and 18th centuries, English churches regularly made collections to ransom British slaves in North Africa. For a brief overview see Paul Johnson, The Birth of the Modern (NY: Harper-Collins, 1991), chapter four, beginning p286. Dalembert 01:29, 8 March 2006 (UTC)[reply]
Learn reading comprehension; that's not what I was referring to. I was referring to whites being held by black Africans as slaves. The context was justifying slavery through both parties holding the other in slavery. Surely you're not disputing that the number of white Europeans being held in slavery by black Africans was completely insignificant in comparison to the opposite situation? --BluePlatypus 12:58, 8 March 2006 (UTC)[reply]
You wrote "Africa", not "subsaharan Africa" or "black Africans". I responded to your assertions as written because I had no way of knowing that wasn't what you meant. Dalembert 17:23, 8 March 2006 (UTC)[reply]
I think that you can expand on that second difference. In the classical world, the slaves (I think) were an integral part of the home culture. Slaves were visibly necessary to the comfortable life to which those with political power aspired. In the European case, slavery may have been held to be necessary (indeed it was argued the sugar industry would collapse without it) but it was not obviously necessary to everyday comfort among the expanding electorate. They were therefore happier to consider abolition because any inconvenience would have been someone else's problem. Notinasnaid 22:43, 7 March 2006 (UTC)[reply]
Roman and Greek and Persian (and even "Barbary") slaves could purchase their freedom or serve out an allotted term. Enslaved Africans in America could not. Further, the children of slaves in Classical societies were not slaves. The children of enslaved Africans were slaves as well. The manumission of slaves by price in the Classical world was highly regulated, and a master could not choose to disobey. Enslaved Africans were manumitted only when the slave holder felt like it, which was rarely. Quality of life is something else: it isn't quantifiable. Largely, the "field slave" in the US suffered greatly because of the type of labor. Plantation economies were vicious to all and fatal to most, so there was no special hatred of Africans necessary for there to be unending misery. The closest analogy in the Classical world would have been slaves in the mines -- a fate usually considered worse than death. On the other hand, enslaved Africans on the domestic side had the same "part of the family" affiliations that Greek slaves in the households of Roman nobles would have had. However, there is no way to quantify what the loss of self-determination is worth, nor the utter hopelessness of a slave system that had no way out, such as the ones in the New World. N.b. when indentured servitude and slavery existed side by side (the early years of the Virginia colony, some of the Carribean colonies, and Suriname), the indentured servants (fellow Europeans) were treated worse than the enslaved Africans, but they knew that they would get free. The Africans knew that they never could. Geogre 04:05, 8 March 2006 (UTC)[reply]
There are some overstatements in your answer. African slaves could purchase their freedom in America -- though it didn't happen often, to be sure. John P. Parker is an example of one slave who did purchase his freedom: as a blacksmith, he was able to save money (skilled slaves sometimes keep some of their earnings) and purchase his freedom. And the statement that plantation slavery was "fatal to most" is misleading: work on U.S. plantations was hard, but not so fatal as to prevent the natural increase of the slave population from generation to generation. (Slaves represented a sizable capital investment, which slave owners were loathe to squander.) Slavery was evil, but not necessarily deadly. In the decades before the U.S. Civil War, slaves in the U.S. had a longer life span than Irish peasants, which I suppose says more about misery in Ireland than about slavery. --Kevin Myers | (complaint dept.) 04:52, 8 March 2006 (UTC)[reply]
Well, hang on. I agree with you that slaves were a large capital investment. In fact, they were so enormous an investment that having two or three slaves meant that one was extremely rich ($10,000 per slave in 1840), and therefore there was little interest in any plantation owner being a Simon LeGre. However, I was referrring to the plantation economy, and not plantations in the US. In Barbados, Jamaica, Central America, and early on in the US, people died on the plantations. The tobacco plantation after around 1800 was not so much, but the sugar cane plantations slaughtered their populace, owner and slave alike. Part was due to tropical disease. Part was due to heat and hydration. Part was due to accidents. Part was due to malnutrition. In the 1600's, indentured servants got their headright after 7 years in the colonies in America, and yet I think only about 20% of them lived that long. Conditions improved vastly as more slaves were put to work, as medicine improved, and as swamps were drained, etc. They improved also when the owners were residents, and not living in London and drinking the profits and demanding ever more. At any rate, the point that I wanted to make about buying one's freedom was that most southern states in the US had no statutory requirement or set price. In the Classical world, such laws existed. (Absolutely no argument about the fatality of the early capitalist economies vs. the plantation economies in lifespan. Freed slaves themselves noticed this. Again, though, no one can set a price or value on self-determination.) Geogre 13:42, 8 March 2006 (UTC)[reply]
Slaves weren't that expensive. That price seems to be adjusted for inflation. See TRADE RESTRICTIONS AND FACTOR PRICES: SLAVE PRICES IN EARLY NINETEENTH CENTURY US (caps theirs) which has similar values but they are 2003 dollars, not early 1800's dollars. Rmhermen 19:11, 8 March 2006 (UTC)[reply]
You may be right. I think I got my information from Genovese, but that's ancient and hotly debated. I think the general point would survive, though: slaves were too expensive for overseers to be killing them willy-nilly (which is one reason they mutilated slaves as punishment, I suppose). Geogre 22:39, 8 March 2006 (UTC)[reply]

March 8[edit]

Moveable Feast[edit]

I read the above article with interest but still do not understand why Easter (the "deathday" of Jesus) is a moveable feast whereas Christmas (the "birthday" of Jesus) is celebrated on the same day each year.

The date of Easter is based on the date of the Jewish feast Passover. The date of Passover is derived from the Jewish calendar (rather than the Gregorian calendar) which is is lunar rather than solar. It is based on the time it takes for the Moon to orbit the Earth (28 days) rather than the Earth to orbit the Sun (365 days.) Because these numbers do not divide exactly into each other a certain date in the Jewish calendar does not always fall at the same point in the Gregorian calendar each year. Consequently Passover (always on the 15th of Nisan in the Jewish calendar) can fall on a variety of different days in the Gregorian calendar. Is that clear?--Cherry blossom tree 00:23, 8 March 2006 (UTC)[reply]
Conversely, the date of Christmas is based on winter solstice celebrations such as the Roman Saturnalia. Our calendar is derived from the Roman calendar and is a solar calendar, so it's natural for Christmas to have a fixed date. Although Christians see Christmas as a celebration of the birth of Jesus and it's sometimes thought of as his birthday, it isn't really. --Anonymous, 00:47 UTC, March 8, 2006.
Many people believe that Jesus's true birth date was around February/March, though documentary evidence is slim, to say the least. Grutness...wha? 01:05, 8 March 2006 (UTC)[reply]

Hang on! Easter is not the "deathday" of Jesus. It is the resurrection day! Good Friday is the anniversary of the passion. Otherwise, yes, it's because of the lunar calendar. Christmas, on the other hand, is an assigned celebration based on taking over the Roman Saturnalia. No one ever claimed to know the date of Jesus's birth, so one date was as good as another, in a sense. The only hints to when it was occur in the nativity description on the Gospel of Luke. There, we find that the shepherds come down from the hills. So, why were they up in the hills? Would they have gone up there in winter? Since their flocks were chasing the shoots of grass, it's likely that the time was spring or summer. Other than that, there is no very good evidence for any one particular date. Meanwhile, it was an act of conscious syncretism to take over the Saturnalia/allow Christians to celebrate without being killed during the persecutions. Geogre 02:22, 8 March 2006 (UTC)[reply]

what is the best way to[edit]

--205.188.117.12 03:21, 8 March 2006 (UTC)[reply]

With your bare hands. But don't tell anyone I told you that. ☢ Ҡiff 04:46, 8 March 2006 (UTC)[reply]
Go straight down High Street until you reach the bridge, then make a right turn. You'll come to a level crossing - go over than and take the third left. Keep going for half a mile and you're there! Grutness...wha? 06:48, 8 March 2006 (UTC)[reply]
  • I can't believe you are answering this like that! It's clear he shouldn't do it, period. :) - Mgm|(talk) 09:33, 8 March 2006 (UTC)[reply]
  • Two tutus too many to tout. Geogre 14:09, 8 March 2006 (UTC)[reply]
  • Amway? --Halcatalyst 04:38, 9 March 2006 (UTC)[reply]
  • My way? DirkvdM 08:57, 9 March 2006 (UTC)[reply]
  • No, the highway. Not High Street, the highway. --Halcatalyst 21:37, 9 March 2006 (UTC)[reply]

british columbia[edit]

What role does the legislative branch in B.C. play in the parliamentary system?

See request for clarification on your talk page. Essentially the legislative branch of the provincial government is arms length from Ottawa (the Federal government). While the provincial government recieves funding from the Federal government, the involvement is mostly dealth with meetings and requests for stuff - if you need any further clarification please let me know -- Tawker 03:34, 8 March 2006 (UTC)[reply]
  • I really didn't ask this question, someone just put it under my header--205.188.117.12 04:34, 8 March 2006 (UTC)[reply]

African American?[edit]

I am editing under a sock to remain anonymous I will be reading this under my normal login name so don't put a message on my talk page, post it here.....Both of my parents are white but my mother was born on a military base in africa. So in a sense she is african american. Does that make ME african-american in a legal sense?--M1X3D 04:23, 8 March 2006 (UTC)[reply]

That depends on your point of view. To the government, you'd be an American that was born in Africa. If you'd like to think of yourself as an African American, then you can go ahead and think that way. It's all pretty much pointless since if you believe that we are all descended from people that originated in Africa, that would make everyone an African-whatever. Dismas|(talk) 05:30, 8 March 2006 (UTC)[reply]
(edit conflict)I assume that you are in the United States. The "legal sense" of African-American, as far as I know, is that there is no exact legal definition. Most people in the United States do not include Euro-African-Americans in the category of African-Americans. Many people who self-identify as African-American would probably be offended if you presented yourself as one. If this question is in regard to receiving benefits or special treatment as an African-American, it almost certainly doesn't qualify. Oh, I just noticed the military base part. If it was a U.S. military base with an A.P.O. address, it is still considered part of the U.S. and not foreign soil. So no, I don't think you are African-American under any societal standard. —WAvegetarianCONTRIBUTIONSTALKEMAIL 05:32, 8 March 2006 (UTC)[reply]
Is Charlize Theron African-American? User:Zoe|(talk) 16:43, 8 March 2006 (UTC)[reply]
Actually there is an unanswered question on that articles talk page about whether she ever became an American citizen. Does anyone know? Rmhermen 18:58, 8 March 2006 (UTC)[reply]
  • White South Africans are Africans. White people who have citizenship anywhere in Africa are Africans. M1X3D's mom, on the other hand, was born on a military base -- and since that fact is totally useless unless it's an American military base, that means that M1X3D's mom was born an American. It's worth remembering that the USA's notion of granting citizenship to anyone born within its jurisdiction is by no means universal, so if, say, Katie Holmes were to give birth to Xenu Cruise while on vacation in Malawi, Baby Xenu is not Malawian unless the law of Malawi specifically says that all babies born in its territory are citizens.--M@rēino 00:01, 9 March 2006 (UTC)[reply]
The definition of "African American" would depend on the body or organization that needs to differentiate people by race. For example, the California State Personnel Board defines "Black/African American' as "a person whose origin is any of the Black racial groups of Africa". That would seem to exclude white South Africans as well as Arabs of north Africa. -- Mwalcoff 00:09, 9 March 2006 (UTC)[reply]
  • OK, I guess this is a test of how closely we read. The fact that your mother was born on a military base in Africa (or anywhere else) has no bearing on your race, legally or otherwise. --Halcatalyst 04:44, 9 March 2006 (UTC)[reply]

Nazis?[edit]

Since they believed they were a hyper evolved master race of superman, couldn't one say that they were in fact history's strongest supporters of darwin? Not exactly the kind of legacy evolution would want people to think about--Somebodye6436346 05:17, 8 March 2006 (UTC)[reply]

No. They weren't history's biggest supporters of Darwin. They believed themselves to be superior, yes, but I don't think Hitler believed he had evolved from Jews, Blacks, and Gypsies. I'm actually amazed that an i.d. supporter would stoop so low. You may be interested in Godwin's law, which answered this question before I did. —WAvegetarianCONTRIBUTIONSTALKEMAIL 05:36, 8 March 2006 (UTC)[reply]
The connection between Darwin and eugenics is a direct one. --Kevin Myers | (complaint dept.) 05:42, 8 March 2006 (UTC)[reply]
Answer to Somebodye: Your argument is spurious - saying that X supports theory Y rarely says anything useful about Y. Hitler's national socialist ideology had strong links to Christianity, and Hitler himself was a devout Christian [11]. So, by your argument, this 'reflects badly' on Christianity. — QuantumEleven | (talk) 08:24, 8 March 2006 (UTC)[reply]
Evolution isn't about "this species is better than this species". Evolution is strictly a theory that species evolve and/or go extinct over time. An extension is that species either evolve or go extinct due to changes in their environment. Eugenics is the political interpretation that one species is better than another species - which is completely subjective and irrelevant to the theory of evolution. From a strictly biological point of reference it is not a valid argument as you can't argue that humans are "better" than i.e.. gorillas. Humans are suited to their environment - and gorillas to theirs. If a cataclysmic event occurred wiping out civilization it is feasible that gorillas would be more successful than humans - hence you can't say that one species is better than another since succes largely depends on the environment in relation to the specifications of the species. Celcius 09:05, 8 March 2006 (UTC)[reply]
Well, it's possible Hitler was a Christian (by some definition), but he certainly wasn't a christian. JackofOz 09:31, 8 March 2006 (UTC)[reply]
  • Given that the Nazis' racial theories have no basis in genetic and evolutionary fact, the answer is simply no. --BluePlatypus 12:37, 8 March 2006 (UTC)[reply]
First, if anyone is trying to argue guilt by association, it's a fallacy, and a really obvious one. Second, Nazism's racial views were expressions of social Darwinism. Third, although Nietzsche cooked up the concept of the ubermensch, his was more metaphysical than racial. Fourth, Social Darwinism's real roots were in the United States of America, and they have a deep, deep, deep root in contemporary American culture. ("Competition will determine the best X" is a form of it. "Every child can grow up to be President, so any child who grows up to be poor and addicted is weak and nasty and doesn't deserve our money" is a form of it.) Geogre 13:50, 8 March 2006 (UTC)[reply]
Just to clarify, though: apart from inspiration, Social Darwinism doesn't have much to do with Charles Darwin or Evolution. If you ask me, Social Darwinism is a sociopolitical theory crafted to justify social inequality. It's unscientific in two ways 1) It was developed from the foregone conclusion that inequality is natural and good and 2) It's supported by little empirical evidence. That's not how Darwin worked. As hard as it may be for some fundamentalists to believe, Charlie didn't have "Destroy Genesis" at the top of his to-do list when he set out on his voyage. Darwin reached his theory by reasoning from the evidence, not by thinking up the theory and then fitting evidence to it. --BluePlatypus 19:43, 8 March 2006 (UTC)[reply]
For that matter, The Origin of Species had nothing in it that would have bothered Genesis. We need Wallace's theories to get to man, and then Darwin later did The Descent of Man. However, per above, Darwin was living in a time and with a church that wouldn't have had any substantial problem with the idea that Genesis is 100% true and not literal. Geogre 20:32, 8 March 2006 (UTC)[reply]

One poster against the EU showed Hitler saying "United Europe? My idea!" Hitler was all for it, so it must be bad. And anyone who drives a Volkswagen is a fascist of course. DirkvdM 09:01, 9 March 2006 (UTC)[reply]

It should be illegal to make comparisons to Hitler and/or Nazism while trying to get your point across! It's so boring - come on ... you can do better than that! What about Stalin? Mao? Slobodan? Don Quijote? Anyone else - just stop saying Hitler all the time. We get it! He was bad! Celcius 01:59, 10 March 2006 (UTC)[reply]

Public Domain works that contain copyrighted characters?[edit]

I noticed at Public Domain Torrents that there are episodes of Tom and Jerry in public domain. Even though Tom and Jerry are still owned by Time Warner can I legally edit these episodes and lets say splice the character animations from the background of these films and make them do pornographic things in an adult movie I am working on?

These are not MGM's Tom and Jerry from William Hanna and Joseph Barbera, but Van Beuren's Tom and Jerry. A similar question had also been asked before elsewhere. Lupo 10:31, 8 March 2006 (UTC)[reply]
OK, how about Betty Boop, Flash Gordon, or other works that you would need to license to create new works of but their older works are in public domain?
I think these characters are owned by people who can afford much better lawyers than you, and who are not above using the threat of expensive legal action to cause you serious inconvenience, whatever the exact legal situation. They are especially likely to do this if they see their brand being abused in a commercial and pornographic way, it seems to me. Also, characters might be protected by trademarks, which do not expire like copyright. Tread very carefully. Notinasnaid 11:47, 8 March 2006 (UTC)[reply]
  • Every time you need to ask about the legal situation surrounding something you want to do which you're not sure about, simply don't. It saves you a lot of problems. - Mgm|(talk) 09:57, 9 March 2006 (UTC)[reply]

Name of famous 'Greek' footballer whose parents fled the junta.[edit]

Hi, A while ago I read about a football / soccer player whose left wing parents fled the Greek junta after WWII for the Soviet Union. He grew up in his new country but returned as an adult and played at the highest level. However, he was not allowed to play for Greece - perhaps because he had already represented his birth country.

Does anyone know his name?

Thanks

Damian.

I don't know the player you have in mind, but if he had already represented the USSR st full international level then FIFA rules would have prevented him from representing any other country. Until they play at full international level for one country, a player can choose to be eligible to play for the country of his own, his parents', or his grandparents' birth. This is why the Republic of Ireland fielded so many English-born players in the 1980s, since Jack Charlton was expert at employing the "grandfather rule"; there's an English-born player who is the first white person to play for Trinidad and Tobago in something like 60 years, because his mother happened to be born in Port of Spain in colonial times. -- Arwel (talk) 03:40, 13 March 2006 (UTC)[reply]

find an author[edit]

i'm trying to find the name of the british authoress - one of her books became the idea for the TV series "21 Jump Stree." thank you, K

Stephen Cannell claimed he got the idea from a real police unit in L.A., not a book. --Kainaw (talk) 16:08, 8 March 2006 (UTC)[reply]

Misuse of verb "would" in "if" clauses in modern American speech[edit]

As a retired English teacher I am concerned about the frequent misuse of the verb "would" in journalism and common conversation. An example is "If I would have known that earlier, I could have done something about it." Another misuse of this clause is in the use of the past tense when conditional is called for, for example, "If I knew this earlier, I would have done something." There are other variations of this problem, and it leads me to believe that verbs are not being taught in schools any more.

If no one has written about this linguistic phenomenon, I will voluteer to do some research and provide an article. Is this a matter of concern for anyone other than ancient grammarians?

Thanks Margaret McDavid

While you're about it, perhaps you could write about the difference between 'may' and 'might'. Up until about 10 years ago everyone used these words correctly, whether or not they were the type of people to be interested in grammar. But nowadays the use of 'may' when 'might' is demanded has become frequent (especially among football commentators: 'If Beckham had arrived in the box earlier, he may have scored'). Maid Marion 15:30, 8 March 2006 (UTC)[reply]
My college newspaper editor was always going on about the difference between may and might. But I wrote Barbara Wallraff, The Atlantic Monthly's language columnist, and she told me it's nothing to get hung up about. Then again, if people more people were to revert to using may and might differently, it would be a helpful distinction. -- Mwalcoff 00:01, 9 March 2006 (UTC)[reply]
English is a constantly mutating language. Usage is assumed correct by those who use it and is often adopted to become correct. I try to ignore bad English, I agree that it is difficult when it is on the news. Actually, what actually bothers me actually the most is actually having newscasters who actually have a degree in some form of media who actually feel the need to actually use the word "actually" over and over. --Kainaw (talk) 16:05, 8 March 2006 (UTC)[reply]
"Would have"? These days, it's pronounced and even spelled "would of". User:Zoe|(talk) 16:44, 8 March 2006 (UTC)[reply]
If i knew yall was so worried about this kind of things i would of wrote teh article my own self. Amirman 17:04, 8 March 2006 (UTC)[reply]

The first instance is a homophonous misunderstanding, but the second instance seems to me to be a valid subjunctive mood. The subjunctive in English has been dying for some time, now, unfortunately. One commonly hears, "If I was you" and other violations of the subjunctive. Now, we can just relax and be descriptive and say that the battle is lost already, or we can fastidiously correct every mistake. In fact, linguists have been watching the misuse of the subjunctive in common speech, but I cannot say whether or not anything is definitive. After all, hypercorrectiveness shows up from time to time -- often with silly results (e.g. the pronunciation of the /t/ in "often") -- so there is some chance that the subjunctive will make a comeback. However, what seems to be involved is a further simplification of the to-be verb in English. Listen out for the misuse of number in "where's" and "there's": it's common and shocking. I have heard "There's many reasons why the mortality rate is declining" on CNN, and I've heard other "there's" with a plural on multiple occasions from everyday speakers and from professional speakers, so one assumes that this is part of the further simplification and flattening of the to-be verb. Geogre 17:13, 8 March 2006 (UTC)[reply]

I think the problem with the second example is tense, not mood. It should be "If I had known this earlier, I would have done something." —Keenan Pepper 20:16, 8 March 2006 (UTC)[reply]
Thanks. You're right, of course. (And here I am taking mood altering drugs to avoid tense!) Geogre 20:33, 8 March 2006 (UTC)[reply]
Margaret, I understand that such usage grates but its an evolving language. Young people will determine the next stage in the grammatical development of the English language and they have a different culture from the rest of us, in dress, manners, etc and language is part of that. At the moment both forms co-exist (if I had/if I would have). Which form wins in the long-term is anyone's guess, but generally we live in a society where young people want to imitate their "cool" contemporaries not their elders, so my money would be on the latter. If so, in fifty years time "If I had" will be the one regarded as incorrect. Jameswilson 23:50, 8 March 2006 (UTC)[reply]
In the meantime, the existing rules apply. Contrary to Keenan Pepper, I believe it has everything to do with mood and nothing to do with tense. It’s all about where the subjunctive mood applies and does not apply. “If” tells us that what follows in that clause is not something that has happened but something that might yet happen, or something that could have happened but didn’t. That something is not where the subjunctive applies. The subjunctive applies to the consequence of that something happening (the second clause), which is inherently conditional because it depends on something that by definition has not occurred. The first clause therefore requires the indicative mood, and the second clause requires the subjunctive mood. The writer has used the subjunctive mood in both clauses, which is the problem. JackofOz 00:06, 9 March 2006 (UTC)[reply]
There's an Australianism which is at least as bad as Margaret's example. "If I had have known that earlier" is frequently heard over here. Sometimes they even compound the error and write "If I had of known that earlier". (Sometimes I weep for my people.) JackofOz 04:58, 9 March 2006 (UTC)[reply]
Brits do that too, I'm afraid. If I'd of had a pound for every time I'd heard that... --Heron 22:03, 9 March 2006 (UTC)[reply]

We have an article on Formal written English and on various dialects. Perhaps we could use one on English usage controversies or something like that, You could start it, Margaret, and I'm sure lots of people would join in the editing. You have some good material here already. --Halcatalyst 05:17, 9 March 2006 (UTC)[reply]

Hindu undivided family[edit]

There is a concept called 'Hindu undivided family' in Indian tax system. I just want to know whether such a concept is --good / favourable for hindus OR --unfavourable and unfortunate for hindus OR --it has some benifits and some disadvantages for hindus.

If you can tell what are the advantages or disadvatages, it is good.

Do you know why such a concept has been introduced?

(I know this question may relate to India and not many people in USA would know about this, but anyway I am posting. May be some Indian or other citizen who knows answer to this may answer this question.) Thankyou.

This article explains why the the founders of modern India believed that national laws should sometimes be different for individuals of different religions in India. The writer thinks its a bad idea. Only a small bit on different tax regimes for families of different religions in para 7. Jameswilson 00:09, 10 March 2006 (UTC)[reply]

Manifest Destiny...myth, or reality?[edit]

what do you guys think? was the United States truly destined to govern and expand from sea to shining sea? were the natives destined to be pushed off their land? did america really have the god given right to push the natives off their land, and run over their rights, all in the name of "progress?"

~angrodnenharma~

  • Speaking as someone who was not only alive in 1845, but is in fact the living breathing reincarnation of jesus christ, yes, it was absolutly my will--Gsauce 19:57, 8 March 2006 (UTC)[reply]
  • could you please keep the sarcasm out of this? this is serious. i have to write a paper on what at least 10 people think about manifest destiny, so i would really appreciate it if some people would be serious, and give me an answer.

~angrodnenharma~

  • That's funny, because I have to write a ten page paper about people who insert bbb into random sections of reference desk markup code--64.12.116.72 20:28, 8 March 2006 (UTC)[reply]
There is a problem with attempting to know the mind of God, as Job found out. Unless you were with him in laying the foundations of the world, it's a bit of hubris to say what his plan was. I think that those who resort to the idea of destiny or fate or God's will to legitimize an ongoing ideology are almost always masking economic desires as divine imperatives, and "Manifest Destiny" was one such case. (The doctrine even has a theological argument buried in it -- a theology that only a Puritan could love. The rest of us don't necessarily agree that the history of the world is a direct transcription of the divine will. It must be God's will, but it needn't be God's will in each particular; for saying so would diminish free will.) Geogre 20:37, 8 March 2006 (UTC)[reply]
It's obviously made up. The Nazis also claimed they had a god-given right to take over land they claimed was rightfully theirs.
Slumgum 20:45, 8 March 2006 (UTC)[reply]

All of you are confusing manifest destiny as a moral imperative and manifest destiny as a description of an inevitable and obvious outcome. Manifest in this sense simply means "obvious". The expansion did not occur as a result of a moral imperative. It occurred as a result of individual people being individual people. The concept of manifest destiny was a 19th century description of the process. In some contexts the phrase can be construed as a post hoc justification; in others simply as a recognition of inevitability. And mention of the nazis disqualifies anything further Slumgum has to say by Godwin's law. A strong argument for inevitability of expansion can be made, given the circumstances of relative cultural strengths and population densities. Those who want to argue that it was morally wrong should be prepared to go defend the southern border of the US from the continuing waves of immigrants from the south who have exactly the same individual motivations as the expansion from Europe. Grow up, guys: peoples dont migrate by "group minds" with "group consciences". Dalembert 21:21, 8 March 2006 (UTC)[reply]

I've never heard of Godwin's law before, but reading the article made me chuckle. The article Manifest Destiny claims that a suggestion of divine intervention is involved, even if you don't.
Slumgum 21:48, 8 March 2006 (UTC) Thanks for taking it with a chuckle. Dalembert 23:21, 8 March 2006 (UTC)[reply]
I was confusing nothing. Several presidents, as well as the only president of the CSA, regarded the expansion as, indeed, a divine imperative. Individual people are curbed by their governments or encouraged with incentives (land lottery, anyone?) To suggest that the westward migration was just a case of "sh*t happens" is so utterly mechanistic as to be anti-historical. Furthermore, the 19th century is when this was occurring, so saying that it was a 19th century justification is to say that it was an announcement of an ideology. Secondly, the theological argument is not only implicit but explicit. How do you know that God wants you to take this land? Well, you can have a vision, or you can have a holy leader. For the Puritans, and for the various descendant churches, God's will was directly transcribed in history. History was a case of eminance. Therefore, the partial historical fact could indicate what the Divine plan was. Other theologies would have had none of this. For example, for Greek churches, and for the Greeks of the early church for that matter, history and divine will moved by moments of direct correlation, but never by a linear narrative. "In those days the time was come for" would not be progressive, but rather conjunctive. (Auden and Isherwood discuss this well in The Dyer's Hand.) Geogre 22:36, 8 March 2006 (UTC)[reply]
And I did not propose a "mechanistic" explanation or imply that people are lemmings. People make individual choices to their apparent advantage. Why did all the major human population migrations occur? Some occurred as one movement with one leader and one inspirational narrative, but most of the largest occurred over decades for a variety of reasons and motivations. Manifest destiny is a phrase used in many ways even in the 19th century, but mostly as an after-the-event justification, divine or "natural". The idea that the US was blessed by God or entrusted with a civilizing mission by God certainly can be traced back to the 17th century and was alive and well in the 19th, but the concept of "manifest destiny" explicit and popular mainly after coast-to-coast domination was nearly complete and was at best a minor, not a major inducement to migration. Dalembert 23:21, 8 March 2006 (UTC)[reply]
  • thank you for all the help, people, you may continue disscussing if you wish, but i have recieved all of the other information that i need. does anyone object to having some of their ideas put in a paper, along with their user name? if so, please respond, soon oh yeah, i changed my user name, it is not angrodnenharma anymore, it is:

MysteriousStranger 02:13, 9 March 2006 (UTC)[reply]

All text contributions made to Wikipedia are licensed under the GFDL, including those on this page, so even if we all yelled and screamed at you, you'd still be entitled to use the information from here provided the licensing requirements were followed. :) GeeJo (t) (c)  17:57, 9 March 2006 (UTC)[reply]
  • hey geejo....some times its nice ta ask, dude. thanx for all the feed back ppl, it really help.

MysteriousStranger 19:48, 9 March 2006 (UTC)[reply]

I will leave all religious claims aside and concentrate on the inevitability of the hunter/gatherers being replaced by farmers/herders. Native Americans simply could not compete with the new technology brought by Europeans. The only way they could have hoped to keep the land is if they adopted European technology and rapidly increased their population. European diseases prevented this from happening, however. Even if all governments had banned their citizens from taking Native American's lands, they would only have slowed, not stopped, this process. StuRat 05:43, 10 March 2006 (UTC)[reply]

Uhhhh. Ok, first, most of the eastern tribes were agricultural. Second, most adopted European technology as soon as they could. Third, many set up independent democratic governments (incl. most famously the Iriquois and the Creek Indians and the Cherokee). So, they had the language, the literature (in the cases of Creek and Cherokee) and culture, and they had the technology and were agricultural. So what is the evolutionary explanation, again? Was it inevitable in Canada? Was it inevitable in Australia? In fact, the process of colonization does not mean genocide. It does not mean replacement of one people with another. Had there been no gold rushes, would there have been "inevitable" western expansion? Had there been expansion, did it have to be genocidal, or could it have been accomodating? Again, all of these arguments strike me as reductive, mechanistic, and rather apologetic. Geogre 20:52, 10 March 2006 (UTC)[reply]
Replacing hunter/gatherers with farmers/herders doesn't necessarily mean they are different people. One way it can happen is by the hunter/gatherers themselves changing into farmers/herders, which did occur in some cases. The disease issue is not so easy to overcome, though. A larger population coming into contact with a small population will almost always have diseases for which the small population has no immunity. As for westward expansion being all about gold, no, it wasn't. There was westward expansion long before the 1848 discovery of gold in California and most who went west weren't gold miners even after that. StuRat 22:52, 14 March 2006 (UTC)[reply]

US Armed Forces hand salute[edit]

When I was a petty officer in the US Navy, I was taught only one form of the hand salute, with the upper arm horizonal and the elbow as far to the side as possible.

Twice in the past month, I've seen members of the US Armed Forces — an Army and an Air Force sergeant — give surprising (to me) but identical version of the hand salute. Both brought their stiffened right hands up the front of their bodies to their right eyebrows, but kept their hands in front of their faces, almost completely covering their right eyes. Their forearms were nearly vertical and in front of the right sides of their chests. In both cases, the situation was very formal (a funeral and an awards ceremony); both sergeants were wearing gloves. In neither case were they in close surroundings; both had plenty of elbow room.

Is this a standard form of the hand salute? If it is new, when did it begin? Under what circumstances is it used instead of the wide-armed version?

(Please don't refer me to the Wikipedia article on salutes — it says nothing about this form, and in any case I wrote most of it.)

Thanks!

➥the Epopt 19:17, 8 March 2006 (UTC)[reply]

In the US military, a salute should have the line from the shoulder to the elbow parallel with the ground. The placement of the hand is up for debate. I served in the Marines. The thumb points straight out, making the hand as flat as possible (not curled into the palm as some other services require). The tip of the index finger touches the eyebrow (or the brim of a cover if worn). The hand is tilted with the pinky lower than the index finger so that it partially covers the right eye. In formal circumstances, the salute is altered. Instead of a quick pop one-motion salute, the elbow is raised first, then the hand. The hand is tilted so it almost completely covers the eye. Sometimes, the head is nodded slightly. Then, the hand is snapped down with the elbow stationary. Finally, the elbow is lowered. As with most marching, it is just for appearances, not function. --Kainaw (talk) 01:33, 9 March 2006 (UTC)[reply]
Sounds very similar to the Navy salute I was taught. The salutes I saw kept the elbow close to the body so that the forearm and hand were almost vertical, not at the nearly-horizontal angle you describe. And they weren't snappy, they were slowly and smoothly raised, held until returned, then lowered slowly and smoothly. I assumed the first one was simply wrong, then I saw the second, identical one in completely unrelated circumstances, and started thinking it was deliberate. ➥the Epopt 02:29, 9 March 2006 (UTC)[reply]
I was taught by the US Army to salute the same way you guys were. But I've seen this slow-motion, hand in front of eye salute Epopt is describing -- I figured it was some kind of ceremonial thing unrelated to everyday use. I always thought Navy guys did a more casual, less horizontal salute, but that may just be the Hollywood version. --Kevin Myers | (complaint dept.) 03:02, 9 March 2006 (UTC)[reply]
Oh, sailors tend to salute (and march) very casually, but it's because there's not much saluting (or marching) underway at sea, not because we didn't learn any better. ;-> ➥the Epopt 04:16, 9 March 2006 (UTC)[reply]

What happened to this page's header template? It just shows up as a redlink?--64.12.116.72 19:49, 8 March 2006 (UTC)[reply]

  • Ah, nevermind, some idiot but bbb infront of the /, I fixed it, so it's ok now--64.12.116.72 19:51, 8 March 2006 (UTC)[reply]

whyz?[edit]

Why are mice so stupid, but mooses aren't much better?--MickeyMoose 23:28, 8 March 2006 (UTC)[reply]

Because your mom! Black Carrot 23:30, 8 March 2006 (UTC)[reply]
Ahem. Mike H. That's hot 00:02, 9 March 2006 (UTC)[reply]
Why assume mice are stupid? Why assume moose are also? I've never seen a mouse or a moose buy a lottery ticket. --Kainaw (talk) 01:27, 9 March 2006 (UTC)[reply]
Speedy Gonzales was a genius. Bullwinkle, not so much. --Kevin Myers | (complaint dept.) 03:07, 9 March 2006 (UTC)[reply]

Pinky may be stupid...but the brain isn't...don't generalize! ;) --Cosmic girl 03:54, 9 March 2006 (UTC)[reply]

  • You're not called MickeyMoose for nothing. Go and think real hard why Mooses aren't much less stupid than mice. Deep inside you know. - Mgm|(talk) 10:02, 9 March 2006 (UTC)[reply]
Mice aren't stupid - they have sufficient intelligence to survive - that's all you need. What more do you want? Intelligence to perform maths and/or wage war with sophisticated killing-tools?Celcius 01:44, 10 March 2006 (UTC)[reply]

The two main things that make intelligence a valuable trait (and thus one that is passed down) are hunting and being hunted. Animals that do neither, like the dodo bird, are, or in this case were, really stupid. Social animals also require a certain degree of intelligence to keep track of all their relationships. StuRat 05:23, 10 March 2006 (UTC)[reply]

March 9[edit]

Jorge Luis Borges[edit]

What is the name of the novel by Borges that talks about some 'blue marbles that refuse to be counted changing in cuantity every time'?.--Cosmic girl 03:53, 9 March 2006 (UTC)[reply]

mmm... that sounds familiar... The Library of Babel perhaps? Grutness...wha? 06:44, 9 March 2006 (UTC)[reply]

thnx! :D.--Cosmic girl 23:37, 10 March 2006 (UTC)[reply]

Maybe they changed colors too ... are you sure he had not bought an m&ms box ? --DLL 22:23, 9 March 2006 (UTC)[reply]

lol!.--Cosmic girl 23:37, 10 March 2006 (UTC)[reply]

  • I'm pretty sure that's not in Library of Babel. But I don't know which one it is in -- it doesn't trigger any memory for me, and I've read most of the well-known Borges pieces. I'll do a quick run over the title of my collected short stories volume and see if anything jumps out at me. --Fastfission 00:11, 11 March 2006 (UTC)[reply]

Thank you! I apreciatte that! :D --Cosmic girl 02:36, 11 March 2006 (UTC)[reply]

Blind leading the blind (HELP)[edit]

What does blind leading the blind mean int he context of the death penalty? please help me! thanks!

just a guess, but it may be a reference to the fact that - in order to punish someone for murder, the state itself becomes a murderer by killing the prisoner. Grutness...wha? 11:40, 9 March 2006 (UTC)[reply]


That phrase doesn't make sense in that context. It means someone with no knowledge of something trying to educate someone else about it, like Rain Man teaching someone how to flirt.
To the original questioner, try this link: http://www.phrases.org.uk/meanings/67150.html It's hard to see what relevance this has in the context of the death penalty. You'll need to explain the context in more detail if anyone is to help you on this. Maid Marion 15:48, 9 March 2006 (UTC)[reply]
And for the record.. the phrase is actually quite incorrect in many ways. I have often seen blind people showing other blind people how to navigate through a city street without the use of sight, something they probably can do better than a sighted person. Sometimes the blind leading the blind can be a good thing! But anyway this has nothing to do with your question. --Fastfission 01:34, 11 March 2006 (UTC)[reply]
  • People advocating death penalty are blinded by the idea of revenge. Those who listen to them are blinded by ignorance (personal research).
  • Retaliation is desired in most societies, giving a sensation of security. Plenty of Jesuses came to tell us to give the other cheek, religions were created that forgot the message. Only when forgiveness exists may we expect security and peace. Politicians need courage and insight to say no to such violence, if they are blind they lead people downwards. --DLL 11:03, 11 March 2006 (UTC)[reply]

Death date of Otto Böhler[edit]

Can anyone find the death date of Otto Böhler, an artist who created many silhouettes of famous late-nineteenth century composers? Thanks, Makemi 17:00, 9 March 2006 (UTC)[reply]

Anybody?...Anybody?... It would be useful so that I can figure out whether his stuff is in the Public Domain, so it can stay in a couple articles. If anyone has access to Grove's Dictionary of Art, I'm pretty sure he would be in there. Makemi 17:17, 10 March 2006 (UTC)[reply]
I did a few searches with no more luck than you had. I'm thinking this is a question for the German language Wikipedia. I see you have a little German; you could try there. That's what I would do. Or go to a (good) library to look at their Groves. --Halcatalyst 04:04, 12 March 2006 (UTC)[reply]
Thanks. Yeah, I tried the German Wikipedia, no luck. When I get to the library (no longer in University, so it's harder) I'll look it up, I just want to get it before Orphanbot deletes the image. Makemi 05:58, 12 March 2006 (UTC)[reply]
Probably 1913

Confirmed here Böhler, Otto 11.11.1847 - 05.04.1913 Or 1911? Hope this is good enough for the bot.--HJMG 10:46, 13 March 2006 (UTC)[reply]

captain cook tongan visit[edit]

I am trying to find out about a relative of mine who sailed with capt cook and his name was Mariner. he was asid to have become the Kings advisor!!-regards -phil southgate

This website refers to a 15-year-old boy named William Mariner who was stranded on Tonga for four years. He did not sail with Captain James Cook, he was on board the Port-au-Prince in 1806, several years after Cook's visit to the islands, and was a survivor when the ship was attacked by the islanders. Finau Ulukalala, the leading chief, apprently took a liking to him and so this is probably the "Kings advisor" you are referring too. Mariner wrote a book about his adventure called Mariner's Tongan Islands, published in 1817. More information here. --Canley 00:35, 10 March 2006 (UTC)[reply]

is jermine o'neal muslim[edit]

The preceding unsigned comment was added by 194.79.99.10 (talk • contribs) 19:33, March 9, 2006 (UTC).

I looked up info on Jermaine O'Neal and I couldn't find anything about his religion because all of the 'bios' I found were really just player accomplishments. Since all I have to guess by is his name I would guess that he isn't. I know a lot of black american muslims that go to a local masjid and the vast majority of them have arabic names. This is, however, original research and is good only as a speculation. -LambaJan 04:51, 11 March 2006 (UTC)[reply]

Queen Juliana[edit]

http://en.wikipedia.org/wiki/Wilhelmina_of_the_Netherlands

In this article it claims that the birth of Queen Juliana, on April 30, 1909, was considered something of a miracle.

I heard that a famous Rabbi has promised Queen Wilhelmina, Queen Juliana's mother that she will have a child.

Any information on this subject?

That "something of a miracle" line should absolutely be sourced, because otherwise, births are entirely unremarkable in the context of occurrence. As for a "prophecy" that a married woman will have a child, that's only slightly more impressive than my prediction that this answer will, in fact, be visible. — Lomn Talk 19:36, 9 March 2006 (UTC)[reply]
Yes, I read that too, and assumed it wasn't meant to be read literally (as in, eg. Wilhelmina being 70 years old at the time, or something like that). But there's nothing about what was so surprising about the birth. This needs not just a source but an explanation. JackofOz 22:17, 9 March 2006 (UTC)[reply]

Here's Queen Juliana's obituary in English from Radio Netherlands. It says: "Seldom was the birth of an heir to the throne so eagerly awaited as the arrival of Juliana on April 30th 1909. Her mother, Queen Wilhelmina, had remained childless during the first eight years of her marriage to prince Hendrik van Mecklenburg-Schwerin. As historian (and royal biographer) Jan Kikkert points out, the long-awaited birth of an heir to the throne brought a sense of relief." Presumably this is what the person who wrote "miracle" was thinking of. --Anonymous, 23:30 UTC, March 9, 2006.

Right. Maybe somebody should make it slightly less breathlessly POV. JackofOz 02:23, 10 March 2006 (UTC)[reply]
Consider it done. Incidentally, I also considered the "70 years old" possibility, but she was about 29 when she gave birth, so no big deal there. — Lomn Talk 02:36, 10 March 2006 (UTC)[reply]

national and regional governments[edit]

Our article on forms of government (truly an impressive list) has a section on Forms_of_government#By_approach_to_regional_autonomy. In connection with Sovereignty, I wonder: what are the different ways (in different countries) that national and regional governments relate? How is the power distributed? What are the nexuses of politics? Are there any forms which appear to be the most successful? How? Why? Insights appreciated. (Curiosity only -- no book in the making :-)) --Halcatalyst 22:19, 9 March 2006 (UTC)[reply]

I think if you picked thirty countries at random, you'd find thirty different ways in which power is distributed between the different levels of government. Whatever the balance though, one important point is whather the central government can overturn decisions taken regionally that it doesnt like (or doesnt think are in the general national interest). Jameswilson 00:23, 10 March 2006 (UTC)[reply]
They would range from the total absence of any provinces to cases of feudalism, where the national government, if it even exists, is insignificant relative to the power of local tribal leaders/warlords. StuRat 05:15, 10 March 2006 (UTC)[reply]
Not based on any factual knowledge, I'd say that where there is no democracy the national government is likely to be overbearing. Other than that, there is the number of levels (in the Netherlands there are five: EU, national, provincial, municipal and in some cities burroughs) and (in a democracy) whether the government at a certain level is elected directly. In the Netherlands, nationally, there are direct elections at all levels, except for the 'house of lords', which is elected by the provincial governments. And at the EU level there is the parliament, which is chosen directly, and the Commision and Council, which consist of national leaders (so indirect) and these have to somehow work together. France and Germany have similar combined systems.
Another answer is that a higher government makes more general rules that the lower governments fill in to adapt to local circumstances and add to where it doesn't cover those circumstances. As far as I know this always works top-down, but I think the bottom-up approach (anarchism?) makes more sense. This is also a problem with the EU, which is criticised for making rules that are way to specific, as exemplified by the way too long and specific constitution. DirkvdM 10:32, 10 March 2006 (UTC)[reply]

Pyrechmen?[edit]

From a German manuscript from the 1540s, treating the antiquity of chivalric combat and duelling etc., I read "Pyrechmen" or "Pirochmen", and "Degmemnum" or "Degmemien". The variants are from two copies of the text, all readings are reasonably certain. Pyrechmen is a hero who defeated Degmenum. The context is ancient literature, they are mentioned alongside Hector killing Ajax, etc. Does anyone know who these people are? dab () 22:23, 9 March 2006 (UTC)[reply]

  • In Latin or German? Geogre 03:27, 10 March 2006 (UTC)[reply]
    • well, it's the Renaissance. The text is in German, but the Classical proper names are inflected in Latin. dab () 08:02, 10 March 2006 (UTC)[reply]
  • The closest reference I have at hand is an 1848 Lempriere's Dictionary. The reason that's not laughable is that Lempriere includes people who are not particularly mythological. The Oxford Dictionary of the Classical World (unabridged or nothing) would be the next logical stop, but my college is fool enough or poor enough to have only abridgments. Geogre 20:47, 10 March 2006 (UTC)[reply]
  • The book is Herodotus. In Latin translation, Herodotus is saying that Pyrechmen was the dictator who was in Aetolia, who defeated one Degmenum Epeum. Lempriere should have it. As soon as I get the book from my office, I'll tell. Geogre 13:54, 11 March 2006 (UTC)[reply]
My copy of Herodotus (Oxford Classical Texts, ed Charles Hude) has what appears to be a comprehensive index of names, in which neither of these guys is listed. So I think the Google reference may be suspect. By the way, I'm not sure why people are referring to 'luctator' as dictator, unless I'm missing some joke. The word refers to someone engaged in a luctus, ie a struggle, so therefore 'contestant' (perhaps gladiator?) The words 'singulari certamine' of course mean 'in single combat', which does tie up with the original question. Maid Marion 09:22, 14 March 2006 (UTC)[reply]

hitler?[edit]

You probably hit enter without realizing it would post the question with no body. If your question isn't answered by our article on Hitler, try again. —Keenan Pepper 23:22, 9 March 2006 (UTC)[reply]
I wonder if it'd be possible to implement a redirect to the search page in these cases. With a warning, of course. ☢ Ҡiff 05:27, 10 March 2006 (UTC)[reply]
Probably difficult - and anyway, some people post one-line questions in their question headers. — QuantumEleven | (talk) 13:24, 10 March 2006 (UTC)[reply]
See this bug report. The idea is to have an error message when enter goes hit too quickly - or a special form to enter questions in WP:RD. Also please vote for that bug fix. --DLL 10:41, 11 March 2006 (UTC)[reply]

March 10[edit]

Engineers' title[edit]

What is the internationally recognized abbreviation for an engineer, eg Er. or Ing. etc

I'm pretty sure it's "Ing". Celcius 01:40, 10 March 2006 (UTC)[reply]
There's "PE" for a certified professional engineer, which is cross-discipline. Otherwise, it's far more common to specify by branch, such as "CE" for a civil engineer or "ME" for a mechanical engineer. Unless you mean the people who drive locomotives? — Lomn Talk 02:32, 10 March 2006 (UTC)[reply]
That would require international standardisation of what it means, and I don't think there is such a thing. For example, in the Netherlands there are two such titles, Ir. for university levels and Ing. for the lower level. DirkvdM 10:36, 10 March 2006 (UTC)[reply]
As far as I know, there isn't one. If there were, it wouldn't make sense in every language, anyway. Plus, as Dirk said, there are so many variations of Engineer (not just by subject, but also by qualification) that a global definition is very difficult. For instance, in the UK, engineers are classified by their affiliation with a professional body (such as the Institute of Mechanical Engineers) and 'rank' within that organisation (Associate, Member, Fellow...). In Germany, you are a different 'kind' of engineer depending on whether you went to a 'university' (Universität) or 'technical university' (Fachhochschule). I'm sure there are many more examples. — QuantumEleven | (talk) 13:23, 10 March 2006 (UTC)[reply]
In the US, professional societies only have clout for their members if they offer a recognized certification; for example, Project Management Institute (PMI) does but Institute of Electrical and Electronics Engineers (IEEE) doesn't (although IEEE technical standards are peerless). In fact, with PMI, you don't have to be a member, but the certification pricing make it advantageous to be one at least for a year. --Halcatalyst 03:27, 11 March 2006 (UTC)[reply]

Sino-Euro Foriegn Relations[edit]

I have yet to find an article on Wikipedia concerning any Sino-Euro Foreign Relations. If there is any website which gives information on this subject, I would like to find it. Thank you.

--Swang 01:23, 10 March 2006 (UTC)[reply]

There's not an actual article on this topic, but there are some close ones: European Union arms embargo on China, Anglo-Chinese relations, Sino-British Joint Declaration and the broader Foreign relations of the People's Republic of China. --Canley 22:50, 10 March 2006 (UTC)[reply]
and if you want earlier history there's Sino-Roman relations, which is a fascinating article. —Charles P._(Mirv) 00:19, 11 March 2006 (UTC)[reply]

Designer of 1979 subway map[edit]

I just started an article on John Tauranac. The bio in the back of the book I was using says he was the Chief Designer on the committee that designed the new NYC subway map in 1979. However, this website disagrees. Who actually designed it? Cantara 01:55, 10 March 2006 (UTC)[reply]

I don't know that there's actually a disagreement. What that web page says is that the committee was "chaired initially by Fred Wilkinson and latterly by John Tauranac"; the "chair" of a design committee could mean the same thing as the "chief designer". It says that Michael Hertz "executed the ideas that emanated from the committee". --Anonymous, 02:22 UTC, March 10, 2006.

Is "Aisakos" the Roman version of the Greek name "Aesacus" in mythology?[edit]

Hello,

I am trying to find out the answer to a question and was told I could submit my query to a wikipedia volunteer for assistance.

In Greek Mythology, there was a character named "Aesacus", which I believe was Greek. Yet it also lists his name as "Aisakos".

As many names were taken by the Greek names were taken and turned into Roman counterparts, is "Aisakos" the Roman version of the name "Aesacus"?

Thank you for any assistance you can offer.

Sincerely,
Dino Vannucci
[ E-mail removed for the sanity of your inbox ]
www.vannucci.net

The other way round - Aisakos (Αίσακος) is the Greek name, Aesacus is the Latinised version. David Sneek 06:57, 10 March 2006 (UTC)[reply]
Anytime you see a K in such a context, it's Greek; Latin had almost no use for K. --jpgordon∇∆∇∆ 17:12, 11 March 2006 (UTC)[reply]

registration of the name of a political party[edit]

At Talk:2006 Dutch municipal election the following story came up about the Leefbaar parties in the Netherlands. The various local parties are named 'Leefbaar Roterdam' and such, with the name changing with the name of the municipality. Here's a copy of what someone wrote there:

A lot of local parties named Leefbaar .... but unrelated. The best example of this is Leefbaar Den Haag, which i believe is a party made by 3 people who met in some kind of mental instutition, and had a homeless guy on their 4th spot. They never campaigned or put up posters, got 4 seats in 2002 (Democracy at work!), clowned around every council meetings, and spend most of their allocated party budget on a car. (Story as remembered from TV)SanderJK 23:25, 9 March 2006 (UTC)[reply]

Brilliant story, but it raises an interresting question. Can one register the name of a party to prevent others from hijacking it like this? And would that work with a name that changes from location to location and the base of which is an ordinary Dutch word? DirkvdM 09:38, 10 March 2006 (UTC)[reply]

You can register the name of a party, like Leefbaar Nederland or Leefbaar Utrecht, but I don't think you can claim the rights to a single word, in this case "leefbaar". Leefbaar Den Haag is definitely the funniest example of the misuse of the adjective "leefbaar", but in other cities there have been problems as well; there is no Leefbaar Oss, because the name was registered by members of the VVD and PvdA, Leefbaar Roermond is a VVD trademark too, the rights to Leefbaar Rotterdam were initially owned by the left-winger who later formed Duurzaam Nederland. David Sneek 15:26, 10 March 2006 (UTC)[reply]
I'll add that to the Leefbaar article. DirkvdM 07:40, 11 March 2006 (UTC) Then again, there's this site, so I'm not sure any more about the Den Haag story, so I've left that out (mentioned in on the tak page, though). DirkvdM 08:01, 11 March 2006 (UTC)[reply]
There are examples in Britain too. Its often (allegedly) deiberate dirty tricks. Or you can select a candidate with a similar name. There was one famous general election in Westbury where one of the minor parties had a candidate who was called Thatcher. He got 3.0% of the votes (compared with the handful he would have got otherwise); most of them had meant to vote Conservative. Fortunately for the sitting MP it was a safe seat. Jameswilson 02:52, 11 March 2006 (UTC)[reply]
This reminds me of something that alledgedly happened in Curacao, I believe. People were bribed into voting for the leader of list one. Due to analphabetism, the bottom candidate for the last list got many votes. :) DirkvdM 07:40, 11 March 2006 (UTC)[reply]
In Chicago, Jesse L. Jackson tried to run against Jesse Louis Jackson, Jr., the incumbent, banking simply on the name similarity factor. Rmhermen 02:46, 13 March 2006 (UTC)[reply]

International political parties?[edit]

Are there any international political parties? I know national parties form international alliances (especially socialist parties I believe), but are there any international 'organisations' that have national (or regional) political parties, so the same idea, but reasoning the other way around? In the EU, for example, it would make sense for an initially EU party to start having national branches, wouldn't it? And, in continuation of the above question, would it be a good idea for such international parties to register their name in al countries, to prevent it being hijacked? Or can they only do that when they actually participate in elections? DirkvdM 09:46, 10 March 2006 (UTC)[reply]

Would you trust a political party whose headquarters were in another country? Well, I don't think most people would. So each political party is careful to [seem to] be automomous. The Green Party of course is widespread in Europe, but I think they share a name and broad aims, rather than an administrative centre. Notinasnaid 10:03, 10 March 2006 (UTC)[reply]
Well, that would apply to specific EU parties, wouldn't it? And a headquarter doesn't have to be localised - it can be on the Internet. A Wikiparty perchance? Hey, let's start a political party! :) But my main question is 'does something like this exist?' DirkvdM 10:40, 10 March 2006 (UTC)[reply]
I think the closest thing to an international party would be something like the World Socialist Movement. What distinguishes it from many other international federations is that all the member parties subscribe to the same 8 principles. The question of different national policies doesn't really arise, since they do not advocate reforms or other alterations of the existing system (insisting on socialism and nothing but). In a sense they are the same party, but in different countries. Mind you, they are all a little on the small side... Mattley (Chattley) 10:53, 10 March 2006 (UTC)[reply]
What about the EU Parliament internal party groupings that national parties belong to? AllanHainey 15:45, 10 March 2006 (UTC)[reply]
Indeed: European political party. They're somewhat odd 'parties' though- you can't just go down to a branch and join. Markyour words 17:40, 10 March 2006 (UTC)[reply]
The only one I can think of is Sinn Fein, which has elected members in the Republic of Ireland and in Northern Ireland (and in the British Parliament, but they refuse to take up their seats there). -- Arwel (talk) 03:52, 13 March 2006 (UTC)[reply]

Rafael Mendez[edit]

There is no article on Refael Mendez, considered by many to be the best trumpet player of the 20th century. Many other trumpeters are mentioned, including Maurice Andre, and even Bix Biederbecke. Why not Mendez? The preceding unsigned comment was added by 216.243.98.161 (talk • contribs) .

Probably because noone's gotten around to creating one. You're welcome to do so yourself upon registering, though. Or if you prefer not to register, you can place a request at Wikipedia:Requested articles. GeeJo (t) (c)  18:04, 10 March 2006 (UTC)[reply]
If you do create one, make sure to include a disambiguation link to Rafael Alfonso Umaña Mendez GeeJo (t) (c)  18:12, 10 March 2006 (UTC)[reply]
There exists already a very short stub: Rafael Méndez. You don't need to register to expand it, just click "edit". David Sneek 20:11, 10 March 2006 (UTC)[reply]
Diacritics are such a bore ! Why can't WP search react as googl search does, ignoring them ? --DLL 10:29, 11 March 2006 (UTC)[reply]
Because most languages use them. English is an aberration ;-) There was a major flutter over this a while back, and it seems a consensus has still not been reached, see Wikipedia:Naming conventions (use English). The easiest solution seems to be to create the article using the proper diacritic marks, and then create a redirect to it from an article title omitting all diacritics so all those people with English keyboards can get to it easily. So, in this case, we could have Rafael Mendez -> Rafael Méndez. — QuantumEleven | (talk) 13:42, 13 March 2006 (UTC)[reply]

Politics Idiom[edit]

what mean of the Politics Idiom "Technoqrate"

Technocrat: "An individual who makes decisions based solely on technical information and not personal or public opinion. This definition is usually applied to political decision makers who use this method to guide their actions." David Sneek 20:57, 10 March 2006 (UTC)[reply]

Alcoholism[edit]

My mum's ex-partner (of 13 yrs) suffers from alcoholism. I'm not going to go into all the boring details but everything he touches turns to crap. He's devastated my mother and his mother and inconvenienced a lot of people inbetween. His habbits have affected my grandparents and myself too.

He's 40 years old and his mum (who he's currently living with) will have to sell her house and go into 'sheltered accomodation' due to debt. Why isn't the government doing something? Illegal substance abusers get help, but he's going to end up homeless. Mum says attempts to 'sanction' him failed and he has to want to be treated. Years of intoxication have totally screwed his mind and he thinks he's just fine.

Alcoholics are a negative force on everyone around them and I want to make the government care and do something. According to this website some organisation was supposed to be met by a health minister and she didn't even show up. I pay her salary! I want her fired. >:O --Username132 21:00, 10 March 2006 (UTC)[reply]

I am assuming you are in England (remember, Wikipedia is on the Internet - which is global). What do you want done? You didn't ask for anything in particular: money, housing, medical treatment... As for the health minister, is she an elected official? If so, don't vote for her next election. I've always hoped the British were smarter than the Americans when it came to politics. In America, people feel it works best if you don't vote and then complain that you hate all the elected politicians. --Kainaw (talk) 21:12, 10 March 2006 (UTC)[reply]
On the side issue of voting, in a Westminster parliamentary system, a small proportion of the public may have some say as to whether a particular person is elected to the parliament, depending on where they live (some upper houses are not even elected but appointed/hereditary). However the public has no say as to who becomes a minister. They are chosen by the Prime Minister from the pool of parliamentarians available. JackofOz 22:04, 10 March 2006 (UTC)[reply]
His mother seems to have assumed responsibility for his debts. Why? She has no such responsibility under law. She's enabling his behavior and should get herself to an Al-Anon meeting. [12], [13] - Nunh-huh 22:17, 10 March 2006 (UTC)[reply]

What I want is the authorities to be obliged to fix alcoholism the same way they are with addiction to heroine. The government should intervene. They have failed his mother who has been forced look after him. Supposing I sent a letter to my MP, expressing my disatisfaction with government policy, supposing the MP gets next to no letters - does he a) respond to the issue because there are no other letters to deal with or b) ignore the issue because there aren't enough letters re: alcoholism? --Username132 00:31, 11 March 2006 (UTC)[reply]

Most MPs hold a regular surgery in the constituency where you can meet him face-to-face. Call the House of Commons switchboard (020 7219 3000), and ask for your MP's office. Jameswilson 03:45, 11 March 2006 (UTC)[reply]

Your MP should respond if he/she is half decent. That's not to say they can do anything useful. A similar approach is to talk to your borough councillor. They have more time to spend with you and may be able to find out what local services are available that could help you. You can identify your borough councillor through this site. --Pontificake 17:44, 11 March 2006 (UTC)[reply]
The government has no obligation to deal with heroin addiction, or any other type of addiction. They choose to do so because it is illegal, & therefore treatment serves to (try to) reduce the level of illegality), & to reduce the social/criminal problems associated with it. They may not take this same attitude (or to the same extent) with all social problems. Though it should be noted that the U.K. government does fund, directly & indirectly, a number of social welfare organisations, some of which do deal with alcoholism. In any case it could reasonably be argued that as alcohol is legal your mums ex is responsible for his own behaviour and shouldn't receive any special treatment because of his own excesses with a substance which the vast majority of the population is able to handle responsibly. AllanHainey 10:40, 13 March 2006 (UTC)[reply]
In the U.S., politicians have found that they can panic people over "drugs", meaning heroin, cocaine, marijuana, etc. Thus, disproportionate amounts are spent in that area, including the resources devoted to pursuing and imprisoning users of small amounts of marijuana. Meanwhile, alcohol creates more problems than all the rest combined. While I agree that the government has no formal obligation, the allocation of resources is irrational, except from the point of view of attracting votes from the sheeple. JamesMLane t c 00:52, 16 March 2006 (UTC)[reply]

Catholic Churches[edit]

Need list and addresses of all Catholic Churches in Krsko, Slovenia Asap....please....leah...........

There's no Krško listing at this masstimes.org Slovenia church listing by locality, but if you know the area you should be able to find something. --Halcatalyst 01:17, 11 March 2006 (UTC)[reply]


American Express, L A Webb[edit]

Who or what is the origin of the name "L A Webb," frequently seen in sample credit card advertisments for American Express?

MSTCrow 01:57, 11 March 2006 (UTC)[reply]
There are a lot of Webb family members at American Express. Lisa A Webb is an executive in their advertising department. --Kainaw (talk) 15:02, 11 March 2006 (UTC)[reply]

March 11[edit]

Drug used by U.S special forces to prevent defecation.[edit]

Dear Sir / Madam,

I have bought a game by the name of "Vietcong Purple Haze" and in the manual, there are annotations of jargon words used by U.S special forces in Vietnam. One word that particularly caught my attention was "CORK" and its definition: "a drug to prevent defecation, used in the field with small teams". May I ask what is the non-slang word for that drug? Who started using it and when? Do they still use it today? Why do they use it for?

Sincerely, Matt714.

Probably just some anti-diarrheal drug, maybe even over-the-counter. I searched Google for defecation drug military and found Turkish rhubarb root and antiepileptic drugs. It seems that chemical warfare includes nerve agents that induce defecation. Anything that affects the nerves can be nasty stuff. Be careful. --Halcatalyst 03:16, 11 March 2006 (UTC)[reply]
BTW, you can pick up a lot more of the Special Forces jargon at Special Forces Jargon and Terminology. --Halcatalyst 03:18, 11 March 2006 (UTC)[reply]
The military is a slang factory and always has been. Typically, it's the organ of the state that introduces the most loan words, to start with, and then there are all the interesting acronymns (e.g. SNAFU), and then there are the desperation-born slang terms (e.g. British infantry term "Jesus boot" -- meaning that a place is surrounded by water and one would need special abilities or miracles to get to it). I wonder if this "Cork" isn't military humor, though: put a cork in your anus to prevent being scared sh*tless, etc. I doubt it would refer to anything real. Geogre 18:42, 11 March 2006 (UTC)[reply]
Never having been in combat, or even experienced it more directly through gaming, I wonder... would there be a real reason to avoid defecation on combat patrols? Probably. No time for that; there may not be a time (that you could count on) until the mission is over; the mission is "scheduled" for four hours, but it may last 8, 12, 16... you know how war is. So maybe a cork would be a good thing to carry. --Halcatalyst 03:53, 12 March 2006 (UTC)[reply]

So, what's up this weekend?[edit]

...anything exciting? —Preceding unsigned comment added by Halcatalyst (talkcontribs)

The Formula 1 season starts, but I don't know if it'll be exciting.
Slumgum 01:37, 11 March 2006 (UTC)[reply]

Dang, I wanted to be anonymous. Well, I can report that WP wouldn't let me post just the title, so I guess there have been some changes implemented in that area. As for my weekend... nuthin exciting. --Halcatalyst 01:57, 11 March 2006 (UTC)[reply]

Check out personalDNA.com. It's a new, free personality assessment devised by a group of New York University psychology PhDs. The interface is very cool. I came out as "animated inventor". --Halcatalyst 03:03, 11 March 2006 (UTC)[reply]

In Cedar Rapids, which you're close to according to your website, there's a 'Springfest' arts and crafts sale at the Hawkeye Downs Speedway. I guess that means formula 1 won't be too exciting. lol. I love your stories, btw. -LambaJan 03:59, 11 March 2006 (UTC)[reply]

Heh. My son-in-law is best friend of one of the winners on that track. Formula One it ain't! But good sport for those who like it. --Halcatalyst 04:50, 11 March 2006 (UTC)[reply]
I took that quiz. It called me an 'encouraging leader.' -LambaJan 04:53, 11 March 2006 (UTC)[reply]

I'm curious as well. What's up this weekend as well as next week? It's spring break time and I'm all by my lonesome at USF. Mike H. That's hot 05:09, 11 March 2006 (UTC)[reply]

Not exactly something you're likely to be able to see from Florida (well, maybe satellite TV channels...), but the Commonwealth Games start early next week. So if you';re into sports that may be worthwhile. Grutness...wha? 07:11, 11 March 2006 (UTC)[reply]
Not especially, but thanks. I just wanted something to do outside, like something around downtown Tampa. If I wanted TV, I might as well just lock myself in my room and watch All My Children all week (and don't think I wouldn't!). Mike H. That's hot 08:11, 11 March 2006 (UTC)[reply]
Well how about this, then? Grutness...wha? 08:25, 11 March 2006 (UTC)[reply]
Naked! Bikes! Two of my least favorite things! Well, I do thank you for trying. :) Mike H. That's hot 09:28, 11 March 2006 (UTC)[reply]

Anybody watching March Madness? --Halcatalyst 15:41, 11 March 2006 (UTC)[reply]

Go Heels! Well, yes, you could say so. Go Heels! However, the NCAA tournament hasn't started yet. On the other hand, Go Heels!, the ACC tournament is going on, and that's pretty good all-day fun. Geogre 16:00, 11 March 2006 (UTC)[reply]
  • Oh. I thought MM started before the betting began. But fan madness is forever ;-). --Halcatalyst 18:40, 11 March 2006 (UTC)[reply]

TELEVISION[edit]

I need to know what the first aired tv show was and where. thank you, judysatp

Depends on what you mean by a "TV show", and for that matter, "aired". The Book of Firsts by Patrick Robertson mentions these among others:
  • October 30, 1925: First TV transmission from one room to the next by John Logie Baird in London, England.
  • January 27, 1926: First public demonstration of Baird's TV.
  • April 7, 1927: First public demonstration of TV in the USA and the first anywhere to feature a professional performer, as A. Dolan, Irish-American comedian, performs "a short act of monologue and song". This is transmitted from AT&T in Whippany, NJ, and viewed by an invited audience 22 miles away in New York.
  • August 22, 1928: First news broadcast. Station WGY in Schenectady, NY, reports on the nomination of Alfred Smith as the Democratic candidate for President.
  • September 11, 1928: First television play, again on WGY: a 40-minute adaptation of The Queen's Messenger by J. Hartley Meyers, with a cast of two.
TV in those days used mechanical scanning systems with very low resolution: for that last broadcast the screen showed just 24 scan lines, less than 1/20 the number on a modern TV. German television with 180-line resolution began in 1934, although it was not officially inaugurated until the next year; 405-line electronically scanned broadcasting to the general public began on the BBC on November 2, 1936.
--Anonymous, 08:35 UTC, March 11, 2006.
The BBC's first experimental tv broadcasts actually started on August 20th 1929, with transmission of the first television play to be seen in Britain, "The Man With the Flower in His Mouth" being on July 14, 1930, though as (who? someone) said, the first high definition signals weren't sent until late 1936. There are a few more pioneering dates at History of television#Broadcast television, and trawling through Category:Years in television will probably produce more info still. Grutness...wha? 11:41, 11 March 2006 (UTC)[reply]

Please ID a novel[edit]

In a sixth grade class a novel was assigned for us to read, in which the characters were two boys. The novel was written in the first person view of one of the boys.The boys decide to go swim in a river despite the danger. One of the boys wanders off and the other can't find him. I don't remember his reaction, but he starts to think that it's easy being dead. After a while the police arrive at the narrator's house. Towards the end of the book, the main character tries to wash off the dead fish smell because it reminds him of his missing friend. At the end of the book the narrator asks his father if his friend went to heaven and his father says he probably did. The story probably doesn't make sense to you all because there's gaps in my memory. Anyway, anybody know anything else? KeeganB

A novel by Borges and H. G. Wells ? --DLL 22:14, 12 March 2006 (UTC)[reply]

Is that some kind of joke? KeeganB

Yes, it was, Keegan - recently we have had a whole truckload of mostly nonsensical questions about Luis Borges, so DLL was just venting a bit, you'll have to forgive him :) As for HG Wells, see the very similar question a bit further down the page. Sorry, we're just having a bit of fun, but I'm afraid I don't know the answer to your question either. — QuantumEleven | (talk) 13:31, 13 March 2006 (UTC)[reply]
Yes, forgive me Keegan, thank you 11. There oughta be a search bot dedicated for this. Something exists beginning with drawings and finds images. Why not begin with events and find stories (news, books, films, biographies). --DLL 23:17, 13 March 2006 (UTC)[reply]
I actually remember this book! I never know these obscure cultural questions. You are looking for On My Honor (ISBN 0899194397); it was a Newbery Honor book. Superm401 - Talk 03:28, 16 March 2006 (UTC)[reply]

Celebrity on London markets![edit]

Dear everyone!

I work for the Norwegian London guide www.dinlondonguide.no. We are working on an article for a Norwegian magazine about the Ridley Road Market, The Shepherd’s Bush Market and the New Caledonian Market, also known as Bermondsey Market.

I was wondering if any of you know if there are any British or international famous people that has been seen on these Markets?

As for example Mr.Jamie Oliver has said that he often buys his ingredients at the Borough Market.

I therefore wonder if you have seen, read or heard about any famous people/celebrities that have been seen on the Ridley Road Market and The Shepherd’s Bush Market?

I also wonder if you know if the New Caledonian Market ever has been used in a British movie/film? I know I have read this somewhere, but I don’t know the name of the film.

Please help us out here if you have any information.

We would really appreciate it.

Thanks a lot for your help.

Please post an answer here or email me at [email removed]

Looking forward to hearing from you.

Mvh. www.dinlondonguide.no

Ole Andreas Auflem Ansvarlig redaktør

E-post: [email removed] MSN Messenger: [msn removed]

Evil state leaders[edit]

Following the death of Milosevic, which murderous ex-leaders other than Saddam Hussein are still alive?

How do you define murderous? Some will claim Bush is murderous because people died in Afghanistan and Iraq. Others will claim Clinton was murderous because he didn't go to war after repeated acts of terrorism over seas, allowing plans to escalate to the World Trade Center attack. So, murderous is a loaded term in politics. You need to be very specific. For a less partisan view, you can look at various Human Rights websites. --Kainaw (talk) 15:07, 11 March 2006 (UTC)[reply]

I think, he means people like Saddam. Osama is still alive. He was the leader of Afganistan.Suraj vas 15:09, 11 March 2006 (UTC)[reply]

Very true, and I didn't intend to be too specific. Idi Amin was the first name that came to mind but then I discovered he was already dead. I was also curious about those 'dodgy' ex-leaders who are living with state protection in the West (e.g. Baby Doc in France). Was Osama the leader of Afghanistan? I thought he was just given extensive protection by the Taleban. --Pontificake 15:14, 11 March 2006 (UTC)[reply]
Osama was not the leader of Afghanistan. He was not even a member of the Taliban (who ruled Afghanistan). --Kainaw (talk) 15:43, 11 March 2006 (UTC)[reply]
Joseph Kony AFAIK rules a small part of Uganda - he's pretty nasty. Celcius 23:05, 11 March 2006 (UTC)[reply]
Murderous evil state leaders don't tend to retire or get voted out. It usually takes wars or coups to oust them
Slumgum 23:20, 11 March 2006 (UTC)[reply]
It's all very subjective... Bush is still in power, so he ain't an ex leader (same with Robert Mugabe). However we seen to still have such ex-leaders as Augusto Pinochet, Charles Taylor, and various Argentinian military leaders (such as Jorge Rafael Videla). Those with similar political views to mine might also like to add Margaret Thatcher, ut YMMV. Personally, I'd say Pinochet and Taylor would be top of the heap. Grutness...wha? 01:07, 12 March 2006 (UTC)[reply]
What does 'people like Saddam' mean? Like Kainaw said, define 'murderous'. If you mean leaders whose decisions have led to deaths then just about any US president would qualify, but, to stress the problem here, Churchill sent quite a lot of people to their death, but that was for a 'good cause'. We all 'know' that this was a good cause, but who is to decide that ultimately? Also, Eisenhower was another major one that chain of command, but became president only after that, so does that count? Or is it about causing deaths in one's own country? Maybe Dutch minister Verdonk would count after she resigns? Smaller death toll, but also smaller country. Should that be taken into account as well? And does 'leader' mean sole leader, as in prime minister, or does a minister count too? Simple lists are nice and give you a sense of overview, but they usually lie. Sorry about being so Dutch. :) DirkvdM 06:56, 12 March 2006 (UTC)[reply]

What was this novel?[edit]

I read it in 2000, in the sixth grade. It involved a struggling family, where the children were home alone while there mother worked two jobs. One day the electricity in the entire city goes out, and somehow the protagonist (a teenager, I think), his sister (maybe) and everyone else in the town take to roaming the country in packs... or something like that. Ring any bells with anyone?

Freak lightning storms around the world which leave all electronic equipment in the affected areas incapacitated. Ray leaves Rachel alone, and goes to investigate. Before he does, he finds Robbie and tells him to stay with Rachel. The family hits the highway, trying to get to away from the assault. They spend a night in the basement ... Is it a Well's novel ? --DLL 22:12, 12 March 2006 (UTC)[reply]
Not HG, sorry, try again. alteripse 22:27, 12 March 2006 (UTC)[reply]

Christopher "Kit" Carson[edit]

Hi, Did Kit Carson do any oil paintings during his lifetime? Thanks, --Mumphree2004 15:43, 11 March 2006 (UTC)[reply]

No. He did them all well after his lifetime. Sorry. I couldn't help it. Anway, the Kit Carson article makes no mention of painting. Also, any search you do will be difficult as so many places are named after him. So, you will turn up oil paintings of Kit Carson Creek, or Kit Carson Tavern, or Kit Carson Bluff... --Kainaw (talk) 15:50, 11 March 2006 (UTC)[reply]

Oakland Alamed & Berkeley Ferry[edit]

Have an old print (unsigned) showing a protrayal of San Francisco Ferry Building 1885 but the front of the Building shown in the print says Oakland Alameda & Berkeley Ferry. Print shows ship masts in background and horse drawn carriages in front of the building. Print also shows a clock but not as tall as the clock shown on the San Francisco Ferry sites. Info on Ferry sites also shows building was completed in approx 1896. Do you have information as to what I have here?

Thanks

P Ford

  • A search on Google using "Oakland Alameda & Berkeley Ferry" yielded nine results. Yahoo coughed up four. Try that and see if you get the results you're looking for. --Halcatalyst 03:46, 12 March 2006 (UTC)[reply]
You say "but the front of the Building shown in the print says Oakland Alameda & Berkeley", as if that is contradictory to its being in S.F. It seems natural to me that a ferry terminal in S.F. which provides service to Oakland, Alameda, and Berkeley would say the latter on the front of it (for the convenience of potential passengers). -R. S. Shaw 04:16, 12 March 2006 (UTC)[reply]

Hate fads[edit]

I was wondering if you guys have any idea of where and how some hate fads originate. For example, since sometime last year, the new, hot, "cool" fad is to hate everything "emo" (whatever that's supposed to be, really.) I was curious because I could see this particular fad spreading, apparently it appeared first on the USA and spread over the internet and then to everywhere. So, you guys know of any papers or subjects I could read to know what is known about how mindless hate spreads on human populations? I'm pretty confident this sort of meme has been studied already. ☢ Ҡiff 23:06, 11 March 2006 (UTC)[reply]

The question is not so much how they originate (anybody can do it) but how they catch on. In this regard it's like stock market trends or clothing fashions or, um, March Madness office pools. There must be some sociologist around here who can enlighten us about these things :-). --Halcatalyst 03:43, 12 March 2006 (UTC)[reply]
I thought "originating a fad" had the "catch on" implicit, afterall, it's not a fad if it doesn't catch on, right? ☢ Ҡiff 04:00, 12 March 2006 (UTC)[reply]
Could someone write an article on hate fad, so I can understand the dicussion? :) Does this include the recent apparent hate of Muslims? Also, is hate supposed to be cool? Ah, I think I get it. Is it about this 'I'm cool because I don't give a shit' attitude? (Which I only know from tv series, by the way, so I'm not sure how real it is.) DirkvdM 07:02, 12 March 2006 (UTC)[reply]

What the author seems to be asking about is just one of those commonplaces of youth. There are plenty of psychological explanations of it, and sociologically it's kind of basic. In groups under stress (and adolescence is a disease state, IMO, but it's a stress state by anyone's point of view), enemies become very important, as one defines one's position in terms of identification (what you value and what you like), expertise (knowledge of what's really true, really good, really bad), and power (what you defy, what you denounce). Adolescents have to prove that they know more than each other, have to prove that what they identify with is better than what the others identify with, and that what they despise is important to despise. (Notice how girls don't seem to go through this as much? Notice how they don't tend to get into the "Death Before Disco" or the "Heavy Metal, not Punk" or the "Rap, not Rock" stuff? why you figure that is? Evolutionary biology would suggest that they go through a different hell of competition and ego differentiation.) So this year it's "emo is bad." Ok. I suppose that replaced "Goth is stupid?" I suspect that what the questioner is referring to is fairly localized, dependent upon the rising or in place power group in the population. The mechanism for spread is person to person, and two or three admired (usually because of physical attractiveness) persons can make a given population pick something else to "hate." It doesn't mean much, ultimately. (Although I still say death before disco.) Geogre 13:41, 12 March 2006 (UTC)[reply]

Not really, no. This doesn't seem as specific as I'd have expected for most hate fads I'm aware of, hence my interest after this particular "emo hate fad". I could clearly notice it permeating pretty much everywhere on internet, and spilling out of it and ultimely reaching my small, stupid city here in Brazil, which is something very unlike some other forms of general hate I know. The emo hate fad just spread too widely as far as I could tell. The hate against muslims is also a hate fad, yeah, but that one is easy to trace, and that one seems to fit your locality theory better, just like the hate fad over here against United States in general, George W. Bush, and furries on internet in general. I'm quite aware of those socialogical aspects pointed out by Geogre. They're true, but what truly happens is a lot more superficial than that, I think, that's why I pointed it out as a fad, really. People don't hate anything, they just say it to look "cool", to appear as they have a strong opinion about something, and above all, they like to act in accord to certain groups in order to feel part of something, even if it's just some silly unfounded "hate" against anything. People sacrifice true critical sense and their power to have opinions so they can just get along with their social life easily. This is a deplorable behaviour, IMO, and literally destroys a potentially interesting and, in a level, meaningful person, but it happens everywhere and it's a pity. Like Geogre also pointed out, this all usually starts when a select group of influential people do it. So, I was wondering if there was any interesting papers out there discussing this sort of subject from a scientific point of view, with statistics, tracing routes, showing age tables, etc. I just want any example of such a study. ☢ Ҡiff 23:17, 12 March 2006 (UTC)[reply]
Not really a paper but Malcolm Gladwell deals with this sort of thing in his book The Tipping Point. AllanHainey 10:56, 13 March 2006 (UTC)[reply]
That's a good book. I also think that Charles Mackay's venerable Extraordinary Popular Delusions and the Madness of Crowds is in play, although he doesn't understand why, either. I would argue that the Internet can act like a very large adolescent group, and a few forceful satires can ruin the reputation of something very quickly, and then, for others seeking to establish credibility and in-group status, they can latch onto the powerful sneer. The fact that this is a type of popular music means, I should think, that we're dealing with the late adolescent age group, just globally. (I'm not even sure what people mean by the term "emo." Is that supposed to be Iron and Wine, or is it supposed to be Galaxy 500? It's such a sloppily and hastily defined genre that it gets easy to 'hate,' because you can make nearly anyone (Coldplay) fit.) Geogre 11:41, 13 March 2006 (UTC)[reply]

March 12[edit]

concerning queen victorias royal yacht[edit]

the name was HMY victoria& albert trying to find who was her captainof her royal yacht;it was in the 1840s could be 1850s; i believe his name wasJohn Jago &his son was the cabin boy. would like this verified.

  • I searched Google using key words queen victoria yacht "Victoria and Albert" and got about 45,000 hits. You might do the same and then follow up on some of them to see if you can find the answer to your question. --Halcatalyst 03:38, 12 March 2006 (UTC)[reply]

There were three royal yachts named the "Victoria and Albert". The first was in service from 1843 to 1853, and was captained successively by Lord Adolphus Fitzclarence (bastard son of King WIlliam VI by Mrs. Jordan), William Crispin, and Joseph Denman [14]. It was renamed the Osborne in 1854. The second was in service from 1855 to 1899, captained by Joseph Denman and Hugh Campbell; the third from 1899. I don't see any Jagos. - Nunh-huh 04:39, 12 March 2006 (UTC)[reply]

Jails[edit]

Hello. my name is Holly. My law class is planing on having a field trip to a jail closest by, Manitoba. I was wondering if there is any jails in Manitoba that we can visit?--142.161.238.21 07:19, 12 March 2006 (UTC)[reply]

According to our Corrections Canada article, there are two prisons in Manitoba: Rockwood Institution and Stony Mountain. According to Correction Canada's website they are both in the same place: Stony Mountain, Manitoba, which is in the Winnipeg Capital Region. moink 10:20, 12 March 2006 (UTC)[reply]
Those are the federal prisons. There are also provincial jails which are listed at [15]. --Metropolitan90 02:35, 13 March 2006 (UTC)[reply]

Relationship between brothers and sisters[edit]

Why does sexual relationship between brothers and sisters not prevalant? What is the reason behind that?

It seems to be hard wired in the brain to reject the sibling as a sexual partner. This is also true of boys and girls raised as if they were siblings but who aren't. Incest is very bad in pretty much every way, but it is very, very bad genetically. Birds, fish, reptiles, amphibians, and other mammals all stay away from siblings for mating. The aversion is fairly basic, therefore, so asking "why" is kind of beside the point. See incest, if you must. Geogre 13:45, 12 March 2006 (UTC)[reply]
Specifically this aversion is referred to as the incest taboo. There are a number of theories about it. --Fastfission 14:50, 12 March 2006 (UTC)[reply]
The incest article mixes up incest with rape and sexual child abuse, which are rather different things. I would say that the situation is more complex. It's true that there is a psychological mechanism against sexual interest between siblings, but this does not come into effect in cases where siblings grow up apart from each other. The genetical issues of inbreeding may cause problems for the population as a whole over a long term (and so may explain the psychological mechanism against sex with siblings), but have no immediate effect -- children from brother/sister relationships are not disabled or sick any more often than other children, as far as known. Also, marriage between first cousins is common in many societies, and even though first cousins are only one degree less related than siblings, no bad consequences have been found. --Chl 15:04, 12 March 2006 (UTC)[reply]
Yeah, the incest article is disappointing. It shows signs of having been edited by them. It bugs me no end to see the apologists for inappropriate sexual conduct weave a line in here, a line in there, in all our articles, and then the reverts are partial. This is one of those cases. Someone who edited it only wanted to talk about how it might not be that bad to have sex with one's own child. Fighting for staying on the topic and maintaining the most widely researched line takes all one's energy, though. Geogre 21:27, 12 March 2006 (UTC)[reply]
Yet another reason to get "stable versions" up and running. A large enough part of the community can concentrate on an article long enough to get an acceptable version, then publish that version. Careful inspection for sneaky edits would need to be done only occasionally when a new version needs to be published. -R. S. Shaw 22:47, 12 March 2006 (UTC)[reply]
It is not true to say that no bad effects of close-relationship parents are known. See this where they are surprised that 1st cousin marriages "only" nearly double the risk of birth defects. Rmhermen 02:26, 13 March 2006 (UTC)[reply]
Adding to Georgre's comment: Birds, fish, reptiles, amphibians, and other mammals all stay from siblings for mating. This assumes there are other mates available. Animals will mate with siblings quickly if there are no other options. This happens regularly when breeding animals that mature quickly (mice, hamsters, rabbits, hedgehogs...) You have to quickly identify the males and females and separate them or you will soon have more babies. --Kainaw (talk) 18:03, 13 March 2006 (UTC)[reply]

Incest is bad in the short run, by dramatically increasing the possibility of recessive disorders, like hemophilia. However, it can, in rare cases, lead to extraordinary abilities, so is sometimes used in breeding. Also, in the long run, it tends to lessen the frequency of bad recessive genes in a population by "bringing them to the surface", where they kill those individuals with a double set of bad chromosomes, or prevent them from reproducing, thus preventing those bad chromosomes from being passed down. Allow me to demonstrate using "B" for a bad chromosome and "g" for a good chromosome:

      INCEST EXAMPLE            NON-INCEST EXAMPLE
   Bg-+-Bg      gg-+-gg        Bg-+-gg      Bg-+-gg
      |            |              |            |
 BB Bg Bg gg  gg gg gg gg    Bg Bg gg gg  Bg Bg gg gg

In both cases the result is 4 of 16 bad genes in the next generation, or 1/4 bad genes, just as in the parent's generation. However, in the incest case, the BB individual dies or is unable to reproduce, meaning that only 2/14, or 1/7 of the genes eligible to be passed down are bad. Thus, in this example, incest has removed almost half of the bad genes in the group studied. StuRat 17:05, 17 March 2006 (UTC)[reply]

The Three Sisters[edit]

I'm writing up on the three sisters; maize, beans and squash. I am wanting to know why the north-eastern tribes relied on these? Computerjoe's talk 15:29, 12 March 2006 (UTC)[reply]

This amaizing article you are writing should squash the competition, after which you can toot your own horn. :-) StuRat 17:18, 17 March 2006 (UTC)[reply]
"Primitive" cultures and social units tend to rely on food at hand that offers the most advantages in terms of ease of cultivation, nutritional value for the effort, length of storage, versatility of preparation, taste, and cultural tradition. No supermarkets, no long-distance food trade networks. Game and fish were also relied on as protein sources in areas where they were plentiful. See Jared Diamond's Guns, Germs, and Steel for an interesting comparison of indigenous plant foods in the New World versus the Old World. alteripse 18:57, 12 March 2006 (UTC)[reply]
Nice use of speech marks - avoiding POV - well done! A better word might be along the lines of ancient? Computerjoe's talk 19:18, 12 March 2006 (UTC)[reply]
No, for example the Romans had a well-developed trade network moving enormous amounts of grain from Africa to Italy, so Ancient isn't the word: the best word I could think of is primitive. The quotes were in deference to those who think primitive is inherently pejorative, but I would argue that is false pc-ness anyway. alteripse 19:23, 12 March 2006 (UTC)[reply]
You raise a valid point. Personally, I have no arguement of your choice of wording - I was (attempting) to make an anylisis. Primitive is most certainly not the right word, but I doubt there's a better word. Lets invent one. Brazilliontyhuh?? Computerjoe's talk 19:52, 12 March 2006 (UTC)[reply]
(Sorry, couldn't resist poking my big nose in here). The PC Lords have a permanently busy schedule, however we are not linguistic martyrs (well, I'm certainly not). My dictionary gives 14 primary meanings for "primitive", including "characteristic of early ages or an early state of human development", and "of or pertaining to a race, group, etc, having cultural or physical similarities with their early ancestors". The word seems perfectly suited to its context, and did not need quotes. But you've taken a bigger-picture approach, which affects personal style. The quotes suggest that you're from the "Willing to participate in the process of compromise about language change" school. I'm a member of that school, but in this particular case I would have been guided more by the principle of "It's not the writer's responsibility to take all other possible scenarios into account when choosing the next word". JackofOz 00:21, 13 March 2006 (UTC)[reply]
My nose sees it like yours. alteripse 01:39, 13 March 2006 (UTC)[reply]

December 5, 1988[edit]

I don't believe in reincarnation but I'm curious this is my birthdate so if you can go to all the trouble and are willing, I would like to know all the people listed on Wikipedia who died on December 5, 1988. I know it's a difficult thing to do but that's why I said if you're willing... ;-)

Nobody's listed to have died on December 5th 1988 on this page; however in the Category:1988 deaths there are hundreds - but I've set a bot to look through them. Please wait. Computerjoe's talk 18:08, 12 March 2006 (UTC)[reply]
My bot has searched over 500 articles in the above category, and brought up no response. Nobody notable died on your birthdate. Computerjoe's talk 18:59, 12 March 2006 (UTC)[reply]
Noone that has had Category:1988 deaths added to their article, anyway. GeeJo (t) (c)  20:28, 12 March 2006 (UTC)[reply]
The only other option is to go through the million articles, which is nearly impossible. Computerjoe's talk 20:41, 12 March 2006 (UTC)[reply]
Also reincarnation might take time. You do not die one day to be borne the same, do you ? What about the choice of good parents ? --DLL 21:53, 12 March 2006 (UTC)[reply]
Found no one in "Find a grave", nor in the 100 first googl results. Go on if you want ...
I seriously think no-one of any notability died on your DOB. Congrats. Computerjoe's talk 22:06, 12 March 2006 (UTC)[reply]
Why isn't there a Category:January 1 births or 364 similar categories? I think there should be...almost enough to create one.
Slumgum 22:35, 12 March 2006 (UTC)[reply]
I think that Category:January 1 births should be something like Category:New Year's Day births. --Revolución hablar ver 03:09, 13 March 2006 (UTC)[reply]
What about January 1#Births? JackofOz 03:18, 13 March 2006 (UTC)[reply]

I did a Google search for the phrases "december 5 1988" and "5 december 1988" on site en.wikipedia.org. The number of hits was manageable, and none of them referred to a date of death, which is fairly good evidence that nobody is recorded in Wikipedia as having died on that date.

However, an unrestricted Google search for the phrase "died december 5 1988" does produce a number of hits, starting with a major general (2-star general) in the US Air Force. No doubt if you try other variations such as "death on 5th dec 1988" -- just the four points of variation I've shown give you 16 variations of the phrase to search on, and no doubt there are others -- then you will find other instances.

--Anonymous, 6:02 UTC, March 13, 2006.

Umm, wouldn't you want to find someone who died the night you were conceived? Wouldn't that be when the metempsychosis and/or transmigration would have happened? There are people who think the soul doesn't enter until the first breath, of course, but if you're looking for any chance of being someone famous redux, you might want to assume the soul adheres at conception. Geogre 11:45, 13 March 2006 (UTC)[reply]
Unfortunately, it is practically impossible to determine the precise date you were concieved, unless you have parents who a) don't mind sharing, b) don't have sex very often, and c) kept a diary of when they did, all you'll get is an approximation. Incidentally, I found I share my birthday with The Game and Teemu Tainio, and am not sure whether to be happy or sad. Proto||type 16:43, 14 March 2006 (UTC)[reply]
The one I will claim for my "twin" on June 23 is Giambattista Vico. Anybody James Joyce liked can't be all bad. No, I take that back. But Vico is very interesting. --Halcatalyst 01:09, 15 March 2006 (UTC)[reply]

Puerto Rican Immigration to Hawaii in early 1900's[edit]

My grandparents left Puerto Rico in the early 1900's to, reportedly, pick pineapples in Hawaii. They were among hundreds who traveled to Hawaii for the purpose of acting as peons in Hawaiian agriculture. I have found many articles dealing with the hardships and travails that these immigrants faced. There are no articles,however, that list the names of the individuals who actually arrived in Hawaii or what their ultimate fortunes turned out to be. I do not know whether my grandparents had other children other than my mother who they left behind in Puerto Rico. This begs certain questions: Do I have extended family in Hawaii? If so who are they and how do I contact them? Jomundo d 18:44, 12 March 2006 (UTC)[reply]

when it gets down to specific individuals, this is the kind of research that you'll have to wind up doing yourself. Hawaii was annexed to the U.S. in 1898, became a self-governing territory in 1900, and a state in 1959. Chinese contract laborers came as early as 1852 to work on the plantations. The Reciprocity Treaty of 1876 allowed for Hawaiian sugar to be exported to the U.S. duty-free. The plantations responded to pressure not to bring more Chinese immigrants by hiring those from other places: Portuguese were brought to the islands in 1878, Japanese in 1884, Puerto Ricans in 1900-1901, Koreans in 1903, and Filipinos in 1906. Hawaii has birth and death records beginning in 1853, and they are mostly complete after 1896. These records are available from the State Department of Health, Research and Statistics Office in Honolulu. The Mormon Family History Libraries have microfilm copies of the death records. The Hawaii State Archives' collection of early Hawaii marriages covers marriage from 1826 to 1929, and is indexed from 1826 to 1910. Hawaii had parital censuses in 1866, 1878, 1890 and 1896, and is covered in the 1900 and 1910 U.S. Federal Censuses. The Roman Catholic church in Hawaii can be contacted at the Diocese of Honolulu, Chancery Office, 1184 Bishop Street, Honolulu, Hawaii 96813, and can provide direction on finding which parish may contain pertinent records. Passenger lists for persons arriving in Hawaii have been microfilmed from 1843 to 1900 and are on microfilm at the Hawaii State Archives. It may be useful for you to obtain the services of a local genealogist, who will be familiar with and better able to access the records. Good luck! - Nunh-huh 22:33, 12 March 2006 (UTC)[reply]

macroeconomics[edit]

How would you ouline how a market system answers the three basic questions of what, for whom, and how goods will be produced? and how would you differentiate among the opinions of economists, politicians and citizens regarding the government's role in the economy?

In a democracy, the economists have the facts, the politicians have the power to implement economic policy and carry it out, and the people (theoretically) set the direction. --Halcatalyst 19:13, 12 March 2006 (UTC)[reply]
I guess it would be better to say that politicians define economic policy and civil servants carry it out. --Halcatalyst 20:14, 12 March 2006 (UTC)[reply]

Hi[edit]

Hi i was reffered by Tzom, i think thats his name, i am enquiring about the contact details for Mr. Michael Winterbottom the producer of Road to Guantanamo, and many other movies, even his agents contact details would be of great help, please get back to me, anisahmed at 69 dot hotmail dot com

by the way i respect wikipedia so much, im suprised no one i know really uses it, i will spread the word, because you people are more interactive,

thanks Anis

Interactively did I try unformatting your email. Please read our advice (top of the page). --DLL 21:50, 12 March 2006 (UTC)[reply]

March 13[edit]

theology and mythology[edit]

if we take as a given that the bible is a reliable source of information to begin with. i want to ask about the theological reliability of the book "the conflict of adam and eve with satan" how deep is this book founded in scripture (if at all)? alot of the bible is written by people who claim to have been given visions from god that they were to write down and pass on. can anyone help me out?

refrences to sources of information would be useful, as well as other books alike to "the conflict..."

thanks, sam hawthorn

86.140.216.202 00:04, 13 March 2006 (UTC)[reply]

I can't find a book with that exact title on amazon.com or bn.com (online bookstores), so can't help you out. These sources carry all books in print and many out of print. --Halcatalyst 03:36, 13 March 2006 (UTC)[reply]

You won't find it there :) It's one of the "lost texts" - see our article Conflict of Adam and Eve with Satan. I wouldn't really know how to answer this question, though, because it seems to me very much of the form "given that all sheep are purple, how can we get green wool?", the Bible being such a clearly unreliable sources of information, particularly as regards Genesis and Revelation, the only two relevant parts of the book as regards Adam, Eve, and Satan. Grutness...wha? 05:51, 13 March 2006 (UTC)[reply]

The Bible is as much a source of historical information as most books of its type and provenance, which is to say quite a bit, but it's not an AP report or a anythng like a history book today. Personally, I don't believe that Genesis was written to be history, and the book of Revelation is frankly that, a "prophetic message" (Rev. 1:3). Take it literally at your own risk. With a few exceptions, biblical prophets were not predicting the future but delivering messages for their own generations. Later readers could well take heed, as they should of any wisdom from the past. --Halcatalyst 21:19, 13 March 2006 (UTC)[reply]
Reliability is in our world, not in those events. First we have Adam with Lilith. Then poof goes Lilith and there is Eve. They beget only sons and one of them is killed, and so on.
Milleniums after, jew priests and fathers of the churches deliberately ignore some events. Those events, related or forgotten, are rather seen as myths today : very interesting as they tell much about us when we try to find something in them. --DLL 23:11, 13 March 2006 (UTC)[reply]
Is this topic going to turn into a harangue? --Halcatalyst 05:08, 14 March 2006 (UTC)[reply]

Normans[edit]

http://en.wikipedia.org/wiki/Normans

In that article it says Normans were Norse and Gallo-Roman.

My questions are:

1. Gallo-Roman... does that mean the Celtic tribes of Gaul were, like Britain, just under Roman governmental rule (just soldiers there, really), and that the Gauls/Franks were still a seperate people from the Roman people? I believe the answer is yes, i'm just making sure.

2. Were the Normans, "blood" wise, that invaded England mostly Norse?

I'm interested in knowing this because the Normans, along with the Anglo-Saxons and Danes, make up the English race, since they all three mixed with each other so thoroughly.

London 03:43, 13 March 2006 (UTC)[reply]

The Romans conquered Gaul, administered it, imposed their language, and mixed with the people to the extent that several centuries later, they were "one people, language, and culture", but genetically primarily derived from previous Gallic ancestry. The Normans conquered England, administered it, imposed their language and laws, and mixed with the people to the extent that several centuries later, they were "one people, culture, and language", but genetically primarily derived from previous English ancestry. The Normans, like the Romans, were of mixed ancestry, including Norse and old British Celtic (which is why the region is called Brittany). alteripse 06:19, 13 March 2006 (UTC)[reply]

The second question is "yes." William of Nor-mandy was William of Norse Man Land. I.e. the Nor-men were "Vikings" who had conquered a region of present-day France. His troops were similarly norsemen. These norse were not actually vikings, in the literal sense, because they were part of an invasion trend rather than raids. At any rate, some of the Norman troops were Gallo-Roman, but the nobles William brought with him were one generation away from Norway. Geogre 11:49, 13 March 2006 (UTC)[reply]
The Normans ("North Men") got a lot farther than England and Normandy, all the way down to the Mediterranean. They repulsed the Saracens from Sicily and Malta. A very interesting and largely unknown story. --Halcatalyst 21:02, 13 March 2006 (UTC)[reply]
Northmen did not conquer Normandy. There was a sort of bargain between the French and Viking kings : "Do not row up to Paris for pillage, I give you the march of Normandy and you will guard us against Angles, Saxons and Danes (from Britain) pillage." The then duke of Normandy referred (feudalistically spoken) to the French whatshisname king. --DLL 23:02, 13 March 2006 (UTC)[reply]

"At any rate, some of the Norman troops were Gallo-Roman, but the nobles William brought with him were one generation away from Norway." - So in other words, those who migrated/conquered England from Normandy ("of Norman origin") were of almost pure Norse blood?

Also; Alteripse- Are you saying Gauls were a mix of Celtic and (what people now would call) Italian? I'm sorry if I seem rude, but i've never heard such a thing(s) in my life- i've heard the Roman soldiers occupied and "protected" their land in Britain, but not that they mixed to that extent, if at all (same goes to the Gauls).

Read again. Romanized Gauls after 400 years were still mainly genetically celtic, but with an admixture of Roman (italian) genes. They also considered themselves romans and used roman language, law and administrative structures until the empire collapsed. There is nothing controversial about what I am saying. My point was that the Norman relationship to England and the Roman relationship to Gaul were quite similar but your initial questions suggested to me that you hadn't seen that yet. alteripse 01:28, 14 March 2006 (UTC)[reply]

Again, the main reason i'm interested in this is, the people who moved from Normandy mixed so thoroughly with the English people, and are now (and even relatively shortly after they conquered/inhabited Britain)woven just as tightly into the English bloodline as the Anglo-Saxons and Danes. -London 23:42, 13 March 2006 (UTC)[reply]

That would seem to be a solid statement. For a while the French (Norman) Court kept itself above the Anglo-Saxon mob. They didn't speak their language and couldn't be bothered to learn. They were the conquerors, after all. One result was that Anglo-Saxon literature, which had been highly developed, pretty much ended; and when English rose again as a literary language it was quite different. Among other things, it had a distinct French infusion. I think the literary developments parallel those of human breeding. Members of the Court married among themselves (and maybe some with nobility from overseas). Of course, they produced a few bastards with Anglo-Saxon maids. Eventually some lines of Normans merged with the locals; eventually, it was all of them. Without any evidence I would guess the mingling was pretty much complete by 1400, about the time of Chaucer's Canterbury Tales. That's the way things go everywhere. --Halcatalyst 00:29, 14 March 2006 (UTC)[reply]

""Yes, its easy to telescope time when you are talking about the distant past. The transition from two separate groups of conqueror and conquered to "complete mingling" LOL took over three hundred years - ie, like from 1700 to now. It wasnt quick. Jameswilson 02:58, 14 March 2006 (UTC)[reply]

Indeed, it wasn't until Chaucer's time (late 1300s) that English was used to produce laws. As he said of one of the characters in the Canterbury Tales, "she spake the Frenssh of Stratford-atte-Bow", implying that Parisians would laugh at it. -- Arwel (talk) 16:05, 14 March 2006 (UTC)[reply]

When you say after 400 years of Roman occupation the Gauls had an "admixture" of Roman genes, do you mean actual Gallo-Roman-Normans with Roman blood and ancestry would be considered (parallel with our modern terminology) "minorities?" Thast, in other words, even those with SOME Roman genes were still mostly Norman/Celtic? I may've found something to answer myself: on the "Normandy" page, it says this- "The Normans were a mixture of the INDIGENOUS Gauls and of the Viking invaders ... "

P.S. Thank you all for correcting my assumption that the Normans and Anglo-Saxons mixed earlier than they most probably did.

And yet one mroe question- Where Normandy now is, were the Breton tribes the Celtic tribe that "made up" the Gauls? I'd read something about it, but i'm not sure if I believe the Gauls and Bretons were seperate, though of the same "blood," or race.-London 23:25, 15 March 2006 (UTC)[reply]

The Roman occupation of Gaul didnt change the genetic stock that much because, although it lasted a long time, there was never a mass migration of people from Italy. Also the soldiers and administrators who were stationed there could be from anywhere in the Roman Empire, from Portugal to Syria and beyond, so the "new blood" was by no means all (or even mostly) Italian. Jameswilson 23:55, 15 March 2006 (UTC)[reply]

London- that "The Normans were a mixture of the indigenous Gauls and of the Viking invaders ... " sounds right to me- I don't believe the Normans that invaded England were at all part Roman, or any Norman for that matter. The reason I think that is because the Normans invaded France all the way to Paris, so the King of France, not the ruler of the Roman Empire at the time, granted them the land called Normandy- I believe the governing people there were Norman/Gaulish "soldiers" or the equivalent of "policemen," =P. "The Normans were of mixed ancestry, including Norse and old British Celtic (which is why the region is called Brittany)" - In other words, I agree with that statement- the Normans were a Norse-Celtic race. - Leif

There is little reason to say that Normans were Norse and Celtic because of Brittany. They were Norse and French. Brittany was the province west of Normandy and had even been at war with the Normans in the 930s. Whether the French contributions was of Gaulish (Celtic) or of Frankish (Germannic) stock may be hard to tell. Rmhermen 18:22, 16 March 2006 (UTC)[reply]

Ah! If only there was solid proof of what they were (other than mostly being Norse). So it's more probable that they were Frankish (Germannic) mixed with Norse? -London 01:59, 19 March 2006 (UTC)[reply]

fratricide (fragging) in Vietnam[edit]

How many officers were fragged during the Vietnam war?--Sydneyndon 06:12, 13 March 2006 (UTC)[reply]

What an odd question. Are you referring to family members finding themselves on opposite sides? And then why only officers? DirkvdM 08:50, 13 March 2006 (UTC)[reply]
Apparently it's a new euphemism for killing your allies aka friendly fire though I'd never heard of it before. A particular subset of this is the intentional murder of officers. The thing about killing your own officers with a fragmentation grenade ("fragging") is that presumably the method was chosen (rather than a convenient bullet) because it it indistinguishable from enemy action. Given that, and that for some reason nobody owns up, the question is impossible to answer. Notinasnaid 09:01, 13 March 2006 (UTC)[reply]
A google search on fragging turns up http://home.mweb.co.za/re/redcap/vietcrim.htm which includes a table with estimated fragging deaths per year (totalling 1013) and quotes historian Terry Anderson of Texas A & M University:
"During the years of 1969 down to 1973, we have the rise of fragging - that is, shooting or hand-grenading your NCO or your officer who orders you out into the field," says historian Terry Anderson of Texas A & M University. "The US Army itself does not know exactly how many...officers were murdered. But they know at least 600 were murdered, and then they have another 1400 that died mysteriously. Consequently by early 1970, the army [was] at war not with the enemy but with itself."
Secondly, this word isn't a new term and it does not refer to friendly fire in general, but to specific attempts by enlisted men to kill their officers. http://www.etymonline.com/index.php?term=fragment cites a usuage in 1972:
"Fragging is a macabre ritual of Vietnam in which American enlisted men attempt to murder their superiors. The word comes from the nickname for hand grenades, a weapon popular with enlisted men because the evidence is destroyed with the consummation of the crime." ["Saturday Review," Jan. 8, 1972]
mennonot 09:50, 13 March 2006 (UTC)[reply]
Agreed. "Fragging" meant "fragmenting" and derived from fragmentation grenades and landmines. Someone who was "fragged" was someone who got hit by a handgrenade or landmine, and "fragging" as an active verb was murdering a disliked person. However, how many officers were fragged? No one can possibly know, as the reports would never have been clear, and the men would be unlikely to admit that they had murdered their officer. It's a popular Hollywood myth, though. Geogre 11:52, 13 March 2006 (UTC)[reply]
Not that it really matters, but the new euphemism to which I was referring was "fratricide". Fragging was pretty well covered by the media, and is reasonably well known. Notinasnaid 12:03, 13 March 2006 (UTC)[reply]
Fratricide means the murder of one's brother, which doesn't sound very euphemistic to me. JackofOz 22:29, 13 March 2006 (UTC)[reply]
Surely it's a euphemism if it isn't actually your brother. I know the literal meaning, which is why I found the reference so confusing in this context. Maybe it's just me. Notinasnaid 07:34, 14 March 2006 (UTC)[reply]
A euphemism is a way of avoiding mentioning something undesirable. "Fratricide" is a slightly formal word for murder, specifically of a brother. It's the antithesis of avoiding mention of murder. Transferring the object of the murder from a brother-in-arms to a real brother is not euphemism, but a form of projection. Or it could be a simple case of personification, as in "Band of Brothers". JackofOz 12:41, 14 March 2006 (UTC)[reply]

I wonder how much of this happened during the first World War. It certainly happened the other way around because a lot of soldiers were shot for desertion. And when they had to go 'over the top' their officers had they pistols drawn, pointing at them. And I once saw in a film how this was done in the Soviet army during WWII too (then again, that was a film and a western one at that). It would make sense if the soldiers would then have also said "Fuck it, I'll kill you before you kill me." But then, in those days (especially during the first war) the levels of authority were still firmly in place and the thought probably never entered the soldiers' minds. I suppose it took the liberating sixties to make people actually take that step. DirkvdM 06:03, 14 March 2006 (UTC)[reply]

The Soviets likely would have not only killed the soldiers responsible, but their entire families, too, so this was not much of an option for them. However, during WW1 the soldiers did mutiny and kill their officers, leading the the Russian Revolution, at least according to Dr Zhivago. StuRat 01:27, 16 March 2006 (UTC)[reply]
Russian soldiers did mutiny, but that was at the home front when they refused to shoot striking workers and in stead joined them. At least, that's what started the revolution, afaik. DirkvdM 15:27, 20 March 2006 (UTC)[reply]
The classic WWI film on killing your own soldiers is Paths of Glory. --Halcatalyst 05:11, 15 March 2006 (UTC)[reply]
I should say, the classic film set in WWI. --Halcatalyst 05:33, 15 March 2006 (UTC)[reply]
I think you were OK first time, Halc. "Saving Private Ryan" could be called "a WWII film". JackofOz 13:46, 15 March 2006 (UTC)[reply]
And for a WW1 book on this topic, how about Ernest Hemmingway's A Farewell to Arms" ? StuRat 01:27, 16 March 2006 (UTC)[reply]

ancient egypt[edit]

can you please tell me how ancient egypt's religion affected their economy and what sites would be good to look at. i have tryed some sites and none of them answer my question. thankyou kindly

You can check out Egyptian mythology and see if there's anything that you're looking for. If not, you can always ask again here. Igor the Lion(Roar!) 13:24, 13 March 2006 (UTC)[reply]

Unknown Research Institute[edit]

Does anyone know of a fledgling "Blakeley Research Institute" based in Geneva? I and some of my colleagues have been receiving encrypted mails from someone there for some time already. The cipher was Vigenère, and decryption revealed a threat of some sort to a colleague. I've checked with the police, and they say they're doing everything they can, but meanwhile, we're still getting these letters! Google didn't turn up anything, so it could be a hoax, but just checking to see if there really is no such place. After all, Google has been known to make mistakes. Thanks. Igor the Lion(Roar!) 13:30, 13 March 2006 (UTC)[reply]

Google doesn't make mistakes, but it may leave things out, because not every web site is accessible to web crawlers and bots -- only about 2/3 of them, in fact. OK, maybe Google makes mistakes, but not that kind. They just find what's available, index it, and respond to search requests.
That said, it's hard to say whether you're receiving a real threat or not, though the persistence suggests something serious. I'd say, keep complaining to the police. Since it's international (I assume you're not in Switzerland), approaching higher authorities (e.g., the FBI if you're in the US) might be a good idea. --Halcatalyst 16:37, 13 March 2006 (UTC)[reply]

Sectioanlism[edit]

  • did constitution makers have to make comprimises because of sectionalism? how did sectionalism affect the new nation and decisions?
        MysteriousStranger 14:31, 13 March 2006 (UTC)[reply]
We have a short article on Sectionalism. Have a look at it and follow up on the links, and you'll have a better handle on the questions you're pursuing. --Halcatalyst 16:29, 13 March 2006 (UTC)[reply]
Which constitution makers? AllanHainey 11:50, 14 March 2006 (UTC)[reply]

Literary criticism of author Julio Cortazar's short story "The Night Face Up."[edit]

March 13, 2006 11:36

I have looked everywhere, except where it obviously is and cannot find it. What I am looking for is the short story "The Night Face Up" and any literary criticism that pertained to the story. I have an essay to hand in Monday of next week, so any help would be most appreciated.

Many thanks,

JR

I hope you know about our article on him. It's fairly brief, but it has a number of external references (in Spanish). When I Googled "Julio Cortazar" stories I got a huge number of hits -- over 200,000. Refining that to "Julio Cortazar" "The Night Face Up", the number was reduced to about 14,000. Looks like you have a gold mine there. --Halcatalyst 20:51, 13 March 2006 (UTC)[reply]

TV series duration[edit]

Is there any web site that lists the number of seasons and episodes for various U.S. television series?

Try http://www.tv.com/. It not only covers current shows, but past shows, as well. User:Zoe|(talk) 17:22, 13 March 2006 (UTC)[reply]
The IMDB also has this sort of information, with better coverage of recent or popular shows than others. It started as a list of guest appearances only, so if there were episodes with no guest stars they'd be less likely to be listed. --Anonymous, 18:32 UTC, March 13, 2006.

March 14[edit]

Oldest historical document of a human being[edit]

What is the oldest known record of a human being and in what media was it made in? (By this I do not mean any records of human existence, but a sort of documentation where individuals or humans were first mentioned in). I appreciate any direct answears. --Funper 01:04, 14 March 2006 (UTC)[reply]

Define "document." Is a cave painting, made by a prehistoric man, documenting the hunting afternoon of the tribe considered a "document"? ☢ Ҡiff 01:20, 14 March 2006 (UTC)[reply]
I am guessing they mean the earliest written language document. Would that be Sumerian cuneiform script ? Or perhaps Egyptian heiroglyphics or Chinese characters ? StuRat 04:46, 14 March 2006 (UTC)[reply]
At what point does it become language? You put your suggestions in a nice order of development from pictures to characters. It's probably when the pictures became more abstract that one starts to call them 'langauge'. But that's rather arbitrary. Language is anything that conveys a message and the cave paintings probably did that. Although it may not have been meant for others but as a re-enactment or even preparation (getting ito the spirit of the hunt). Maybe you can only speak of language when it is meant to comvey a message to others who know nothing of what happened (and who can actually get a good idea of that through the message). So the cave paintings were language (we sort of get the point when we see them) but maybe not intended as such. DirkvdM 06:12, 14 March 2006 (UTC)[reply]
I don't think cave paintings qualify as language any more than modern paintings do. The problem is, they mean whatever you want them to mean, not anything specific. A hunting scene could be a record of a specific hunt, a mythical hunt, a desire for a future hunt, etc. StuRat 23:14, 14 March 2006 (UTC)[reply]
What about a novel then? It may have happened, it may be wishful thinking or it may actually happen. But it's language. DirkvdM 08:41, 15 March 2006 (UTC)[reply]
I think possibly the first instance where a person is named would be in the Epic of Gilgamesh. Whether the person named was actually human or divine, however.... -- Arwel (talk) 16:12, 14 March 2006 (UTC)[reply]
I suspect some rather boring records, like receipts for animals or grain paid in taxes, would be older. StuRat 23:21, 14 March 2006 (UTC)[reply]
The cuneiform script you mentioned started off as an accounting language, I believe (the article doesn't seem to mention that, though). DirkvdM 08:41, 15 March 2006 (UTC)[reply]
I believe you are correct. They also used unique marked clay cylinders they wore around their necks to make a "signature impression", which carried legal weight. That's smarter than our current credit cards, which don't always require a signature, allowing for massive fraud. StuRat 18:32, 15 March 2006 (UTC)[reply]
Sepultures ; and painted hands on grotto walls ; they tell us "I was here". True record of a person, still we do not have the names. --DLL 19:49, 14 March 2006 (UTC)[reply]
Do you mean sepulchres, the burial vaults? (Sepulture is the act of burying a person). JackofOz 22:17, 14 March 2006 (UTC)[reply]
sepulture is also a synonym of sepulchre, though I think the former has a slightly "antique" feel when used that way. - Nunh-huh 00:12, 15 March 2006 (UTC)[reply]
OK. Thanks. JackofOz 13:42, 15 March 2006 (UTC)[reply]
For the antique touch : excuse my french (touch). --DLL 20:15, 15 March 2006 (UTC)[reply]

What happens when a college guy tells a prospective girlfriend he's on SSI?[edit]

How would she react? Would she decide to move on to another guy right then and there, or will she continue with him?

Of course the first thing on your mind may be: "It Depends On Her Personality". Perhaps if you were a college girl looking for a boyfriend, also in college, and he told you this, what would you say & do?

And for the guy that doesn't want to tell her about the way he keeps a roof over his head (in other words, about his SSI), what are some good ways to evade her question without lying? --Shultz III 02:31, 14 March 2006 (UTC)[reply]

Evading the question is strikingly similar to lying. If you're on SSI then there's probably a reason for it. Are you hiding that also? If she's aware of whatever condition you have and is ok with it, then she'll most likely be ok with your monetary situation also.
Think about the implicit message you're sending. If you evade these things it sends a message of low self esteem to her. On one or another level she starts wondering how far it goes, the feeling like you have things to hide as well as the actually being able to hide things. If you're just straight up and mention it like it's no big deal when it comes up it sends the message that you know that these things don't make you any less than anyone else. You don't even have to lie about being uncomfortable about it, she'll appreciate and understand it. Just say something like 'my rent? oh I use my SSI checks to pay for it.', 'Yeah, it's kinda nice and it kinda sucks, that's how things go. So what's up with...'
If she's cool about it then cool, if not it's best to get it out of the way as soon as possible. -LambaJan 03:43, 14 March 2006 (UTC)[reply]
How would the guy get it out of the way as soon as possible effectively? Can you provide specifics, please?
by telling the girl it of course. and you already touched the important part yourself, it depends on the girl. you simply can't say they all will do this or that. Boneyard 12:29, 14 March 2006 (UTC)[reply]
By the way, the reason for SSI would be Asperger's Syndrome. With that disorder, it's a whole lot harder to land a job (at least with a living wage) than it is already for normal people. That is one reason SSI is needed. --Shultz III 10:13, 14 March 2006 (UTC)[reply]
That's nothing to be ashamed of. Just let her know straight up. If you don't you're heading straight for giving her a double shock (a small one for your condition and a HUGE one for all the undercover stuff). -LambaJan 02:45, 15 March 2006 (UTC)[reply]

Maybe this hypothetical woman would have more trouble finding out that you've been asking around for a surrogate mother for your children on Talk:Asperger syndrome. Anyway, where you get your money is really your own business, but any woman you may have a relationship would probably want to know that people with Asperger syndrome can't read facial expressions well and have certain social problems. If you find a woman willing to overlook the obvious signs of your disability, she probably won't be shocked to hear about the diagnosis. Don't lie to a woman if there's a chance you could get caught. They hate that. There's no reason, however, to volunteer every piece of information about yourself without being asked, unless there's a chance she's assuming something that's not true. For example, it wouldn't be good if she thinks you're squeaky clean, and you've actually done prison time, or if she thinks you're inexperienced, and you've had plenty of action. In my experience, college women aren't as focused on money, status, and job stability as older women with jobs, and you can get by on looks and charm alone. Brian G. Crawford 04:35, 15 March 2006 (UTC)[reply]

Legislatures and Judiciary of India.[edit]

I want to know the defination of an ordinary bill in an indian parliment session

A Bill is a proposal before a Legislative body for the members to contemplate, ratify and pass (to become law) or reject. Please also see our Section on Indian Parliament. --Tachs 13:41, 14 March 2006 (UTC)[reply]

Looking for type of banking account[edit]

(This question may or may not be suited for this part of the reference desk - feel free to move to Misc if appropriate, just let me know).

This is a multi-part question that involves banking and tax laws. I live in the United States.

So, in high school, my economics teacher mentioned a type of banking account that one sets up for one's child's future college education. What I remember:

  • That money was taken out of one's paycheck before taxes, either a percentage or a set ammount per week. (I may be mistaken about both parts of this.)
  • That this account gained interest, but you do not pay tax on this interest. I do not know if this included state tax - I lived in New Hampshire at the time, and we aren't too fond of state taxes. I now live in New York, where not only do I pay state taxes, but city taxes (....goddamnit).
  • That the account must be used on the assigned child's college (etc) eductaion expenses. That if the money is withdrawn for any other reason, then a huge penalty is payed.

First, does this type of account exist? What's it called? Links?

If it does exist, can I set one up and begin contributing to said account for my very young nephew and niece back in NH? Are there cross state issues? Can the kids' parents also contribute to the same account, or, can they set up an account to which I am able to contribute with the same benefits (pre-tax withdrawal from pay, etc.).

Much thanks for all assistance provided. A piece of cake to whomever digs through the mess above and provides an answer or twelve. Thanks! --Jeffrey O. Gustafson - Shazaam! - <*> 06:26, 14 March 2006 (UTC)[reply]

Google reveals that they're called 529 plans, in that faintly ugly hyper-numerical way you guys have of naming your financial instruments. savingforcollege.com seems to have a wealth of information. From a brief inspection of their FAQs it seems that you don't have to invest in any particular state but can choose the one with the best available plans, and still use them to pay tuition anywhere in the US, and you can in fact invest in many different states and the IRS won't count them against each other! --Bth 09:15, 14 March 2006 (UTC)[reply]
The numerology comes from the fact that such government-sponsored programs are commonly known by the subsection of the law which authorized them. Thus there is also 401(k), a tax-deferred investment program offered through employers. --Halcatalyst 02:18, 15 March 2006 (UTC)[reply]
See also Education Savings Accounts. Neither come out pre tax, but yes you can contribute to the same account their parents do in both cases. Payroll contributions are handled by your payroll department, so you may not have every option, but you may be able to work one out. Publication 970, listed as the external link is your best bet to get the authoritative information on the tax treatment. - Taxman Talk 20:59, 16 March 2006 (UTC)[reply]

Big and Small Government[edit]

Could somebody please explain to me the meaning of both "small government" and of "big government" and provide me with some examples of each so I can understand what they refer to. The entry on Wikipedia is rather vague and I haven't been able to find any simple explanations on the Internet. Thankyou.

"Small government" refers to the idea that government should do the minimum necessary to ensure the security and safety of its citizens (see minarchism and libertarianism for the extreme version). In this view, power always corrupts, individual liberty is best guaranteed by people left to get on with their own lives, and government is a necessary evil that should be minimised. In contrast, "big government" refers to the idea that the government can and should do more for its citizens (a prime example being the provision of a social safety net). Here, some of government's powers are viewed as the best way of providing certain things that those on the small government side would want done by the private sector, individual liberty is best guaranteed by ensuring everyone has a minimum standard of living, and government is viewed as having the capacity to do go−od (though obviously it isn't always in practice). Note that the widespread use of the terms in general political discourse (esp. in the US) is an example of highly successful framing by those in favour of the small government side -- the idea is that pretty much everything from the New Deal on constitutes unacceptably big government. --Bth 09:09, 14 March 2006 (UTC)[reply]

Thanks, that was exactly what I needed.

What is unacceptable is rather POV. A good example of a big government would be State Socialism, aka Communist States. An interresting mix was Nazi Germany, where there was a strong but rather decentralised government, so it didn't all come from the top. DirkvdM 08:52, 15 March 2006 (UTC)[reply]
Yes, it's the POV that the people doing the framing are trying to push -- that was my point. --Bth 10:02, 15 March 2006 (UTC)[reply]

Citing images used in presentations[edit]

If I use an image in a PowerPoint presentation for my statistics class, do I need to cite where it came from? (e.g. clipart that came with MS Office, clipart from a collection I purchased, images from Wikimedia Commons, and pictures from my school's website) Hermione1980 16:08, 14 March 2006 (UTC)[reply]

General rule is that you do not cite public domain artwork (such as Microsoft's free clipart). If you do want to cite it, you do not do it in the presentation. You do it with an accompanying list of citations. If the image is not in the public domain, you should cite the image. Usually this is done in very small text along the bottom of side of the image. --Kainaw (talk) 16:20, 14 March 2006 (UTC)[reply]
Microsoft's clipart is not technically public domain -- the copyright is often ambiguous -- but in general clipart is not cited. Images are, in my experience, not usually cited in presentations unless 1. they are presentations of data that you did not create (i.e. a chart or graph from someone else's data or paper -- good to give credit where it is due), or 2. the author is well-known or well-known-enough that it would be interesting/useful to know who made it. Think of Powerpoint as an elaborate slideshow -- citations are usually not required for presentations, though dropping names is useful for controversial claims or unique claims that you yourself did not come up with. Of course in a written paper this is very different. --Fastfission 16:30, 14 March 2006 (UTC)[reply]
I beg to differ. Whether something is public domain or not is irrelevant to the question of whether you must cite it. Plagiarism is a particularly sensitive issue in academia, and for your own sake, you should provide a citation for every source you have used, in a form dictated by your institution's guidelines. If you need guidance on how to do this, you should ask your statistics teacher: if nothing else, you will have covered your ass by doing so. - Nunh-huh 19:48, 14 March 2006 (UTC)[reply]
I agree with Nunh-huh. Look: you'll very rarely get accused of anything if you cite too much, but failing to cite is a gross offense. When in doubt, cite. If it's not yours, cite. Geogre 21:14, 14 March 2006 (UTC)[reply]
The only reason I'm asking instead of just doing is because I've already turned in the presentation. I'm just wondering if I should send a panic email to my teacher saying "sorry, did this at 1:00 in the morning and forgot to cite the images", or if I can, well, just leave it be. Hermione1980 21:40, 14 March 2006 (UTC)[reply]
Since you feel guilty, yes, let your teacher know. Confession is good for the soul. The real guilt in plagiarism is intent or carelessness. Correct citation and not-too-close paraphrases are skills students need to be taught; they don't come naturally. A lot of teachers punish students who are merely ignorant. OTOH, there are plenty of cheaters, and plagiarism violates the prime directive of education: to get educated. --Halcatalyst 00:57, 15 March 2006 (UTC)[reply]
Citing clip art in a presentation is silly, but sure, check with your teacher. Clip art is meant to be used without citation (see our article on clip art). And I don't think that not citing slides in a presentation is necessarily plaigiarism at all -- again, citation in a presentation is usually used in specific circumstances, when something is clearly attributable to someone else and should be. I write this as someone who has witnessed umpteen million academic powerpoint presentations and have never seen clipart cited. The idea defies the entire point of clip art. --Fastfission 01:15, 15 March 2006 (UTC)[reply]
I've told you a million times to quit exagerrating. --Halcatalyst 02:13, 15 March 2006 (UTC)[reply]
Yes, exaggeration will cause the destruction of the universe. StuRat 18:39, 15 March 2006 (UTC)[reply]
Have you noticed that the word exaggeration sounds like eggification = edification? The universe will end in the smell of rotten eggs. --Halcatalyst 23:14, 15 March 2006 (UTC)[reply]
You must cite Wikimedia Commons clearly with at least 48-point font. :D --69.234.54.17 10:37, 19 March 2006 (UTC)[reply]

Found this interesting[edit]

In village news : "Your participation in the "Micro Wikipedia Survey" at User:Shuo Xiang is much appreciated!"

The survey asks three basic questions and you may want to visit the page and give an advice. This leads me to another quick survey : what is your preferred source of information (WP, Googl, other) and when ? --DLL 19:41, 14 March 2006 (UTC)[reply]

I think this is a wonderful student project and urge everyone to participate, as I have. I believe there is a current discussion of "stable articles" on WP somewhere. Anybody know where, and what the status is? --Halcatalyst 00:47, 15 March 2006 (UTC)[reply]

I don't recall - I know its out there -- but User:Nikodemos's user page has very interesting related material. Herostratus 05:26, 15 March 2006 (UTC)[reply]

Halcatalyst, do you mean Wikipedia:Stable versions? (which seems to have done a couple of test cases but is still mainly a discussion of criteria and techniques) Or are you thinking of the more ambitious Wikipedia 1.0 project? --Bth 08:22, 15 March 2006 (UTC)[reply]

Depends. Whenever I think I might find something in my library, I use that (dictionaries, reference books, travel guides). When it is something 'established' and not prone to pov I use Wikipedia (mostly for scientific and technical stuff). When those fail, I usually Google it, but am very wary of what sites I read something in (knowing how to read between the lines is an essential ability for information gathering). For pov-prone topics I try to use as many sources as humanly possible (and as varied as possible). In this respect it is handy that there are Wikipedias in various languages and that I can read several of them. I wouldn't trust the English article on Cuba, for example, because it'll be too influenced by US editors and assorted maniacs. For this sort of thing I prefer my travel guides (Rough Guides have good background sections). By the way, I also use Google as a spell checker. DirkvdM 09:08, 15 March 2006 (UTC)[reply]

Yes, we need this perusing of other sources. The French WP is quite poor but may help to translate some notions or fill in some red links. I tried Polish, Finnish, Spanish, any WP site with latin charset, you do not have to understand everything to gather something valuable. Googl tools use should be learned soon - preferably before learning to walk, still I expect some free (ad free, public domain) research tools.
When there is a question here in WP:RD, it is more interesting to try and search something than to complain about misuses of our own etiquette. Also, any pov is very rich as it gives plenty of uncensored information, then you have to take it with your own grain of salt.--DLL 20:11, 15 March 2006 (UTC)[reply]
  • I've been to "Le Bistro" on the French WP several times and one or two times got toasted for asking my question. That's one reason why I'm on the side of the questioners here who sometimes get arch responses from us. Over there, too, some people realize it's better just to do your best to help, and that sometimes it's necessary to call the dogs off your guests. No offense intended. --Halcatalyst 22:38, 16 March 2006 (UTC)[reply]

Info on "Shrift"[edit]

When someone went to Shrift, as Juliet does in "Romeo and Juliet", what did that consist of? Was it the same as modern day confession, or was it different in some ways?


well, shrift = confession, though when Juliet went to shrift she wound up poisoned and unrequited. As to whether it was the same as modern day confession, almost certainly not, in particulars, but probably pretty much so in generalities. Since she's a fictional character, though, when she went to shrift, was she going in her fictional time period and locale, or in the time period and locale in which Shakespeare wrote the play (much in Shakespeare is anachronistic in this way)? Eiher way, confession of sins, the imposition and satisfaction of a penance, and absolution would have been involved. - Nunh-huh 00:10, 15 March 2006 (UTC)[reply]
If you know you're going to die, you really need to get to confession. Also, the monk (Brother Lawrence) was the one who had the potion to feign death. If you're about to fake death, there is a chance you'll die, so you need to confess your sins before that happens. Juliet died in a state of grace. This means a lot to a Tudor audience (cf. Hamlet having a chance at Claudius while the latter is at his prayers but refrains, because he doesn't want him to go to heaven), and suicide was not settled as a mortal sin (see, for example, John Donne's Thoughts on Emergent Occasions, written only a few years after Romeo and Juliet, which is in praise of suicide from a clerical point of view). Geogre 10:50, 15 March 2006 (UTC)[reply]

The Beatles White Album[edit]

Could you please tell me where I could find the current value of an Unopened Beatles White Album? This album includes all inserts. I would also like to find out how to identify if it is mono or not. Thank you.

The album was recently reissued on vinyl, so you really should find out whether you have an original or a reissue. If I recall correctly, it's one of the more commonly available Beatles albums that are not compilations. Try a used record store or ebay. Wait until there's buzz about the Beatles again, and the demand and price should go up somewhat. Brian G. Crawford 03:53, 15 March 2006 (UTC)[reply]

Assuming you are talking about an lp, my "Rock & Pop Preis Katalog" (German) lists the following, with prices in DeutschMarks (divide by two for an indication of price in Euro):

Parlophone PPCS 7067 (1LP - black/yellow label) 283 DM
Apple EMI SMO 2051/52 (2LP) 26 DM
Apple EMI 8119 (2LP) 43 DM
Mobile Fidelity Sound Lab MFSL 2-072 (2LP, half speed) 73 DM
Apple BTX 1006 (2LP, mono) 27 DM

So there's quite some variation, but it would have to be a special edition to really be worth a lot it seems. I wonder what selection the 1LP and half speed versions have - they can't contain all songs. While you're looking at Beatles albums, keep an eye out for 'Impression', printed by 'Deutsche Buchgemeinschaft Parlophone 6279', which was given a 1735 DM pricetag. There will probably be just a few of those around. Somewhat more realistic would be 'Please please me (export release), Odeon ZTOX 5550' at 475 DM. And then there's a whole bunch more with prices in the hundreds, but tracking them down would require so much effort and experience it's probably not worth the trouble. DirkvdM 09:26, 15 March 2006 (UTC)[reply]

Can't be a very recent catalog if the prices are in DM? Anyway, I found a guide here here, seems the PPCS 7067 is the first pressing, and most valuable. (The low price above is probably because it's missing two sides?) Anyway, it seems the price range is quite extreme depending on the serial number. --BluePlatypus 14:37, 15 March 2006 (UTC)[reply]
The "half speed" designation for the MFSL release refers to the mastering process, not playback, so it would indeed contain all songs. (In fact, if an LP were to be played back at half speed, it could conceivably contain twice the content.) Unfortunately, our Mobile Fidelity Sound Lab article doesn't contain a lot of detail on the mastering process, and what information is there is marked as questionable. --LarryMac 17:48, 15 March 2006 (UTC)[reply]

March 15[edit]

Tommy Douglas[edit]

Article about T. C. Douglas includes no mention of his wife, Irma. I would like to know if she is still living, and some details of her life.

Wayne Sheldon

I got 49 hits on Google when I searched for "T. C. Douglas" irma. Try it and see. --Halcatalyst 05:05, 15 March 2006 (UTC)[reply]
She was alive as of 1991. http://www.saskschools.ca/~stdom/g12main.html -- 142.214.8.69 14:22, 23 March 2006 (UTC)[reply]

Richard Nixon on Idi Amin[edit]

Does anyone remember that Richard Nixon quote that went something like, "Idi Amin? He's nothing but a goddamn cannibal asshole. He'd eat his own mother. Christ! He'd eat his own grandmother!" I can't seem to find the exact words. Maybe I'm thinking of something Hunter Thompson wrote, so who knows how reliable it is. Brian G. Crawford 00:32, 15 March 2006 (UTC)[reply]

Can't answer the question, but it sounds like Nixon in private: brilliant and nasty. --Halcatalyst 00:49, 15 March 2006 (UTC)[reply]

Primitive attempt at reverse engineering[edit]

A co-worker related a story that might be an urban legend, but I wanted to know if it's true. It relates to a crashed or scrap commercial airliner that was copied by a technologically backward society, perhaps the inhabitants of an isolated island, with the belief that it would work as long as its shape was correct. Does this sound familiar to anyone? Any ideas on where to look for more information?--Joel 00:35, 15 March 2006 (UTC)[reply]

See cargo cult. alteripse 00:45, 15 March 2006 (UTC)[reply]

Thanks!--Joel 22:57, 17 March 2006 (UTC)[reply]

Shane[edit]

For those who read the novel, in Ch. 9 was there a person screaming (I don't notice any)?. If so, what did he do (where did he go)? And after they pinned Shane down, why couldn't he punch him twice? And why did Marian want Joe to stop fighting and let Shane do it all himself? Confusing book. --hello, i'm a member | talk to me! 00:43, 15 March 2006 (UTC)[reply]

The Tecora[edit]

Hi. I'm looking for any info about a Portuguese slave ship called the Tecora. Could anyone please help me out here? It was a very infamous one, from what I hear in the 1997 movie Amistad & was functioning around 1840. --User:EricSpokane

  • Well, a Google search provides a number of useful-looking sites, including the Amistad Research Center, which might provide some helpful info. --jpgordon∇∆∇∆ 03:57, 15 March 2006 (UTC)[reply]
  • I just added several paragraphs to the entry for the Tecora.

Jews in 20th Century America[edit]

How did Jews find jobs in the 1900s? How did their observance of sabbath refrain from interupting work hours?

It depends very much on the particular Jew. :) I for example, do not keep Shabbat at all. Others, such as Hasids, work only for fellow Jews which allows them to observe shared customs. Many others worked in businesses that did not require Saturday work, such as some office jobs. Still more simply made up any missed work on Sundays, which was the day of rest for everyone else. I hope this helps, but there is not one simple answer. Superm401 - Talk 03:47, 15 March 2006 (UTC)[reply]

They found work the same way as any other minority facing discrimination, by taking jobs no one else wanted, by running their own businesses, by getting jobs from others in their group, and by entering established professions that guarantee work, like medicine, dentistry, and law. Brian G. Crawford 04:45, 15 March 2006 (UTC)[reply]

When you talk about observing sabbath, you're talking about Judaism as a religion, not about being a Jew by descent. A distinction often overlooked. DirkvdM 09:42, 15 March 2006 (UTC)[reply]
That's also talking about a particular subset of the religion. Durova 20:34, 15 March 2006 (UTC)[reply]

aborigines[edit]

Please help we require information about the following:

Kinship and Family structure within the aboriginal culture.

Also Aboriginal bush food and preparation.

That sounds like homework. Geogre 10:53, 15 March 2006 (UTC)[reply]
See Aborigine. -LambaJan 20:56, 15 March 2006 (UTC)[reply]

Death Penalty[edit]

Which crimes impose the Death Penalty? thankyou!

In which country or countries? AnonMoos 08:23, 15 March 2006 (UTC)[reply]
Thankfully none here in the Netherlands. Not even desertion, I believe. DirkvdM 09:43, 15 March 2006 (UTC)[reply]
Amnesty have a list of what the situation is in different countries, but for those that have it they don't list which crimes it's applied to. --Bth 09:54, 15 March 2006 (UTC)[reply]
Contrary to popular belief, the United Kingdom has not entirely outlawed capital punishment. It can still technically be applied for treason and 2 other crimes whose names escape me. So the UK definitely imposes the death penalty. But were you asking about countries that actually apply the death penalty, that is, actually carry out executions? Things can change, as they did when the USA reintroduced capital punishment in the mid 1970s. But for the foreseeable future, the UK is not going to execute anybody, for any reason, in my opinion. JackofOz 13:39, 15 March 2006 (UTC)[reply]
That was the case until relatively recently (arson in the royal dockyard was the famously obscure offence, but may have been a bit of an urban legend) but is now in the category of common misconception itself. The Human Rights Act of 1998 was amended to completely revoke the death penalty for all offences. See Amnesty again, here. --Bth 13:56, 15 March 2006 (UTC)[reply]
Ah, excellent. Thanks. JackofOz 14:05, 15 March 2006 (UTC)[reply]
ISTR that Lese majeste (sp?) was the other supposed capital offence. Grutness...wha? 06:16, 16 March 2006 (UTC)[reply]

Murder and treason (at least) in the U.S. (not all states have capital punishment, but the federal govt. does) --Nelson Ricardo 01:44, 16 March 2006 (UTC)[reply]

Please see Use of death penalty worldwide --69.234.54.17 10:32, 19 March 2006 (UTC)[reply]

Marijuana[edit]

I have several questions I could not find the answers to.

1. How much does the War on Drugs cost New York State annually?
2. Has there ever been any evidence that terrorists use marijuana money to fund terrorism?

Yes, but they are only able to make money off drugs because they are illegal. So, politicians who keep drugs illegal are at least partially responsible for providing terrorists with a way to make money. If drugs were legal, then pharmaceutical companies would get rich off them, not terrorists. StuRat 17:59, 15 March 2006 (UTC)[reply]

3. Is it possible to convert marijuana into a pill, and if so, how expensive is it?

A good herbalist can turn any botanical into a pill. Pills predate Western medicine by centuries, and probably existed before writing, although the old-fashioned ones don't look as pretty and aren't measured very carefully. I've heard that originally hashish was made by keeping marijuana in one's shoe until the resin clumped together as a pellet. Without accounting for the raw material, this sounds inexpensive in the extreme. --Joel 23:34, 17 March 2006 (UTC)[reply]

4. How much money would New York State make by legalizing marijuana and producing it?
5. What would be some major benefits of legalization of marijuana?
6. Has the Federal Government ever denied funding to a state because of a violation of a federal law?

Thanks.

  1. Millions of dollars.
  2. Poppies and Coca are the main illegal crop used to fund terrorism, not marijuana.
  3. The active ingredient, THC, can be synthesize and put into a pill, but marijuana itself cannot exactly be put into a pill. The drawbacks of THC in pill form are that: a. THC is one of many active compounds in marijuana that provide relief to a variety of conditions. b. It is easy to regulate your dose when smoking.
  4. Millions of dollars
  5. Decriminalizing marijuana is smart for many reasons.
  6. Yes.

--Shadarian 14:48, 15 March 2006 (UTC)[reply]
(I made your list easier to read. — QuantumEleven | (talk) 10:25, 16 March 2006 (UTC)) [reply]

3. Not quite a pill but see Sativex - AllanHainey 15:10, 15 March 2006 (UTC)[reply]
1) No idea, ask the NYSDoCJS. 2) FARC, for instance, does trafficing in marijuana, although mainly coca. 3) Yes, the active substance in marijuana, THC is already available in pill form. It is cheap when legally produced. 4) I seriously doubt the state would produce it. Why would they, even if it were legal? They tend to let the drug companies do it. 5) Presumably making more legal resources available for other crimes. And medicinal uses, which don't require full legalization, however. 6) If a state law violates federal law, it gets smacked down in SCOTUS. If it doesn't but isn't clearly legal either, you can end up with a situation like in California. --BluePlatypus 15:24, 15 March 2006 (UTC)[reply]
1) Impossible to say. Consider a cop who pulls over a drunk driver who then turns out to have some drugs on him. Is the pay for that cop included in your cost of the war on drugs?
2) Yes. In Indonesia there are gangs who are well-connected to terrorism and raise funds through kidnapping, drugs, and pirated videos. Basically, it is asking if criminal activity helps fund other criminal activities.
3) It depends on what you want. If you are trying to make rope, the pill form is rather useless. If you are trying to get high, the pill form works. If legalized, the drug companies would certainly price it around the same level as Viagra.
4) None at all. It would cost money. For example, the cost of police/courts/jails for handling drunks is greater than the income from taxing alcohol. Plus, we'd be paying extra since it will take at least two stoner cops to do the same job as one who isn't stoned.
5) None. What is the major benefit of legalized gambling or alcohol? Maybe some head of a current drug gang will raise his sons to become politicians as the Kennedy's did after their illegal alcohol business was legalized at the end of prohibition. But, I wouldn't consider that a major benefit.
6) Yes - many times. In the 80's, some states raised the speed limit beyond Federal standards. The Feds denied funding for interstates in any state with too high a speed limit. So (after the next state election) the speed limits were reduced again.
All in all, this is the same set of questions that have been raised for over 40 years now. It comes down to: If we legalize a crime, the crime goes away! Why not legalize murder. There are many people I'd like to kill. If it was legal, I'd be right out there doing my share. It would be a great benefit too. Just give me a few days and I'd get rid of a lot of people who waste a lot of our time and money. But, alas, some people are more concerned with getting stoned instead of cleaning up the gene pool. --Kainaw (talk) 16:23, 15 March 2006 (UTC)[reply]
Are you equating marijuana with murder ? Hardly. At worst I would equate it with tobacco and alcohol, both of which are legal. And even if a thing is bad, that doesn't necessarily mean it should be illegal. Adultery isn't typically illegal, because of the obvious problems with enforcement. The same logic should apply to marijuana. StuRat 18:07, 15 March 2006 (UTC)[reply]
You got me all wrong. I was simply saying that if we are going to make a current crime legal based on the idea that making it legal will reduce crime and create a benefit to society, then I would prefer it to be murder. I promise, I won't kill the pot heads - well, not many of them. --Kainaw (talk) 19:25, 15 March 2006 (UTC)[reply]
Ever consider that somebody might be waiting for that to occur so they can murder you ? :-) I also fear you are falling into the circular logic trap: "anything which is illegal is bad, and all bad things should be illegal". That argument essentially means that nothing should ever be legalized, like driving a car without having to have a person on foot in front of it waving a red flag. Yes, that was the law at one point in some places.StuRat 00:59, 16 March 2006 (UTC)[reply]
I'm sure there are. I currently have a nice collection of enemies. But, if you want to play around with bad comparisons (such as murder and marijuana)... Ever consider that your kid's bus driver (assume you have a kid if you don't) is waiting for marijuana to be legalized so he can get really zoned out before driving all the kids to school? There's always a paper tiger argument. I tend to find them funny - especially when you can add: Won't someone please think of the children!!! --Kainaw (talk) 01:07, 16 March 2006 (UTC)[reply]
"Driving under the influence" would still be illegal, as it is for someone driving under the influence or other legal drugs, like alcohol. StuRat 01:12, 16 March 2006 (UTC)[reply]
Actually, medicinal THC would undoubtedly be far cheaper than Viagra. First, Viagra is patented, which means no competition and higher prices for that reason. Second, Viagra had development costs which the Pfizer no doubt needs to recoup (although they no doubt have by now). Third, Viagra is just much more expensive to produce, since it requires some complex organic synthesis. THC can however be extracted from cannabis, which is a far simpler process. Taking those factors into account, the cost of producing THC should be more comparable to Aspirin or Caffine than Viagra. --BluePlatypus 17:43, 15 March 2006 (UTC)[reply]

The pill you're looking for is Marinol. It's apparently as well liked by regular marijuana users as nicotine gum is by smokers. Brian G. Crawford 16:43, 15 March 2006 (UTC)[reply]

Legal drugs are the main funding of crime. Either that or gangster movies lied to me. But back to original question 4, I think New York (world) would definetely benefit from legalizing illegal drugs. First since they are already so popular, it would be a very profitable legitamite business like cigarettes which in turn fuels the economy. Taxes would not be used to controlling drug trafficing and keeping drug dealers in jail so less taxes. Organized crime would become less powerful and there would be alot less drug related murders. Also like previously mentioned people are afraid of people getting high and driving. Well basically illegal or not, driving under the influence is a personal standard and you cant stop it. People who drive under the influence do not really care if the substance they are using is legal or not. Tutmosis 03:06, 16 March 2006 (UTC)[reply]
Saying that illegal drugs fund terrorists therefore we should legalize drugs and terrorists will be out of money is just plain ignorant. What some fail to realize is that economically, anything illegal generates an overhead value of perpetrating crime, and that is what the terrorists are exploiting. Take away the crime from producing an (arguably) harmless drug like marijuana, and they simply adjust to dealing worse things like oxycontin or heroin or whatever. What would you rather have, a healthy market for illegal marijuana and the knowledge that some terrorists may have made money off getting a patriot high, or a legal market for marijuana, and a flood of lower cost higher risk drugs like cocaine or x-tacy because terrorists still gotta make a buck, and they will do it however they can... Replace terrorists for organized crime in the previous statement and the effect is the same. Not that I agree with drug use, or our (America's) drug policy, or anyone in general, but I think this is an interesting aspect that deserves some attention. --Jmeden2000 22:40, 16 March 2006 (UTC)[reply]
Yes, I would expect them to move into other criminal fields, but the fewer fields open to them the better. Marijuana, in particular, can be grown just about anywhere, so is difficult to control. Other drugs, particularly methamphetamines, are quite difficult to produce, so could be easily stopped if we would just make the effort (for example, by requiring prescriptions for pseudoephedrine products, from which meth is made). And terrorists trying to move into new criminal markets may get some major trouble from the current drug lords in those fields, who will not appreciate the competition. You could even get an unholy alliance where the current drug lords report the terrorist to the DEA to get them shut down. Limited law enforcement agency resources could also be focused on the serious public health issues, not marijuana. StuRat 01:34, 17 March 2006 (UTC)[reply]

Adelaid (Adelaide?) Stevenson[edit]

I am trying to find a book about or book by or a website relating to him. I believe he was a politician during McCarthy era.... any help given would be most appreciated.

Is Adlai Stevenson who you were looking for? --Bth 15:05, 15 March 2006 (UTC)[reply]
Yes, and he's not from Adelaide, Australia. StuRat 17:47, 15 March 2006 (UTC)[reply]
Adlai is of Hebrew origin, and its meaning is "my ornament." Also represents an Aramaic contracted form of Adaliah (Hebrew) "God is just." --DLL 19:48, 15 March 2006 (UTC)[reply]
Offhand, I can't think of anybody else (well-known) with that name. Are there any? Are there variants of the name? --Halcatalyst 23:10, 15 March 2006 (UTC)[reply]
Google came up with a few. I wonder how many are named for the Stevensons? User:Zoe|(talk) 23:27, 15 March 2006 (UTC)[reply]
Adlai also appears to be a rare surname -- [16]. User:Zoe|(talk) 23:31, 15 March 2006 (UTC)[reply]
Well, there was his grandfather, Adlai E. Stevenson, who was a U.S. Vice President. JackofOz 23:42, 15 March 2006 (UTC)[reply]
There's AES I (Vice-President), his grandson (AES II Governor of Illinois), his son (AES III, Senator from Illinois), his son (AES IV, a Chicago newscaster) and his son (AES V, aged 12)! There was also an Adlai Stevenson Hardin, another grandson of AES I, and his son, ASH Jr, a judge in NY. Don't know of any unrelated Adlai's, though. The name seems to have entered the family via AES I's maternal grandfather, Adlai Osborne Ewing. - Nunh-huh 03:45, 16 March 2006 (UTC)[reply]
Wow, I'd a laid odds against there being so many. StuRat 01:15, 17 March 2006 (UTC)[reply]
The gr'father's bio is now renamed Adlai E. Stevenson I, BTW.
--Jerzyt 06:43, 29 December 2007 (UTC)[reply]

The Zacharias Moussaoui trial[edit]

I've been following the Zacharias Moussaoui trial from a distance (other-side-of-the-atlantic distance) and from what I understand the prosecution is trying to award him the death-penalty because he could have come forward and revealed the 9/11 plot. This is correct, yes?

What I'm wondering is that isn't that argument legally dubious due to the fifth amendment? What I'm saying is that if you want to prosecute him as an accomplice to the 9/11 plot, fine, but can you really prosecute someone for not speaking out when doing so he would incriminate himself? Oskar 17:25, 15 March 2006 (UTC)[reply]

I believe he was found guilty of assisting in the plot, not withholding info. However, in the sentencing phase, they are arguing that by withholding info he caused deaths, which would make him eligible for the death penalty. And yes, this is shaky ground. I suspect he will get life in prison, instead. StuRat 17:51, 15 March 2006 (UTC)[reply]
To an outside observer it seems very strange indeed.... Why don't they use the argument that he should get the death-sentance because he actually caused deaths by helping int the planning, not just assister by not saying anything? Can't you get the death-sentance for just assisting a crime under federal law? Thanks for the answer btw Oskar 18:03, 15 March 2006 (UTC)[reply]
His claim, which seems to be supported by the evidence, is that he was training for a future repeat of 9-11, and not for 9-11 itself. As such, this makes him ineligible for the death penalty, since he didn't actually cause any deaths but was only planning to do so. The prosecution is therefore arguing that he at least had knowledge of 9-11, so did cause deaths by withholding info, and is thus eligible for the death penalty. Yes, it's a stretch. And no, I don't expect it to convince the judge. StuRat 18:13, 15 March 2006 (UTC)[reply]
They don't have to convince the judge - they have to convince the jury. All death sentences in the U.S. have to be handed down by a jury in a separate phase of the trial after the conviction. The judge only decides what may or may not be presented as evidence. Rmhermen 19:19, 15 March 2006 (UTC)[reply]
That's not entirely true, the judge has the power to pull the death penalty off the table. Some jurisdictions also allow the juries to determine guilt but leave the sentencing to the judge. Also, in other cases, the defendant may opt to bypass the jury and have the judge decide. This is sometimes felt to be good for a hated defendant who is technically innocent (due to a warrantless search, for example). In such cases, juries are more likely to find them guilty, anyway. StuRat 00:54, 16 March 2006 (UTC)[reply]
I don't believe that your statements are correct since Ring v. Arizona in 2002. See [17] Rmhermen 17:47, 16 March 2006 (UTC)[reply]
My reading on that decision is that the jury must find the defendant guilty and decide if all the conditions exist to make the defendant eligible for the death penalty. However, the judge typically still has the final word on whether the death penalty is actually imposed. That is, unless there was a mandatory death sentence for a crime, which, AFAIK, is never the case. StuRat 01:09, 17 March 2006 (UTC)[reply]
Thanks for the clarification, guys. Oskar 21:56, 15 March 2006 (UTC)[reply]

child neglect[edit]

what effect would child neglect have on children between the ages of 0- 16?

A simple Google search gives some good info. Here's a site with some detail on the different kinds of neglect. The effects can be physical (malnutrition, injury, etc) and emotional (low self-esteem, psychological damage, etc). -- Наташа ( UserTalk ) 20:58, 15 March 2006 (UTC)[reply]
Child neglect redirects to Child abuse, but that article is woefully short on neglect, as is the abuse template. Seems like systemic bias supported by focusing the template on abuse. - Taxman Talk 20:43, 16 March 2006 (UTC)[reply]

Music in East Germany[edit]

Were records from bands such as Kraftwerk, Neu! and Can (and other krautrock) available in East Germany, or had the groups in question been banned? —DO'Neil 21:01, 15 March 2006 (UTC)[reply]

I believe they were permitted, although in practice it was no doubt pretty difficult to get your hands on the records (as with anything imported). This page says Kraftwerk even tried to get permission to play there but were denied, supposedly because they needed to bring computers. --BluePlatypus 00:41, 16 March 2006 (UTC)[reply]

meaning of a snoop dogg song lyric[edit]

i have asked everyone i know and i have been wondering forever any help would be most gratful in telling me what the lyric means (i treat a bitch like 7-up never have never will) i know it is crazy but it is in my head to find out what this means thanks for your help in finding the answer

I can't believe I'm answering this. 7-up, the beverage, had an advertising slogan about caffeine: "Never had it, never will." It never had any caffeine, and it never would. One assumes that Mr. Dogg is suggesting that he has never had the lady in question, in some sense, and never intends to "have" her, either. Geogre 22:01, 15 March 2006 (UTC)[reply]
7-up makes some think of Snow White (the princess). --DLL 18:12, 16 March 2006 (UTC)[reply]

i think he's talking about the caffeine line but more in the sense that he's a player and will never let a 'bitch' make him settle down.

Well the whole thing goes:

A-hah, niggaz be brown-nosing these hoes and shit. Takin bitches out to eat, and spendin money on these hoes, knowhatI'msayin? I treat a bitch like 7-Up I never have I never will I tell a bitch like this Bitch, you without me is like Harry Melvin without Bluenotes You'll never go platinum

So, I guess, just like 7-up never had caffeine, Snoop will never take a b*&#ch to dinner or spend money on her. Does that mean he takes men out to dinner and buys men things?

Healthy / Unhealthy lifestyles[edit]

Can you please list at least 5 aspects of a healthy lifestyle and 5 aspects of an unhealthy lifestyle. Finally, please comment on the advantages and disadvantages of both aspects

I'm sure we can. Please do your own homework. Geogre 22:02, 15 March 2006 (UTC)[reply]
Lifestyle diseases offers some useful information. --Halcatalyst 22:29, 16 March 2006 (UTC)[reply]

sociology[edit]

What is meant by identity, how is identity formed, and to what extent can we shape our own identity

Well, the article Identity (social science) isn't too helpful in itself, but it does contain links to some articles that are more thorough, including self-image, self-concept, and self (psychology). Chuck 00:13, 16 March 2006 (UTC)[reply]

Identity can also be formed through reflection following Doing Our Own Homework.

I get tired of these frequent admonitions to questioners to do their own homework. Can't asking questions be part of homework? Or are they only allowed in the classroom (where nobody wants to, I wonder why)? How about we get our noses out of the air? --Halcatalyst 22:25, 16 March 2006 (UTC)[reply]
P.S., not that it's never appropriate to say it, but do we have to scold? --Halcatalyst 22:27, 16 March 2006 (UTC)[reply]
I think the "Do your own homework" is not a way to discourage the slacker students but rather to discourage the other helpdesk people from rewarding the slacker. I.e. I don't think the lazy kids ever even look again (if they knew how to find the help desk page in the first place -- they are, after all, lazy to start with), but rather to stop other people from going ahead and doing the homework for the lazy kid and thereby rewarding him or her, if he or she does manage to find the page. It's not that I mind answering an obvious question, but I do not like the idea of helping a cheater prosper. Having spent all those years in college not cheating and watching cheaters win, and then all these years teaching and having to waste my time coming up with clever clever ways of preventing the cheats, I darn sure don't want to be part of a system that rewards the incurious. Geogre 02:36, 17 March 2006 (UTC)[reply]
By the way, not that I think anyone's saying it's not, but I think Chuck's answer was right on target. He directed the person where to find the answers without spoonfeeding. This is exactly what a reference librarian would do, and I think it's a great service. Makemi 02:44, 17 March 2006 (UTC)[reply]
Agreed, although I understand despairing at people who haven't found the Search box. (And sociology is a social science rather than one of the humanities, so, if we wanted to be picky and despairing....) Geogre 10:38, 17 March 2006 (UTC)[reply]

Parole[edit]

My friend was sent to jail for 3 and a half years and I was wondering is he able to get parole? If so when?

That depends very much on what jurisdiction he is in and what he was convicted of. Could you provide some more details? Chuck 00:10, 16 March 2006 (UTC)[reply]

Demonic possession[edit]

Lately there's a lot Demonic possession incident covered in Indonesian media (TV), with dozens of people affected. Is there such cases in any other countries (Demonic possession which affect a group of people)? roscoe_x 00:02, 16 March 2006 (UTC)[reply]

See Salem witch trials. User:Zoe|(talk) 01:36, 16 March 2006 (UTC)[reply]
Dont take it as a fact but I believe in Italy there are alot of posessions. Tutmosis 03:08, 16 March 2006 (UTC)[reply]

March 16[edit]

Searching for 3 movies[edit]

I have a couple movies I cant seem to find or remember, brief description below:

  1. Some woman has a gorilla as a pet in a suburban setting and the movie basically revolves around her and the monkey relationship.
  2. Set I believe in new york city, some monster goes around ripping hearts out. 2 or 1 cop tries to catch it. Final battle is in a water sunken subway.
  3. Some tv eletrician is a serial killer. Cops go and get him at his store. He skins one of them and wears his skin. Eventually half way through the movie gets caught. Gets the death penalty, gets electrecuted and posses some bystander lady.

Thanks, this movies are early 90's, late 80's. The monkey one i have no idea. Tutmosis 03:33, 16 March 2006 (UTC)[reply]

  1. I can think of either Buddy or Born to Be Wild.
  2. Not sure, thinking about it.
  3. is it Ghost in the Machine? ☢ Ҡiff 05:08, 16 March 2006 (UTC)[reply]
The first one also sounds like Bedtime for Bonzo, but that's a chimp and a gorilla/future president Reagan. The third is a combination of Silence of the Lambs and Fallen. In the latter, Denzel Washington plays a policeman chasing a demon that goes from body to body. Geogre 11:49, 16 March 2006 (UTC)[reply]
Umm I dont think any of those are it guys...Tutmosis 14:41, 16 March 2006 (UTC)[reply]

I'm not sure on the first two but the third one is the movie Shocker staring Mitch Pileggi of X-Files fame. Pardon my lack of formatting skills http://imdb.com/title/tt0098320/plotsummaryNigelthefish 18:56, 16 March 2006 (UTC)[reply]

The first one is most likely Mighty Joe Young. The last one sounds very much like Fallen. --Kainaw (talk) 19:00, 16 March 2006 (UTC)[reply]

Second movie is Split Second http://www.imdb.com/title/tt0105459/, set in London with Rutger Hauer. My favorite action Halloween movie. JK

First movie could be "Max mon amour" (1986), directed by Nagisa Oshima and starring Charlotte Rampling. Please let us know once you have found what you are looking for and let's find out what it was... Good luck!

how can I email the author of this article?[edit]

To Help Wikipedia Ref. Librarian From peterwshelley

'''QUESTION: WHO WROTE THIS ARTICLE? I WOULD LIKE TO EMAIL THEM ABOUT MORE INFO SOURCE FOR INTERSTER TV SHOW

Removed whole cut-and-paste of the article Television in South Africa and your email address...
In answer to your question, no one person wrote the article. If you click on the history tab at the top of the Television in South Africa article, you can see a list of the Wikipedia users who have contributed to the article. If you want to add information about Interster, just click on the "edit this page" tab and add the information yourself! That's how the amazing Wikipedia works! --Canley 06:47, 16 March 2006 (UTC)[reply]

Sigurd[edit]

This is kind-of a strange qustion. What version of Sigurd story is considered better, the Edda poems or the Volsunga saga? KeeganB

It depends on what you mean by "better," but the Volsunga Saga had much the larger effect on music and literature. It was a sensation in the later 19th century. The eddic version is pretty brief, and I'm sure it influenced a number of other Icelandic writers, but the Volsunsaga is the one that influenced poets, novelists, and composers in other languages and nations. I would also argue that it's better from a literary point of view, since it's closer to what we expect in terms of narrative literature. Geogre 11:52, 16 March 2006 (UTC)[reply]

Osama bin laden[edit]

How many countries do/did want bin Laden captured or killed? or how many countries are/ was helping to get bin Laden?
USA wanted him to be killed for sure.Suraj vas 10:05, 16 March 2006 (UTC)[reply]

The United States would prefer to have Osama captured alive. If killed, he becomes a martyr. The U.N. considers him a wanted criminal - but to be specific about countries that are after him, the following countries have had direct acts of terrorism on their land related to Osama: The U.S., U.K., France, Spain, China, Israel, and Indonesia. Afghanistan and Pakistan are both actively hunting for Osama (being paid to do so by the U.S.). Russia still has a healthy dislike for him ever since they were driven out of Afghanistan. The Saudis claim to have disowned him and claim that they would arrest him if he came back. That is the position of the more powerful countries. The less powerful ones tend to follow the views of whichever country is providing them with the most money and/or military. As for powerful countries that support Osama: Iran, Palestine, Indonesia (I list it in both sections because it is a very divided country), and various political/military groups around Africa. But, so summarize... Osama bin Laden is not wanted only by the United States. It would make a good movie though. I wonder if Michael Moore reads this? --Kainaw (talk) 13:52, 16 March 2006 (UTC)[reply]
Indonesia does not support Osama, rather it is Osama and affiliates who trained and funded terrorists and subversives there, they kinda exploited internal turmoils in Indonesia for their fundamentalist goals. Indonesia, which is the largest Muslim-majority country in the world, could have been a strong partner with US against terrorism because of its moderate Muslim population, unfortunately, US currently isn't paying much attention to that region. --Vsion 09:34, 19 March 2006 (UTC)[reply]
You missed Yemen, Tanzania, Kenya, Jordan, Saudi Arabia were attacks occured - probably more still. Rmhermen 18:36, 16 March 2006 (UTC)[reply]
  • Just about every government on the planet wants him captured. While he may have some support in Palestine and other Arab nations (But not particularily much in Iran; Osama doesn't consider Shiites to be true muslims). Not every country is actively participating. It's not like there's much Iceland or Bolivia could do, except promise to grab him if they see him. But it's safe to say there's no government that would publicly harbor him. Even Libya, a traditional haven for terrorists, has swung around considerably in the last years. Saudi Arabia wouldn't just arrest him, they'd execute him for his terrorism in that country. (In case you missed it, Bin Laden's first and foremost goal has always been to unseat the Saudi government.) If there was any government prepared to harbor him, he'd already be in that country. Given that there isn't, what remains is what he's probably doing, hiding around in Afghanistan-Pakistan where he has popular support and people prepared to hide him. --BluePlatypus 14:24, 16 March 2006 (UTC)[reply]
What about Venezuela? --Username132 (talk) 21:37, 16 March 2006 (UTC)[reply]
Seriously? Chavez has nothing at all to gain from helping Bin Laden. While leftie South Americans aren't crazy about "yanquis", they hardly condone killing them in the name of a fundamentalist interpetation of a religion they don't have, in order to promote a political agenda that runs totally opposite to their own. Chavez anti-USA rhetoric and Castro-cuddling is mostly that: Rhetoric. It scores points with the demographic that supports him. The same goes for a lot of the anti-Chavez rhetoric in the USA. That doesn't mean Chavez isn't an asshole, who's continually extended his powers and weakened democracy and free speech in his country. Although he does still enjoy popular support. But that's the same situation as with the USA's trusty ally Vladimir Putin. In any case, the USA is still buying oil from Venezuela, literally "business as usual". Also, keep in mind the rest of the world doesn't have the grudge against Chavez the USA does, apart from voicing concerns about democracy there (as in Russia). But they'd have a heck of a lot more against him if he started harboring terrorists.
Any country that would give Bin Laden a safe haven would undoubtedly be hit with economic sanctions, just like Libya was. And I just can't think of any country where Al-Qaida's popularity would outweigh the economic disaster incurred by helping them. (Not to mention a possible invasion) --BluePlatypus 23:02, 16 March 2006 (UTC)[reply]
Unless they had nuclear weapons, I think any country that publicly harbored bin Laden would most definitely face a US invasion, just as Afghanistan did. StuRat 00:55, 17 March 2006 (UTC)[reply]
And Iraq, Sadam may have to been able to shine clinton's cock when it came to slaughtering his own people, but by supporting and cheering for bin laden, he crossed a lineHow cme tkn4 01:56, 17 March 2006 (UTC)[reply]
Iraq never harbored bin Laden, and, AFAIK, never publicly supported him either. If you have proof to the contrary, please enlighten me. Others who did cheer on bin Laden, like the Palestinians, not only were not invaded or bombed by the US, but actually received US financial support, at least until they went and elected a self-admitted terrorist organization (Hamas) to lead them. StuRat 17:27, 17 March 2006 (UTC)[reply]
Hamas are not self-admitted terrorists; they regard themselves as freedom fighters. What's the difference? You tell me. --Halcatalyst 00:18, 18 March 2006 (UTC)[reply]
Using suicide bombers to intentionally blow up busloads (or restaurants) full of civilians is the difference, and this is what Hamas admits to doing and apparently plans to continue doing, especially if they can get more funds from Europe to buy more bombs. StuRat 04:14, 19 March 2006 (UTC)[reply]
Granted, the other side (us) doesn't send out suicide bombers, although our side causes the same type of civilian death and destruction (in spades), refering to this as the regrettable but important, nay essential, collateral damage.
Is it your contention that, since civilians are sometimes killed accidentally during military operations by Western powers, those Western powers have a duty to fully fund Hamas, to give them an opportunity to intentionally blow up busloads of civilians, just to be fair ? StuRat 13:31, 21 March 2006 (UTC)[reply]
  • So, what's the difference between terrorists and freedom fighters, other than one's point of view? --Halcatalyst 05:13, 20 March 2006 (UTC)[reply]
Terrorists would be "those supported by people who make no distinction between targeting civilians and killing them while targeting military targets". - Nunh-huh 11:04, 20 March 2006 (UTC)[reply]
Also, the chances of achieving the group's goals should be considered. Since the Hamas goal of driving all Jews out of Israel has 0% chance of success, killing people for that purpose is just plain murder, not a "military campaign". Their argument that murdering civilians is somehow for "defense" is also absurd, as Israeli attacks increase following Hamas attacks, they don't decrease. StuRat 13:40, 21 March 2006 (UTC)[reply]
Actually Saddam never supported bin Laden, or vice versa. There were no ties between the secular Iraqi governmnet of Saddam and Al'Quaida. Infact if I remember right Osama once asked the U.S.A. (when they were still allies) for permission to assassinate Saddam but they refused. AllanHainey 13:19, 17 March 2006 (UTC)[reply]

Gelato[edit]

hi guys im am doing an italain project for italy and was wondering in which part of sicily does gelato come from

thanks renee

This webpage suggests it originated in Greece or China ~1000BC, though it was significantly developed by a Sicilian called Francisco Prokopioo in the C17th. These two pages suggest that the answer to your question might be "Mount Etna" -- in ancient times, its higher reaches were the only source of frozen water in that climate. --Bth 10:51, 16 March 2006 (UTC)[reply]
The guy created the "café Procope" in Paris, France. His story may be searched thus also. --DLL 18:08, 16 March 2006 (UTC)[reply]

The oldest trick[edit]

What is the oldest trick in the book? Ohanian 11:46, 16 March 2006 (UTC)[reply]

Oh, no! We're not falling for that. Asking hypothetical questions is the oldest trick in the book! Geogre 11:54, 16 March 2006 (UTC)[reply]
One candidate for oldest trick in the book might be Harriet Sohmers Zwerling, whom the New Yorker magazine recently (13 March 2006) called the city's grande horizontale. "Which is wonderful," she said. "I'm seventy-seven fucking years old." --Halcatalyst 16:50, 16 March 2006 (UTC)[reply]
  • In magic, the oldest trick in the book is considered to be the Cups and balls. First performed in ancient Egypt. But one Dedi did a decapitation trick around that time too. - 131.211.210.17 10:44, 17 March 2006 (UTC)[reply]

Israeli National Health Service[edit]

Can anyone please inform me whether an Israeli citizen who has lived outside Israel for twenty years comes to Israel, can benefit immediately from the Israeli National Health Service. Thank you.

Surely the Israeli National Health Service can tell you. Presumably they have a site under http://www.gov.il somewhere? --Anonymous, 00:00 UTC, March 17, 2006.
They have a contact page. Superm401 - Talk 04:46, 17 March 2006 (UTC)[reply]

What does "Hame Baham" mean?[edit]

I think it's farsi. If no one knows the answer, can anybody direct me to a place where I could find an answer?

Question also found in the language reference dsk. --DLL 18:06, 16 March 2006 (UTC)[reply]

The new revelation[edit]

I have read the book called The New Revelation. I would like to submit the book as article. Anyone can help me? —This unsigned comment was added by Ahanibal (talkcontribs) 15:36, March 16, 2006 (UTC).

It is easier than you think. Create a user account (it is free!). Then, click on The New Revalation and create the new article.--Kainaw (talk) 14:53, 16 March 2006 (UTC)[reply]
Or better still, click on The New Revelation. Bear in mind that you have to write an article about the book, using a neutral point of view. You cannot copy any part of the book onto this web site (except for very limited and clearly marked quotations). The article needs to say something worthwhile (e.g. beyond just listing the author and date), or it will just be deleted. (P.S. I see someone has already created this article, but it will be deleted very soon.) Notinasnaid 15:02, 16 March 2006 (UTC)[reply]
Copyright constraints can be ignored for this text though. At least if it's the Arthur Conan Doyle book, since that's no longer copyrighted. --BluePlatypus 16:28, 16 March 2006 (UTC)[reply]

Suffragan of Prague[edit]

When did Olomouc become suffragan of Prague? It should be sometime in the fifteenth century I think, but anything more specific would be greatly appreciated. I think that suffragan is a sort of religious authority one city's religious leaders have over another city's religious leaders, but of course in that period this was a lot more meaningful for political things than it is now. Thanks, Makemi 17:08, 16 March 2006 (UTC)[reply]

Finally figured it out. In case anyone's interested, Suffragan usually refers to a kind of sub-bishop, but can also refer to a sub-diocese, under the archdiocese. Olomouc became suffragan diocese of Prague at the same time Prague became archdiocese, in 1344. I was reading something totally unrelated (about notation) and it did one of those "When Olomouc became suffragan of Prague" and I became thoroughly lost. Makemi 17:45, 16 March 2006 (UTC)[reply]
This site is best for this (if I may say so) rather obscure subject. As I understand it suffragans are proper bishops with proper dioceses in the Catholic Church. (In the Anglican Church they are not. They come below bishops and look after a part of the diocese). Jameswilson 23:59, 16 March 2006 (UTC)[reply]
Yes, they are "real" bishops, but they are under the authority of the Arch-bishop. The Anglican church doesn't have quite as thorough a hierarchy as the Roman Catholic church. In this particular case it was a bit more confusing because I thought it might be getting into the whole Utraquist/Hussite issue. And yes, it's really obscure, which is why I couldn't figure it out at first. Thanks, Makemi 01:43, 17 March 2006 (UTC)[reply]

Is it better to say...[edit]

...whilst adenoviruses have a larger cloning capacity (about 35 kb) (Bob et. al., 2009).

or

...whilst adenoviruses have a larger cloning capacity (about 35 kb; Bob et. al., 2009).

I'm sure I've seen it done the second way before... --Username132 (talk) 19:17, 16 March 2006 (UTC)[reply]

Either version is OK but the second reads more easily, so I'd go with that one. Do people still say "whilst"? JackofOz 19:43, 16 March 2006 (UTC)[reply]

Well, I'd say the first. When you use parenthetical reference, the parenthetical expression has to be the reference. I.e. you can't mix your words with your citation. Geogre 21:21, 16 March 2006 (UTC)[reply]

I concur in and with Geogre; though surely neither usage is altogether wrong, the latter is generally disfavored. With respect to "whilst", few in the US use the locution (I happily count myself amongst their archaic number), but many in the UK, I think, still use the term (see, e.g., [18]). Joe 21:30, 16 March 2006 (UTC)[reply]
Yeah, in the UK we use both, although I think I generally favour 'whilst', since that's the first word to spring to mind as I attempt to string a sentence together. It's really weird that it's considered so old in the US! --Username132 (talk) 21:34, 16 March 2006 (UTC)[reply]
I've always been interested in that difference, which is why I just went out to dictionary.com to see what I could find. It seems whilst comes from Middle English whilest, alteration of whiles, whiles. See whiles, it commanded, so I did. You'll have to go see for yourself what I learned. From a linguistic standpoint, it looks like the two usages simply evolved differently. I see no reason to regard either as old-fashioned, since they're both very much alive in different places. --Halcatalyst 22:19, 16 March 2006 (UTC)[reply]
"Whilst" is pretty much dead in colloquial language in Australia, but it crops up in formal contexts. On public transport, you'll see notices like "Please do not converse with the driver whilst the tram/bus is in motion", and "Look both ways when alighting from the carriage". This seems to be an attempt to use "proper" language, which I recognise is often required from a legal standpoint (even though plain English would usually be more effective). But "whilst" is no more formal or appropriate than "while". I think the writers are just trying too hard to appear to be bureaucratic, and going overboard. JackofOz 00:06, 17 March 2006 (UTC)[reply]
"Alighting": That's another word I daresay you'd never hear in the US. It has such an airy sound. I was going to say "light and airy." Contrasts with "get off," doesn't it? :P But the latter has Anglo-Saxon claims. --Halcatalyst 01:19, 17 March 2006 (UTC)[reply]
Oh, and carriage. That's sooooooooo nineteenth century. Well, I guess we all have to go back to our roots. It's just when we carry them around with us.... ;-) --Halcatalyst 01:21, 17 March 2006 (UTC)[reply]
All this talk of "getting off" and "roots" is giving me some ideas for the weekend .... JackofOz 01:45, 17 March 2006 (UTC)[reply]
I've virtually shouting rows with my former PhD supervisor over the use of while/whilst. He's an Aussie and hates whilst, calling it archaic. I'm a NZer, and the word is still used here and has a subtle difference in that "while" has an implication of "at the same time" whereas "whilst" implies "although". These are only slight nuances though; for most purposes the two terms are virtually synonymous. Grutness...wha? 08:09, 17 March 2006 (UTC)[reply]

"While" performs both functions on this side of the Tasman.

  • (although) "While I've read this book, I haven't read that one". We'd never say "whilst" in that case.
  • (at the same time) "I fell asleep while I was reading this book". This is the only sense where "whilst" would be possible. But very uncolloquial. JackofOz 08:20, 17 March 2006 (UTC)[reply]
  • It's exactly the same in the US. So, did you guys emigrate from America? --Halcatalyst 00:12, 18 March 2006 (UTC)[reply]
  • Today's Trivia Surprise: "The English language didn't actually start in America." :-) JackofOz 00:57, 18 March 2006 (UTC)[reply]
The English language wasn't invented in America. It was just perfected there. Geogre 17:32, 18 March 2006 (UTC)[reply]

federal empolyment for a convicted felon[edit]

If someone is a convicted felon and sex offender. Are they precluded form working federal civil service?

First, what country are you referring to? Assuming it is the U.S., it would depend on the clearance required for the job. --Kainaw (talk) 21:25, 16 March 2006 (UTC)[reply]

I Dig Those Blues...[edit]

The beginning of the song 'Harden My Heart' by 'Quarterflash' features a short instrumental that I really dig... who does Blues like this? --Username132 (talk) 21:30, 16 March 2006 (UTC)[reply]

What? Please suitly emphazi your question. Luigi30 (Ταλκ) 21:41, 16 March 2006 (UTC)[reply]

Orson Welles film "Othello" film in Ireland in 1955[edit]

Thank you so very much for all your guidance.

I am working on my Ph.D. dissertation on the films of Orson Welles. It has come to my attention that Orson Welles filmed an Othello using British and Irish actors during the year of 1955.

The actors involved were: Robert Coute, Michael MacLiammoir, Suzanne Cloutier, and Michael Laurence. My research has provided little information regarding the "bios" and any clear "head shots" taken of the above-mentioned cast members. They would prove to be immensely helpful.

Should you possess any reviews, bio information, or pictures of the above, I would be deeply grateful.

I have never used this service and am a wee bit confused about not leaving my email address. I shall leave all pertinent information for you.

Most sincerely,

DeLaine Morrow <email removed>

I wonder about the propriety of one's researching a doctoral dissertation at the Humanities Reference Desk of Wikipedia, but, in any case, perhaps the IMDB listing may provide more information, at least as to the filmographies of the actors whom you mention. Joe 00:00, 17 March 2006 (UTC)[reply]
Hatcatalyst suggests that my assessment that it was rather odd for a doctoral candidate to be posing queries about his/her research topic at the Reference Desk was perhaps overly harsh, and, having considered the several factors involved, I am inclined to agree and consequently strike through that text; I surely hope that I haven't turned anyone off to using the Reference Desk, which thought I also conveyed in an e-mail to DeLaine (although of course DJ is correct in writing below that infrequency of usage militates against one's leaving an e-mail address). Joe 02:23, 17 March 2006 (UTC)[reply]
We don't recommend leaving emails becase 1) hardly anybody replies using them and 2) it's just asking to get huge amounts of spam. Replies will be left on this page. You should return and view it periodically. DJ Clayworth 00:02, 17 March 2006 (UTC)[reply]
Pretty well known film, The Tragedy of Othello: The Moor of Venice - in 1952 it won the Grand Prize at Cannes. It has been available in a restored version on video since 1992 and on videodisk since 1995. Rmhermen 00:02, 17 March 2006 (UTC)[reply]
IMDB gives some present-day reviews of it which are interesting. DJ Clayworth 00:05, 17 March 2006 (UTC)[reply]

March 17[edit]

Axioms[edit]

Hi, I read the article about axioms but I don't understand the examples...since I only see the symbol of an empty set followed by other symbols I have never seen before. can someone give me a couple of 'non-mathematical' (lingüistic and philosophical) examples that mean the same thing as the examples in the article? I mean, translate them somehow? because I have no clue what those mathematical sentences mean. :|.--Cosmic girl 01:11, 17 March 2006 (UTC)[reply]

Discrete mathematics is a logical disipline which deals with real world quantities rather than abstract ones, if that helps. It's not big on axioms, though. --Halcatalyst 01:25, 17 March 2006 (UTC)[reply]
That said, all logic and mathematics (for example, Euclid), used to be expressed in words rather than symbols. You could say that the major premise in Aristotelian logic is an axiom. To cite the well-worn example,
All men are mortal.
Socrates is a man.
Therefore, Socrates is a mortal.
IMHO, though very useful in sorting things out, this sort of logic, and all logic really, doesn't create new knowledge, only realizations. I guess you could call me analogical. Just don't call me anal. --Halcatalyst 01:35, 17 March 2006 (UTC)[reply]
Well, an example of an axiom in ordinary words would be "If A is exactly the same thing as B, then B is exactly the same thing as A". As you see, it's something so simple that it can be considered self-evident. Once you have a couple of these (a system of axioms) you can start combining them and eventually turn up with conclusions that aren't that self-evident at all. That's basically how math works. --BluePlatypus 02:09, 17 March 2006 (UTC)[reply]
The original idea of an axiom was that it would be something self-evidently true. When Euclid wrote his great treatise on geometry, he used axioms like "Two things that are equal to the same thing are equal to each other"; "if equals are added to equals, the sums are equal"; and "any line can be extended indefinitely in a straight line" (although he also used one axiom that was not so self-evident -- the parallel postulate -- evidently because without it he could not prove things he wanted to prove).
However, mathematicians later came to believe that while axioms should be as simple as possible, there is no need for them to be self-evidently true; what matters is whether, if you assume a particular set of axioms, you can derive a consistent system of mathematics containing interesting, non-obvious results. For example, it turns out that by assuming alternative axioms contrary to the parallel postulate, you get either of two interesting systems of non-Euclidean geometry. So in math, an axiom now is simply something you choose to assume as true for the purpose of proving other things. If it happens to relate to something in the real world, that's nice, but it's not necessary.
Outside of math, though, people are likely to expect anything called an "axiom" to be self-evidently true.
--Anonymous, 04:00 UTC, March 17, 2006.

Nice, thanx!, do you think that Axioms are tautologies? I mean...they can only justify themselves within logic, but never provide a basis for logic besides and beyond it's inner consistence? (meaning a WHY is it this way instead of something else)...is it this way?.--Cosmic girl 17:35, 17 March 2006 (UTC)[reply]

Not really. A tautology is more like a statement that can be boiled down to "If X, then X". You can get somewhere with "justifying" axioms because you can examine the consequences if you adopt or don't adopt certain axioms (there's lots of fascinating stuff in mathematics like this, for instance the parallel postulate mentioned above or the axiom of choice). Different geometries with different basic sets of axioms all "exist" in some sense and apply in different circumstances. --Bth 19:52, 17 March 2006 (UTC)[reply]

I C, thnx! :).--Cosmic girl 22:09, 17 March 2006 (UTC)[reply]

By the way, it turns out that the parallel postulate is basically a way of saying that the geometric figures are being drawn in a flat plane. Those were the sort of figures that Euclid wanted to consider. One type of non-Euclidean geometry is spherical geometry, which concerns figures drawn on the surface of a sphere. Instead of three straight lines, for example, a triangle is formed from (arcs of) three great circles, for example. If Euclid considered such figures, he would have considered them as part of 3-dimensional space and relating to things outside the sphere. But in spherical geometry you can study the surface of a sphere as a self-contained 2-dimensional object. Some theorems of Euclidean geometry still apply (the ones you could prove without using the parallel postulate), while others do not. --Anonymous, 00:45 UTC, March 18, 2006.

Osama bin laden and the 9-11 attacks[edit]

By taking American lives during, early in the 2000-2004 term, a time when George Bush was still bogged down by clinton's mind boggling security failures and shortsights, that he was trying to promote a democrat to the office by making Bush look weak? Ironic isn't it that even with democrats going all out playing the national security card, it worked to the opposite goal, and the American people will probably never again be foolish enough to surrendor the country to a democratic anything, and Bin Laden's goal of a democratic presidency will have failed, permanetly forcing his party of choice, the democrats, out of all 3 branches of government?--Demos ocracy 02:01, 17 March 2006 (UTC)[reply]

Is this a rhetorical question? If not, the answer is "No, not really.". --Canley 02:22, 17 March 2006 (UTC)[reply]
Notwithstanding my general agreement with Canley's response and my general dislike of the use of the Reference Desk to advance arguments through questions, I would note that it is not settled that bin Laden would prefer a Democratic administration and surely not settled that his pronouncements in mid-2004 aided the Kerry campaign (see, e.g., Bush's recent concession that bin Laden's late October speech likely helped the Bush campaign).
Or maybe bin Laden really wants the Republicans to win and thought that by attacking the U.S., he would get people to think just like you and vote for the GOP! My guess is bin Laden doesn't really care who is the president of the U.S. as long as he's an "infidel." -- Mwalcoff 02:59, 17 March 2006 (UTC)[reply]
I remember back in the 1960s some New Left activists were hoping the Republicans (or whatever they considered right-wingers) would win so the world could see how terrible things were and then the Revolution would occur. I always thought that was an insane way to approach politics. Why not just Revolution Now? Or do it the regular way (democratically). Otherwise, sit down and shut up. --Halcatalyst 03:17, 17 March 2006 (UTC)[reply]
But I guess OBL doesn't have either option. So he's found a middle way: Pardon me if I object. I'm anti-authoritarian, so how could I like people who kill you if you don't agree with them? Or even if you do, randomly? --Halcatalyst 03:23, 17 March 2006 (UTC)[reply]
Or maybe he doesn't even want people to vote for the GOP, but is rather just trolling wikipedia in an attempt to make Republicans look bad? --BluePlatypus 04:27, 17 March 2006 (UTC)[reply]
Tout est possible. I don't pretend to understand how such minds work. But this would be an odd place to do it; do you suppose he's getting his jollies because we're having this conversation? --Halcatalyst 05:10, 17 March 2006 (UTC)[reply]
In the news today, the US was in a small minority voting against improvements to the United_Nations_Human_Rights_Council on the grounds, according to Ambassador John Bolton, that the move doesn't go far enough. He insists that the problem be solved completely or not at all. To me, this is the same kind of political rigidity as is exemplied by the New Left, and countless other ideologies I could name of all political stripes. --Halcatalyst 05:27, 17 March 2006 (UTC)[reply]
I think there's something to both theories:
1) Allowing the group opposite to your own to take power temporarily can, eventually, discredit them and put your group back in power. A good example would be communism. After the experiment in the Soviet Union and China, most people inside and outside those countries have been convinced that offering no economic incentive to work is ultimately bad for a country. Had those experiments never been done, we would still have a world full of revolutionaries wanting to try communism. A current example might be Muslim fundamentalism. Living in a country where all the laws are "made by God" might sound like a paradise until they see it in practice. In Afghanistan, women being beaten in the streets for what some unelected thug thinks is inappropriate clothing, and art, literature, music, and even kites being destroyed by religious nutjobs quickly convinced much of the population that this was not quite the paradise they had in mind.
2) Refusing to accept a compromise position and insisting on getting your full demands sometimes works, and sometimes fails. Accepting the compromise and then pressing for more later, also sometimes works and sometimes fails. StuRat 14:44, 17 March 2006 (UTC)[reply]
But for this to work, it would have to assume that Bin Laden knew that Bush would have won the 2000 election, which, as we recall, was not at all clear even after the election had finished. The 9-11 attacks took years of preparation, and the idea that the outcome of the national elections ten months before played any part in their execution seems very unlikely. --Fastfission 16:33, 17 March 2006 (UTC)[reply]
Also, to influence an election they should attack right before the election, as was done to influence the election in Spain. If they allow too much time, it allows for a counter-attack, like the Afghanistan invasion, which makes the current government look good. StuRat 18:06, 17 March 2006 (UTC)[reply]
While i have to agree that political agendas don't belong on wikipedia, I am inclined to offer what's hopefully an unbiased opinion. If you look at the current situation, and say that someone is getting what they want, it would be OBL and Al Qaieda, and not the US. Consider the effects of 9/11. Americans were spurred into international actions that compromised the country's image. Americans made sweeping changes to national policy costing billions of dollars a year in direct expenses, and possibly trillions of dollars over all. While it's impossible to say what the american democrats would have done, since hindsight is indeed 20/20, it would appear that the actions of the republicans in response to 9/11 have lead to an overall loss to America, and that translates into a gain for terrorist agendas. The argument presented as the topic of this thread is nothing more than partisan bickering, something that only serves to further divide Americans (on top of being poorly worded and logically fallacious). What Americans would do if they were clearly concerned with moving the entire country forward, would be to think critically about national and international actions, and demand accountability from elected officials, both democrat and republican. Until that sort of thing becomes more commonplace, both political sides will be able to push selfish agendas and noone (but them, arguably) will win. --Jmeden2000 06:32, 19 March 2006 (UTC)[reply]
I'm pretty sure the Democrats would have still overthrown the Taliban in Afghanistan, but not invaded Iraq. Then again, any Republican President other than Bush would likely have also invaded Afghanistan, and not Iraq. Bush had personal motives for wanting to get Saddam, as he tried to have Bush's father killed. StuRat 14:43, 19 March 2006 (UTC)[reply]
I don't quite buy that, for one thing it implies that he might care for a person other than himself, ie his father, two implies that he has a long term memory that reaches all the way back to when his father was the president--152.163.100.72 02:09, 20 March 2006 (UTC)[reply]

Politics[edit]

What is the difference between a premier (as in Gorbachev or Putin) and a prime minister (as in Blair)?

It's explained in the Premier article. Putin is the President though, and is referred to as such. --BluePlatypus 02:34, 17 March 2006 (UTC)[reply]


Question about Georges Briard[edit]

Is there any way to view a deleted page? I am CERTAIN that I read an article about Georges Briard a few days ago on Wikipedia and now it is gone. He was a designer of china about 50 years ago. All I can figure is that the article was deleted. Is there a way to view it? 72.79.9.197 02:36, 17 March 2006 (UTC)Anon[reply]

There was no article under that name, no. However, if you know the name of an article, you can ask about it at wp:dr. Be prepared to assert the importance and usefulness of the article in some dispassionate and verifiable way. Geogre 02:40, 17 March 2006 (UTC)[reply]

The link worked, but I don't see anywhere to ask/post the question. Help! 72.79.9.197 02:45, 17 March 2006 (UTC)Anon[reply]

I looked in the deletion log and didn't see anything for either Georges Briard or George Briard. Are you sure this is the same spelling? Remember that capitalization and diacritics can change where an article is. If the article wasn't deleted, it won't be useful to go to deletion review. Makemi 02:48, 17 March 2006 (UTC)[reply]

It is Georges Briard (with the "s" - just type it into a search engine and you'll get 1000's of hits). The Wikipedia article on him was very good in how it explained his real name and how he changed it to this when he went into design work etc. Emmigrated from France etc. A thorough little article (maybe 1500 words). Anyway would it help to know that now that it is Thursday, it may have been as many as 10 days ago that I read the article?? 72.79.9.197 02:53, 17 March 2006 (UTC)Anon[reply]

I don't think the deletion log is archived that frequently, so it would still be logged even if it was deleted 10 days ago. I see that he's an glass artist, could the information been under another article, such as one about his company (if he had one), studio, or artistic movement? Perhaps if you browse the categories Category:Glass and Category:Glass art. Makemi 03:05, 17 March 2006 (UTC)[reply]

Hmmm, I am certain the article was about him b/c when I entered the name, I had a choice of selecting an article about him or about a dog breed called "Briard." Georges Briard is perhaps still even alive according to http://www.sgcd.org/child_2004.html 72.79.9.197 03:14, 17 March 2006 (UTC)Anon[reply]

No sign of it. Maybe it was a site like Wikipedia. If your browser has a history feature, you could use it to see what pages you were visiting a few days ago. Notinasnaid 13:06, 17 March 2006 (UTC)[reply]

Literary Critcism[edit]

I am doing an essay about the Great Gatsby in my high school english class. The paper was going along fine until I started to look for sources of literary criticism. Most relevant links from google scholar are under protection by JSTOR. Can anyone help point me in the direction as to where I might be able to find sources of literary criticism of the Great Gatsby on the web? -Ridge Racer 04:12, 17 March 2006 (UTC)[reply]

You're going to have to use paper resources, if you want anything worth reading. There are sites that have literary criticism, some of it even moderately intelligent, but you would be well advised not to use them. Your teacher is going to prefer print criticism, and, to tell the truth, so are you. The barrier to publication with the web is low. That's good, in that it means a world of plenty, but it also means that there is a world of plenty of junk. Good with bad sit side by side. Unless you feel capable of discerning the expert from the blowhard, the professor from the fellow high school student, the unprotected literary criticism is going to be fairly worthless to you. Hie thee to the library. It's going to be better for you and your grades, and there will be no temptation to accidentally committ plagiarism. Geogre 10:31, 17 March 2006 (UTC)[reply]
By the way, if you want to read actual scholarly journals and books online and you're willing to pay a small subscription fee, investigate Questia.com. It's a very serious site and entirely academic (i.e. not one of these cheat sites). I assume you already know about the cheater sites, so we needn't mention those -- they're unfit for a bibliography. Public domain sources will be few to non-existent due to when the novel came out. You're stuck with print or a Questia online version of a print source. Geogre 10:36, 17 March 2006 (UTC)[reply]
Also, a number of libraries do have onsite access to JSTOR, so if you found some articles which look good to you on there, just take a note of the bibliographic info and then hie thee to the library. Makemi 20:16, 17 March 2006 (UTC)[reply]

Thanks for all of the suggestions, but I managed to grab some books that were on reserve in the school library for over the weekend. I gotta tell my teacher not to reccommend going online for the reasons listed above (which I learned the hard way). Thanks. -Ridge Racer 19:49, 19 March 2006 (UTC)[reply]

Lanai[edit]

The only information under "lanai" is about one of the Hawaiian islands. Would it also be appropriate to have a short reference to lanai as "a type of patio"?

Thanks. 04:18, 17 March 2006 (UTC)66.171.48.138

Yes. A search here (box appears in every page) shows :
  • An airport
  • An island
  • A city, each related ;
  • A thrush (kinda extinct bird)
  • A house style here, also shown in Disney's tiki room ... Be bold, else someone shall create the disamb page and all before you. --DLL 17:20, 17 March 2006 (UTC)[reply]
Done. I had a few minutes while here at work :-) See Lanai (disambiguation). Dismas|(talk) 21:11, 17 March 2006 (UTC)[reply]


Relationship between Steve Jobs, and Bill Gates.[edit]

Thank you for taking the time to review my question.

I have read the biography of Bill Gates, and the biography of Steve Jobs on Wikipedia. I have also read the History of Microsoft Windows. I have seen very few references to the stealing/copying of the first Operating System used by Windows. I remember it saying that Windows strangley resembled the OS released by apple, but there is a great talk about Microsoft stealing whole concept of the OS from apple. I also remember seeing that Apple had in fact saught legal action in this area, but I still have not found any definate answer as to whether Microsoft stole it all from Apple. Is this true? Did Microsoft steal its techknowlogy from Apple, thus propelling itself passed Apple in the coorperate world? Or are a ton of people seeing a coincidence and just acting on what they know, rather then what happened? It seemed to me that Bill Gates built the windows system around MS-Dos, which he had in fact purchased legally, just his OS that he compiled onto the hardware was just similar to Apples "Apple III" OS. Thank you for your time and consideration.

Nathan Sorensen (email removed)

Actually, they both stole it (the visual part) from Xerox, who largely invented the modern GUI for their Xerox Alto computer. But it all depends what you mean by "steal". Do Apple and Microsoft imitate each other's look, innovations, and features? Yes, constantly, and in both directions. Do they steal programs and code? Nobody seems to have said so. Do they steal other forms of intellectual property? This is open to debate, and there have been law suits on whether the idea of a "look and feel" can be protected, or whether good ideas can just be used to improve everyone's products. Notinasnaid 10:39, 17 March 2006 (UTC)[reply]
The reason that Windows looks a bit like Macintosh is because Apple CEO John Sculley made an agreement with Microsoft allowing them to use Macintosh's GUI elements in the first version of their OS. A loophole, however, allowed Microsoft to continue using these elements in any future versions of Windows. :( Hmm... Wikipedia says this about it:
Microsoft threatened to discontinue Microsoft Office for the Macintosh if Apple did not license parts of the Macintosh graphical user interface to Microsoft for use in the Windows operating system. Sculley agreed under the pressure, a decision which later affected the Apple v. Microsoft lawsuit.
Interesting... —OneofThem 17:22, 18 March 2006 (UTC)[reply]

David Bowie Lyrics - Ashes to Ashes[edit]

On the 1979 Number One hit 'Ashes to Ashes' there is barely audible chanting by Bowie behind the second verse and Middle Eight. What are these background vocals on David Bowie Ashes to Ashes saying?

Paul, Hammersmith please email to [ E-mail address removed for your inbox' sanity ] --86.137.88.214 11:04, 17 March 2006 (UTC)You stupid spam-monkey.[reply]

Everyone in the world is going to e-mail you (twice over it would appear),

kablam dan

He chants the main lyric. "I never did good things/ I never did bad things/ I never did any thing/ Out of the blue, oh no." On Scary Monsters he does that in other places, as well, where the second singer is echoing (sometimes in Japanese) the main lyric. Geogre 22:38, 17 March 2006 (UTC)[reply]

Reference to Dali in song by Bowie?[edit]

I've heard that there is a reference to a painting by Salvador Dali in the song Life on Mars? by David Bowie. Is this true, if yes, which painting?

Beacharn 19:56, 17 March 2006 (UTC)[reply]

Hm. not sure - the lyrics could refer to any of a dozen paintings by Dali... you're not thinking of Diamond Dogs, which mentions a girl wearing a Dali brooch, are you? Grutness...wha? 05:47, 18 March 2006 (UTC)[reply]

perisan,chinese and american unicorns culture differences?[edit]

I found alot about american unicorns but can't find any on perisan and chinese, can you please explain what i'm doing wrong. I'm sure all cultures have some kind of unicorns, i remember reading that somewhere but can't find any information. please help thank you. [removed e-mail]

I assume you're referring to the article Unicorn which mainly deals with European unicorns, but the section Unicorn#Unicorns in antiquity gives a little bit more of a multicultural view. There is also the article Qilin, about a mythical beast which could also be called a unicorn. The problem is that mythical animals tend to have a wide variety of names, and are not all necessarily linked historically or conceptually. Basically, just because a mythical beast in China has a horn, it won't necessarily be called a unicorn in English sources. see also Karkadann, ElasmotheriumMakemi 20:30, 17 March 2006 (UTC)[reply]

Confucius say: "The truly wise man never plays leapfrog with the unicorn." StuRat 00:56, 19 March 2006 (UTC)[reply]

March 18[edit]

discussion board[edit]

What's the difference between an online reference desk and a discussion borard board? Does it matter, here? --Halcatalyst 00:21, 18 March 2006 (UTC)[reply]

Is this a rhetorical question, Hal (if I may call you Hal)? —Keenan Pepper 01:09, 18 March 2006 (UTC)[reply]
  • You may call me Hal but not Hallie (makes me want my momma.)  ;-) Yes, it's a serious question. If it were a rhetorical question, I'd have paused for about a second and a half and then given you my answer. --Halcatalyst 05:59, 18 March 2006 (UTC)[reply]
Depends - can he call you Betty? Grutness...wha? 05:49, 18 March 2006 (UTC) (with apologies to Paul Simon)[reply]
Ah, in that case, you have just made it more of a message board by asking that question. My personal, unstudied and not-so-experienced-with-the-reference-desk-opinion is that the purpose of the reference desk is for people who actually have something to say say something. And hopefully, what they will say is either: here's where to start looking for the answer; or: do your homework; or: this is the answer, and here's where I found it. Makemi 06:05, 18 March 2006 (UTC)[reply]
Well said, Makemi. Also, the Reference Desk is more structured than a message board because it's divided into askers and answerers (at least for any given question), and because randomly changing subjects is frowned upon. And no, you can't call me Betty. (?) —Keenan Pepper 06:14, 18 March 2006 (UTC)[reply]
They are referring to the Paul Simon song "You Can Call Me Al" off the album Graceland, the text of which is here, note the similarity between the names Hal and Al. Makemi 06:20, 18 March 2006 (UTC)[reply]
As Keenan noted, the Reference Desk is for asking questions. A message board is for messages. That message may be a question. It may be a statement, like "TV sucks when there is basketball on half the channels." It could be for exposition, like "A freddled gruntbuggly spat on me and then lurdled off after a rather frudled bee." Such postings are not very welcome on the Reference desk because they are not questions. Also, few teenagers post homework questions in the message boards. --Kainaw (talk) 20:47, 18 March 2006 (UTC)[reply]
Actually, I was really referring to my 2001 filk (parody) of "You can call be Al", which, for those desperately bored, can be found here (damn difficult song to sing or parody, that one. Weird polyrhythms...). Grutness...wha? 01:33, 19 March 2006 (UTC)[reply]
  • Here's my view. The reference desk is for asking questions. Anybody can ask, including those who answer them. When that happens, it can legitimately be called a discussion board, and it legitimately is a discussion board as well as a reference desk. Also, there can be some horsing around among the regular "answerers." This is normal, something you see in the workplace, for example, maybe even among reference librarians. If it becomes nothing but kidding around, then it becomes more and more like your garden variety discussion board, of which there are as many around the Internet as you'd ever care to discover.
  • So, the question is, really, what do we want here? Well, I'm satisfied with what we've got. But I think we should be conscious (and maybe appreciative) of what we've got. By the way, there was a good reason I didn't pose this question on the reference desk discussion page: because very few contributors here participate there.
  • Thanks for your responses. I hope people keep responding for the seven days before the topic disappears into that great ghost town, the Archives. --Halcatalyst 04:56, 20 March 2006 (UTC)[reply]
  • For those of you who want a fancy name, this here's a meta discussion. --Halcatalyst 05:02, 20 March 2006 (UTC)[reply]

Weird cartoon[edit]

Could someone explain this cartoon to me? I get most of the PBF ones but this one just baffles me. —Keenan Pepper 06:10, 18 March 2006 (UTC)[reply]

I'm not sure of the whole context, but the joke seems to be that the girl is blind. Note that her eyes don't seem to have pupils. Makemi 06:14, 18 March 2006 (UTC)[reply]
Why is her reflection in the mirror weird, then? —Keenan Pepper 06:24, 18 March 2006 (UTC)[reply]
I think it's trying to point out how poorly done her makeup is (the goal of most peoples' makeup is not to have huge circles of blush and eyeshadow, although there is a certain aesthetic which values that). Makemi 06:29, 18 March 2006 (UTC)[reply]
I think that the girl is a vampire. Vampires do not cast shadows or have reflections. In the mirror all we can see is her clothing and her make-up. Pontificake 08:30, 18 March 2006 (UTC)[reply]
Doesn't seem that strange to me.. The girl's mother complains about her daughter's use of make-up. The girl complains that the other girls get to use it, thinking that she's opposed to the use of make-up in general. But it turns out that she's just against it because the daughter is so bad at it, thus a humorous situation arises. The humor is heightened by the realization that the daugher is apparently a vampire (and has no reflection in the mirror), so she literally can't see what she's doing. (explaining why she was so bad at it. --BluePlatypus 14:37, 18 March 2006 (UTC)[reply]
Ah. Vampire. Gotcha. —Keenan Pepper 17:01, 18 March 2006 (UTC)[reply]

yeah, the girl is obviously a vampire. you can tell by their stereotypical pointed clothing or by the mom's pointed canines in the upclose pic.-drtyhobo

  • Ze aczent iz also ze tell tale, sterotypical eastern european accent, as in romania, more specifically transalvania, dracula, and so forth--152.163.100.72 02:06, 20 March 2006 (UTC)[reply]
Actually, the stereotypical Dracula accent is a Hungarian one, mostly thanks to Béla Lugosi. Although Bram Stoker's Drac was supposedly a Szekler, but OTOH had a Romanian-sounding name. (Inspired either by the Drakulya family or the "Draculea" nickname of Vlad Tsepesh) --BluePlatypus 15:35, 20 March 2006 (UTC)[reply]

'critiques on surrealism' paragraph[edit]

hello, i am writing a dissertation on surreal film and was wondering whether you can give me the exact source of your 'critiques of Surrealism' paragraph. (feminists have in the past critiqued the surrealist movement for being a fundamentally male movement and a male fellowship.....)

i would really appreaciate if you could let me know as soon as possible what book or website this was taken from. thank you. i would need the original source and not the wikipedia reference.

If it's not sourced in the article (by means of a reference or something), then I'm afraid we can't help you. While we ask editors to cite their sources, not everyone does it all the time. You can try to leave a message on the article's talk page, or else search for the phrase in question on Google. Good luck! — QuantumEleven | (talk) 12:57, 18 March 2006 (UTC)[reply]
You can also look through the history of the article to find who wrote that section, then ask them directly. -LambaJan 21:20, 18 March 2006 (UTC)[reply]

Chinese and Japanese Kamasutra[edit]

There is that Indian Kamasutra. Apart from that, even the chinese and the japanese have similar books originating from their country. Can you name what is Chinese kamasutra called and what is Japanese kamasutra called?

The Japanese "Kamasutra" is called Shijuhatte: Shijuhatte Sexual Positions ≈ jossi ≈ t@ 04:14, 20 March 2006 (UTC)[reply]
  • [A system-proscribed spam URL has been removed here to restore the page's editability] --Jerzyt 06:42, 29 December 2007 (UTC) The Japanese Kamasutra] - 3d-illustrated guide to Shijuhatte (various sexual positions practiced in Japan)[reply]

priory of sion[edit]

Is priory of sion a real organisation operating since past protecting secrets about the holy grail and other secrets?

It's a fictional organization operating to make Dan Brown lots of money. Of course, some people believe they exist. Those people are called "conspiracy theorists". --BluePlatypus 18:28, 18 March 2006 (UTC)[reply]
Is "conspiracy theorist" a synonym for "whacko"? --Nelson Ricardo 18:47, 18 March 2006 (UTC)[reply]
That's just what "they" want you to think. :) --BluePlatypus 21:29, 18 March 2006 (UTC)[reply]
It's obvious you guys are a conspiracy. --Halcatalyst 23:24, 20 March 2006 (UTC)[reply]
One may, incidentally, consult our article and the attendant links. It seems fairly clear that Jimbo is actually the Grand Master. Joe 23:00, 18 March 2006 (UTC)[reply]

philosophy[edit]

is it better to be a hhappy pig or an unhappy human? —Preceding unsigned comment added by 69.235.209.13 (talkcontribs)

Depends on your philosophy. --Slumgum | yap | stalk | 21:10, 18 March 2006 (UTC)[reply]
It's a false dichotomy: I know plenty of happy pigs who walk on two feet. Geogre 21:28, 18 March 2006 (UTC)[reply]

The Ides of March[edit]

Last Wednesday, March 15, it occurred to me to wonder why this date (fatal to Julius Caesar in Shakespeare's play) was called the ides and whether the 15th of every month was so designated. I Googled and found out that only the months of March, May, July and October have the 15th so designated. All other months in the ancient Roman calendar have the ides on the 13th day. Can someone eplain this anomoly further? Thank you. ElizaK

The months with 31 days have the ides on the 15th. The months with 30 days have the ides on the 13th. --Kainaw (talk) 21:47, 18 March 2006 (UTC)[reply]
It's not that simple. The Ides were on the 15th in March, May, July, and October, which are all 31-day months. I think I remember reading that this was a holdover from the earlier form of the calendar, but I forget the details. --Anonymous, 01:50 UTC, March 19, 2006.
What is not that simple!? It cannot be any simpler. Roman months were either 31 or 30 days. I clearly explained what day the ides fell on for each month length. --Kainaw (talk) 02:51, 19 March 2006 (UTC)[reply]
Ah, I get it. First, I was writing in a hurry and meant to say "which are not the only 31-day months" or something like that. Point is, in the Julian calendar there were 7 months with 31 days, same as today, but the Ides were on the 15th only in the 4 months I named. Second, I was assuming we were talking about the Julian calendar because I think that's the one people usually mean when they speak of the Roman calendar, and also because the question explicitly referred to Julius Caesar. But Kainaw was actually talking about the "earlier form" of the calendar that I spoke of, which in fact is covered in the Wikipedia article, and the statement is correct for that. --Anon, 05:57 UTC, March 19.
The Wikipedia article also gives the dates of the Kalends and Nones for each month, in which one may also be interested. Joe 23:04, 18 March 2006 (UTC)[reply]

population[edit]

what city/borough/village has the lowest population in the US?

There are a few with zero, such as S.N.P.J., Pennsylvania, which has only part-time residents. -- Mwalcoff 00:18, 19 March 2006 (UTC)[reply]
For those with populations, Parker, Pennsylvania (pop. 799) calls itself "The Smallest City in the United States". However, Ismay, Montana only has 26 people. --Kainaw (talk) 00:35, 19 March 2006 (UTC)[reply]
An odd thing about the US is the way distinctions are made between "city" and "town." Parker is saying it's the smallest city, whatever that means. As far as I know, it means incorporated as a city and not as a town. Probably that depends on the laws of a particular state. All I know is that I live in a city of 3500 souls and my brother lives in a rapidly-growing town of 180,000. I'm sure such distinctions are also artifacts of historical development. He lives near Phoenix, Arizona. --Halcatalyst 04:40, 20 March 2006 (UTC)[reply]
"There are 948 cities in Iowa of which more than 870 belong to the League of Cities. Population sizes range from quite small (Beaconsfield, pop. 11) to quite large (Des Moines, pop. 198,652)." "Cities in Iowa." However, there are no localities formally called towns that I know of. --Halcatalyst 04:45, 20 March 2006 (UTC)[reply]
It does depend on the state. In Ohio, any municipality with 5,000 or more people at the latest decennial census is a city, and any municipality with fewer people is a village. I know of one city that went from village to city to village and back to city over three censuses. (They never bothered to change the stationery when the population dropped below 5,000.) -- Mwalcoff 05:30, 20 March 2006 (UTC)[reply]
The Cedars, California has a (year-round) population of 1. (5 in the summer.) {{subst:User"CliffHarris/sig}} 00:15, 25 March 2006 (UTC)

March 19[edit]

Toshitaka Nomi[edit]

Hello I'm New here. I was answered by Nutinasnaid, to which I say yes; I have been unable to contact the ABO-World web-site@the mentioned location which is why in the first place I am contacting you instead.. I am very interested in advancing my knowledge of the material complided by Toshitaka Nomi and his Blood Type Human Studies Institue. Can you shed any light I why I cannot contact the ABO-World web-site? Thank You for your help John C. Revi

It appears to work fine for me (although I can't read Japanese). Perhaps explain the problem in more detail?--inksT 02:36, 19 March 2006 (UTC)[reply]
The page I saw was in English: http://www.human-abo.org/english/message.html. I seem to recall there was no aforementioned location - you never told us the web site - so we may all be talking at cross purposes. John, can you confirm you have tried to e-mail, to telephone, to fax, and to write a letter, and all of these have been successful? Or are you saying you cannot even visit this we page? Please be clear and tell us what web page you mean, and how you have tried to contact. Notinasnaid 10:33, 20 March 2006 (UTC)[reply]

death punishment[edit]

i want to have some opinions and arguments against Death punishment. —Preceding unsigned comment added by 70.55.55.32 (talkcontribs)

You have to make your own opinion. Slumgum | yap | stalk | 02:04, 19 March 2006 (UTC)[reply]
Or, if you just want to copy other people's opinions, try reading up on capital punishment (no, it's not the penalty for using all uppercase letters, although perhaps it should be.). StuRat 02:38, 19 March 2006 (UTC)[reply]
You cannot merely consider arguments against the death penalty - you must also consider the death penalty itself to keep the question in balance and thus not arrive at an unbalanced conclusion. I suspect the greatest argument against lawful execution is that the sentencing and executing authorities can, and have, at times been demonstrably wrong, and have killed (executed) innocent people in the process. But conversely, those unfortunates who have been wrongly convicted, sentenced, and executed, hopefully in the earnest belief that they were in fact guilty as charged, and sentenced and executed according to law, can never offend again, if in fact they ever did. I recall a TV discussion involving a late British Lord Chancellor (Lord Hailsham aka Quintin Hogg) on this very subject in which he declared that it was far better to let 99 guilty murderers walk free than to erroneously hang (execute) one innocent man. But I also recall hearing a counter-argument that suggested it might be better in a civilised but endangered society to hang (execute) all one hundred, including the one innocent man, so as to make the community a safer place. The justification ran along the lines that the 99 guilty men would never offend again; and the one innocent man would meet his maker sooner rather than later, and would be rewarded in heaven/paradise/eternity for having sacrificed his life in contribution to the betterment of the society on earth he left behind.
You may be amused by the article "n guilty men", a study of how the "it is better to let n murderers free than kill one innocent man" theory has changed over time... Shimgray | talk | 21:26, 19 March 2006 (UTC)[reply]
If you say you want to have an opinion against the death penalty then it seems you already have one. :) DirkvdM 15:55, 20 March 2006 (UTC)[reply]

political question[edit]

What would happen if the United states pulled out from the middle east?

would there be an increase in terrorist activity or would the politcal situtaion change?--06:51, 19 March 2006 (UTC)

I'm sure that the simple act of removing an American presence from the middle east would count as "changing the political situation". Also, what is it exactly that the US is "pulling out"? Money? Troops? McDonalds?--inksT 09:25, 19 March 2006 (UTC)[reply]
My crystal ball says these will happen
  • Civil war in Iraq
  • A fundamentalist power will take over Iraq,
  • Iran will be nuclear-armed
  • Russia moving in to fill up some of the power vacuum
  • re-emergence of Al-Qaeda, more terrorists will be recruited, trained and deployed around the globe
  • Saudia government in danger of being overthrown. Other secular or western-support regimes are also in danger of being overthrown.
  • US diplomacy is at its lowest, because it has failed in its obligation as an occupying force.
  • Clash of civilisations.
  • Worst of all, oil prices will go up, 69.234.54.17 10:19, 19 March 2006 (UTC)[reply]
Seems to me like most of those will happen even if the U.S. doesn't pull out.--Fastfission 21:41, 19 March 2006 (UTC)[reply]
Make that have happened when they 'pulled in' (well, several, anyway). DirkvdM 16:01, 20 March 2006 (UTC)[reply]
Terrorism is in 95% of the cases aimed against foreign occupation. Same here. But the civil war will continue for some time, so local terrorism (if you wish to call it that) will also go on, but terrorism against the US will become les likely. DirkvdM 16:01, 20 March 2006 (UTC)[reply]

Ships 1750[edit]

i need information on ships in 1750. conditions onboard, punishment onboard etc. i aso need information of how they changed between 1750-1900. please help.

  • Are you referring to conditions onboard "Middle Passage" slave ships? If so, here is some quick information:
  • Low water rations, which lead to severe dehydration in slaves (symptons include hallucinations, anorexia, low metabolic rates, and extreme difficulty swallowing)

    "Dr. Joseph Cliffe, a physician in the Brazilian trade, claimed that when he was on shipboard, 'the want of water' was so great among the slaves that they were not brought on deck because 'if they were to see water alongside," thirst would drive "a great number of them to jump overboard without considering that it was salt water.' When questioned about the amount of water given the slaves, Cliffe stated that in one case water only in the amount of a 'teacup-full' was given every three days."

-Stanley L. Engerman and Joseph E. Inikori, The Atlantic Slave Trade: Effects on Economies, Societies, and Peoples in Africa, the Americas, and Europe. (Durham, NC: Duke University Press, 1992), 322.
  • Low food rations, which lead to severe malnutrition.
  • Slaves were huddled together in the dark lower decks of the ship. Diseases like dysentary, yellow fever, smallpox, and typhoid ran rampant in these cramped holds. Men were kept shackled at all times. Brutal punishment would come to any slave who challenged the masters' authority.
  • As a result of these horrid conditions, mortality rates during the Middle Passage from the mid-17th century up until the early 19th were always high for slaves and crew alike (who presumably caught the diseases carried by slaves and were only given rations with a slightly higher margin in quantity). Estimates range from 13 and 33 percent for any given journey. During the 19th century, increased "efficency" came as a result of better designed ships which could carry more fresh water. Engerman and Inikori again write:

    "Finally, there is the question of the declining middle passage mortality of slaves, from the first decades of the eighteenth century to the first part of the nineteenth century, from close to 25 percent to around 5 to 10 percent. This decline must have been the result, not just of the construction of specialized ships intended. for the trade, as has been suggested, but of the construction of specialized ships designed to carry more water than their predecessors and very likely to catch more water from rainfall as well."

-Stanley L. Engerman and Joseph E. Inikori, The Atlantic Slave Trade: Effects on Economies, Societies, and Peoples in Africa, the Americas, and Europe. (Durham, NC: Duke University Press, 1992), 330-331.

Additional links:

http://www.pbs.org/wgbh/aia/part1/1narr4.html

http://www.melfisher.org/henriettamarie/middlepassage.htm

Also see the film Amistad for graphic depictions of the conditions onboard slave ships.

Compliments of User:Jrousso

Detailed answer, but not to the user's question. He asked about changes between 1750 and 1900, which certainly excludes slave ships. I assume he meant conditions for sailors, probably not conditions for passengers. He also did not specify whether he meant naval vessels or civilian. A lot of things changed between 1750 and 1900. Some of the things included increased size of ships, change from sail to steam with consequent reduction in size of crew needed for a ship of particular size, abolition of frequent corporal punishment and press gangs for sailors in most navies, shorter voyages because of steam with less isolation from the rest of the world because of telegraph access at various ports, elimination of scurvy, increased dietary variety and quality for several reasons, including refrigeration... Perhaps the inquirer might narrow the question a bit if he wants more? alteripse 12:11, 20 March 2006 (UTC)[reply]

I ignore people who shout, so I didn't read your question. DirkvdM 16:02, 20 March 2006 (UTC)[reply]

Professor Victor Gold[edit]

Can anyone please tell me which organisation brought Victor Gold, then aged 16 (who was later Professor of Chemistry at King's College London University) over to England from Austria in 1938. Thank you.

Born in Vienna, Austria, Victor Gold was educated at the Vienna ‘Realgymnasium’ until the age of 15 when, following the Anschluss, he was sent to the UK by his family.[19] ≈ jossi ≈ t@ 04:12, 20 March 2006 (UTC)[reply]

Thank you. But did his family sent him to England privately or through some ORGANISATION such as the Kindertransport?

Village Pump[edit]

While Wikipedia has a community forum called Village Pump, I can't seem to find the article for Village Pump. Am I just not finding it, or does the article ironically not exist (yet)? Cheers --TG

Moved your question to the right date, as it should be. — QuantumEleven | (talk) 13:52, 19 March 2006 (UTC) [reply]
The Village Pump is located here. Technically, articles are only located in the main namespace, so it's not an article, more of a discussion page (actually, it's six discussion pages....) Oskar 14:01, 19 March 2006 (UTC)[reply]
If you are looking for a discussion of the village pump (an article on it rather than the space itself), you shouldn't find one. The reason is one of the guidelines: no self-reference. Wikipedia articles are supposed to avoid referring to Wikipedia itself. There is an article called Wikipedia, and it's relatively uncontroversial, but other self-reference violators (Criticisms of Wikipedia and History of Wikipedia) get more heat. Other than those, there should be no articles on ourselves, or at least very few. Geogre 16:10, 19 March 2006 (UTC)[reply]
It's acceptable to have articles on Wikimedia (or parts thereof), when those parts are notable. That isn't a violation of Wikipedia:avoid self-reference. Superm401 - Talk 22:40, 19 March 2006 (UTC)[reply]

Perhaps the questioner is referring to village pumps in general, rather than Wikipedia's Village Pump. In which case, village pump (with no capital letters) would be the place to look. Grutness...wha? 07:04, 20 March 2006 (UTC)[reply]

correction. I would have thought that was the place to look, but that seems to redirect to WP:VP. We do have the article pump, though, which seems to be largely about rurlar water pumps. Grutness...wha? 07:05, 20 March 2006 (UTC)[reply]

Historical Speech on Education. "All education is to the glory of God!"[edit]

During my school days a teacher once showed me a speech or article a minister wrote. It was once a public exam question. In the article, the minister talked of education and its privatisation and the resulting question of the use, beneficiaries and goal of education. The last sentence of the work was approximately "... All education is to the glory of God!" The minister in question was Irish or Scottish and the work was published in the 1920s. Please bear in mind that my recollection is approximate and I am not sure of anything (perpetually), except the last sentence of the speech. Thank you in advance!

The Jesuits speak of Ad Majorem Dei Gloriam "to the greater glory of God" quoting St. Ignatius Loyola. Maybe this can help. ≈ jossi ≈ t@ 04:09, 20 March 2006 (UTC)[reply]

--Thanks. Will look into it and report. Don420 16:36, 24 March 2006 (UTC)[reply]

explanation of cryptic crossword clue[edit]

Hi, I'm a novice cryptic crossword fan, and I came across the following clue in the Guardian Weekly. It was no. 4 down and the compiler was Janus: "Nothing cat-man can do for this marsupial". The answer was OPOSSUM. Can anyone explain how this is obtained? The Mad Echidna 17:00, 19 March 2006 (UTC)[reply]

Here's my theory. The leading O can be taken as a zero, therefore "nothing." T. S. Eliot wrote Old Possum's Book of Practical Cats, so Possum would be the "cat-man." The "can do for" part is assigning equivalence, thus "nothing" + "cat-man" = O + POSSUM = OPOSSUM. And of course, an opossum is a marsupial. --LarryMac 17:26, 19 March 2006 (UTC)[reply]
If I saw a clue like that I would just roll up and play dead. StuRat 17:57, 19 March 2006 (UTC)[reply]
Heh. I love doing cryptics, but my wife has convinced me not to read her the clues and answers; she says they make her head explode. --jpgordon∇∆∇∆ 18:00, 19 March 2006 (UTC)[reply]
For the record, I never would have gotten that if I'd been the one doing the puzzle. Having both the answer and the clue to work from made it a little easier to work out. --LarryMac 18:15, 19 March 2006 (UTC)[reply]
Would I be Janus, I would roll upside down in my sepulchre (still keeping one face up and one face down) with this clue. I ain't got any better, however. --DLL 21:50, 19 March 2006 (UTC)[reply]
Just as an aside: I think one would be quite likely to get this clue, without understanding the really obscure part (the cat man reference). You know it's 7 letters, to start with. Almost all cryptics split into a literal and cryptic part; the most likely literal part is either "nothing" or "marsupial". Well, nothing could be things like zero, null, nill, nihil, but I can't think of a seven letter nothing. So let's try marsupial: a seven letter marsupial. I could think of wallaby, kangaroo...no good. I was hoping for a simple alphabetical list, but List_of_Australian_marsupials is arranged taxonomically. Still, the word opossum leaps out and is seven letters so worth looking at. At this point, I'd see the "o" could mean nothing and decide that was good enough and fill it in. (Footnote: I see wallaby actually does have 7 letters. If I'd have noticed, I would have played with it for a while, but decided I couldn't fit it in).Notinasnaid 09:55, 20 March 2006 (UTC)[reply]
As another aside, the Guardian also had a clue a couple of weeks ago: "Primal Australian creature". The answer was "marsupial", because it is an anagram of "primal Aus". Some of the logic behind these cryptic clues is far, far more convoluted than the above. --Shantavira 13:46, 20 March 2006 (UTC)[reply]
You also get an extra hint in the fact that marsupials could be said to be primative mammals found mainly in Australia. Primal Aus gives you the letters, the clue as a whole gives you a.. er.. clue.

Plans for post-war Europe[edit]

Does Wikipedia have a page for Nazi Germany's plans for post-war Europe? Have searched, have not found. The Generalplan Ost page is pretty sparse. Thanks! Jørgen 20:04, 19 March 2006 (UTC)[reply]

The Nazi's didn't want there to be a post-war, as far as I can tell. After Europe, it was on to the world, and that kind of makes things take a while...Superm401 - Talk 22:41, 19 March 2006 (UTC)[reply]
That'd probably depend on the outcome of the war, Hitler being a megalomaniac, the more, the better. That doesn't mean he didn't have certain plans and priorities. He wanted to unify all 'Germanic' people under his rule. People like the Norwegians would be allowed to keep their own languages and cultures, but German would be their first language. The others wouldn't be treated that way. Following the doctrine of Lebensraum, the East (in particular Russia) would be used to give the Germanic peoples room to spread out and colonize. The indiginous and slavic peoples would be reduced to slaves or serfs, and afforded little education. (I believe Hitler said something to the extent of "All they need is enough German to read a road-sign, and the knowledge that the Germans are their masters", I think it was in Sereny's excellent "Albert Speer: His Battle with Truth") The capital of all this would be "Welthauptstadt Germania", which Speer actually made plans for. --BluePlatypus 18:11, 20 March 2006 (UTC)[reply]
Thanks for your answers. I've read Fatherland, which gives a striking description of the new capital. As far as I understand, there is no page for this topic on Wikipedia, and from what I've (failed to) discover on the Web, there is not enough info around to create one. After all, though these people liked thinking great, they were wildly insane, and plans may not have been entirely consistent... Jørgen 18:48, 20 March 2006 (UTC)[reply]
Also compare the Nazi plans for post-war Europe with the Allies' plans, which weren't made until the Yalta Conference, once victory was just a matter of time, and the Potsdam Conference, after the war had ended. The Nazis also were waiting until it became apparent that their victory was guaranteed, before finalizing their plans, but that time never came. StuRat 19:40, 20 March 2006 (UTC)[reply]
This does not directly address your question, but any good discussion of this issue ought to consider Philip K. Dick's novel The Man in the High Castle which is set in a world where Nazi Germany & Japan won WWII. Dick's visualization won him a Hugo award.-- llywrch 02:50, 23 March 2006 (UTC)[reply]

claudia muzio[edit]

My mother informed me years ago she named me after Claudia Muzio. I recently acquired three photographs of Claudia Muzio in costume, 8x10 addressed to my aunt and my godfather, both of whom were opera impressarios, and my mother was an opera coach for many years with the SF Opera Company. I was born in 1932 and my mother claimed she missed opening night at the new opera house.

I'm asking if these photographs are of significant important to any society and/or monetary value. They are originals and are dated 1933.

I'm no expert in opera collectibles, but a quick eBay search turned up one picture of Muzio [20] which was selling for US$4.25 with two bids. Not wildly popular by any means, but there are at least some people interested in it. I suggest you take ask at the local opera house (if there is one) who deals in opera collectibles and talk to them. --George 03:32, 20 March 2006 (UTC)[reply]

Study on Experts[edit]

I saw in the news a few weeks ago someone had done a long-term study of how accurate were the forecasts of experts, asking a number of them questions and tracking over a number of years whether their predictions came to pass. Does anyone have a citation to this study?

If you can provide a link to a related news article (and more importantly, the name of the corresponding author), I could probably track it down quite easily. It's always bothered me how news stories on published research rarely provide a reference to the publication. --inksT 21:30, 19 March 2006 (UTC)[reply]

I'm not aware of any study that crosses all fields, but am aware of a study where stockbroker's picks were compared with a monkey's stock picks, and the monky did better. In general, an expert's predictions could be quite accurate or highly inaccurate, depending on how specific the question is. If a meteorologist is asked to predict if there will be any hurricanes next year anywhere, I would expect accurate results, if asked to predict exactly where each hurricane will make landfall, a year in advance, I would expect pathetic accuracy. StuRat 14:58, 20 March 2006 (UTC)[reply]

StuRat makes a good point. There are also lots of articles that look at this idea in various contexts. I've seen a few about economists' projections of interest rates. Beyond a short period of time the predictions were more wrong than right, and the predictions couldn't even tell more than 50% of the time even the direction of changes, much less specific numbers. Of course everyone knows predictions of this type are very difficult, so it's just meant to be illustrative. - Taxman Talk 17:44, 20 March 2006 (UTC)[reply]

I think you're looking for Expert Political Judgment: How Good is It? How Can We Know? by Philip Tetlock. Here's the publisher's site. From the aforementioned site: "Tetlock first discusses arguments about whether the world is too complex for people to find the tools to understand political phenomena, let alone predict the future. He evaluates predictions from experts in different fields, comparing them to predictions by well-informed laity or those based on simple extrapolation from current trends." —Seqsea (talk) 05:28, 21 March 2006 (UTC)[reply]

March 20[edit]

Parliamentary sovereignty in Australia.[edit]

Hi, I am a first time user of this space, so I am not sure I am in the right place.

My question is: Can it be said that Parliamentary sovereignty exists in Australia? Is Parliament the ultimate law making body?

Thankyou. Northcoastbaby.

The Commonwealth of Australia is a constitutional monarchy. ≈ jossi ≈ t@ 04:06, 20 March 2006 (UTC)[reply]

For the student, what you will have to find out for yourself is if actions of the lower house must be ratified by the English House of Lords or by the Queen of England. In Canada, which shares the Commonwealth root, the right to unilaterally make its own legislation was approved for the House of Commons in the early 80s. Prior to that, everything had to be sent up to the House of Lords--meaning that things like the national budget had had to be approved by the mother country. In Australia's case it may be different. Also, can the Australian parliament negotiate its own treaties, internally & externally? Or can they only be resolved by the head of state (which in Canada, at least, is the Queen). Make sure you clearly understand who is the head of Government and who is the head of State and you will get an A. --24.80.70.174 22:06, 20 March 2006 (UTC)JK[reply]

That may be a little misleading. Actions of the Australian lower house (the House of Representatives) do not have to be "ratified" by anybody. The Australian Parliament as a whole makes laws. The three elements of the parliament are: the House of Representatives, the Senate, and the Queen of Australia. The House and the Senate agree on changes to the law, and the Governor-General representing the Queen of Australia gives Royal Assent. Australian law has nothing whatever to do with either the House of Lords or the Queen of "England" (and not even with the Queen of the United Kingdom). JackofOz 06:09, 21 March 2006 (UTC)[reply]
See also Statute of Westminster 1931, Statute of Westminster Adoption Act 1942 and Australia Act 1986. --Canley 01:58, 21 March 2006 (UTC)[reply]

Lifehouse's song "You and Me"[edit]

I was watching an episode of Cold Case with my mom. At the end of the episode, "You and Me" by Lifehouse was played. My mom says that at the end of every episode, a song that was released and/or made during the time that the mystery took place is played. The mystery for that episode took place some time during the 1990s! I recently searched various Lifehouse Websites and I couldn't get any information about when the song was made. The Wikipedia says that the song was produced in 2002. When was it made?

I am eminently confident that the Wikipedia entry is correct in noting that the song was produced in '02 and released as a single in '05. Perhaps this website, which purports to enumerate the musical selections for each episode, may be of assistance to you in ascertaining whether, in fact, concomitant to the end of each episode is music contemporary to the resolved crime (although I should note that the site doesn't seem to list the song). Joe 04:14, 20 March 2006 (UTC)[reply]
In my experience watching that show, the songs used during the "flashback" portions are contemporary to the timeframe. At the end, however, they are usually showing something happening "currently" so the song could be from any time period. Last night, they played Depeche Mode's Precious during one scene in the middle of the show, but it was during a "present-day" scene, even though they were investigating a 1980 cold case. --LarryMac 14:42, 20 March 2006 (UTC)[reply]

lost city[edit]

I am trying to find the name of a city that no longer exists that I saw pictures of recently. I believe the city was somewhere in asia and had apartments seemingly stacked very high on eachother loosely like shoeboxes. I'd like to know more about the city but can't find information about it anywhere. I'm pretty sure the name started with a C and was something like Carrageena (?) Thank you for any information you can provide, Charlotte

Have you checked at Lost city? Are you thinking of the Forbidden City, by chance? The Jade Knight 04:00, 20 March 2006 (UTC)[reply]
Is it Angkor Wat, Cambodia? --Vsion 04:37, 20 March 2006 (UTC)[reply]
Carthage? In Spanish, the name "Carthage" became Cartagena, which is still there. -- Mwalcoff 05:26, 20 March 2006 (UTC)[reply]
Çatalhöyük (also spelled Catal Huyuk and other various ways) was one of the world's first cities. It was in whats now Turkey and had stacked apartments. See this site for an artists image of the buildings. You can also search Google image for "Catal Huyuk" and find others.--Bkwillwm 05:42, 20 March 2006 (UTC)[reply]

rights[edit]

what right are you given by law in a civil case that protects you personal information
Tell in which countrySuraj vas 09:51, 20 March 2006 (UTC)[reply]

jugalbandhi[edit]

I need definition of the work jugalbandhi. email removed

It's not in the OED. Do you know what language it is? The Jade Knight 07:25, 20 March 2006 (UTC)[reply]

According to the article about Mangalampalli Balamuralikrishna, it's a kind of jam session between two musicians in Indian music. --Canley 07:31, 20 March 2006 (UTC)[reply]

I am sure its in Hindi ciz i am an Indian.Suraj vas 09:49, 20 March 2006 (UTC)[reply]

Current economic sanctions[edit]

is there a list of the current economic trade sanctions currently enforced by the UN and US?--orphan frequently 06:16, 20 March 2006 (UTC)[reply]

US trade sanctions
Here are a current list of US trade disputes under section 301. This is by no means a comprehensive list of US sanctions, let alone UN ones, but it's a start. --Rwiedower 18:49, 20 March 2006 (UTC)[reply]

history of production.[edit]

Hello Could someone help me with the proper words for the modes of production, manually production, manufacturing and mere facturing.

Thanks Christian Blom

Don't know the answer, but thanks for expanding my vocabulary with the word "facture". JackofOz 12:09, 20 March 2006 (UTC)[reply]
I see no need to be rude, which that could easily be construed as. - Taxman Talk 17:39, 20 March 2006 (UTC)[reply]
Well, I am curious as to what the asker meant by that word, aren't you ? StuRat 18:50, 20 March 2006 (UTC)[reply]
Being accused of rudeness does not sit well with me at all, Taxman. I'll happily apologise if I was unintentionally rude, but I'm struggling to see how what I wrote could be construed that way. If my thanks were obviously meant to deny the existence of the word facture, then you would have a case for accusing me of sarcasm. But such an intention was not there, and cannot reasonably be read into it. For the record, I had never heard of facture before, so I checked it out and discovered it's well established. Thanking the questioner was my natural and very courteous way of expressing my delight in learning a new word. JackofOz 14:24, 21 March 2006 (UTC)[reply]
Well it would be a little rude if like me you erroneously thought that it wasn't a word. Then you would just be mocking the asker. I didn't infer from your answer that you had actually gone and looked it up, and I didn't. It seems that's twice in a couple days I've been sharper than I intended, so I'll try to improve that. Sorry. - Taxman Talk 20:34, 22 March 2006 (UTC)[reply]
No hard feelings. JackofOz 01:54, 23 March 2006 (UTC)[reply]

Following the links in Manufacturing and Production should give you more than enough to get started. Respond to this again if you have specific questions after reading the relevant articles. - Taxman Talk 17:39, 20 March 2006 (UTC)[reply]

Noun: facture fakchu(r) The manner of making an artistic work. For those who would like to know. This sounds like a French word to this Canadian. Actually this question might find a better response on the Languages page, eh?

(random trivia) It means "receipt" in French. — QuantumEleven | (talk) 07:46, 22 March 2006 (UTC)[reply]

history world war1 and2[edit]

I have a closed head injury and have proublems understanding some readings so looking for it in terms i nay understand. What were the causes effects of world war 1 and 2. Please list these seperate from each other. and any usefule sites that i may check out would be great as well. Thank you for yor help

This is a very big subject. Millions of pages have been written on these subjects.Also, historians have spend years arguing about these points, though some things are agreed by most people. So, to keep it simple, I recommend starting with just one of these questions: causes of World War I. Please read World War I#Causes (scroll down to section 2, Causes, if you need to), and if there is anything in there that you don't understand, please let us know. Once you are happy with that we can move on. Notinasnaid 13:24, 20 March 2006 (UTC)[reply]
If you're having trouble reading this Wikipedia, you may also want to try the Simple English Wikipedia. They have a decent, albeit simplistic, overview of the beginning of the war at simple:World War I. Superm401 - Talk 15:00, 20 March 2006 (UTC)[reply]

Nahum Gilboa, Isreal artist.[edit]

I am seeking information on how to find his oil paintings, their sales price, if he is still alive and painting, and is their a Web site for him? Any of the information would be appreciated.

I found an art gallery that seems to show his work. They offer a biography, but it doesn't make clear his current status. I suggest you ask the gallery. Superm401 - Talk 15:04, 20 March 2006 (UTC)[reply]
Most sites list his birth date (1917) but no death date, so I suspect he's still alive. Grutness...wha? 06:39, 21 March 2006 (UTC)[reply]

mediocrity[edit]

Does our public educational system promote mediocrity?

Yes, it encourages people to ask questions on online reference desks, which is a mediocre research technique at best. Please do your own homework (see notice at top of this page). Superm401 - Talk 14:55, 20 March 2006 (UTC)[reply]
  • I guess you're saying all of us here who answer questions are mediocrities. Or that people shouldn't consult encylopedias for answers. --Halcatalyst 18:00, 20 March 2006 (UTC)[reply]
And be specific (ie, which country) and sign your post. Any more rules he's broken? DirkvdM 16:09, 20 March 2006 (UTC)[reply]
He (assuming it's a he) didn't sign his post. :) — QuantumEleven | (talk) 16:54, 20 March 2006 (UTC)[reply]
  • To the questioner. Your question is seeking an opinionated answer. We don't provide those here on Wikipedia. At least, that's not our function. --Halcatalyst 18:00, 20 March 2006 (UTC)[reply]

Also at what level of education? If you are in search of a specific topic to write about, consider the spelling bee. This is a tortuous 'educational' device designed to weed out mediocrity, while simultaneously breaking down any notion of cooperation within the group. Plenty to consider there.

National Policies[edit]

What is a National Policy?

It is the policy of a nation. --Kainaw (talk) 16:27, 20 March 2006 (UTC)[reply]
It can also be a policy that applies nation-wide, eg. in an organisation that has offices or activities in many parts of the nation. JackofOz 04:37, 21 March 2006 (UTC)[reply]

Accounting[edit]

is it impossible to satisfy the needs of different users with a single set of published financial statements?

Yes. And no. — QuantumEleven | (talk) 15:27, 20 March 2006 (UTC)[reply]

This sounds like a third year business question. Financial statements could be a good start, also the GATT page.

Yes. Everyone will require different figures to suit their own particular needs and tastes. And as everyone knows, there's no accounting for tastes. Grutness...wha? 06:42, 21 March 2006 (UTC)[reply]
It depends of countries ; the mandatory report to the tax administration is often enough for other users. When it comes to floating or mergers, the details required by the stock market authorities are far more heavy : the same goes when you want to trade in foreign countries. Please narrow your homework ? question ... --DLL 18:00, 21 March 2006 (UTC)[reply]

South Dakota abortion ban[edit]

I have a question about the recent ban of abortions in South Dakota, which was clearly a direct attack on Roe v. Wade. My question is, since it is so abundantly clear that it is unconstitutional under Roe, does this case even have a chance of getting to the Supreme Court? I mean, wont it be struck down in a much lower federal court, and then simply be denied appeal? Don't you need some minute detail that the Supreme Court have to "test" whether it's covered under Roe, and then they might strike it down? IA obviously NAL, but this is how I have understood it.

Also, isn't there a majority on the court to uphold Roe, that is isn't Kennedy, Souter, Ginsburg, Breyer and Stevens all for it? Oskar 16:30, 20 March 2006 (UTC)[reply]

We'll just have to wait and see what happens. Previous Supreme Court decisions have overturned before, as with Plessey v. Ferguson (1896). In other words, what is "constitutional" is continually a matter of dispute. During his confirmation hearings, John Roberts declared the right to an abortion was "settled law." This is not at all the same thing as saying the 1973 Roe v. Wade decision was constitutional. --Halcatalyst 17:46, 20 March 2006 (UTC)[reply]
Yes, I do understand this point, that Roe v. Wade might get overturned, that's not what I mean. What I mean is that, for lower courts, don't they have to follow the precedents that the Supreme Court sets, and if they do, this law will almost certainly be struck down. And if it is, how can they get an appeal when the law is so blatantly in violation of a supreme court precedent? Oskar 17:54, 20 March 2006 (UTC)[reply]
Only catching bits of the S.D. law on the news, it appears that it requires a minor's parents to be notified before a minor has an abortion. If that is correct, it is not a "direct attack" on Roe v. Wade. Roe v. Wade stated that an abortion is a matter of privacy and that the public didn't have to be notified (such as a "women seeking abortion" section of the newspaper). It is not uncommon to take issues that are allowed for adults and restrict them for minors. --Kainaw (talk) 18:15, 20 March 2006 (UTC)[reply]
I don't think that's correct. I believe the law bans abortion, with an exception for the life of the mother, but without exceptions for incest and rape. StuRat 18:44, 20 March 2006 (UTC)[reply]
You are correct. Utah's new law is the one I was describing above. --Kainaw (talk) 19:05, 20 March 2006 (UTC)[reply]

If another court struck down the law, South Dakota or Utah could appeal to the Supreme Court or petition for a writ of certiorai (different ways of getting the case to the Court; which one would be correct depends on some technicalities). The attorney seeking Supreme Court review would be representing that the arguments "are warranted by existing law or by a nonfrivolous argument for the extension, modification, or reversal of existing law or the establishment of new law . . . ." (Rule 11 of the Federal Rules of Civil Procedure). That test would be met if the brief said, "This Court's decision in Roe v. Wade should be overturned." (I support reproductive rights, but I don't think that position would be frivolous.) Depending on the specifics of the case, the appeal might instead take a less sweeping approach, calling only for a modification of Roe. The Supreme Court could hear the case if it wanted to. JamesMLane t c 22:17, 20 March 2006 (UTC)[reply]

Thank you, this was precisely what I was wondering. Oskar 22:43, 20 March 2006 (UTC)[reply]

I don't know what its called...[edit]

I know that every us citzen can legally marry one couple in their life time but whats the term called?

"marry a couple"?
Actually, the initial assumption is flawed. Civil marriages must be officiated by an agent of the state (most commonly a judge or an ordained minister). While I'm unable to find the specific definition of the appropriate agent (it probably varies by state anyway, as that's not a matter of federal legislation), an agent has no upper bound on the number of marriages he or she can perform. In short, a given US citizen can legally marry either (a) no one or (b) any number of couples. — Lomn Talk 18:29, 20 March 2006 (UTC)[reply]
one couple?
Your syntax would lean against this, but are you referring to the concept of monogamy as opposed to polygamy? --Rwiedower 18:41, 20 March 2006 (UTC)[reply]
Not just monogamy. The question added "in their life time". So, even with a divorce, you cannot remarry. --Kainaw (talk) 19:09, 20 March 2006 (UTC)[reply]
That would be a life sentence without parole....<g> - Nunh-huh 23:45, 20 March 2006 (UTC)[reply]
That still qualifies as monogamy, doesn't it? The prohibition on remarrying following a divorce is based on the view that marriage is divinely ordained and cannot be dissolved by human intervention. Divorce does not change this. JackofOz 04:28, 21 March 2006 (UTC)[reply]
I was trying to clarify the question. Reading it yet again, I see that it has nothing at all to do with monogamy. Monogamy means that you have one sexual partner. The question states that you may legally marry once in a lifetime. It says nothing about sexual partners. This is closely related to the Catholic belief that you may marry once and only once (but you can have sex with as many people as you like as long as you go to confession). OK - stop bashing religions. You know they are very sensitive. --Kainaw (talk) 14:01, 21 March 2006 (UTC)[reply]
I'll chime in once more: my answer above is based on a reading (and an urban legend I've heard) that a person is legally empowered to perform one marriage (as opposed to being the subject of one marriage). — Lomn Talk 17:17, 21 March 2006 (UTC)[reply]
Searched the net for "valid marriage" : it seems like you may get married thus : "The Society of Friends or Quakers is excepted from the requirement (of a license to marry) in some of the states, and in other the parties may have recourse to the publication of banns instead of securing a licenses". [21]". I do not see a case where you would need some average citizen instead of a civil officer. --DLL 17:54, 21 March 2006 (UTC)[reply]

Etheum?[edit]

Frm a "Life of Saint Judith":

"They have overthrown Cananeum the king, Jebusee, Pheresee, Eneum, Etheum and Amoreum, and all the mighty men in Esebon"

I can gather Cananeus = Canaan, Jebusee = Jebusites, Amoreum = Amorites, but what about "Pheresite, Enite and Ethite", and "Esebon"? Any idea? dab () 17:59, 20 March 2006 (UTC)[reply]

No idea about the other ones, but google gives Esebon = Cheif city of the Ammonites Oskar 18:10, 20 March 2006 (UTC)[reply]
That's the people of Ammon, not the mollusc :P Oskar 18:12, 20 March 2006 (UTC)[reply]
AKA Heshbon, I believe. --BluePlatypus 22:44, 20 March 2006 (UTC)[reply]
Ohh, ohh, and maybe Pheresees are the Pharisees. This is fun, it's like biblical Clue :D Oskar 18:14, 20 March 2006 (UTC)[reply]
Could Etheum refer to Ethiopia ? StuRat 18:34, 20 March 2006 (UTC)[reply]
Go to the everlasting source, (this one curioulsy kept by catholics), not church commentaries. Judith 5:20 where Ahior tells to Holophernes : "So they overthrew the king of the Chanaanites, and of the Jebusites, and of the Pherezites, and of the Hethites, and of the Hevites, and of the Amorrhites, and all the mighty ones in Hesebon, and they possessed their lands, and their cities." [22]
It looks like Exodus 3:7 : "And I have promised to bring you up out of your misery in Egypt into the land of the Canaanites, Hittites, Amorites, Perizzites, not pharisees, Hivites and Jebusites—a land flowing with milk and honey." [23]. Then what do we miss ? --DLL 22:25, 20 March 2006 (UTC)[reply]
More likely the Hittites, "Hettheum" in the Vulgate. "Pheresee" is probably Perizzites given the context, "Ferezeum" in Latin. As for "Eneum" it's a typo, should most likely be "Eveum", the Hevites. All these guys are mentioned together in Exodus 23:23,
For mine Angel shall go before thee, and bring thee in unto the Amorites, and the Hittites, and the Perizzites, and the Canaanites, and the Hivites, and the Jebusites: and I will cut them off.
--BluePlatypus 22:44, 20 March 2006 (UTC)[reply]
Oh, dang, DLL beat me to it. Oh well, but I guess two references beats one. --BluePlatypus 22:47, 20 March 2006 (UTC)[reply]

thank you all! so we have Hittites, Perizzites and Hevites, with Esebon the only article we might be missing. and Esebon = Heshbon; case solved. dab () 22:37, 21 March 2006 (UTC)[reply]

Government Language Programs[edit]

Assuming they do not already speak the language, how do employees of government agencies (such as the State Department)learn the necessary languages? I realize that most of these employees are probably hired already speaking the foreign language required for the position, but what about a current employee who is transferred and most speak a new language? Any special programs, etc? Thanks, Ryan

When it is your job to learn a language and you are sent to classes all day and you are not allowed to converse in any language other than the language you are learning, it isn't hard. When in Beijing, I surprised myself by pressing elevator buttons with numbers in Chinese characters without counting up to floor 17. --Kainaw (talk) 20:55, 20 March 2006 (UTC)[reply]
See the Foreign Service Institute homepage. -- Mwalcoff 01:11, 22 March 2006 (UTC)[reply]

Timeframe of Draughts and Checkers[edit]

Hello,

In the New England Colonies of 1600s and early 1700s would our game of Checkers still have been called Draughts? Or Checkers?

Thanks, Mona

[email removed]

Probably draughts...my quick google search came up with these sites http://www.acfcheckers.com/origin.html and http://www.indepthinfo.com/checkers/history.shtml. If you are a writer, the first site has a number of historical facts listed. --24.80.70.174 21:32, 20 March 2006 (UTC) JK[reply]

Draughts, absolutely. That remained the term to 1730, at least. (Now, find out about quoits.) Geogre 02:55, 21 March 2006 (UTC)[reply]
In the OED Supplement, the earliest cite for "checkers" referring to the game is from 1712. This is in the "Letter-book" of Samuel Sewall, who according to Wikipedia had lived in New England since 1661. It seems entirely plausible to me that the term could have been used there in the late 1600s and not recorded in print until 1712. Of course this doesn't mean that people didn't also say "draughts". --Anonymous, 05:22 UTC, March 21, 2006.
Words are generally in use before they appear in print, of course, but I find "draughts" the term until the 1730's in England. I suppose it's possible that "checker" was a regionalism or an Americanism, in which case the questioner could conceivably use "checkers." I just can't see that as being the most common term. Geogre 18:05, 21 March 2006 (UTC)[reply]
Draughts is still the name used in the UK. Even when the box is clearly labelled checkers, http://www.gamesonboard.co.uk/s/all+draughts/203 PhilipPage 23:46, 21 March 2006 (UTC)[reply]

Constitutional amendments[edit]

Can you give me some information on the first 4 amendments of the constitution, what are the legal radifications? The legal/historical circumstances which led to the creation of these amendments? Thanks so much.

Are you thinking of a particular constitution, or just the first four amendments to any constitution? Markyour words 23:09, 20 March 2006 (UTC)[reply]

Excuse me, information on the United States Constitution's Amendments.

See United States Bill of Rights. --Metropolitan90 23:42, 20 March 2006 (UTC)[reply]

See First Amendment to the United States Constitution, Second Amendment to the United States Constitution, Third Amendment to the United States Constitution, and Fourth Amendment to the United States Constitution. If those articles left something out, or don't answer your questions, feel free to ask about those specific problems here! - Nunh-huh 23:43, 20 March 2006 (UTC)[reply]

President James Buchanan[edit]

Do you have, or where can I go to find out what school(s) James Buchanan attended prior to coll 23:27, 20 March 2006 (UTC)

Our article on James Buchanan says "He moved to Mercersburg with his parents in 1799, was privately tutored and then attended the village academy and was graduated from Dickinson College, Carlisle, Pennsylvania." The Complete Book of U.S. Presidents, William A. Degregorio, p. 212, says, "Buchanan learned the fundamentals at common schools and studied Latin and Greek at Old Stone Academy in Mercersburg, Pennsylvania, in preparation for admission as a junior to Dickinson College in Carlisle in 1807." But for his father's influence, he would have been expelled from Dickinson in his junior (first) year as a disciplinary problem... - Nunh-huh 23:40, 20 March 2006 (UTC)[reply]

entry on Alberto vargas[edit]

Hello,

This is the first time that I added information to an entry (Alberto Vargas - my uncle). I dont see the addition under Alberto Vargas. It was 3 paragraphs long with some references to his work. Is it lost in virtual space? Do I have to redo it?

Thanks for your help, Yolanda

I've removed your e-mail address so it doesn't get spammed. I don't see any changes to that article since February 19. Perhaps you hit the "show preview" button instead of the "save page" button? So yes, it seems it's lost.
Sometimes your browser may load an old version of a page from cache rather than reloading a changed page, in which case you can see the actual new page by "reloading" it (the specific command depends on the browser) or just by waiting a while and coming back later. But that doesn't seem to be the problem in this case. - Nunh-huh 00:52, 21 March 2006 (UTC)[reply]
The last change was on Feb 19th, 2006. I suspect you hit the "Preview page" button instead of the "Save page" button. If so, try again. StuRat 01:08, 21 March 2006 (UTC)[reply]
Or maybe 'Show changes' was pressed instead - it is a slightly ambiguously-named button.
Slumgum | yap | stalk | 15:23, 21 March 2006 (UTC)[reply]

March 21[edit]

Clarkson[edit]

Fine, I search for Clarkson [not seardh, just return] and the maopr British person in natislavery stuff does not come up - and you have an article on him. Why??????? —Preceding unsigned comment added by Dumarest (talkcontribs)

Not sure what words were intended by "maopr" and "natislavery". --Slumgum | yap | stalk | 00:19, 21 March 2006 (UTC)[reply]
I am a dyslexic typer, so allow me to translate:
     maopr       = major
     natislavery = anti-slavery
     seardh      = search
StuRat 00:48, 21 March 2006 (UTC)[reply]
Well, searches don't search by "importance", they search by spelling, so it's no surprise that you don't get results in order of "importance" - besides which, we'd never agree on what that order might be. Presumably you are using "Go" and getting our "Clarkson" disambiguation page; Thomas Clarkson wasn't on it, but he will be now. - Nunh-huh 00:58, 21 March 2006 (UTC)[reply]
I think the real question is: "Why is it when I get to the Clarkson disambiguation page, I can't figure out how to click the 'edit this page' button so I can add Thomas Clarkson?" I checked it - clicked the 'edit this page' button and added Thomas Clarkson. I didn't see any problem. --Kainaw (talk) 01:01, 21 March 2006 (UTC)[reply]

Reason for Congress Recess[edit]

Could you please explain why U.S. Congress is in recess from October through December 2006? Is this repeated yearly? What do the Congress members do for those months?

Thank you!

Here's a hint. Elections are in November. -- Mwalcoff 00:39, 21 March 2006 (UTC)[reply]
They are also supposed to go home to remain in touch with their constituents by meeting with them and asking them questions. Of course, some find it difficult to do this while hanging out with underaged hookers in Bangkok. StuRat 00:42, 21 March 2006 (UTC)[reply]
Simple solution - bring the underaged hookers to the U.S. so they can visit both at the same time. --Kainaw (talk) 00:52, 21 March 2006 (UTC)[reply]
Ah-ha, the real reason for Bush's pro-migrant-worker stance is revealed! (BTW, does StuRat have a specific Congressperson or persons in mind, or is he just casting general aspersions?) --Bth 10:56, 21 March 2006 (UTC)[reply]
Just general aspersions. I suppose I shouldn't accuse them of going to foreign countries to have sex with underage hookers, when plenty probably do that right here at home. "Buy American !" :-) StuRat 12:19, 21 March 2006 (UTC)[reply]

Elections being every two years, will there still be an October to December recess in 2007?

Yes, because underage hookers can be found in Bangkok every year. StuRat 01:12, 21 March 2006 (UTC)[reply]
Yes, because the seats are staggered. Only half the house is up for reelection in 2007, the other half in 2008, so everyone gets the campaigning/fundraising time. Plus, it can take months to get your hands into a lobbyist's pockets. Geogre 02:58, 21 March 2006 (UTC)[reply]
Actually, no. No congressional elections occur in odd-numbered years. You seem to be thinking of the Senate, where approximately 1/3 of the members come up for election every 2-year cycle to make up their 6-year terms. — Lomn Talk 17:19, 21 March 2006 (UTC)[reply]
In 2005, there was no Oct.-Dec. recess. The only long recess was in August. -- Mwalcoff 00:29, 22 March 2006 (UTC)[reply]

Underdogs[edit]

This might count as psychology or sociology or cussedness of human nature... I've often wndered why the following happens - any ideas? When people watch a sports event featuring no teams that they are fans of, they tend to automatically start rooting for the underdog. Say (to use an example from English football) a Man Utd fan is watching a match between Chelsea and Rotherham - they're far more likely to start cheering on Rotherham. I suppose part of it may be that they're throwing their support behind a team perceived as less of a threat to their own team, but I doubt that that would be a strong enough motivation in many cases. Any suggestions as to what causes this underdog support? Grutness...wha? 07:59, 21 March 2006 (UTC) (who spent some of yesterday cheering on a Papua New Guinean swimmer in TV coverage of the Commonwealth Games)[reply]

Perhaps because if they put themselves in the losing team's shoes, they would like somebody to be rooting for them. Do unto others etc .... (btw, a lovely word for a champion of the underdog is infracaninophile). JackofOz 09:27, 21 March 2006 (UTC)[reply]
Did you just make that word up on the spot, Jack...? :) — QuantumEleven | (talk) 10:41, 21 March 2006 (UTC)[reply]
Not at all, but I'm flattered you imagine this might be the case. I would love to be that creative, but I must rest content with the fame I derive from my other gifts. Credit goes to Christopher Morley, who coined it in the preface to "The Complete Sherlock Holmes" (1930). I notice there are 17,500 Google hits for this word, but I first became aware of it in "Mrs Byrne's Dictionary of Unusual, Obscure and Preposterous Words", a book no serious verbophile should ever be without. Its author is Josefa Heifetz Byrne (daughter of Jascha Heifetz). Infracaninophile has now given rise to infracaninophily (the act or practice of supporting underdogs), infracaninophobe (one who is afraid of underdogs), and ultracaninophile (one who favours those who are in power). JackofOz 13:21, 21 March 2006 (UTC)[reply]

Thank you, mwalcoff and Geogre (sic) for your real help. One reason is that the non underdogs because of their apparent superiority often behave arrogantly. Furthermore, vast majority of people can see themselves as part of a group of regular underdogs than they can see themselves as elites. Furthermore because the odes are stacked against them, just by pursueing what their pursueing the underdog can be seen as behaving with courage. Even if an underdog losses his courage is remembered, more than the fact that he lost. For example the defenders of the Alamo are international heros even though they lost, and Santa Anna is no ones hero and sometimes is thought of as a villian. In addition their many famous cases where the underdog one, most history most famous cases have been victories by an underdog. Small fundemtalist Jewish sect and their nonJewish adherents took on and latter dominated the largest Empire and military machiene of their time (Christans V. Romans). The American Revolution pitted a small group of farmers against british Empire. The founding of isreal pitted the powerfull oil wealthy Arab world agaist a international wandering race, with no money and little international support. Finally many of the world greatest leaders stories are true underdog stories. Many of the worlds greatest individuals, where considered intellectually, physically socially, economiclly inferior Napoleaon as Cadet was considered an underdog because unlike his fellow cadets, he has and Italian and not from an aristocratic background yet he is remebered as the greatest military men. Eisteien probably had a learning disability, yet is now considered histories greatest genouses. Lincoln came from a very humble background and became Americas greatest leaders. Teddy Roosevelt, as a child was consided sickly yet became a great, soldier, president, and fitness ethusiase. Their are recent living examples. John Major a recent British PM is Highschool drop out and worked as bus driver in his youth yet became PM. Even the US Bernard Kerik the son of Prositute and her pimp, a man who lacks a BA (though probally has at least some post secondary education) reached the highest level in americas national security apparatus.

USA PATRIOT Act renewal[edit]

I need to update several Patriot Act articles (in particular Title II. However, I'm finding it tough to find the amendment on THOMAS. Can anyone point me to the right spot? If so, I can update the article. - Ta bu shi da yu 08:47, 21 March 2006 (UTC)[reply]

Edward Jacob FSA[edit]

Dear Sirs, I have added an article on Edward Jacob FSA. You state that this is original research. It is not. I have added several sources such as the Dictionary of National Biography, yet the article still has not appeared. Please adivse. Kenneth W Jacob

( saving Mr Jacob's inbox from being utterly swamped by spam )

Looks like a good article ! I hope you didn't copy it from anywhere, though, as that would be a copyright violation. Some comments:
1) You may need to pick "reload" or "refresh" on your browser so you can view the new version, versus the cached version.
2) Your references aren't in the Wikipedia format. See Winston Churchill#Notes and references for the proper format.
3) Once satisfied, you can remove the {{OriginalResearch}} text from the top of the article so that template will disappear.
4) I don't think the "FSA" belongs in the title of the article. We generally only use the person's name as the title. One way to rename the article is to create a new article named just Edward Jacob, copy everything over, then change Edward Jacob FSA to:
#Redirect [[Edward Jacob]]
5) Your article seems to lack an introduction. It should give a brief overview of the reason for his fame, and contain the title of the article (his name) in bold text.
6) I notice his father had the same name. This would be a problem if his father ever gets a Wikipedia entry, too. If you don't think that is ever likely to happen, then we don't need to distinguish between their names, however.
7) It may take several weeks until the Wikipedia index is updated so your article will be found when you hit the "Search" button. Until then, you must type the exact name of the article or redirect, then hit the "Go" button, to find the article.
StuRat 11:52, 21 March 2006 (UTC)[reply]
For #4, don't do it as StuRat describes, but use the "move" function (button just to the right of "history"), as it preserves the edit history. — QuantumEleven | (talk) 13:15, 21 March 2006 (UTC)[reply]
The edit history is still preserved with my method, it's just under the old article name, where those changes actually took place. You can also add a comment when creating the new article: "See Edward Jacob FSA for old edit history." StuRat 13:51, 21 March 2006 (UTC)[reply]
True - but it's desirable to keep the entire edit history in one place, splitting it up makes no sense as the two edit histories would still be talking about the same article. See Wikipedia:How to rename (move) a page.
Anyway, in this particular case, the point is moot since I decided to Be Bold and do it myself :) — QuantumEleven | (talk) 18:01, 21 March 2006 (UTC)[reply]

Research of informations[edit]

(Removed post -> Don't post on multiple Reference Desks, see the Miscellaneous Desk for question. StuRat 10:47, 21 March 2006 (UTC))[reply]

I have had no luck searching on the Internet, so hopefully one of you can help me here! I wanted to edit the Mrs Dale's Diary article to give the name of the TV professor the daughter Gwen became engaged to in the final episode, from 1969. All I can gather is that the actor's name is "John Justin," and I have no clue (and apparently nobody else does either) as to what the character's name was. Hopefully a former listener from the UK can help this Wikipedian! Mike H. That's hot 11:24, 21 March 2006 (UTC)[reply]

arthouse film?[edit]

would you classify "american beauty" as an art-house film? state reasons.

i have read the article on art film, but still have no real understanding or grasp on the classification..

if it could be cleared up, would be appreciated. gelo 12:05, 21 March 2006 (UTC)[reply]

I think rather than getting us to directly answer your homework it would be better if we try and help you understand the article. Can you start with which bits are least clear to you? Let's focus on the first paragraph, but realise that this isn't a rigid category; it's subjective and answering it you'll need to show you understand the way people use the term. Notinasnaid 12:09, 21 March 2006 (UTC)[reply]
When doing homework like this, you should "compare" and "contrast". If you want to argue that it is an arthouse film, find similar films that are well-known arthouse films and compare it to them. If you want to argue that it is not an arthouse film, find similar films that are well-known non-arthouse films and compare it to them. The term arthouse is not discrete. "Pulp Fiction" was an arthouse film that went mainstream after a few weeks of release. So was "The English Patient". Many in the film industry tend to label any independent film or Mirimax film as "arthouse". That has nothing to do with content - just the production. --Kainaw (talk) 13:57, 21 March 2006 (UTC)[reply]

sorry. but this ISNT homework.. im genuinely wanting to know. you probably thought that because of "state reasons". but no, this ISNT homework, honestly. gelo 06:22, 22 March 2006 (UTC)[reply]

Well, even if it isn't homework, as Kainaw and Notinasnaid said, classifying a film as "arthouse" is largely a matter of opinion. There are some for which it's obvious - for instance, Armageddon is most blatantly not arthouse. But for many films close to the dividing line between arthouse and non-arthouse it's largely a matter of opinion. American Beauty has some arthouse elements in it, but according to some definitions, arthouse films cannot be as popular as American Beauty was. In the end, you have to make up your own mind about it. Also, remember that whether a film is classified as "arthouse" or not doesn't make an iota of difference in the end, it's just a label. — QuantumEleven | (talk) 07:29, 22 March 2006 (UTC)[reply]


The problem is that the term "arthouse" derives from the 1960's, when a type of theater in the US opened up that would show films that ordinarily weren't shown. They showed "art films" as opposed to popular films. These independent, isolated movie houses, almost always located near universities and in large cities, developed a set of distributors. The theaters themselves never cooperated to create the "arthouse film," but the genre came from the distributors who would serve art houses with films. The films could be popular, but they had to be unpopular at first -- otherwise the arthouses wouldn't show them because they didn't want to try to compete with the corporate theaters. Arthouses served a niche, and they lived and died by being very faithful and exclusive to it. American Beauty had a major distributor and a large advertising budget, and therefore it can't be an arthouse film. That said, people have begun to use the term "arthouse film" for films that are like the films that used to show only in arthouses. American Beauty satisfies that by having an avant-garde director, disturbing subject matter, naturalism, and light plotting. Geogre 11:32, 22 March 2006 (UTC)[reply]

I feel that the previous is not an accurate definition, as what is an "arthouse" film is at this point and perhaps always has been , entirely subjective. The English Patient was turned down for production by 20th Century Fox even though its producer (Saul Santz) and director (Anthony Minghella)had great success and could hardly be disputed as great film makers (read: The Unbearable Lightness of Being ) and both the novel and novellest are many a prize winner. But the film on screen as it seems in this article we judge them by, was every bit worthy of the scope and wide breadth of Hollywood film making - they called it the Casablanca of 1996, and an "intimate epic". I do not consider this film "art house". I consider it well made and 20th Century kicking itself in the rear for not producing it. But just because something does not have a major distributor and a large advertising budget does not make it more or less anything. The distribution unfortunately reflects on the success of many films, as does the large or small budget, but it always comes down to the making itself. If American Beauty had less of a budget and the same actors, it may have had less sound quality ( but I doubt it ) but would not have been less of a film, so by the above's definition, it is budget that makes a film "art house" or "Hollywood"? I cannot think so. The films spoken of were not unpopular, but had less opportunity due to less of a budget, to distribute and acquire funding and support. Those who ran the art houses to show these films were the visionaries and probably perpetuated the creation of the term "art house" because the films were being shown in their theatres and they believed in them and loved them or at the very least had an audience that would come to see them and spend their money in their theatres, but the quality was no more nor less than has succeeded or failed in Hollywood. My favourite year of the Academy Awards was the year The English Patient received best picture among many other awards, as it was standing up with so many independent films and the only "Hollywood" film was Jerry Macguire, and I think this woke Hollywood up. The art should stand on its own, and as it was proven in that year, it did. And both American Beauty and The English Patient do this, whether the distributors have the intelligence to recognise it or not, and the producers have the foresight to take on the project when they read the script. In the end, I must ask what is it in the content of a film that makes it "arthouse", for now that a certain director has gained Hollywood success but continues to make the same type of film, are we going to judge him and say it is no longer of the same spirit just because he is being backed by MGM or Miramax? We need only to look at the content of the films we have mentioned to ask the question. Is it disturbing subject matter and an avant garde director and natural acting and lighting that make it "avante garde"? Should we even bother to perpetuate the term anymore when film making is where it is at and we have so many films to choose from and enjoy? Does avante garde mean raw and real? I am a sceptic of Hollywood, believe me. But do not think we should be snobbish and make another category, as other films have always been out there... Ckalliope 08:13, 28 March 2006 (UTC)[reply]

1812 overture[edit]

In the late sixties, a british popgroup wrote a song called "Night of fear" (I think). The first line og text is: "Silent night has turned to night of fear". The point is that Tsjaikowskis 1812 overture was part of the melody - What was the name of that popgroup?

Is this it ->[24] ? StuRat 12:53, 21 March 2006 (UTC)[reply]

Recorded by The Move http://www.youtube.com/watch?v=1ghFMlYyh3A

understanding / Cortazar's work 'House Taken Over'[edit]

House Taken Over by Julio Cortazar is about the political strife in Argentina but many things still confuse me, such as the knitting which seems so important to the story. There are other symbolisms which I do not understand.

I haven't read that one, but knitting has obvious connotations of creation, motherhood and integration, for a start. The mothers of the disappeared are a vocal group in Argentinian society, so he might perhaps have had an eye on that. Markyour words 22:42, 21 March 2006 (UTC)[reply]

Help with Fine Art...[edit]

i have 2 burning questions that i would like on helping information about if at all possible, or peoples opinions...

1) Discuss the implications of the way that public galleries arrange and display modern art.

2) Is there still an avant-garde?

Any information at all that people an help me out with would be very much appreciated.

Thanks again! --212.219.28.91 14:24, 21 March 2006 (UTC)[reply]

This sounds very much like homework questions. They are intentionally very broad; part of the skill of writing the essay is that you should try to figure out a manageable strategy for answering. For the first, clearly you are going to have to find out about how public galleries arrange and display modern art; does your city *have* an art gallery featuring modern art which you can peruse?
For a start on your second question, have a look at Avant-garde. --Robert Merkel 14:39, 21 March 2006 (UTC)[reply]

Loretta Lynn & June Carter Cash[edit]

Is Loretta any relation to June Carter Cash???

Well, everybody is ultimately related to everybody else, but I know of no documented relationship between these two. Loretta Lynn is the sister of Crystal Gayle, if that's any help. JackofOz 16:13, 21 March 2006 (UTC)[reply]
No relationship that I'm aware of, though they're both members of country music royalty. AllanHainey 14:18, 23 March 2006 (UTC)[reply]

Phallocrypt or necklace?[edit]

Read about the koteka and lonkalonka on WIKIPEDIA. The description of the lonkalonka as a phallocrypt hung from male genitalia seems very peculiar, as I have seen an image of one with a twisted thong made of human hair. But to me it seems more like a necklace or ornament for the upper arm, as the thong is too short to get around the waist, and how it could be hung from the genitalia and called a phallocrypt, is beyond me. I have never seen pictures of aborigines wearing these lonka lonkas, so how do you know this is a phallocrypt? Someone who can explain how they are fixed to the body or lead me to an image so I am no more in doubt... Are there pictures anywhere of the ornaments of jade and hardstone hung from the genitalia in South America, in other words, is this a fact? Nothing surprise me about garments, but seeing is believing... The koteka, on the other hand , has got an historian/entropologist to suggest that they could have been worn by people here in Scandinavia during the bronze age due to the many rock carvings of males with axes and phallos. And why not..? Thank you. Steinar Ådland

  • I see what you refer to in the Koteka article. (It might have been better to put this comment on the discussion page there, by the way.) The information you refer to was added by an anonymous editor who apparently could himself use some editing . . . "lonkalonka" refers to the type of carved and decorated shell, not the type of item the shell is made into. I have seen a picture of one of these with a 23" "strap" attached, so I'm not ready to give the anonymous editor the lie just yet. I'm moving a copy of this discussion to the relevant talk page in the hope that your question will be better addressed there. Crypticfirefly 04:09, 23 March 2006 (UTC)[reply]

Therston Guast?[edit]

where can find one?--152.163.100.72 18:37, 21 March 2006 (UTC)[reply]

I find no matches for that name in Google, are you sure it's spelled correctly ? StuRat 20:29, 21 March 2006 (UTC)[reply]

Josefsburg, Galacia[edit]

I am trying to determine if the town of Josefsburg in the former state of Galacia in the Austro-Hungarian Empire still exists and what it might be called now. It was located about 40 miles southwest of Lemberg, (now L'viv) near the Dneister River. This area is now part of southwest Ukraine. A nearby town previously called Brigidau is now apparently called Laniwka. Was Josefsburg perhaps destoyed in WW2.

--Vic Thomas 18:41, 21 March 2006 (UTC)VicT.[reply]

This site mentions three Josefbergs in Galicia. I havent got a detailed map to hand to check where they are in relation to Lviv. Presumably Josefsberg was a village in one of the three raions - Drohobych, Strochabiez or Stryi. Jameswilson 00:39, 22 March 2006 (UTC)[reply]

If you go to maps.altasua.net and zoom in real, real close to Letnia (the closest town of any significance), you can see a teeny village called Korosnytsia (Коросниця). This was an alternate name for Josefsburg, according to this old map. If you do a Google search on Korosnytsia, you'll find other people who have been looking for the same village. According to this page, the village has 144 people. -- Mwalcoff 01:07, 22 March 2006 (UTC)[reply]

Thanks, I very much appreciate the assistance. Oh by the way, I can't seem to link to the maps.altasua.net. Is th URL correct?

--Vic Thomas 16:31, 24 March 2006 (UTC)Vic Thomas[reply]

Works on my computer. Make sure to click on the British flag if you're not up on Cyrillic and Slavic languages. -- Mwalcoff 23:47, 24 March 2006 (UTC)[reply]

Band that combines traditional music of three Abrahamic religions[edit]

Thanks very much in advanced for any help! I'm searching for a (possibly new age) band that makes music which combines the traditional music of the three major Abrahamic religions, i.e. Judaism, Christianity, and Islam. Gregorian chants and the like... Mostly instrumental with some vocal hymns. All I can remember is that I followed a link from somewhere onto their website, and that the website had a brownish/reddish background colour, I also listened to some clips provided on their website, I remember a Gregorian hymn, and also a female vocal from the Islamic musical tradition. Lastly, I think their band was formed around 2002-2003ish, sorry that I don't remember much. I've tried to look them up on Google, Yahoo, and Amazon, but unfortunately to no avail. Any help/leads would be much appreciated! --Shibo77 19:16, 21 March 2006 (UTC)[reply]

-- This might not be what you were initially looking for, but is very much along the lines of what you are describing: The Foundation for Universal Sacred Music. They comission several pieces of music each year from composers who - regardless of religion or region - write sacred music. Good luck on your initial search. Don420 14:33, 23 March 2006 (UTC)[reply]

Are you aware of this Wikipedia talk:Censorship ?[edit]

For myself, I would like to say that the method is not innocent. The subject is truly important : there is one talk page and twoscore people discussing auto censorship for one million (counting non active users). Will you give your advice ? --DLL 20:02, 21 March 2006 (UTC)[reply]

Major to Minor[edit]

I know the C major scale's relative key is A minor. To get from one to the other (respectively) you must go down 1 1/2 steps. Is this a hard-fast rule? Like F major is is D minor, E flat major is C minor? schyler 21:48, 21 March 2006 (UTC)[reply]

Yes, that's exactly the way it works. But I wouldn't call the interval "1 1/2 steps" - it's actually 3 semi-tones. JackofOz 23:32, 21 March 2006 (UTC)[reply]
I'd call it a minor third. —Keenan Pepper 03:23, 22 March 2006 (UTC)[reply]
I'd call it 300 cents. Or 316 cents. Or something like that. But just because I enjoy being pretentious anonymously. Seriously. --George 05:42, 22 March 2006 (UTC)[reply]

Was Margaret Beckett Official Leader of the Opposition and Leader of the Labour Party?[edit]

The template Template:UK Labour Party Leaders (for Labour Party leaders) lists Margaret Beckett between John Smith and Tony Blair. But the Tony Blair page succession boxes list Tony Blair as the successor to John Smith both as Labour Party leader and as Official Leader of the Opposition. There has been discussion of this on the talk page for Margaret Beckett, and I have asked about this on the talk page for Tony Blair, but have not recieved any answers yet. Does anyone here know? I suspect she was acting leader of the Labour Party, but that you can't be acting leader of the Official Opposition (there either is a leader or there isn't one). Carcharoth 22:29, 21 March 2006 (UTC)[reply]

She was acting leader of the Labour party and therefore leader of the opposition. What's the problem? Markyour words 22:38, 21 March 2006 (UTC)[reply]
She should not be listed as Leader of the Labour party. She was deputy leader when John Smith died, and therefore acting leader. Why is thistemplate so named anyway!? There is no such position as president of the Labour Party in the UK. The title is Leader - see the NEC page http://www.labour.org.uk/nec Jooler 22:47, 21 March 2006 (UTC)[reply]
Quite right, between May and July 1994.PhilipPage 23:59, 21 March 2006 (UTC)[reply]
She doesnt count as leader - just a caretaker until the next leader was chosen. And agreed, there is no such title as Labour Party President. Jameswilson 00:47, 22 March 2006 (UTC)[reply]
The correct terms appear to be "Chairman of the Parliamentary Labour Party" and "Leader of the Labour Party". The posts have been split between two people since 1970. For further details see here. Carcharoth 12:19, 22 March 2006 (UTC)[reply]
Carcharoth's assumption that "you can't be acting leader of the Official Opposition" is incorrect. She would have been correctly described as "Acting Leader of the Labour Party" and "Acting Leader of the Opposition". When the Leader of the Opposition goes on holidays, or is sick, the Deputy Leader of the Opposition becomes Acting Leader until the Leader returns to duty. When the Leader dies, the Deputy becomes Acting Leader pending the election of the new Leader. If the Deputy is elected to the top job, they would cease being Acting Leader and become Leader. JackofOz 01:41, 22 March 2006 (UTC)[reply]
I agree in the case of Leader of the Labour Party, where there are doubtless clear rules covering who is leader and acting leader at any one time. Though somone has pointed out here that the Labour Party rules at the time drew (and probably still do draw) a distinction between temporarily unavailable (holiday, short sickness) and being permanently unavailable (death, resignation). This is standard practice to avoid a power vacuum. In the case of John Smith dying, Margaret Beckett became leader. The wheels were then set in motion to have an election. The case for Official leader of the Opposition is more complex. This is a salaried job and official title. I will need to follow up the relevant legislation, as stated below, but see the comments on what happened in WW2 here. Carcharoth 12:00, 22 March 2006 (UTC)[reply]

An acting leader is a kind of leader. If you really want to know whether she was officially Leader of the Opposition, then you should consult the relevant legislation: it seems to be governed by the 1937 Ministers of the Crown Act (possibly since replaced) and various parliamentary standing orders. Markyour words 11:08, 22 March 2006 (UTC)[reply]

Thanks for the replies. I do have one further question... If Margaret Beckett was officially leader, then did she stop being deputy leader, and if so, who took over that role in the interim period between John Smith's death and Tony Blair's election and subsequent appointment of John Prescott as his deputy (or was it a joint ticket)? Carcharoth 12:19, 22 March 2006 (UTC)[reply]

From a reply I wrote on Talk:Tony Blair last night:
It's a difficult issue and brings to mind the constitutional disagreement in the US over whether a Deputy President became the President on the death of the incumbent, or became the Acting President, or remained the Deputy President but with Presidential powers. Formally, the Labour Party rules at the time of John Smith's death said this (rule 5 (4)): "When the party leader, for whatever reason, becomes permanently unavailable, the deputy party leader shall automatically become party leader until a new party leader is elected". So Margaret Beckett was actually Leader of the Labour Party, and in fact during this period material was indeed issued which identified her as such. David | Talk 22:33, 21 March 2006 (UTC)[reply]

One more comment, taken from this source is: "On two occasions the death of the Leader of the Labour Party has led to a Deputy Leader becoming Leader pending the election of a successor. The Leader from 18th January 1963 to 14th February 1963 was George Alfred Brown (b. 2nd September 1914, d. 2nd June 1985). The Leader from 12th May 1994 to 21st July 1994 was Margaret Mary Beckett (b. 15th January 1943). Mrs Beckett was the first woman to be the Leader of the Labour Party." This makes it perfectly clear that Margaret Beckett was Leader, if only briefly and in a caretaker role. She did stand in the ensuing election, but from memory was never thought to be a serious contender. That quote also shows that George Alfred Brown needs to be added to the list of Labour Party Leaders. What I don't understand is why Jooler said, quite categorically, that Margaret Beckett shouldn't be listed as leader. This is just wrong! Carcharoth 13:44, 22 March 2006 (UTC)[reply]

The George Brown you want is George Brown, Baron George-Brown. PS the author of that source is really very good and someone should give him a job paying lots and lots more! David | Talk 14:15, 22 March 2006 (UTC)[reply]
Thanks. I did find that George Brown, Baron George-Brown page when making the changes, and I thought it was quite funny that a George Brown has already been leader of the Labour Party! :-) Carcharoth 14:20, 22 March 2006 (UTC)[reply]
Ah. I've just read this and this. I see what you mean now! ;-) Congrats on such an impressive website, and thanks again for the advice. Any advice on where to get confirmation of the loose ends of this discussion would be much appreciated. Carcharoth 15:17, 22 March 2006 (UTC)[reply]

And a final response, mainly to JackofOz's comment that I was incorrect to assume that you can't be acting Leader of Her Majesty's Loyal Opposition, is a quote from the PDF document here PDF link and here Google HTML link: "The Speaker’s decision on the identity of the Leader of the Opposition is final (Ministerial and other Salaries Act 1975 (c27) s2 (2))" - though this seems to be with reference to who draw the salary. I would guess that if Margaret Beckett didn't draw the salary, she was nominally the Leader of the Opposition (ie. informally recognised as such), but not officially unless the Speaker recognised her as such. For what it is worth, the list at the bottom of that document does not include the interim Labour leaders (Brown in 1963 and Beckett in 1994) as Leaders of the Opposition. My guess is that the post was vacant until the new leader was elected, with maybe the interim leaders nominally filling the role for a few months for the purpose of parliamentary procedures, but not drawing a salary. That is just a guess though. Wouldn't know where to begin confirming that.

Based on the above discussion, I've tidied up the Labour Leader and Official Leader of the Opposition entries I can find in Wikipedia, but may have missed some. Where is the best place to archive this discussion, I wonder? It seems to cover several pages! Apologies for turning a reference desk question into a discussion! :-) Carcharoth 14:20, 22 March 2006 (UTC)[reply]

Seems I was wrong (shock! horror!). I was assuming the Australian experience mirrors what happens in the UK, but apparently not entirely. But there is an analogy with our Prime Ministers. Following the deaths of John Curtin and Harold Holt, Frank Forde and John McEwen respectively were appointed Prime Minister, not merely acting PM. The appointments were not meant to be indefinite, but only for as long as it took the party concerned to get over the shock and organise itself to elect a new permanent leader. Because of this caretaker role, they are sometimes referred to as "acting Prime Minister", or their names are even sometimes left off lists of Australian Prime Ministers, both quite incorrect. JackofOz 01:47, 23 March 2006 (UTC)[reply]

Immigration[edit]

Could you please tell me the U.S.A's current immigration laws.

I can't, but someone at The U.S. Citizenship and Immigration Services probably can. --Kainaw (talk) 23:27, 21 March 2006 (UTC)[reply]
See Title 8, Chapter 6 of the US Code. -- Mwalcoff 01:12, 22 March 2006 (UTC)[reply]

March 22[edit]

Parliamentary Sovereignty in Oz.[edit]

Hi, Thanks for your previous comments. I guess that I am also asking about the role of the High Court of Australia in the whole law making process. The High Court is able to over turn laws made by the legislature of they are unconstitutional... does this mean that you cannot say that parliament is the ultimate law making body in Australia?

The High Court cannot make laws, only the Parliament can do that. The High Court can overturn laws, which I suppose means the previous law (if any) is reinstated, but I'm not enough of a lawyer to say just what happens when a law is overturned. The High Court cannot act on its own volition. It must wait to be asked to consider the constitutionality of laws. In my view, that means the parliament is the ultimate law making body in Australia. JackofOz 05:31, 22 March 2006 (UTC)[reply]

Jack, what about the Common Law courts. They too make laws don't they? This doesn't affect the Parliamentary sovereignty bit, but does that affect the fact that the parliament is the ultimate law making body? Just trying to get my head around it! Thanks Northcoastbaby.

In my view no court can make a law. They can interpret laws, and their interpretations often become precedents for the way future cases are decided. If the parliament doesn't like a court's intepretation of a law, they can amend the law to clarify the intent. Courts often have wide discretion, often virtually none (such as where mandatory sentencing applies), but they are always ultimately subject to the law, which is not their creature but the parliament's. JackofOz 01:17, 25 March 2006 (UTC)[reply]
To add to this, when the parliament amends a law to clarify intent, or makes any other law that clashes with common law, the parliamentary law overrides common law to the extent of the incompatibility between the two. I think. The Mad Echidna 20:10, 28 March 2006 (UTC)[reply]

Hi again, I know that this is not the right place to ask this, but I have been trying to find how to write up my own profile like your fabulous one, JackofOz, and am unable to get the right information. Thanks, Northcoastbaby.

Thanks. Come and talk to me at my talk page, here. JackofOz 14:43, 27 March 2006 (UTC)[reply]

Canadian Prisons[edit]

Hi. I'm writing a research report on my hypothesis on the future of Canadian prisons. I was wondering if you would know where I can find information on who decides when to build one? I am somewhat pressed for time, and have been through so much information at this point that my kids are about ready to pull my hair out for me! I would appreciate the help, even though this information is probably only going to rank in the intro / context, it is still important to me. Thanks. 207.216.159.184 05:12, 22 March 2006 (UTC)[reply]

I would imagine the Department of Correction, or whatever it's called there, would submit a request for a new prison to the legislature. Depending on the law there, it would then either be decided upon by the legislature or put up to the voters as a referendum. Maybe somebody else can give you more specifics. StuRat 05:45, 22 March 2006 (UTC)[reply]
Corrections is generally a provincial matter. For example, in Ontario, it would be the Ministry of Community Safety and Correctional Services, although I would assume a decision as big as where to be a prison would require Cabinet approval. -- Mwalcoff 04:21, 24 March 2006 (UTC)[reply]

College project, I need help from women[edit]

I was told this was a good place to come for help. I'm taking a course in Islamic studies, and I'm doing a project which requires the assitance of women. I've already used all of my friends who are willing to take part in this.

The project has to do with how Muslim women dress. If you're a woman who thinks she may be interested in helping me, please take a look at my website.

http://www.geocities.com/rachelborange

Men, please don't respond or email me unless you know a woman who's interested. It would just be a waste of time. Thanks to everyone interested. Rachel Orange 08:25, 22 March 2006 (UTC)[reply]

add text[edit]

hello how can I add text because I have a lot of information about a men named ewald marggraff I was one of his best friends for 14 years he died and we belive he has been murdered so I have a lot to tell about him

There already is an article about him, Ewald Marggraff, so you can edit that. But please remember that wikipedia articles must be written from a neutral point of view and should not present original research or theories. David Sneek 10:40, 22 March 2006 (UTC)[reply]
If someone has the time, that article needs some severe editing. Markyour words 11:14, 22 March 2006 (UTC)[reply]
Yeah, I noticed. I'll see if I can get some work done on it. --JoanneB 11:17, 22 March 2006 (UTC)[reply]

in the begining......[edit]

does anyone know why God created light twice in genesis? cos( correct if wrong) on the 3rd day he created the sun,moon and stars, the source of light before electricity and natural source of today, but on the first day he already said let there be light? can anyone think of what the bible could be referring to? --Crazypinkster 12:15, 22 March 2006 (UTC)[reply]

It seems that the creation myth in the Bible started out as between 2 and 5 stories, which were then brought together when the Bible was constructed. Two of them are called the J- document and P-document (what, no articles on these ?). This has led to some discrepancies in the order in which things are supposed to have been created. Also, the people at the time may not have understood that the light in the sky during the day is diffracted sunlight (this is the only source of light on an overcast day). So, to them, it made perfect sense that the Sun would be created on one day and this "sky light" would be created later. StuRat 13:11, 22 March 2006 (UTC)[reply]

Here is the theory that there were 4 stories from which the first 5 books of the Bible were written:

A form and redaction critical approach to the Pentateuch that argues four distinct authors constructed the work; J = the Yahwist, E = the Elohimist, D = Deuteronomist, P = Priestly.

JEPD theory came from the form criticism of the 18th century which began to look at units of Biblical texts in terms of their oral forms before they were recorded in written format. Redaction criticism then began to examine how authors might link these story forms into a single text that supports the argument the author wishes to make.

In the Pentateuch two major names are used for God, YHWH (or Yahweh or Jehovah) and Elohim. Some sections of the Pentateuch exclusively use the name Elohim for God (Gen 1) while others exclusively use YHWH (Gen 2). YHWH tends to be spoken of more in anthropomorphic terms (he walks in the garden and creates with clay, Gen 2), while Elohim is described in more majestic, distant terms (he creates by his voice, Gen 1). This lead some to propose that two different authors, one preferring to use YHWH for God’s name, and another who preferred Elohom, recorded various stories in the Pentateuch and that later editors assembled these stories into the Pentateuch we have today.

Formally, the four author theory is as follows:

The Yahwist - a 10th or 11th century B.C. wrote what is now called the J Document. The Elohist – a 7th century author added new material to the stories older YHWH stories. The Elohim stories present a more elevated and advanced God. The stories are called the E Document. The Deuteronomist – 2 Kings 20 tells the story of Israel’s King Josiah recovering the Law and founding that it commanded that worship should only take place in Jerusalem. JEDP theory proposes that in fact Josiah had the Deuteronomist create these laws and add them to the Pentateuch in order to justify his desire to limit worship to Jerusalem. These additional laws form the D Document. The Priestly Group – the temple was destroyed in 586 B.C. by the Babylonians. While Israel was in exile and after they returned, the Priestly group added additional laws to the Pentateuch. This is known as the P Document.

StuRat 13:24, 22 March 2006 (UTC)[reply]


Crazypinkster, StuRat has given you one possible answer (the documentary hypothesis explains the possible historical origins of the Bible). If, like myself, you don't assign any divine truth to the Bible, StuRat's further extrapolations are a reasonable guess. If you, however, believe that the Bible is divinely inspired, you may place a different interpretation on things. One possible approach is to accept Genesis as a poetic account of creation; the question then arises, I suppose, is there any special metaphorical significance to the part you describe. For that, you're best off consulting your priest as to what it means. AFAIK there isn't any expert theologians who answer questions here on the RD (if there are any in the woodwork, please come out of it...). If you're asking from the perspective of Biblical inerrancy, again, ask your priest. I'd personally like to hear the answer myself... ---Robert Merkel 13:29, 22 March 2006 (UTC)[reply]
From a poetic perspective, light has to exist before there is a lit star. You have to have the concept and entity before you have any given iteration. God created light, and then he set up the sun and moon which would have the aspect of light to them. Poetically, the authors knew primary vs. secondary qualities, and they especially knew of a difference between an essence and an existence. In fact, in that regard the Hebrews were way ahead of almost everyone. Geogre 13:56, 22 March 2006 (UTC)[reply]
And do poets say you can have light without any source ? Scientists most certainly don't. StuRat 17:07, 22 March 2006 (UTC)[reply]
Yes. Light would exist without a source as a concept, and to any idealist would exist as an essence without existence. Geogre 20:51, 22 March 2006 (UTC)[reply]

Manifestos[edit]

May anybody tell me the "manifestos" of the following:

  • Christianism as a whole, Catholicism, Protestantism
  • Buddhism
  • Islam (Qur'an?)
  • Jewism

Thanks in advance.

See Religious text. --jh51681 15:52, 22 March 2006 (UTC)[reply]
These religions don't have 'manifestos' as such. They do have foundational documents. For Christians (all kinds) that would be The Bible. There are also a number of different Creeds, which are statements of belief. DJ Clayworth 18:06, 22 March 2006 (UTC)[reply]
While I'd agree that the Qur'an is the foundational text for Islam, I wouldn't say that the Bible is that in the same sense for Jews and Christians. Rather, the sacred scripture grew out of their communal experience. The closest thing I can think of as a "manifesto" in Judaism is the beginning of the Decalogue: "Then God delivered all these commandments: I, the LORD, am your God, who brought you out of the land of Egypt, that place of slavery. You shall not have other gods besides me...." (Exodus 20:1 ff.). For Christianity, Jesus' declaration of his mission, quoting Isaiah, will do: "He unrolled the scroll and found the passage where it was written: 'The Spirit of the Lord is upon me, because he has anointed me to bring glad tidings to the poor. He has sent me to proclaim liberty to captives and recovery of sight to the blind, to let the oppressed go free, and to proclaim a year acceptable to the Lord.'" "Today this scripture passage is fulfilled in your hearing," he continued (Luke 4:16-21). Pretty manifestational in both instances. --Halcatalyst 02:16, 24 March 2006 (UTC)[reply]

Zambia[edit]

Can you plese tell me who wrote the article on Zambia and when/ THanks . I need it for my APA sytle reference page.

Use Special:Cite to look up citation styles for Wikipedia articles. --jh51681 15:56, 22 March 2006 (UTC)[reply]
More than 100 edits for this page ; a good average. The "cite" link simply says "Author: Wikipedia contributors". This means that it is a collective work, as almost every WP article. --DLL 21:13, 22 March 2006 (UTC)[reply]

Paul[edit]

I have an old text here, talking of an epistle of Saint Paul, reporting Paul to say something like "you see that those who fight and compete for worldly fame [meaning athletes or gladiators] abstain from the lust of the flesh". I was so far unable to identify the epistle referred to here. I was searching for terms like "lust", but nothing came up. Does Paulus say anything of the kind, anywhere? dab () 16:27, 22 March 2006 (UTC)[reply]

This is probably not what you are looking for: "No one serving as a soldier gets involved in civilian affairs—he wants to please his commanding officer. Similarly, if anyone competes as an athlete, he does not receive the victor's crown unless he competes according to the rules." (2 Timothy 2:4-6) (biblegateway.com) Thuresson 17:52, 22 March 2006 (UTC)[reply]
hm, at least it mentions athletic competition. KJV has
No man that warreth entangleth himself with the affairs of this life; that he may please him who hath chosen him to be a soldier.And if a man also strive for masteries, yet is he not crowned, except he strive lawfully.
I searched KJV, but I didn't think of searching for "warreth" or "strive". I think this may well be what I'm looking for, turned to his own purposes by my author. Thanks, dab () 13:38, 23 March 2006 (UTC)[reply]

Most famous pope[edit]

Which of all the popes would be considered the most famous.

That depends on your definition of "famous"; in the sense of "which pope is currently known and recognized by most people", my bet is on John Paul II simply because he was around for so long that almost everyone currently alive instantly thinks of him when thinking "Pope" -- Ferkelparade π 17:25, 22 March 2006 (UTC)[reply]
In the sense of "which pope has been known and recognized by most people all-time", my bet would be on Saint Peter. --Metropolitan90 05:57, 23 March 2006 (UTC)[reply]
The only qualification I would make to that statement is that there is utter disagreement as to whether Peter is properly counted among the popes at all. Even the Vatican is equivocal about it. See a long debate about this issue. JackofOz 09:25, 23 March 2006 (UTC)[reply]
I don't think the Vatican has any qualms about enumerating Peter as the first pope. - Nunh-huh 05:44, 24 March 2006 (UTC)[reply]
This is a perfectly reasonable question and I don't think Peter is the answer, since he's not really famous for being pope per se. Gregory XIII probably has had the most direct effect on the world, with Pius IX and John XXIII just behind. I am blameless 05:34, 24 March 2006 (UTC)[reply]
Fame is a very fleeting phenomenon and an ill-defined concept. What is the measure of fame? Somebody who was world-famous 50 years ago might be a nonentity today. The question was not about all the popes who have ever been famous (because that would include rather a lot of them). We can really only look at popes who are famous today. Ask the "man in the street" the name of the Pope after whom the Gregorian calendar is named, and maybe 1 in 50 will tell you the correct answer or be able to give any details of how, when or why it came about. His impact is undeniable (or the reform named in his honour is), but Gregory XIII himself is not famous today. My gut feeling is that most people would immediately mention John Paul II, with John XXIII coming a distant second (he'd get a mention from middle-aged people or older, who remember the Vatican II reforms such as introduction of the vernacular). None would mention Benedict XVI, who is very much an unknown quantity almost a year after his election (in utter contrast to his predecessor who made his mark very early on) and looking like becoming one of the most nondescript and irrelevant popes in history. Very few would talk about John Paul I, Paul VI, Pius XII or any of the earlier popes including Pio Nono. It makes me wonder why we spend so much time sweating blood over our Wikipedia articles on these gentlemen. But I guess it keeps us off the streets. (Thus endeth today's rant from Jack). JackofOz 06:25, 24 March 2006 (UTC)[reply]

Theme song[edit]

I was wondering if anyone knows who sings the theme song to Extreme Makeover Home Edition, and also the name of it.

Answer:

The band O.A.R. (Of a Revolution) sings the song, and the title of it is Unbelieveable. (Not 100% positive on the title, but I found more than one referance to it) :)

President visiting Congress[edit]

On the West Wing they said that before the President can give the State of the Union address the Congress must first issue him an invitation to attend. Is this true, and if it is does it derive its heritage from the right of the House of Commons to exempt the monarch from its sessions?

I don't believe the former is true (I expect it's a distinction of "does" rather than "must"), nor is the president required to give the State of the Union address in person -- there was a 100+ year gap (virtually the entire 19th century) in which it was not so delivered. However, this likely falls under the various rules set forth by the House and/or Senate for their own operations, and I've not found those.
As for the roots of such an invitation being from the House of Commons: perhaps indirectly. However, since it would be found only in the internal rules, and not in the Constitution proper (which makes no reference to the issue at all), it would seem to be a minor cause at most. Also, the VP is free to attend any and all Senate sessions in his role as Senate President, so the physical separation of Executive and Legislative isn't really a big deal. — Lomn Talk 20:45, 22 March 2006 (UTC)[reply]
Still, as a matter of courtesy, I'm sure the Congress would appreciate it if the President doesn't walk in uninvited, just as the President wouldn't expect Congressmen to just walk in to the White House uninvited. I suppose that if the President had the right to walk into Congress and talk whenever he wanted, he could deliver a Presidential filibuster, and just keep talking until Congress goes into recess, say to prevent them from overriding a Presidential veto. StuRat 00:22, 23 March 2006 (UTC)[reply]
Please don't say that too loudly, or else someone might hear you and get inspired...not that he's ever actually vetoed anything...Brian Schlosser42 14:02, 24 March 2006 (UTC)[reply]
That's probably a fairly moot point because the President already has a lot of other, more effective ways to influnce Congress. The White House press room alone give him an unusually useful way to get his message across, especially if presidential press conferences are so rare that any all the media outlets will jump on any occasion that presents itself. So, if the President wants to stop Congress from passing a law he's going to veto, the public may know about it even before the Congressmen do. sebmol 07:53, 28 March 2006 (UTC)[reply]

An island named "Gab-uh-boo-too"[edit]

I'm looking for the correct spelling of the name of an pacific island which is roughly pronounced "gababootoo". I seem to recall it was involved in the Pacific War during World War II, but I'm not sure of the pronounciation or the spelling. Does such an island actually exist? -Alecmconroy 17:57, 22 March 2006 (UTC)[reply]

I guessed that the most likely spelling was Gababutu and tried a Google search on that. There were two hits. One was irrelevant, but the other was to an item at a "movie nitpickers" web site, referring to the 1965 WW2 movie In Harm's Way. The fact that there were no other hits suggests to me that the place is a fictional one, made up for the movie (or for the novel it was based on, Harm's Way by James Bassett). Alternatively, it could be that it's a real place but the spelling is seriously wrong. (I tried a bunch of other spelling permutations, assuming that the A's and the consonsants were right. The only one that produced any Google hits was Gabbabootoo, but all the hits turn out to be finding copies of the same old message which appears to be using it as an example of a fictional distant place. But if it was a real place with a seriously wrong spelling, this sort of technique wouldn't help.) --Anonymous, 20:30 UTC, March 22, 2006.
Most Pacific languages (certainly all Polynesian ones) only allow a small number of possible consonant-consonant combibations of letters, so the double b spelling is unlikely. Gababutu sounds and looks believable, but the G and B rule out almost every Polynesian language. So if it were a real place and your spelling is close, then it would more likely be in Melanesia or Nicronesia. This would make somewhere from the Solomons through to the Marshalls the most likely locations. Mind you, Anon is almost certainly right about it being fictional. Grutness...wha? 06:44, 23 March 2006 (UTC)[reply]

Nepenthes Coccinea[edit]

Can anyone give me complete directions on the care, feeding, deadheading, etc of this plant. thank you...68.194.131.37 23:22, 22 March 2006 (UTC)[reply]

Good place to start for advice on the care of any carnivorous plant is Peter D'Amato's The Savage Garden (Berkeley: 10 Speed Press, 1998) ISBN 0898159156 Further questions can be asked & answered on the Carnivorous Mailing list. -- llywrch 02:35, 23 March 2006 (UTC)[reply]

morality?[edit]

why do so many people feel the need to rebel against Christain moral values? Isn't it true enough that these are universal to most people, and that any decent society should have a good idea of right and wrong?--Mytrjf 23:37, 22 March 2006 (UTC)[reply]

Is this a chat room or an encyclopaedia? Maid Marion 08:28, 23 March 2006 (UTC)[reply]
No, most people don't believe in Christian morality as defined in the Bible, particularly these parts:
  • If you feel that God commands it, it's OK to murder your children.
  • Women are to be subservient to men.
  • Slavery is OK.
  • If you feel that God commands it, you may commit genocide against your enemies.
Then again, many people also don't believe there is anything wrong with fornication or adultery. Then there are all sorts of new rules, not found anywhere in the Bible, which some church officials seem to have made up and declared to be "the will of God":
  • No abortion.
  • No birth control.
  • No genetic engineering/cloning.
  • No science in general.
StuRat 00:05, 23 March 2006 (UTC)[reply]

totally agree.--Cosmic girl 22:37, 25 March 2006 (UTC)[reply]

I don't think we need the indignation of StuRat's comments to point out that some Christian values are not universal. Monogamy, for one. And premarital chastity. And of course, even within Christianity, there are a lot of disagreements over what moral values should be. The Southern Baptists have a drastically different view of morality from the Unitarian Universalist Association. -- Mwalcoff 00:35, 23 March 2006 (UTC)[reply]
To be fair, many people don't consider the Unitarian Universalists to be Christian. --Serie 01:02, 23 March 2006 (UTC)[reply]
StuRat is clearly upset over this issue—one of the controversial aspects of Christianity (like all religions with ancient religious texts open for interpretation) is that various passages are open for interpretation—for example, the commandment to "multiply and replenish the earth" (which is Biblical), which some interpret to mean that contraception is bad (and other such things). The Jade Knight 01:27, 23 March 2006 (UTC)[reply]
Those first four things that StuRat listed aren't found anywhere in the Bible. Also, who said that science and religion must differ? They don't! They complement each other! Anyway, people rebel against Christian morality for the sake of rebelling. They want to feel special by opposing something. —OneofThem(talk)(contribs) 20:11, 23 March 2006 (UTC)[reply]
As for the first four things, here is my evidence:
  • God commanded Abraham to sacrifice his son, Isaac. Thus, it was expected that anyone who felt they were ordered by God to murder their children should do so:
(Gen. 22) 1Some time later God tested Abraham. He said to him, "Abraham!" "Here I am," he replied. 2Then God said, "Take your son, your only son, Isaac, whom you love, and go to the region of Moriah. Sacrifice him there as a burnt offering on one of the mountains I will tell you about."
This commandment is clearly not written for women, but for men, with women as their property:
"Thou shalt not covet thy neighbor's house, thou shalt not covet thy neighbor's wife, nor his manservant, nor his maidservant, nor his ox, nor his ass, nor any thing that is thy neighbor's."
That last example also covers slavery, where "manservant" and "maidservant" were euphemisms for slaves. They sure weren't paid employees and weren't free to leave. Also, with slavery widespread in biblical times, you don't see any commandment against it, do you ?
As for being willing to commit genocide if God commands it, we have this command from God:
(1 Samuel 3) "Now go, attack the Amalekites and totally destroy everything that belongs to them. Do not spare them; put to death men and women, children and infants, cattle and sheep, camels and donkeys.' "
Saul then went and killed everyone but spared the King of the Amalekites, Agag, and God's response was:
(1 Samuel 10) Then the word of the LORD came to Samuel: 11 "I am grieved that I have made Saul king, because he has turned away from me and has not carried out my instructions."
StuRat 01:49, 25 March 2006 (UTC)[reply]
I recommend you read The Abolition of Man by C.S. Lewis, particularly the appendix, Illustrations of the Tao, for his interpretation of this. By and large, many values seem to permeate most (but never all) major cultures of the world. Some Sociologists argue otherwise, but usually contrary examples are minor or small fringes of society—rarely a substantial proportion of the world. However, these values, then, are not "Christian" values, properly, but universal values—things like not murdering or stealing, the importance of honouring one's parents, etc. The Jade Knight 01:24, 23 March 2006 (UTC)[reply]
Why do people rebel against any moral code? Because they can.
How are people introduced to a moral code? Thru their parents, teachers, religious leaders & other adults -- all of whom are human & likely to act hypocritically or otherwise set a bad example. A minister rails against premarital sex -- then it's discovered he's gotten a teenaged girl pregnant; a businessman boasts that hard work is good for everyone -- only to omit the fact he took over the business his father built up. And then there's the case where a kid decides to express her/his own individuality by attacking everything her/his parents believe in.
So why do you ask? Do you honestly want to know why they reject a moral code, or are you seeking approval to look down on them? People rebel for all sorts of reasons, & the only way you can know what they are is by asking them. Their reasons might just surprise you.
I can't speak about other religions, but one of the foundations of Christianity is the concept of forgiveness. Consider the Parable of the Prodigal Son: a kid demands that his father give him his inheritence, & he pisses it all away on fast living, good-looking women & fun times -- & hits rock bottom. Realizing that he has screwed up his life, he goes home to his father who receives him with great joy -- forgiving all of his misdeeds. The kid needed to express his individuality, & his father understood because he knew his son & did not look for a way to dismiss him as being somehow unclean.
I would hope that a good Christian would care about other people & try to help them find a way to live their lives according to a healthy moral code -- even if it wasn't a letter-perfect expression of what was written in the Bible. However I'm not a good Christian, so I can't tell you this is how you should live. Only you & God know that answer. -- llywrch 03:30, 23 March 2006 (UTC)[reply]
How about the fact the so many Christians are about hatred and judgement? Many are also hypocrites. And the fact the W is a "Christian" is hardly a ringing endorsement of the faith. --Nelson Ricardo 06:33, 23 March 2006 (UTC)[reply]
It can also be argued that certain people rebel against 'Christian' moral values because not only do they not agree with them (see StuRat's comments above), but they resent having morality imposed on them, as it were. You can consider it in a similar vein to listening to heavy metal music as a way of saying "I won't have my CD collection dictated to me". Flippant example, I know, but I wanted to illustrate the point. Additionally, from what Nelson said, certain people, while they may agree with the basic tenets of Christianity (love your neighbour, respect your parents, do not lie, kill or steal...) they feel that past or presents high-profile adherents (whether self-proclaimed or otherwise) of Christianity have tainted the religion through their actions, and they do not want to be associated with these individuals. So, they may follow a basically 'Christian' morality, but you will never see them in Church or wearing a cross (and you probably won't consider them Christians).
Just something to think about - the world is often not as clear-cut as one might like it to be. — QuantumEleven | (talk) 08:43, 23 March 2006 (UTC)[reply]
As an atheist, I do not consider myself as 'rebelling' against christian values because I have never been a member of the christian church, and to rebel, one must first belong. I also object to christians appropriating the idea of being nice to people and claiming it as their exclusive teachings. Atheists are nice to people because they are nice people, and to be honest, I believe that most christians are nice to people because they are nice people and not because their god has told them to be nice. David | Talk 10:28, 23 March 2006 (UTC)[reply]
I'm not so sure about that. Much as parents sometimes tell naughty kids "Santa won't bring you any presents unless you behave", I believe religions are established to scare immoral citizens into behaving, out of fear that "God won't let you into heaven unless you behave". (The other main reason for religion, of course, is a way to give the leaders power over the masses.) To me, being nice only when you think you are being "watched" doesn't really count for much. StuRat 11:10, 23 March 2006 (UTC)[reply]
This whole question of "being nice" as an expression of one's religious values is the greatest furphy of all time. Was Jesus being "nice" when he upended the money lenders' tables and got stuck into them with a whip? Was Peter being "nice" when he cut off Malchus's ear in the Garden of Gethsemane (John 18, 10-11)? Was Abraham being "nice" when he was prepared to cut his son's throat with a knife? Was the slaughter of thousands in the crusades, or jihad, "nice"? "Nice" is a middle class value that we've all been gulled into believing is terribly important when we're talking face to face, but please feel free to gossip and criticise people as soon as their back is turned. That double standard is commonly practised by members of all religions, and of none. JackofOz 12:46, 23 March 2006 (UTC)[reply]

Christian moral code? What does this moral code entail exactly? Where does it leave the Taoists, Buddhists, Hindus and atheists? That's of course assuming that Muslim and Jewish moral codes are similar to the Christian one.

  • Any moral code can be warped - the Christian crusaders and the Spanish inquisitors were very moral by Christian standards - for their time. The Christian moral code is flexible - not absolute. See Joseph Kony.
  • There is no evidence that religious moral code is somehow better or superior to the moral code instilled in humans throughout childhood. Argue otherwise and you're saying i.e.. Japanese are less moral than Christians. Crime statistics favor the Japanese moral code in this regard. Celcius 10:44, 27 March 2006 (UTC)[reply]
  • Also - Christians don't have a monopoly on certain moral values. "Don't kill people" and "Don't steal stuff" wasn't invented by Christians. Ok, "Don't eat shellfish" - I think you got that one but I'm quite certain that you'll find most of the "Christian" morals embedded in earlier societies and religions. I'll get us started and claim that the Roman law complex had laws which corresponded to at least some of the Christian "laws". So "Don't hurt people" and "Don't steal stuff" is in fact Roman morals. I'm certain pretty much most of what Jesus told you to do and not do is covered by some law or religion at some point - Christian morals are copyvio. Celcius 10:52, 27 March 2006 (UTC)[reply]

Christian morals? Yeah right! At leats in Australia we don't claim to be Christian and then enforce the death penalty. And it has been my experience that Christians are the most judgemental, homophobic, exclusive group on the planet. I think that if Jesus really ever did exist, he would be turning in his grave if he knew how people behaved in his name. Northcoastbaby.

I love that delicious piece of circular irony, Northcoastbaby. According to Christian belief, he did turn in his grave, get up and leave, just as he said he would. Which us what makes him special, and why anyone cares about what he said or did, or what's been done in his name. JackofOz 14:34, 27 March 2006 (UTC)[reply]

Location of Swords in 17th century British Estate[edit]

I'm trying to find out where dueling swords (such as a rapier), when not attached to a person, would have been kept in a typical 17th century English manor. Was there any tendency to keep them in certain places, in that regard? The Jade Knight 01:17, 23 March 2006 (UTC)[reply]

Modern estates would likely have them displayed on the wall, but that's not likely back when the swords were still in use. I suspect dueling swords, in particular, would be kept in a case, to keep them dry, so they wouldn't rust. It might be a rather ornate case, with a felt lining, as opening the case and removing the swords would be part of the rather formal dueling ritual. A lock would also be likely, as the owner wouldn't want an angry servant getting at his weapons. StuRat 01:32, 23 March 2006 (UTC)[reply]
Thanks for the suggestion. It makes sense to me, at least. I wonder what the historical fact is, however. The Jade Knight 09:02, 23 March 2006 (UTC)[reply]

Look at Googlimages with those keywords "swords case" --DLL 20:38, 24 March 2006 (UTC)[reply]

2 pages of nothing relevant. The Jade Knight 20:50, 24 March 2006 (UTC)[reply]

Oriah Moutain Dreamer[edit]

What is the legal name of Oriah Mountain Dreamer, the author of the poem, "The Invitation" ?

On her home page, there is no suggestion that's not her real (legal) name. Why would you think it isn't? --Halcatalyst 02:02, 24 March 2006 (UTC)[reply]
It does say on one page I spotted what inspired her to change her name, but it doesn't say what it was previously. --jpgordon∇∆∇∆ 20:25, 26 March 2006 (UTC)[reply]

need assistance[edit]

I am doing a persuassive essay on the mdical use of marijuana. I have found plenty of things to back up its use but I am looking for at least one organization that works to fight its use (they are aganist the use). I also might want more information on why people are aganist it such as the side effects. If you dont have anything on this site maybe you can point me in the right directon. my e-mail is amandafloros (at) msn.com

thanks for your help! this is a great site I found a lot of information to back my essay up on this site and I have recommended it to others in my classes.

As to your second query (and quarry), Mike Pence, I think, has been one House member frequently to express objections to the use of medical marijuana. His most remarkable moment on the floor of the House apropos of med marijuana was his explanation that Americans shouldn't speak of med marijuana because--this is no joke--a group of girls in his district, upon finding that one of their number was seriously ill, gave her pot to smoke, thinking it a palliative; in the absence of emergency care, profound trauma befell the girl. The debate on a 1998 anti-medical marijuana resolution in the House of Representatives might offer some info as to arguments against medical marijuana (see also the Wikipedia article Medical cannabis and this Wiktionary article). Joe 06:17, 23 March 2006 (UTC)[reply]
The arguments I most often hear are:
1) It's a gateway drug, leading to more serious drugs. There is a "cause and effect" problem here, however. It may well be that those with a tendency to use drugs will end up using serious drugs, but will first try milder drugs, like alcohol, tobacco, and marijuana; whether they are legal or not.
2) The "slippery slope" argument. This says that legalizing mild drugs will lead to legalizing serious drugs. This is a rather specious argument, however. Did the repeal of Prohibition lead to the legalization of other drugs ?
3) It can distract people from performing critical tasks. This is true, but so can alcohol and countless over-the-counter medications with the "don't drive or operate heavy machinery" warning, which is routinely ignored. Also note that "driving under the influence" laws could be used as they are, or modified where necessary, to include marijuana intoxication.
This is already happening. In Victoria, drivers can be randomly pulled over and subjected to tests not only for blood alcohol levels but also for being under the influence of a range of prohibited drugs including marijuana. These tests can also obviously apply following accidents. JackofOz 02:17, 24 March 2006 (UTC)[reply]
These are arguments against the general legalization of marijuana, but can also be used against medical marijuana.
StuRat 09:14, 23 March 2006 (UTC)[reply]

Photo of Joe Carr[edit]

My name is Joe Carr and a photo of me is being displayed on a page about me and I do not wish that photo to be there. I have told the person who put it there that I do not want it there and he said it would be removed. Please remove it. I give permission on my website to use other pictures but not that one. Thanks, Joe

Hi Joe. Image:JoeCarr.jpg appears in the article Joe Carr, and it looks like the uploader thought you had given permission on your webpage for its use. Your fastest route is to show that you are who you claim to be and post a request at Wikipedia:Request_for_immediate_removal_of_copyright_violation. I've removed it from the article for the time being. moink 06:47, 23 March 2006 (UTC)[reply]
By the way, what is your objection to that photo ? StuRat 09:26, 23 March 2006 (UTC)[reply]
I've uploaded a new version of the same photo. Under US fair use laws, it appears to be validly included in articles about him, as no free alternatives are known to exist. -- Zanimum 15:40, 23 March 2006 (UTC)[reply]
Who does the picture belong to? or is it in the public domain? Are there places where a person can prohibit a photo of himself from being published (if he did not own the photo)? Not where I live. That's why some people cover their faces when they're being hauled off by the law and newspaper photographers are covering the event. --Halcatalyst 01:57, 24 March 2006 (UTC)[reply]

the commonwealth[edit]

If Australia becomes a republic will we still be able to compete in the Commonwealth Games? If not why, and if so why?

Yes, assuming the Republic of Australia remains a member of the Commonwealth (and I can see no reason why it wouldn't). Elizabeth II is head of state of only a minority of the members of the Commonwealth. David | Talk 10:04, 23 March 2006 (UTC)[reply]
The majority (or at least a very large number) of Commonwealth countries are republics. A few which spring to mind are India, Pakistan, Singapore, Kenya, Uganda, Ghana, South Africa, Tanzania, Mozambique, Cameroun, Nigeria, Sierra Leone, Malawi, and Gambia (there are quite a few more in Africa, but the list was long enough). Grutness...wha? 00:51, 24 March 2006 (UTC)[reply]
Queen Elizabeth II is the head of the Commonwealth, which contains 53 countries, both republics and monarchys. She is the head of the group. However, she is head of state for the commonwealth realms, which include australia, canada and the UK. Her role as head of state for the Commonwealth and Commonwealth Realms is merely symbolic and each of these states will have somebody similar to her to do official deeds. As long of a country stays within the commonwealth or commonwealth realms then officially they should be able to participate in the games even if they changed they way the country is run i.e monarchy/republic etc.

Commonwealth Games Countries[edit]

Why do Scotland and England compete individually in the Commonwealth Games when countries with states like Australia and Canada do not compete similarly, also if Scotland and England are not Commonwealth of Nations member countries as is the United Kingdom then how do they compete legally in the Commonwealth Games? Dale

--Dalecarr 10:16, 23 March 2006 (UTC)[reply]

The divisions within the UK are more autonomous than the states in Australia. For example, Scotland has its own parliament. England and Scotland were also separate nations for centuries. StuRat 10:54, 23 March 2006 (UTC)[reply]
Those examples don't really illustrate the difference, StuRat. All the Australian states have their own parliaments (as do all the Canadian provinces), and prior to 1901 they were all separate British colonies. Scotland was indeed once a separate kingdom, and Wales was a separate principality, and Northern Ireland was a part of the separate country of Ireland. At some point Wales was attached to England, to form the political entity "England and Wales", which I understand still exists. Nevertheless England, Wales, Scotland, Northern Ireland, the Isle of Man, Guernsey, and Jersey all have separate teams in the Commonwealth Games. But at the Olympics they're all just one country, the United Kingdom. It's all very mysterious. JackofOz 12:22, 23 March 2006 (UTC)[reply]
Ok, let me clarify what I meant by Scotland having it's own parliament. If the UK had a parliament, and if England, Scotland, Wales, Northern Ireland, etc. all had their own subsidiary parliaments, then you would have a "federal system" much like in Canada, Australia, or the US. However, since England doesn't have a separate parliament, that's not the case. So, it's more like Scotland is a semi-autonomous region within the UK while England is not. Very confusing. StuRat 04:12, 24 March 2006 (UTC)[reply]
What that means is that Scotland is less autonomous than the Australian states are, not more. The UK Parliament gave Scotland its parliament, and could take it away again (and probably will at some stage). But the Australian Parliament has no power over the existence or structure of any state parliament. That is entirely a matter for the constitution of the state in question (eg. Queensland abolished its own upper house in 1922). JackofOz 00:06, 25 March 2006 (UTC)[reply]
I forgot to mention that the NT and the ACT have a similar level of autonomy to that of Scotland. Their Legislative Assemblies were set up under laws of the Federal Parliament (1978 and 1989). Those laws could be changed, in theory. JackofOz 08:31, 25 March 2006 (UTC)[reply]
For the Olympic Games, the teams that compete have to be those sent by national Olympic committees and as there is only the British Olympic Committee, there is no way that the Olympics would admit England, Scotland, Wales and Northern Ireland separately. There is a long-running issue in Football (soccer) where there has never been a British or UK team and all international participation is by England, Scotland, Wales and NI separately: proposals for a British team have generally foundered because of a fear that it would be a precedent for insisting that the UK sides amalgamate for everything. I suspect the real reason for separate participation at the Commonwealth Games is that it helps the diversity of the winners to break the UK down to lower levels. Remember, the Crown Dependencies also compete separately at the Commonwealth Games. David | Talk 12:55, 23 March 2006 (UTC)[reply]
Scotland and Wales are also somewhat fearful of (even symbolic) further English control, and forcing them to compete as part of the UK would likely meet with some resistance. The Jade Knight 17:23, 23 March 2006 (UTC)[reply]
I think it was more to do with tradition, than real-world politics. And having four seperate orginasations (as they do in soccer f.x.) gives them four votes rather than one. Eivindt@c 17:30, 24 March 2006 (UTC)[reply]

What are these clothes[edit]

What are these people wearing? I've never seen them before. Susie Platterton

http://community.webshots.com/photo/548283256/2564430850078515824wUfXZb http://community.webshots.com/photo/548283256/2576632380078515824VIWhFt http://community.webshots.com/photo/548313534/2593464720086967025hIwAkL http://community.webshots.com/photo/548313534/2241418020086967025qKcBeW

Looks like black spandex zentais (featuring a facial mesh), fluorescent green ties, fluorescent pink gloves, white horned-rim plastic glasses, fluorescent necklaces, and either white headbands with pink flowers or while bowler hats. StuRat 17:18, 23 March 2006 (UTC)[reply]
These are stage performers. They perform on a black state lit with blacklights so the gloves, ties, and such glow brightly. Then, they dance around and the audience is impressed by the glowing clothing that appears to move by itself. --Kainaw (talk) 18:12, 23 March 2006 (UTC)[reply]
Weird stuff, zentai. All of these people have facial mesh though, what fabric would that be? Susie
Look at zentai. It is normally nylon/spandex mesh, which is easy to see through. --Kainaw (talk) 19:25, 24 March 2006 (UTC)[reply]

Plutarch's Moralia[edit]

In our article on the Moralia by Plutarch, we list 7 of 78 titles. It would be nice to have the full list. Does anyone have it, or find it online? dab () 15:43, 23 March 2006 (UTC)[reply]

ok, I just found it, in amazon's "look inside" [25]; help in transcribing is welcome. dab () 15:44, 23 March 2006 (UTC)[reply]
What help do you need exactly? Maid Marion 15:56, 23 March 2006 (UTC)[reply]
Oh, I've just seen the Moralia article - you mean you just want the Latin titles listed and translated into English? If so, happy to do it for you, just shout. Maid Marion 16:18, 23 March 2006 (UTC)[reply]

human family?[edit]

how many humans are there in this? theorically of course?--64.12.116.72 17:44, 23 March 2006 (UTC)[reply]

In "this"? What do you mean by "this"? This room? Just me. This house? Two humans. This city? A few thousand. This state? About 4 million. This country? About 300 million. In the world? Over 6 billion. Theoretically though, just one. I'm the human. Everyone else is a figment of my imagination. --Kainaw (talk) 18:10, 23 March 2006 (UTC)[reply]
According to solipsism, there's only one person, you. Thuresson 19:21, 23 March 2006 (UTC)[reply]
But there are three personalities inside of me...what does solipsism say about that? No matter, I prefer solecisms in any case. :) Joe 20:48, 23 March 2006 (UTC)[reply]
42 - Adrian Pingstone 21:38, 23 March 2006 (UTC)[reply]
Yeah, but we are all ontologically alone, so the other 41 may as well not exist. JackofOz 22:08, 23 March 2006 (UTC)[reply]
We have an interesting article on Humans. I didn't know that besides our sub-species (Homo sapiens sapiens) there was also another branch of homo sapiens, Homo sapiens idaltu. I don't see any reference to it on our pages, but I recall hearing or reading something recently about the discovery of a third branch somewhere in Oceania. Can anybody confirm this? Back to the question, it used to be argued that the Neanderthals were a human species, but apparently that argument was put to rest and they are Homo neanderthalensis. So the answer appears to be that there are two, or maybe three, subspecies in the human family. --Halcatalyst 01:49, 24 March 2006 (UTC)[reply]
Re the Oceania question, are you thinking of Homo floresiensis? JackofOz 02:09, 24 March 2006 (UTC)[reply]
Yup, that's the one, but I see it's only a homo cousin. Oh well, a descendant of homo erectus can't be all bad. --Halcatalyst 05:16, 24 March 2006 (UTC)[reply]

In a 'normal' nuclear family, I guess it's arround 4 or 5.--Cosmic girl 22:02, 25 March 2006 (UTC)[reply]

Deals ...[edit]

Hey all hope you can help me with this. I was reading last week about the Americans buying Manhattan from the Dutch for beads. It got me thinking about other great deals that have taken place. Can anyone else think of any famous/unique/funny deals that have taken place in the last few hundred years? Thank you! Aidan

The Louisiana Purchase and Alaska purchase come to mind. The Jade Knight 17:48, 23 March 2006 (UTC)[reply]
The recent deal in which NBC acquired Al Michaels from ABC in exchange for, inter al., the rights to Oswald the Lucky Rabbit, though not on the scale of the land deals referenced above, is rather amusing.Joe 18:11, 23 March 2006 (UTC)[reply]
See also [this recent deal; it is not uncommon in certain parts of the world for ice hockey and soccer players to be traded for food (this regularly happened, IIRC, in Canada in the early days of professional hockey), but this story is particularly funny in that, after considering that his club viewed him as worth but 33 pounds of meat, the player traded decided to retire, leaving the club to which he was to go out those ever-important 33 lbs. of meat. Joe 18:11, 23 March 2006 (UTC)[reply]
You can eat hockey players? --BluePlatypus 18:24, 23 March 2006 (UTC)[reply]
Why not? Rugby players are edible. User:Zoe|(talk) 02:11, 24 March 2006 (UTC)[reply]

Note that many uneven deals are agreed to under duress. For example, the French sold the Louisiana purchase because they sorely needed the cash for the Napoleanic wars and were, in any case, unable to defend that area from the Americans or, even worse, their enemies, the British (in Canada). StuRat 18:58, 23 March 2006 (UTC)[reply]

Aiden, your history is rather mixed up. There were no "Americans" back then. It was the Dutch who bought Manhattan from the Natives. --Nelson Ricardo 20:07, 23 March 2006 (UTC)[reply]
It's actually worse than that. Nearly every history textbook published since 1880 reports that the Dutch settlers bought Manhattan from the Canarsie Delawares for beads. In fact it was 60 guilders in trade goods, but it's not clear that it was beads [26][27], and it's not clear that Manhattan was the Delawares' to sell.[28] - Nunh-huh 01:45, 24 March 2006 (UTC)[reply]

North Carolina public records[edit]

Template:PD-NCGov quotes a North Carolina statute which may or may not put public records in the public domain. Any informed opinions about this? The statute says that public records "are the property of the people" and are made available free of charge, which certainly sounds like public domain. This template was recently redirected to "no license". -- Curps 18:46, 23 March 2006 (UTC)[reply]

Sounds like public domain to me. StuRat 18:51, 23 March 2006 (UTC)[reply]
Agree with Stu. Joe 20:11, 23 March 2006 (UTC)[reply]
Diagree. "Free of charge" does not mean "no copyright", which is what "public domain" means, in a copyright context. "Property of the people" could be interpreted as "public domain", but that's just one interpretation. You'd need a judge to tell you if that would be a correct one. (Any government property could be said to be "property of the people", which still doesn't give you license to do what you want with it) --BluePlatypus 23:02, 23 March 2006 (UTC)[reply]
I was formerly an NC legal resident, and what it had meant was that all public records were public domain. They were "property of the people" in that no one could make them copyright, as the ownership resided with the people of the state. I.e. you couldn't take state census data, publish it with a shiny cover, and affix a copyright to it and sell it. That material was always public. That's how it was treated and how it was explained in the Introduction to Bibliography class in grad school (in NC). Geogre 03:07, 24 March 2006 (UTC)[reply]

NOTE: This is now being discussed at Wikipedia:Templates_for_deletion#Template:PD-NCGov as well. -- Curps 04:43, 24 March 2006 (UTC)[reply]

  • From the searching I have done, I agree that the template is correct: property of the People=public domain. User:Zscout370 (Return Fire) 06:35, 24 March 2006 (UTC)[reply]

Troll Question?[edit]

I am a troll question, please give me a serious answer-Imat 19:29, 23 March 2006 (UTC)[reply]

42 - Adrian Pingstone 21:38, 23 March 2006 (UTC)[reply]
No, sorry, wrong answer. The correct answer is 23. Ferkelparade π 22:28, 23 March 2006 (UTC)[reply]
If I answer the question correctly, do I get to cross the bridge? --Kainaw (talk) 23:31, 23 March 2006 (UTC)[reply]
Only if you pay a t[r]oll. --Halcatalyst 01:31, 24 March 2006 (UTC)[reply]
No need to be gruff, Kainaw. --LarryMac 14:50, 24 March 2006 (UTC)[reply]

I'm trying to get an idea of why there seems to be no significant minority of British Indians. Did all people of British ethnicity leave after Indian independence? I suspect that the answer is hidden somewhere in our many fine, detailed articles on the history of India, but my attention span is weak. The closest article I could find was Anglo-Indian, but it does not answer my questions. Actually, my best question is not really about the history itself, which I'm sure I could find through a web search, but about Wikipedia. Do we have an article or articles to which British people in India and British Indians could redirect? Thank you. Pissant 21:20, 23 March 2006 (UTC)[reply]

Well, seems most did migrate. This article quotes a figure of 200,000 Anglo-Indians. Not a small number, but certainly enough to disappear among one billion. --BluePlatypus 22:54, 23 March 2006 (UTC)[reply]
They are two separate groups.
  1. Anglo-Indians were the descendants of mixed marriages which were common in the early days but then mostly stopped happening. They often worked on the railways. Many still live in India, some went to Western Australia and other English-speaking countries.
  2. British people living in India on the other hand were always a tiny minority. Less than 0.1% of the population. On independence only a minority "stayed on" - a higher proportion in Pakistan than in India. Jameswilson 23:54, 23 March 2006 (UTC)[reply]
Those of us who live in the eastern states of Australia sometimes wonder whether Western Australia is an English-speaking place. On balance, it probably scrapes in. (Cheers to my sandgroper friends :) JackofOz 00:46, 24 March 2006 (UTC)[reply]

March 24[edit]

Water Problems[edit]

I live in a fourplex and my landlord has recently started landscaping the property. When I asked the contractor where are they going to get the water for the sprinklers, I was told that is was going to be hooked up to my unit. I contacted my landlord to question why do I have to pay for irrigation to a property I am only a tenant at? The responce I got was it is going to be hooked up to my water and the reason why I have to pay for it was because I live here. I do not know how legal this is, but I do know this much... I do not feel I should pay for watering a property I do not own.

Agreed. You would have a good legal case, unless there is some clause in your lease which says he can do that. However, you might find that the amount is so inconsequential as to not be worth fighting over, if it's only a few dollars a month. Depending on your water rates and the humidity and watering schedule, it could be quite a bit or hardly anything. StuRat 00:12, 24 March 2006 (UTC)[reply]
Mr StuRat.. I agree with your thoughts, but there is no lease and the water rate here in South Florida is very high. At this time the sprinkler system has not completed yet, but I am going to keep a close eye on my water bill since I've been living here for over four years and I know what my usual amount would be.
I see. The relatively high humidity in South Florida will at least limit the amount of watering needed. StuRat 00:58, 25 March 2006 (UTC)[reply]

You should consult a lawyer who does landlord-tenant law. In the mean time, keep records of everything. Keep your water bills and communicate with your landlord in writing, retaining a copy of each letter to present in court, if it comes to that. Brian G. Crawford, the so-called "Nancy Grace of AfD" 01:41, 24 March 2006 (UTC)[reply]

You should consider your relations with your landlord as well. If you actually go to court it's not for certain he will be very forthcoming to any future requests. However, if it is a large amount it is of course unacceptable. I would keep records and confront him with it if it is a substantial amount. If he does not comply wait untill the total sum is worth the hassle of court and go for it. Celcius 10:26, 27 March 2006 (UTC)[reply]

Bobby Goldsboro's "Honey"[edit]

I just saw an anonymous contributor ask on the Bobby Goldsboro talk page if the song "Honey" was about a suicide. I always thought it was about his wife who died, but not necessarily someone who took her own life. Is there any definitive ruling one way or the other? All I know is...it's not a "romantic love song" like the article says. ;)

I'm also curious because it was one of my mother's favorites...it was #1 the month she first got married. Mike H. That's hot 05:02, 24 March 2006 (UTC)[reply]

It's about the most sentimental song ever recorded, and it was part of a spate of "dead girlfriend/dead boyfriend" songs -- perhaps the apex of the trend. All we know from the lyric is that she's dead. One assumes that it's cancer or some other dreadful disease that would take a young wife. (Needless to say, "Johnny Angel," "Teen Angel," "Last Kiss," "Patches" (sort of), "Seasons in the Sun," and "DOA" are other "dead object of love" songs.) Geogre 18:06, 24 March 2006 (UTC)[reply]
There are no more than two hints in the song for a would be suicide : "See the tree, how big it's grown " (if she hung herself) and "caught her cryin' needlessly" (was she depressive ?) I don't think it is the case ; thank you for mentioning it, the song is very nice. --DLL 19:42, 24 March 2006 (UTC)[reply]

John Gowen[edit]

Ok. So, after doing a lot of research nad looking at census records and old family documents, not to mention Wikipedia articles (Melungeon in particular), I have ascertained that I am a descendent of one John Gaeween/Gowen/Goyne/Guynes/Going/Goings/Goins (the list goes on). From what I understand, and what I have read, He was born in Angola somwhere around 1615 and imported to Virginia as a teenager, as a slave. In one or more documents, which I cannot seem to be able to find, I read that this man, John Gowen, was the first black man to be manumitted in North America (I assume this is meant to designate the non-Spanish parts of the new world...), the first Freeman in the English-speaking New World. I am curious to know if anyone has heard or read anything about this particular man, or any sources that I should look into?

--Dlayiga 05:23, 24 March 2006 (UTC)[reply]

Note that "North America" usually also includes some formerly Spanish regions, including what is now Mexico, the Southwest US, Florida, and the Gulf coast of the US. Central America (most of which was Spanish) and the Caribbean (some of which was Spanish) are also sometimes included with North America. Of course, in the time frame you mentioned, the Spanish were quite brutal and unlikely to free any slaves. In English and Dutch colonies, I believe slavery was rare at that time, being far before it's peak was reached, with indentured service, for all races, being far more common. I'm not aware of their having been any slavery in French regions. StuRat 09:57, 24 March 2006 (UTC)[reply]
I hope you realize that "Melungeon" is probably the most disputed subject in American genealogy. I notice our article doesn't even mention the recent DNA study which claimed Jewish descents for many early families. Rmhermen 16:35, 24 March 2006 (UTC)[reply]
I know that Melungeons are disputed, I just meant in the sense that I used the article to help tie my Guynes family into the Goins family. I had other sources, however, non-Wiki--Dlayiga 23:41, 24 March 2006 (UTC)[reply]
  • What I've read (not that much) indicates that the Spanish treated their slaves far better than the nortamericanos did. I'd be wary of sweeping generalizations. We have an article, not very detailed, on Slavery in the Spanish New World colonies. --Halcatalyst 21:05, 24 March 2006 (UTC)[reply]
That may be more of a there is nobody left to complain situation. It wasn't until after most of the Indian slaves died/were killed off that the Spanish even needed African slaves. The majority of slaves imported to the Americas went to the Caribbean and South American colonies, where they died off at unbelievable rates. Rmhermen 00:15, 25 March 2006 (UTC)[reply]

The University of North Carolina at Chapel Hill would be the most logical spot to search, as they have meticulous and plentiful sources on the first European and imported settlers in America. Their center for American Studies and Southern Institutes would be good places to query. Goins is a common name in Virginia and North Carolina, and therefore it is likely that the impressing family would have been in the Piedmont region of present-day North Carolina or Virginia. Geogre 18:10, 24 March 2006 (UTC)[reply]

This page includes a "history" of the John Gowens family but please take with a whole handful of salt as almost every one of its conclusions and even some of its facts are denied by other Melungeon researchers. This uses the "any mixed blood person is a Melungeon" theory which is very strongly denied by others. Rmhermen 00:47, 25 March 2006 (UTC)[reply]

Apparently, the U.S. Supreme Court ruled on March 4, 1996 that there is no constitutional right to invoke the innocent owner defense against a civil forfeiture (Bennis v. Michigan, 116 S. Ct. 994, 58 CrL 2059 (U.S. March 4, 1996); "High Court: Constitution Doesn't Require Innocent Owner Defense," BNA Criminal Practice Manual, March 13, 1996, p. 116). Anyone know anything about this? - Ta bu shi da yu 05:25, 24 March 2006 (UTC)[reply]

Yes, it's just as your link suggests. I agree that it's a horrid decision (on par with the recent decision that anyone can be forced to sell their home simply so the government can make money), but do you have any specific question about it ? StuRat 12:59, 24 March 2006 (UTC)[reply]

Pen pals[edit]

Does anyone know any good websites where i can have a christian pen pal without any costs or hazards.

Thanks you

As for safety, as with any web site, don't give out any personal information beyond your first name, and you should be perfectly safe. If, on the other hand, you give out all info including your address and where you keep your key hidden outside, you are taking a serious risk, whether it's a Christian website or not. StuRat 09:51, 24 March 2006 (UTC)[reply]

The Jewish Brigade[edit]

Does anyone have any information about the 5,000 strong Jewish fighting force which fought in WW2 other than the article supplied in site...

The external links section of that page has a lot of links to useful sites. - Akamad 23:45, 24 March 2006 (UTC)[reply]
Try doing a Google search on "Jewish Brigade". -- Mwalcoff 23:45, 24 March 2006 (UTC)[reply]

Margaret Thatcher[edit]

What was the name of the woman who gave margaret thatcher such a hard time about the sinking of the belgrano on the phone in programme on the bbc?

According to this page, it was Diana Gould, of Cheltenham. --LarryMac 22:16, 24 March 2006 (UTC)[reply]

World's Largest TV Networks/Channels[edit]

According to Wikipedia:

"Rede Globo (Portuguese: Globe Network) is Brazil's biggest television network and the number one producer and provider of Portuguese language television programming in the world. It is also the third biggest TV channel in the world, watched by 80 million people daily."

What are the first and second biggest TV channels in the world (and I'm guessing it's commercial, not cable, TV networks that they're talking about)? I can't seem to find a list like that anywhere!

Thanks for any help you can provide!

I notice there isn't a source for that statement. :( It sounds impossible to prove- how do you measure the size of a TV channel? The most viewed channels are probably Chinese and Indian state TV, though you won't get accurate audience figures for those countries. Or do you measure by how many people can receive the channel, even if they never actually watch it? HenryFlower 23:06, 24 March 2006 (UTC)[reply]
This says #1 is CNN, but doesn't take into account viewership. -- Slumgum | yap | stalk | 01:26, 25 March 2006 (UTC)[reply]
Now that site mentions 149 million households capable of viewing CNN. According to this site, China has 361 million households, with 90% national TV penetration. I'm not sure what 'national TV penetration means', but I'd be amazed if there are any households in China which a) watch TV but b) can't view the main state TV channels. Very puzzling. HenryFlower 01:34, 25 March 2006 (UTC)[reply]
Hmmm, the title of the article is "Largest Worldwide TV Network," so I guess that rules out a national TV station. If you're right about China, then one of their stations might well get the most viewers. -- Slumgum | yap | stalk | 01:44, 25 March 2006 (UTC)[reply]
Just for the record, we had a few Brazilian editors who made trouble with innumerable "This is the best in Brazil" articles. We also got the usual run of amateur input. The consequence is that we have had very, very little review and fact checking of Brazil-related articles compared to US- and UK- and Australia-related articles. Therefore, the usual disclaimer that Wikipedia is not definitive should be amplified with regard to pop culture in non-English nations. (In other words, the article may well be incorrect or naive.) Geogre 03:36, 25 March 2006 (UTC)[reply]
To be perfectly honest, lack of sources is so damn common around here (in several medias) that most things are really unverifieable. I have several problems tracking things around here. ☢ Ҡiff 06:10, 25 March 2006 (UTC)[reply]
This isn't the place for it, but I have a sort of law about how much to trust an article. The more common the interest, the more likely the subject has many editors and therefore the more likely that it will be vandalized and the more likely it will be correct. The more esoteric the knowledge, the more likely it will be correct. In cases where the knowledge is popular but there is a barrier to accessing the interested population (Finnish pop culture; pop culture in Zambia), the more likely that it will be random. The democratic editing model is such that esoteric subjects invite specialists, while popular topics invite enthusiastic amateurs. Specialists usually can cite their information, but they tend to be at Wikipedia either as IP's or just temporarily. The pop culture folks tend to police themselves, so long as there is a sufficient population of editors. Geogre 14:23, 25 March 2006 (UTC)[reply]

March 25[edit]

Joe Carr - Part 2[edit]

Hello. My name is Joe Carr and I am writing to you for the second time in the last few days because there seems to be a misunderstanding about the nature and intent of an image of me which his appears on the homepage of my personal webspace http://www.lovinrevolution.org/Bio.htm If you visit that site, to the right of the page you will see a small logo of me with text that says JOE'S BIO & STATEMENT OF IDENTITY. If you click that link, you get to another page, where that small thumbnail appears in a much larger form. And if you visit the website http://en.wikipedia.org/ and type my name into the search box on the right of that page, you will also find that picture of me. The rights to that picture belong to me and I have not authorized that picture's use on wikipedia's website. I have a lawyer friend of mine about whether that picture is in the public domaine, and he says it is not. I would like you to please remove it from your website.

I would also like to mention that this will be the second time that picture has been posted without my permission. I have previously posted a message similar to this one (asking that that photo not appear on your website) in this help section (I am sorry I cannot remember where).After I did this the photo was taken down, but then new one was put back up. I then sent an email to [email protected] asking that it be removed again. I do not really understand why people seem to think I am important enough to warrant my own encyclopedia entry, but apparently some of my past poliltical actions have upset some people enough that they now feel compelled to never let the world forget me. I have received death threats in the past, and this is why I do not want my picture appearing on this website. This is a very serious matter. There is no reason for my picture to be on this website and I have not granted permission for that picture to be used. I trust that you will do your best to make sure this does not happen again.

Thank-you, Joe Carr Olympia, WA

This really doesn't belong here at the Humanities section of the Reference Desk. Have you posted this message on the discussion page of the article itself? That might help. The Jade Knight 02:35, 25 March 2006 (UTC)[reply]
  • Hi, Joe. The place to complain is here. You'll find there an e-mail address to use where someone can actually help you out. This page is a reference desk manned by volunteers, not representatives of Wikipedia. Good luck. --Halcatalyst 03:23, 25 March 2006 (UTC)[reply]
I've removed the pic from the article, but have no way to prevent others from putting it back. You might want to ask an admin to delete the original image and lock the article, to prevent anyone from putting it back. StuRat 03:55, 25 March 2006 (UTC)[reply]
Your original post is here: [29]. StuRat 03:58, 25 March 2006 (UTC)[reply]
If that's your rationale for wanting the picture removed, you might also want to prevent Google from indexing your personal website. Currently a search for "Joe Carr" gives your website as the 4th result (where there are other pictures of you) - the Wikipedia entry for you is the 47th result returned.--inksT 04:07, 25 March 2006 (UTC)[reply]
Joe, the most obvious thing to do is remove all pics from your website immediately. StuRat 04:17, 25 March 2006 (UTC)[reply]
If you're going to try to maintain more than one identity, it's pretty much essential. Ask Valerie Plame. - Nunh-huh 04:23, 25 March 2006 (UTC)[reply]
Someone is playing games with us here. The user who originally added that image to the page has admitted he is posting from User:192.211.25.9, the same IP address that was last used to post here by the person who is claiming to be Joe Carr, [30] aka Joseph Smith of the International Solidarity Movement (ISM). That same IP address has been trying to control ISM-related content in several Wikipedia articles using two different user accounts: User:TroiS6 and User:Jonathansword. It was TroiS6 who added the Joe Carr image to the article in the first place. I don't know what's going on and I don't care, but we shouldn't be taken in by it. If Joe Carr cared so much about hiding his appearance because of death threats, he wouldn't have several photographs of himself on his website. Having said that, I also don't care whether Wikipedia has a photograph of him on the article. SlimVirgin (talk) 04:36, 25 March 2006 (UTC)[reply]
Notwithstanding the many questions raised by Slim, FWIW, Zanimum concluded, upon reading "Joe's" initial post, that the use of the image was fair; in view of his formidable Wikipedia credentials, I don't imagine that anyone, absent compelling reason, should be inclined to disagree (this is not, I should say, argument from authority, but, instead, a recognition that, considering the sundry substantial responsibilities that the community writ large and the Foundation have invested in him, it is likely that his pronouncement is considered). Joe 06:20, 25 March 2006 (UTC)[reply]

Freak Flag[edit]

Periodically I come across references to the word "Freak Flag", as in: "let your Freak Flag fly..." or in some 1960s song lyrics (f. ex. David Crosby)

Question: what exactly is a freak flag, and what does it look like?

Thanks, Peter Paller

I've always taken it as just an idiom as in the idea of presenting a banner or flag to make it obvious that you are not willing to go along with convention. I've never come upon any actual "freak flag" in any of my reading of the 60's counter culture which Crosby lived through. Dismas|(talk) 02:13, 25 March 2006 (UTC)[reply]
Isn't there a flag with a marijuana leaf on it ? The would seem to qualify. StuRat 03:40, 25 March 2006 (UTC)[reply]
This page[31] while it does not source the claim, claims it refers to a man letting his hair grow long. Maybe it could be any "flag" (sign or symbol) signifying hippiness: clothing or whatever. I wonder about the origin of the quote. Esquizombi 05:39, 25 March 2006 (UTC)[reply]
The line from the song that the original poster mentions is from "Almost Cut my Hair" which was written, I believe, by David Crosby for Crosby, Stills & Nash. Dismas|(talk) 14:21, 25 March 2006 (UTC)[reply]
In the context of the "Almost Cut my Hair" song it clearly means that having long hair is an indication to others that the singer is a "freak", i.e. not of the "mainstream establishment". Whether long hair on males would today still count as a "freak flag" is questionable, as times and fashions change, but a better modern analog might be a mohawk, which would signal to most people that the wearer is a "freak" in the sense meant by Crosby, and would carry with it a series of negative and positive effects. --Fastfission 17:56, 25 March 2006 (UTC)[reply]

African ancestry[edit]

I don't know if I'm harping, but I have another question, somewhat related to my prior: What percentage of people who consider themselves "white" have African ancestry? I know that it is probabably a much smaller number than those "white" people who have Amerindian blood, but between the shameful hiding of ancestry and simple ignorance, I imagine it would be difficult to ascertain. I guess percentage is a wrong word, it might be too precise. I don't know if there is even an answer to this. I'm just curious.

--Dlayiga 02:30, 25 March 2006 (UTC)[reply]

If you accept the "Out of Africa" theory, everyone is of African ancestry to some degree :)--inksT 03:27, 25 March 2006 (UTC)[reply]
That's a ridiculous question reminicent of Nazi ideas of "racial purity". If you look white then you are white. That is the only thing the term means in the modern sense. It does not mean "devoid of african ancestry" (as if such a thing were even possible to define). --BluePlatypus 14:49, 25 March 2006 (UTC)[reply]
There are a lot of people who look white (to me) who consider themselves black. Personally I consider a social definition of race to be more interesting and useful than one based on hue. HenryFlower 15:06, 25 March 2006 (UTC)[reply]

Total Participants in Korean War[edit]

How many total participants were there in hostilities during the Korean War? The article desperately needs figures on it. The article has strength figures-- total troops active at any one time. But all told, how many Americans, Chinese, S. Korean, and N. Koreans were actually "in" the Korean war?-Alecmconroy 05:39, 25 March 2006 (UTC)[reply]

biography of a musician[edit]

I remember reading an article in a magazine quite a while ago about a biography about a musician. This musician wrote music that broke a lot of rules and he created his own instruments. However, very little of his instruments survive and there are no recordings. He was a wanderer and died fairly young. I forgot to mention that he lived in the 20th century. Can anybody identify the composer's name? KeeganB

You could mean John Cage --Hughcharlesparker 11:44, 30 March 2006 (UTC)[reply]

Adding a person with the same name as some one already listed in Wikipedia[edit]

hi

just wondered what the wikipedia protocol is for creating a page for a musuician who has the same name as another musician who has a listing in wikipedia already?

thanks

brian

Wikipedia:Disambiguation may prove helpful. The Jade Knight 09:36, 25 March 2006 (UTC)[reply]

yes very (i think!) thanks

Funny Laws[edit]

Does any one know any funny, usual or strange laws around the world?? for example: not sure which one, but one of the asian countries its illegal buying or chewing gum.

That country would be Singapore. They like it very clean there, and will inspect your stall after leaving a public bathroom. -- Mac Davis] ⌇☢ ญƛ. 12:53, 25 March 2006 (UTC)[reply]
Now there's a job to covet: post-poo stall inspector. HenryFlower 19:24, 25 March 2006 (UTC)[reply]
Better to be the post-poo stall inspector than the post-poo stool inspector. Proto||type 14:48, 27 March 2006 (UTC)[reply]
I wonder why they don't just install automatic flush toilets. And just in case someone gets the bright idea of stuffing the toilet, make the stall door unlockable until the toilet flushes correctly-- trapped with an overflowing toilet! :P --Uthbrian (talk) 06:18, 27 March 2006 (UTC)[reply]

This example, from Australian tax law, is proof that the sinister forces we always suspected were at work, are legal:

For the purpose of making a declaration under this Subdivision, the Commissioner may:

a) treat a particular event that actually happened as not having happened; and
b) treat a particular event that did not actually happen as having happened and, if appropriate, treat the event as:
i) having happened at a particular time; and
ii) having involved particular action by a particular entity; and
c) treat a particular event that actually happened as:
i) having happened at a time different from the time it actually happened; or
ii) having involved particular action by a particular entity (whether or not the event actually involved any action by that entity).

JackofOz 14:10, 25 March 2006 (UTC)[reply]

  • Funnily enough, each of the last two US presidents (Clinton vis-à-vis Monica Lewinsky, Bush vis-à-vis Iraq) adopted that power. Joe 20:28, 25 March 2006 (UTC)[reply]

I remember reading that there was a club with a rule that was written into the charter, and could never be changed, which said "Absolutely no pets". Some time later, the attitude changed, and they passed an addendum "Henceforth, dogs shall be considered to be people". Some time after that, they passed another addendum "Henceforth, cats shall be considered to be dogs". :-) StuRat 02:28, 26 March 2006 (UTC) In Denver Colorado it is illegal to store any live stock on the second floor or above. Nigelthefish 20:17, 27 March 2006 (UTC)[reply]

in Cleveland, OH, it's illegal for women to wear patent leather shoes. This is an old law that isn't enforced but has probably not been taken off the books. The reason is that women almost always wore skirts or dresses at the time. -LambaJan 16:38, 1 April 2006 (UTC)[reply]

fascislamism[edit]

Hello! I am Yong-Ju from South Korea. When I read one article of Time,weekly magazine, I found an unidentified word, fascislamism. I could not find its definiton although I have an abridged Oxford English Dictionary,2nd. Please, What does that mean? Is it relevant with the Facism or anything? Please, Help me!!!! Good bye, and Thank you for reading my question. Have a good time.

See Islamofascism

Seems like a portmanteau of "fascism" and "islamism". --BluePlatypus 14:16, 25 March 2006 (UTC)[reply]
It's hard to be sure without the context, but it sounds so strange to me I think it must be a parody of neologisms like Islamofascism. Like muslimonazi. David Sneek 14:27, 25 March 2006 (UTC) P.S.: It's from an interview with BHL. Therefore it is probably just the result of confusion. David Sneek 14:51, 25 March 2006 (UTC)[reply]
It's a more standard portmanteau IMO in that it joins the words at a shared letter, whereas the other adds an "o" which is in neither. Esquizombi 15:18, 25 March 2006 (UTC)[reply]
It's done that way round so it follows the standard form. See cryptofascism, neofascism, etc. Proto||type 14:40, 27 March 2006 (UTC)[reply]

War Memorial dates of World War One[edit]

Embarrassingly, I work at a Tourist Information Centre and I've just been asked a question I couldn't answer. A member of the public asked why our local war memorial listed "European War 1914 - 1919" Now, from the World War One article I've found out that 1919 is carved as that is when the Treaty of Versaille was signed, but why does the memorial say "European War" and not "Great War"? I'm in North-East England. Henriksdal 14:53, 25 March 2006 (UTC)[reply]

I believe they said it that way because it was only a European war until the last couple of years, when the US entered the war. StuRat 15:29, 25 March 2006 (UTC)[reply]
It's hardly right to call it only a European war until the US joined in - there were land campaigns in German South West Africa, Tanganyika (German East Africa), West Africa (Togo and Cameroon), the southwest Pacific (German New Guinea and other places); naval battles and commerce raiding all over the place; Japan was a belligerent on the allied side from very early. -- Arwel (talk) 21:58, 27 March 2006 (UTC)[reply]
You might want to keep an index card with the critical dates:
Assassination of Archduke Ferdinand:       Jun 28, 1914
World War 1 begins:                        Aug ??, 1914
US enters the War:                         Apr  6, 1917
Armistice Day (cease fire):                Nov 11, 1918
Treaty of Versailles (official end):       Jun 28, 1919
Treaty of Lausanne (Turkish independence): Jul 24, 1923
The actual day in August, 1914 on which the war began is unclear, as the war slowly ramped up from colonial disputes to all-out war in Europe. 1923 is sometimes also used as the end of the war, as the war for Turkish independence blends directly into WW1. StuRat 17:05, 25 March 2006 (UTC)[reply]
I would check to find out when the memorial was built (or specifically when that notation was carved) and try to determine the terminology that was in use at that time. --LarryMac 16:48, 25 March 2006 (UTC)[reply]
Thanks guys - some great stuff there for me to start on looking into it! Henriksdal 13:59, 29 March 2006 (UTC)[reply]

GB - House of Lords: Is SIR Bob Geldof a life peer?[edit]

I looked up the list of life peers and found that the life peers in the British House of Lords are all barons or baronesses. Titles like "SIR" given to Peter Ustinov, Peter Geldof or to Mick Jagger - are these life peerages,too? Is "Sir" is the way of addressing barons? Or is the rank "Sir" inferior to that of a baron and does not entitle its holder to sit in the British House of Lords? Thanks!

See the articles on Baron, Life peer, Peerage, and List of Life Peerages. Note that virtually no new hereditary peerages have been created in half a century (since the Labour Party came to power). The Jade Knight 18:53, 25 March 2006 (UTC)[reply]
"Sir" is the title of a knight or of a baronet. Bob Geldof may be referred to as "Bob Geldof KBE", but not as "Sir Bob Geldof", because he is not a citizen of the Commonwealth. Knighthoods and baronetcies are not peerages, but rather are honours. Sam Korn (smoddy) 18:57, 25 March 2006 (UTC)[reply]
Life peers are called Lord not Sir. As Sam Korn says Bob Geldof is one of the relatively few foreigners (ie non-Commonwealth) with a knighthood. IIRC its only an honorary knighthood because of that. Jameswilson 00:16, 26 March 2006 (UTC)[reply]
"Relatively few"... well, there are quite a few. As pointed out, Bob is Bob Geldof KBE since he can't officially be called "Sir Bob" as it is an honorary knighthood (despite what British newspapers seem to think). But Americans given honorary knighthoods, and British people who take US citizenship, cannot even add the letters after their name (owing to some clause in the US constitution, ISTR). Thus it wasn't Sir Bob Hope or Sir Alistair Cooke. Quite a large number of prominent Americans (including a lot of Presidents) have received honorary knighthoods. And - to return to the original question - knighthoods aren't peerages, they are the next level down. Grutness...wha? 00:56, 26 March 2006 (UTC)[reply]
The list is quite long; see List of honorary British knights.
The media have short memories. Whenever someone famous gets an honorary knighthood, the media tell us it won't be "Sir" whoever, and why not. But with Geldof alone, they always ignore their own preachings. I have taken this up with several news organisations, and their response has usually been à la "That's what he's most often called, so that makes it OK for us to call him that. We need to relate to our readers". They could at least meet us half way and call him "Sir" Bob Geldof - but that's too hard, apparently. A lot of readers don't know what quotes mean (or any punctuation at all).
Even if full knighthoods were awarded to foreigners, Americans are prevented by law from accepting foreign honours without the permission of the president. But there's nothing to prevent an American accepting an honorary award from a foreign government. Despite their honorary nature, honorary knighthoods carry with them the right to use the postnominal. The reason American honorary knights (and those from most other countries) don't use the KBE while in their home country is a question of custom, not law. In various foreign contexts they are appropriately referred to as, eg. "Mr Steven Spielberg KBE". JackofOz 05:51, 26 March 2006 (UTC)[reply]
There's absolutely no prohibition against Americans accepting foreign honors. What's prohibited is for employees of the federal government to accept such honors without permission of congress (and congress has delegated granting such permission, so it can occupy itself with other matters....). I think the queen once made a remark to someone who had gained British citizenship (after being made a knight as a foreigner) along the lines of "Oh, how nice, now you can use your 'sir'" - though I don't think that would necessarily be so unless the honorary knighthood were "upgraded".... - Nunh-huh 06:19, 26 March 2006 (UTC)[reply]
I stand corrected. Thanks. But permission is only rarely granted, isn't it? JackofOz 11:28, 27 March 2006 (UTC)[reply]
My impression is that it doesn't come up very often (largely because the British government knows the rules very well and offers honours mostly to those no longer in service to the federal government), but that it would almost certainly be approved if it did. The purpose of the prohibition is to prevent foreign influence in the conduct of government affairs, and an honorary knighthood isn't going to buy a congressman... - Nunh-huh 01:49, 28 March 2006 (UTC)[reply]
FWIW, I think the last "employee of the federal US government" to be so honoured was Colin Powell. Grutness...wha? 13:18, 28 March 2006 (UTC)[reply]

The Esperanto flag.[edit]

How many people would recognise the Esperanto flag and what it means? (It's a green flag, with a white square in the upper-left corner and a green star inside that square). Is it a commonly recognised item?

I would say most people would not recognize the Esperanto flag, nor would they recognize most flags, for that matter. I don't. --Halcatalyst 21:56, 25 March 2006 (UTC)[reply]
I would recognise most flags, but before I saw this, I'd never heard of a language having a flag. Definitely not a commonly recognised item. Slumgum | yap | stalk | 00:01, 26 March 2006 (UTC)[reply]
I suspect you mean most national flags. There are thousands of flags for other organisations. HenryFlower 00:06, 26 March 2006 (UTC)[reply]
I agree. Quite a few languages have flags, but the Esperanto one is the best known of those. Even so it is not widely known. I'd have recognised it, but then again I'm a vexillologist :) (I could probably recognise 2-3,000 flags, but there are some 55,000 listed at Flags of the World alone.) Grutness...wha? 01:03, 26 March 2006 (UTC)[reply]
Yes, I did mean national flags. I'd assume that the mention of flags would, to most people, conjure up the idea of a national flag, or perhaps the regional flag as well - not a language flag. (There's an illustrated list here). I could probably identify 250-300 flags, which is possibly more than most people, but I wouldn't have known the Esperanto one. Slumgum | yap | stalk | 02:04, 26 March 2006 (UTC)[reply]
Not many. Most people don't even know what Esperanto is. I know what it is (and have for quite many years), but didn't know there was a flag until just recently (running across the Esperanto article here). I still probably wouldn't recognize it if it wasn't fresh in-mind, it's kind of generic looking. I'd probably assume it was the flag of some islamic nation, since it's all green. --BluePlatypus 02:08, 26 March 2006 (UTC)[reply]
I doubt many would. And I'd agree that very few would even know what Esperanto is. Though maybe everyone that I've never met is a vexillologist. (yes, I've been dying to use that word in a sentence for some time now).  :-) Dismas|(talk) 03:08, 26 March 2006 (UTC)[reply]
I agree about the flag. But I would have thought Esperanto is widely known as some kind of language. If this is any guide, it regularly crops up on places like TV quiz shows and newspaper general knowledge quizzes. JackofOz 05:12, 26 March 2006 (UTC)[reply]

Does English have a flag? What about other languages? —Keenan Pepper 06:01, 26 March 2006 (UTC)[reply]

Well, Lojban has a logo, but I don't know whether it counts as a flag or not. I doubt that the English language had a flag: it doesn't even have an academy or institute to guard it and publish official rules and dictionaries like other languages do. – b_jonas 15:42, 26 March 2006 (UTC)[reply]
For people who use computers, you often see a flag icon used to indicate your language preference -- Mac OS X seems to be using the Union Jack to represent UK English (or International English as some programs insist on calling it), and the Stars and Stripes to represent American English.
Example
I remember some program displaying a mixed flag, diagonally divided between both US and UK flags to represent the english language, but can't remember whether that was an OS or a website. Ojw 16:31, 26 March 2006 (UTC)[reply]
I'd never seen or heard of this flag before. I did know about the green star as a symbol of Esperanto. Don't some people wear a green star as a lapel pin to indicate fluency in the language? JamesMLane t c 17:09, 26 March 2006 (UTC)[reply]
To answer B Jonas's question - most constructed languages have their own flag: Esperanto; Interlingua IALA; Ido; Volapük; Klingon; Lojban; Glosa; Bolak; Interlingua L.S.F.; Novial; Occidental/Interlingue; and Vikto all have or had flags (see [32] and the links from it - also listed here are the other Esperanto flags). Grutness...wha? 06:17, 27 March 2006 (UTC)[reply]
Could that be because constructed languages are intended as cross-national? --Halcatalyst 21:21, 27 March 2006 (UTC)[reply]
Could well be - or more to the point a flag is a good rallying image for anyone on a specific mission to introduce a new "product" such as a constructed language. Grutness...wha? 00:45, 28 March 2006 (UTC)[reply]
Interlingua IALA does not have a flag or official logo. The flags at FOTW are "conjectural". --Cam 04:18, 29 March 2006 (UTC)[reply]
French uses the flag of the organisation "la Francophonie". --Shibo77 23:57, 30 March 2006 (UTC)[reply]

plane crash[edit]

I was watching the National Geographic channel and a commercial for a show called Seconds to Disaster played. It was featuring a plane crash "caused by the loss of a 5 inch screw." It also said it crashed in to an urban area. I was wondering if anyone knew which plane crash it was talking about. I'm quite sure however it is not the plane that crashed in to the New York Neighborhood a couple years ago. I would appreciate any help. schyler 18:34, 25 March 2006 (UTC)[reply]

Air France Flight 4590 was caused by damage from a loose metal strip on the runway, and crashed in an urban area, but not sure if that's the one you mean (not at an airshow). Ojw 20:20, 25 March 2006 (UTC)[reply]
National Geographic's site has a list of episodes. I'm not sure exactly which one it is, but the flight could have been British Midlands Flight 092, United Airlines flight 232, or EL Al Flight 1862. --Cadaeib 01:31, 26 March 2006 (UTC)[reply]
I knew I'd seen it somewhere - this question on the science reference desk mentions an F-22 Raptor whose "engine was destroyed by ingesting a five-inch metal pin". Ojw 16:15, 26 March 2006 (UTC)[reply]

I think it must be American Airlines Flight 587 and the famous "jack screw"... AnonMoos 21:51, 28 March 2006 (UTC)[reply]

It's the El Al cargo flight - see Bijlmer disaster. The fuse pin that was the likely cause could have been 5 inches (the jackscrew of AA 587 is much larger). -R. S. Shaw 09:39, 30 March 2006 (UTC)[reply]

Japanese Drawings[edit]

What is the name of the old stylized Japanese line drawings that predate manga? They were always of samurai's and half naked women going to public baths. I think I heard they originated in shady taverns where no longer needed samurai's went to gamble and drink. Also, are their any famous artists of this art type?

Ukiyo-e? Shunga? A famous artist was Hokusai. Peter --195.93.60.129 21:06, 25 March 2006 (UTC)[reply]

A priori[edit]

Can I be skeptical about a priori knowledge somehow? or is that imposible and stupid.--Cosmic girl 20:25, 25 March 2006 (UTC)[reply]

'when I analyse the event expressed in the sentence 'I think,' I acquire a series of rash assertions which are difficult, perhaps impossible, to prove - for example, that it is I who think, that it has to be something at all which thinks, that thinking is an activity and operation on the part of an entity thought of as a cause, that an 'I' exists, finally that what is designated by 'thinking' has already been determined - that I know what thinking is.'(Beyond Good and Evil, section 16)

Are you skeptical about a particular piece of knowledge, or the possibility of any a priori knowledge? --Squiddy | (squirt ink?) 20:58, 25 March 2006 (UTC)[reply]

I dissagree too...I've found quite a lot of parallels between Nietzche and me...maybe I'm his reincarnation...lol!. I'm not skeptical about any particular a priori knowledge..it's more like something general...like existence...the natural numbers, logic... and I haven't got any satisfactory answer... I mean... of course those things are TRUE...I just want to know if they have been doubted by which philosophers besides Nietzsche and found to be unprovable meaning they can always be doubted...one a priori notion that I also 'doubt' is 'truth' itself...like, it can never prove itself from within i guess... It has to always be justified...I hope I didn't confuse you :S...but you seem pretty smart Squiddy!, cute username by the way! XD.--Cosmic girl 22:04, 25 March 2006 (UTC)[reply]

First Pope[edit]

who was the first pope?

Saint Peter, Pope Linus and Pope Leo I are sometimes said to have been the first pope. --195.93.60.129 20:51, 25 March 2006 (UTC)[reply]
Unless someone's being extremely tendentious, the correct answer will be Saint Peter. - 00:36, 26 March 2006 (UTC)
Perhaps, but it's fair enough to distinguish "bishop of Rome" from "Pope" and to argue that the latter is a position that did not exist until some time after St. Peter. Geogre 03:45, 26 March 2006 (UTC)[reply]
The question would be a very reasonable one from someone who knows nothing of the history of the papacy. The issue is explained here. But there is a permanent debate going on about it at Talk:Saint Peter and Talk:List of 10 longest-reigning popes. JackofOz 05:03, 26 March 2006 (UTC)[reply]
At least one Catholic scholar holds that the institution of bishops only developed gradually in the early church. The idea of a pope came even later. See Francis A. Sullivan, S.J., From Apostles to Bishops: The Development of the Episcopacy in the Early Church (New York: Newman, 2001). --Halcatalyst 01:37, 27 March 2006 (UTC)[reply]

Himmler[edit]

Would Himmler have succeeded Hitler as leader of the Third Reich had he not betrayed the Führer and/or been found out? Would Himmler have been an effective leader? cassidy

Have a look at Himmler and see what you think. --Halcatalyst 21:43, 25 March 2006 (UTC)[reply]

At the time of Hitlers death, the third reich was about to colapse. Because of his distance from fuerer bunker he might have been logical successor to Hitler, he would be safe from imediate allied apprehension, so it is possible might have endorsed Himler as the leader. Ultimatly Hitler gave the chancilorship to Goebels, and the tilur presidency to Admiral Doentiz. As Hitler was close to Goebels he probally intended Goebels to be be the dictator, and Doenitz to serve as head of state. However Goebels took his own life this put Doentiz in a possition to negotiate the surrender of the Germans. The Germans would have been horedously badly served if a Nazi such as Goebels or Himmler took over as dictator. The war was lost, the Germans was to end the war as quickly possible. Doentiz because of his level head, lack of extreme Nazi ideology, relativly clear concious could do that. Indeed had Goebels or Himler been in charge they might have tried to keep the war going so to avoid their own culpability.

OK, fine, you've answered your own question. Why did you ask it in the first place? --Halcatalyst 01:30, 27 March 2006 (UTC)[reply]

I was just curious as to the opinion of other people. I did not write the above reply. I study the Third Reich for my degree, I have Ian Kershaw, who wikipedia has an extensive page on, as my tutor. I am just finishing a dissertation on the subject of the end of the war and the relationship between Himmler and Hitler. In actuality it is not clear if Hitler did want Himmler to be his successor, even though he was perceived as the natural choice by almost all within the Reich. Certainly Donitz had no idea of the situation. Hitler expressed serious doubt over Himmlers abilities as a leader, for good reason, commenting to Goebbels that he lacked the 'divine spark'. Of course once Himmler's behind-the-scenes negotiations had been broadcast across the world the matter was decided. Thanks for the reply - wikipedia seems like a decent place. cassidy

Had Hitler died before the war turned south, he likely would have been succeeded by Gorring, and after him would have been Hess (except he flew himself to Scotland because he was nuts...). Captain Jackson 23:30, 28 March 2006 (UTC)[reply]


As has already been pointed out, the question is moot since there was practically no Third Reich left at Hitler's suicide. I just wanted to add that Himmler was appointed the leadership of Berlin's last defense, and proved himself woefully inadequate, as he had no military experience. He was sacked/resigned pretty quickly, though. Asav 19:23, 29 March 2006 (UTC)[reply]

Celebrity Deathmatch[edit]

Hi! From which band is the music played during the end credits of Celebrity Deathmatch? Thanks in advance. Peter --195.93.60.129 20:51, 25 March 2006 (UTC)[reply]

The soundtrack, with audio samples, can be found at [33]. You can check those and see if any of them match the song you're looking for. Night Gyr 22:56, 25 March 2006 (UTC)[reply]
Thank you for your help! Unfortunately the tune isn't on the CD - at least I don't recognize it :( Peter --195.93.60.129 22:24, 31 March 2006 (UTC)[reply]

vietnamwar[edit]

what contries were the allies and who were the enemies in the vietnam war?

Perhaps you are asking which countries were allied with which other countries? See Vietnam War.--inksT 21:26, 25 March 2006 (UTC)[reply]
Very apt comment.
At first, I thought the real question was: Who were the "winners" and who were the "losers"? But then I realised they could only have meant: "Who was on our side and who was on the other side?". The answer to that question will depend on where the questioner is from. JackofOz 04:38, 26 March 2006 (UTC)A[reply]

America and it's allies the U.S, South Vietnam, South Korea, the Philipeens Australia. The other side: North Vietnam, the vietcong (a south vitnamess insurgency group), Russia (mainly in a advisory capacity), China (mainly in an advisory capacity), Cuba (mainly an advisory capacity).

Regarding Eve[edit]

The following comment was initially placed on Image_talk:Symbol question.png by a first time editor. I moved it here so it could be answered. J@redtalk+ ubx  21:48, 25 March 2006 (UTC)[reply]

While many of the comments are interesting all as well as mine are in true scientific terms theory. Even though many of the comments state that there were many peopel alive at the time of "Eve" this does not have to be the case does it\? It could be possible that there was only one woman at the time. I understand the bias against an understanding from the "scientific community" against this possibility but to state that it is impossible is subjective isn't it? Therefore to reject this possibility only promotes the agruement that science is biased. Would this not imply then that we are afraid of a view which challenges our theories? Isn't scient supposed to explore "all" possibilities. To neglect one would deny our right to call it science and then it would be religion wouldn't it? --144.137.131.242

Well, while I'd agree with a sentiment that "impossible" is too strong, could we settle on "extremely unlikely"? As a scientist, I would accept that in place of "impossible", and I don't think that doing so changes the MT Eve theory in any significant way. I suppose the use of fully restrictive terms like "impossible" should be limited to statements of logical contradictions - e.g. "It is impossible to have a square circle". However, I think that to suggest that use of the term "impossible" is indicative of a fear or prejudice against religion is mistaken - the word is used to convey a sense of extremes of improbability (i.e Where P approaches 0.000). Like saying it is "impossible" for pigs to fly.--inksT 22:55, 25 March 2006 (UTC)[reply]

If there was only one woman and only one man (or very few men), you would expect to have a problem with a lack of genetic diversity. This does appear to have happened in the case of cheetahs, since they do have a lack of genetic diversity. See Genetic_bottleneck#Humans for an account of a similar event with humans. Unfortunately, this was some 70,000 years ago (far earlier than Biblical accounts) and there seemed to be several thousand individuals left at the time, not just a few. StuRat 01:50, 26 March 2006 (UTC)[reply]

Just because something is possible does not mean it is plausible, and all ideas are not equally plausible. Just because something cannot be known with absolute certainty doesn't mean it can't be known with greater or smaller certainty. And that certainty can be known. Also, science does not in general work by assuming a conclusion and exploring for evidence for it. That is a classical method of pseudoscience, not the actual real science that generates real knowledge. Science starts with observation, then the building of a theory that generates predictions, which then lead to new observations. --BluePlatypus 02:03, 26 March 2006 (UTC)[reply]
There could have been only one woman alive at the time of Eve - as long as you accept that the Book of Genesis is at least partly fictional. If you accept Genesis 4:16-17 as fact then it is clear that Eve could not have been the only woman on earth (unless, that is, she was two-timing Adam with her own son). Even if 4:16-17 is false and Cain had no wife, there are serious genetic problems, since Seth would have had to produce children either with his mother or sister (and no sister is mentioned in Genesis). Whichever way you look at that, there were either serious genetic bottleneck-problems (and somewhat deranged interfamilial relationships) or there were other women at the time of Eve. And, of course, this is all without even mentioning Lilith... Grutness...wha? 05:52, 27 March 2006 (UTC)[reply]
In other words, the things you'd have to accept for there to have been only one woman are not the things you'd like to have to accept given the ostensible moral code of the religion. There's plenty of other problems with assuming only Adam and Eve and their children, but that's a deeper biblical scholarship issue. - Taxman Talk 16:58, 27 March 2006 (UTC)[reply]

March 26[edit]

Bridge laying tanks[edit]

Does in anyone know the name (and details) of the primary American Bridge layer tank in the '80s. --24.247.126.44 03:03, 26 March 2006 (UTC)[reply]

Since the late 80s, we've used the M60A1 Armored Vehicle Launched Bridge (AVLB). Our article on it is a copyvio, but you can read the original at [34]. I'm not sure what the predescessor was, and I haven't been able to find any information on it. Night Gyr 18:40, 26 March 2006 (UTC)[reply]
I started a temp page with Marine Corp info. See M60A1 Armored Vehicle Launched Bridge (AVLB)/Temp. Rmhermen 02:19, 27 March 2006 (UTC)[reply]

Total War Crimes in the Vietnam War[edit]

I've looked everywhere for the total number of war crimes by americans in the vietnam war, and i cannot find it...

You're unlikely to do so, since the definition of a war crime is not concrete. User:Zoe|(talk) 04:12, 26 March 2006 (UTC)[reply]
And aren't you interested in war crimes by other parties during the war ? StuRat 11:42, 26 March 2006 (UTC)[reply]
It's an impossible question, of course. A crime requires a law and a court. At the time of the Vietnam War, there was neither, except for laws and courts internal to each nation. Winter Soldier argued that there were multiple instances of war crimes by US soldiers, and both the South and North Vietnamese were known to have committed multiple war crimes by US legal standards. However, only Mai Lai got prosecuted. So that would leave a list of one warcrime committed by US forces as defined by US law, as documented by US forces, and as prosecuted by US non-military courts. Anything else is smoke and trollery. Geogre 14:25, 26 March 2006 (UTC)[reply]

Every single death on either side would count as a War crime to me.Not to mention the Agent Orange on going genetic problems.hotclaws**==

Freedom in Marriage,Sex, and Having Children[edit]

I have four questions to ask you about freedom in marriage,sex, and having children:

1.How much freedom to people nowadays usually hav in areas such as marriage,sex, and having children? Depends on where in world you go in the U.S. and Europe, with some exceptions people are free to engage in the forementioned activities. A consenting adult can marry any consenting adult of the opposite sex (and in some place of the same sex) regardless racial, religous or socio economic difference. Consenting adults are more or less free to engage in most sexual activities and can limit or not limit the number of children they have. Their are some limitations, many of these limitations are often on public health concerns and insuring that one person by exercising their right their not violating another individuals rights. Some of the limitations relate moral concerns. In a sexual or matrionial relationship both parties must be consenting adults. This of course is meant to preserve rights of a weaker party. Even among consenting adults their are limitations, specifically with marriage. Even among the most liberal societies homosexual marriage, inscest and polygamy. Espessally with the two latter the limitations are based legitament societal concerns, such as disease prevention, and the bring in the world of children who cannot be take car. Beastiality and incest also carry certain public concerns. Some societies are less free. China limits the number of children can have

2.Is it true that people nowadays usually have more freedom in those areas than in the past? In general yes.

3.If so, then why has the law allowed people to have more freedom in those areas nowadays?

4.If so, then is this the reason why marriage and the family are in the condition they are now in the modern society?(I mean, so many divorces,teenage pregnancies,sexual immorality,etc.)

5.If so, then how much freedom should people have and be allowed to have in these areas?(I think this is one of the big questions to do with marriage,sex, and having children nowadays, don't you think?) Media:User:Bowei

Looks like a school assignment. Try the article on marriage and/or ask your parents. Ouro 07:17, 26 March 2006 (UTC)[reply]

I love when they have questions based on you picking a certain answer to the previous question. I've aced tests on topics I was completely unfamiliar with using this method. Or, altrnatively, you can answer NO to question 2, then, by definition, questions 3-5 are not applicable, LOL. StuRat 11:35, 26 March 2006 (UTC)[reply]

Composer/arranger Clair W Johnson[edit]

I have lost my copy of Grove's and need to know any info that is available on Clair W Johnson. Does anyone know anything that might help?

Thanks! sveeb

A I can tell you pretty much anything you would like to know, he was my father. You may contact me directly: [email protected] --K7DJI 13:37, 28 June 2006 (UTC)[reply]
A Google search on Clair W Johnson turned up 469 references. Try it and see if any of them have the information you're looking for. --Halcatalyst 01:22, 27 March 2006 (UTC)[reply]

Appropriate to list band?[edit]

I notice that several bands, including some friends of mine in indie groups are included in Wiki pages. Is it appropriate to add info about my own band? We have 4 albums out and have been written up in Billboard, but I wanted to check before I just contributed.

thanks for any guidance, Craig The Taters email: [email protected] web: www.theTaters.com

It sounds like you would be notable enough but it's probably not a great idea for you or your friends to do it since the article would probably be very one sided. See WP:MUSIC and WP:BIO for more info. Dismas|(talk) 19:17, 26 March 2006 (UTC)[reply]

japanese culture[edit]

I'd like to know more about the culture the japanese have in wearing white uniforms in the workplace either in japan, but mostly in a japanese owned facility in the United States. When did it start and why? I also need references for such. Thanks.

The Japanese are rather obsessed with cleanliness, and white uniforms show any dirt very effectively, making for a good check. Note that American gas stations in the 1950's also featured attendants in white uniforms. This is also why medical staff are given white uniforms to wear. StuRat 21:09, 26 March 2006 (UTC)[reply]
... among other reasons, see white coat. — QuantumEleven | (talk) 08:17, 27 March 2006 (UTC)[reply]
Of course a white coat also conceals white dirt which is really the worst kind of dirt. Celcius 17:26, 29 March 2006 (UTC)[reply]

March 27[edit]

Ptolemies...[edit]

Can anyone think of historical and current uses of the name Ptolemy? People, things, descriptions (Ptolemaic), and so on? Any donations gratefully received at Ptolemy (disambiguation). I think I've covered most of the historical ones, but more contemporary uses of the name would be great. Thanks. Carcharoth 02:09, 27 March 2006 (UTC)[reply]

One of Harry Partch's instruments was called "the Ptolemy". It doesn't deserve its own article, but we could use an article about Harry Partch's instruments. —Keenan Pepper 05:06, 27 March 2006 (UTC)[reply]
Thanks. I've added a section to the Harry Partch article. If it gets big enough, it can be spun out into its own article. There is stuff in the section I labelled biography that should be moved to the new instruments section, if anyone wants to do that. Carcharoth 14:16, 27 March 2006 (UTC)[reply]
there is a chap who turns up on TV now and again who rejoices in the name of Ptolemy Dean link. Makes one wonder about the mentality of the parents. Jameswilson 01:04, 28 March 2006 (UTC)[reply]
LOL! That's funny. I wonder if it is possible to find out which Ptolemy he was named after? I've been wondering whether people in general are more familiar with the Ptolemies that ruled Egpyt, or the Ptolemy who was an astronomer (among other things)? Carcharoth 02:13, 28 March 2006 (UTC)[reply]
I also found an author called Ptolemy Tompkins... Carcharoth 02:37, 28 March 2006 (UTC)[reply]
There's also the Ptolemaic system of astrological houses. - Nunh-huh 01:52, 28 March 2006 (UTC)[reply]
Hmm. Tetrabiblos currently redirects to Almagest. From reading Ptolemy I see that the former covered astrological stuff, while the latter covered astronomical stuff. Looks like that redirect is wrong. Anyone agree? Carcharoth 02:13, 28 March 2006 (UTC)[reply]
I also noticed that problem. I have looked at both books and you are right, they are on different subjects. Plus, they are two separate books, making it like redirecting The Hobbit to Lord of the Rings. Someone needs to un-redirect. Maestlin 18:39, 29 March 2006 (UTC)[reply]
For the moment I have blanked the redirect page so it no longer redirects. I think there are two options for now: (1) Someone writes a stub article; (2) The page is deleted until someone can write an article (thus turning the blue links red). At the moment, what is in Ptolemy#Astrology is sufficient. Copying that into Tetrabiblos is another option, but rather redundant. I've copied the above three comments to the discussion page for Tetrabiblos. Carcharoth 22:50, 29 March 2006 (UTC)[reply]
I don't really anything to add about Tetrabiblos, so unless another volunteer turns up the page probably should be deleted. I can't get to it yet anyway; I am still being redirected to Ptolemy. --Maestlin 00:56, 31 March 2006 (UTC)[reply]

I am the chupacabra!?[edit]

fear me!?--Chupacabrads 03:16, 27 March 2006 (UTC)[reply]

No I don't think anyone does. AllanHainey 11:30, 27 March 2006 (UTC)[reply]

Get a Life,they're kind of fun.hotclaws**==P.S. for $1,958 I will cure you

A Painting of Fate Sitting on a Bubble[edit]

I am looking for the title and artist of a painting that I saw a long time ago. It depicts several nude women in a garden, who I believe represent ideas like "happiness" or "loyalty" (but not those ideas specifically)and one of these women is "fate." Fate sits on a huge bubble in the picture, supposedly representing how she is fragile, etc. It is a renaissance-era painting. I'm sorry I have so few details! Thank you for your help, Sara

undergraduate essays by famous authors[edit]

I am curious to read undergraduate essays, or exam scripts, by famous authors. Does anybody know of printed or electronic source of such a resource?

Highly unlikely. Except for public performances (dissertation defenses), in most European and American schools, at least, student essays and the like are private. When the teacher has the student, he or she doesn't know the student is going to become Victor Hugo or Thomas Woolf. I have had two students who are now famous professional athletes, but their student papers were handed back to them with a grade, and I doubt either wanted to hang onto them and publish them later. On the other hand, public performances like dissertations and dissertation defenses are available, but they're not usually very interesting. The exception may be Kierkegaard's M.A. thesis, The Concept of Irony with Continual Reference to Socrates. This is because academic work is... well... academic and not creative. (T.S. Eliot's dissertation is supposed to be about, as well, and of moderate interest.) Otherwise, juvenalia is generally not available. Geogre 11:35, 27 March 2006 (UTC)[reply]
FWIW, I'm aware personally of at least one case, and anecdotally of two or three others, where a school has retained the early work of someone interesting - usually it's through a particular teacher suspecting the kid has promise, or seeing some unusual merit in the work, and filing the essays and so on on the off-chance. But this material is generally also kept private (at least until someone sells it!), more as a curiosity than as anything published for academic research. I suspect this may be more common with poets, but couldn't say. Shimgray | talk | 12:42, 28 March 2006 (UTC)[reply]
Additionally most universities have very strict confidentiality policies in regards to the work of former students, often not granting access until decades after their deaths and even then not usually giving permission to reprint in full. Most famous authors are probably not interesting in sharing their very early work, either (though there are exceptions to this -- Bertrand Russel has excerpts from his gradeschool papers in his autobiography, lord knows why). Anyway, I think all of this adds up to it being unlikely that there has been any systematic compilation of such things, much less an online archive. --Fastfission 19:47, 27 March 2006 (UTC)[reply]
In fact, Thomas Wolfe is a bit of a test case. His great novel was written about his time at UNC Chapel Hill, and so quite a few people got interested in what he wrote while he was there. No joy. First, the university wouldn't release even his transcript until decades and decades after his death, and all of the literary productions he did that were preserved went silently into a collection per his wishes. There's an author who draws attention to his juvenalia and yet who, perhaps because of when he died and how, did not end up with things shared, even though they were preserved. Geogre 21:20, 27 March 2006 (UTC)[reply]

Allah[edit]

Can the omnipotent Allah send a prophet to humanity in the 21st Century? Ohanian 04:45, 27 March 2006 (UTC)[reply]

If you accept His omnipotence, then of course the answer must be yes. This does not mean, however, that He will or would do so. The mere fact that something can happen doesn't mean that it will happen. Muhammad (PBUH) is still The Last Prophet in Islamic tradition, and will remain that way - possibly even if a new prophet were to arise. Most of the major splits in the church (Judaism to Christianity and to Islam) have been caused by the rise of a major new teacher of the Word. A new prophet would amost certainly bring a major schism in Islam. Grutness...wha? 06:06, 27 March 2006 (UTC)[reply]
Perhaps this prophet is among us: Jimbo Wales. Joe 07:00, 27 March 2006 (UTC)[reply]
Revolution cometh Celcius 10:10, 27 March 2006 (UTC)[reply]
See Joseph Smith, Jr. for a modern prophet of enormous affect. Marskell 12:18, 27 March 2006 (UTC)[reply]
What was so enormous about Smith's externally displayed mood? (wink)JackofOz 14:04, 27 March 2006 (UTC)[reply]
I'd say it's quite certain that they'd remain that way. There already are a number of sects (like the Bahá'í) who've split off as new prophets have turned up. Those aren't usually considered Muslims. And Muslims aren't usually considered Christians, depsite acknowleding Jesus. Neither groups are considered Jews either, although they all agree on Moses, Abraham and friends. Mandaeans acknowledge John the Baptist, but not Jesus or Moses. At any time there's always a bunch of people claiming to be the successor to (insert favorite prophet here). So I don't quite agree with Grutness: In my opinion, a successful scism is more the result of politics than of the existance of a dissident prophet. Martin Luther was successful more because he had powerful friends, less because of his theology, if you compare to the failure of Jan Hus. --BluePlatypus 16:02, 27 March 2006 (UTC)[reply]
"Is there a truly fundamental alteration in theology?" I disagree with Grutness too. I think Muslims couldn't have a split over Mohamed without one party ceasing to be Muslim anymore than I can imagine Christians having a schism over Christ's existence without the deniers ceasing to be Christian. To be Muslim is in large part to acknowledge Mohamed as the final prophet. You don't schism over that; you leave the faith over that. But you may schism over (and I think I'm agreeing with Blue here) whether one Caliphate is to be obeyed, one follower is to be followed, etc. etc.
An interesting question has always occured to me, and is pertinent given the Joseph Smith example: are Mormons Christian? Yes, obviously in that they follow Christ. But no, insofar as Mormonism rather radically introduces a new prophet and interprets Christ through him... That's absolutely not an insult to Mormons BTW! What makes a "new religion"? Somebody post it on the Wiki ref desk? :) Marskell 22:05, 27 March 2006 (UTC)[reply]
Well, the simple answer would be that the LDS church considers itself Christian, and that most other Christian groups don't recognize it as such. A historical attempt to define "Christian" which is still highly regarded is the Nicene Creed. (The LDS's disagreement with it is one of the main reasons most others don't consider them Christian.) --BluePlatypus 16:22, 28 March 2006 (UTC)[reply]
That is apt. A religion is one that considers itself one. For instance, a Baha'i will find any way possible to reiterate that their religion is an independent religion and not a sect of islam (the relationship being roughly similar to the relationship of Christianity to Judaism). This discussion also happened on the Abrahamic religion talk page. It was decided that a religion qualifies as being Abrahamic simply if they consider themselves to. -LambaJan 16:53, 1 April 2006 (UTC)[reply]

Old Theatre in Paris[edit]

I am looking for information about the Theatre De La CiGale in Paris, France. Its' history is what I am interested in. Information would be helpful. Also, any coins they used in the old days for a pass or fare. (Instead of tickets)

A Google search on your terms above turned up 151 references to théâtre de La Cigale, though they seemed to be mostly in French. If you don't read French, but you can specify what information you're looking for, somebody here might be able to help, though we can't do your research for you. --Halcatalyst 21:12, 27 March 2006 (UTC)[reply]

Good advice. But here's some preliminary info. The théâtre de La Cigale is located in the Pigalle in Paris. The name translates to "Theatre of the Cicada (or grasshopper)". It was built in 1887, and has been a theatre, a cabaret, and since 1940 a movie theatre, specializing for a time (the 1970s) in Kung fu movies. It was renovated and reopened in 1987. It sounds like you are looking for historical information to use in writing. What time period were you interested in? [35] [36] If your specific question is, say, how many centimes admission might have been in 1890, you could try writing them at the address on the web-site cited, but I think it's a long-shot. - Nunh-huh 02:09, 28 March 2006 (UTC)[reply]

Hebrews and Phoenecians - Hamites or Semites?[edit]

Ok- from my modicum of familiarity with languages in general and Hebrew and Arabic in specific, my understanding of the Bible and other secular reading on the subject, I have never seen spelled out just what the deal is with languages and ethnicities in the Levant in ancient times. I am half Lebanese and have seen many place names which are pre-Arabic (Canaanite) in their vocabulary there (Kfar____), plus of course almost every town or village there was named before the Arab conquest. eg. Nabatiyeh, Sidon, Tyre, etc

Here's my confusion; as far as I understand... 1)The Phoenecians and pre-Israelite Canaanites were Hamites (Gen.10:15). Canaan was the son of Ham and in turn had these sons: Sidon, Heth, and the Jebusites, Amorites, and Girgashites.

2)But both the Phoenecian and Hebrew (being Canaanitic adopted by Abraham ?)languages are Semitic.

3)Where did the Semitic lang's in this area come from? Are there Hamitic words carried over into Canaanitic/Hebrew, and if not, why not? If so, what are some simple, common Hamitic words that survive? How are they attested to in other Hamitic Languages?

Thank You, Daoud F., Milwaukee, WI, USA(4.252.149.34 07:44, 27 March 2006 (UTC))[reply]

There is tradition, originated in the Bible, saying that people and nations come from three sons of Noah. There is convention nowadays to use a term like Semitic for some languages - cultures - &c. I am not sure that any concordance works ... --DLL 20:12, 27 March 2006 (UTC)[reply]

Have you read Hamitic and Canaanite languages? According to the former, the idea of a Hamitic ethnic/linguistic group is essentially a myth. HenryFlower 00:19, 28 March 2006 (UTC)[reply]

Mona Lisa[edit]

I was just wondering does anyone know the actual price of the mona lisa or is it actually priceless?

In the (unlikely) event that the Louvre decided to sell it, it would go by auction, and the price couldn't be guessed in advance; however, it must have a nominal price for insurance purposes, although I don't know what that would be. I don't think many people would disagree if you called it 'priceless', though. --Squiddy | (squirt ink?) 09:06, 27 March 2006 (UTC)[reply]
Actually it probably won't have a nominal price for insurance. I doubt very much if it is actually insured, most high profile works at major galleries aren't due to the valuation problems, cost involved & the fact that the gallery (if its big & fancy enough) generally feels its at low risk of fire, theft, etc. AllanHainey 11:33, 27 March 2006 (UTC)[reply]
The issue of insurance came up when it was exhibited in the USA in the 1960s. I can't imagine the Louvre permitting it to be sent across the Atlantic without very clear agreements about monetary compensation in the event of damage or destruction. JackofOz 11:50, 27 March 2006 (UTC)[reply]

The original question has been answered, accurately in my opinion. I wouldn't be surprised if it's not insured while it is at the Louvre, since insurance is just one tool in risk management, and they probably feel the money is better spent on preventing theft, fire, etc. And it's not like the 100 million or whatever it would sell for could replace it, so there's not much point in insuring it for that. For moving, you're right it was probably insured, but if not, I agree there certainly was an agreement for compensation. Since it was for a relatively short time, and was likely profitable to exhibit it here, the insurance was probably feasible. - Taxman Talk 16:47, 27 March 2006 (UTC)[reply]

As with most irreplacable items, it is not insured for replacement. It is insured for loss. For the Louvre, it will be insured for expected monetary loss due to reduced revenue and investigation costs. --Kainaw (talk) 18:04, 27 March 2006 (UTC)[reply]

glass slippers[edit]

could cinderella have really worn glass slippers ? if not or if yes, please write a scientific explanation

thank you

Cinderella is a fictional character in a fairy tale, so of course she couldn't really have done anything. In science we posit that fictional characters do not exist; this is widely accepted. Also, "glass slipper" is generally considered to be a translation error for "fur slipper", or a later poetic invention in retelling the tale. Notinasnaid 11:40, 27 March 2006 (UTC)[reply]
This discusses the issue at some length. JackofOz 11:43, 27 March 2006 (UTC)[reply]

i have read that article. it proves that glass slippers is not a mistranslation for furslippers.

the main answer i want is , weather if cinderella ever existed, could she have been able to walk/dance/run with them...... or maybe it can be can anyone dance/run/walk with glass slippers without breaking them or damaing them etc..pls can i have scientific explanation /reasoning

Assuming the glass is unbreakable, it would probably be like wearing wooden shoes because both substances are unbending. Seems uncomfortable but people did. alteripse 12:22, 27 March 2006 (UTC)[reply]

can i have a better explanation pls ??--Pearline 12:27, 27 March 2006 (UTC)[reply]


Glass is a very strong material. Glass is prone to shatter with a strong impact, but glass can readily be made that withstands likely impacts. It's used to make floors that many people can walk on. So there is no reason it couldn't be used to make footwear except possibly it may be uncomfortable or a little heavy. But not too heavy, and probably no more difficult to walk in than very high heels. A google search for "real glass slippers" suggests that people have indeed made them, and many people seek them for a fairytale wedding, but the details are tantalisingly brief. You'll need to make your own decisions about how you present this when you write up your homework. Notinasnaid 12:29, 27 March 2006 (UTC)[reply]

thank u for all ur help . i really appreciate it . i will not ask anymore questions on this cinderella's glass slippers issue.thank you for ur time spent to help me.

I daresay if it is possible to turn rags into a gown and a pumpkin into a carriage, then glass slippers that are comfortable are possible. Looking to science is not right for a fairy tale. Bippity Boppity Boo! Esquizombi 23:45, 27 March 2006 (UTC) "could Cinderella have really worn glass slippers?" As she only wore them for one night I doubt if they were very worn,depends how much of the night she boogied away with the Prince before midnight.hotclaws**==[reply]

Liability[edit]

If I were to have a store that sold devices that purpose were to cheat on exams what is my liability? My idea is a data communication pen that can transmit and recieve ABCDE silently. Would I break any laws assuming I called them something like "spy pens" or something that wasn't stating its true purpose.

In the US, you would have no liability whatsoever. Cheating on a test is not illegal in a legal context. If, however, you skipped over all the FCC regulations concerning electronic devices, you'd have a problem. — Lomn Talk 16:50, 27 March 2006 (UTC)[reply]
This is not necessarily true. A professor's test is copyrighted. Receiving that copyrighted information and disseminating it again would run afoul of copyright laws. Generally, those who used the service would be the ones breaking all sorts of laws, but hosting copyrighted material can get you sued, especially if you set out to do so. In the case of Napster, they argued that their service was not intended primarily to house copyrighted material, that some people used it that way, but that such was not their intent. On the other hand, setting up specifically to grab test questions would probably be tough to explain any other way. Geogre 17:16, 27 March 2006 (UTC)[reply]
There also seems to be a (large) distinction in broadcasting a test question (which can be copyrighted) and broadcasting a suspected answer (which is the impression I get from the original question). Multiple-choice A-E would not, I think, be copyrightable. — Lomn Talk 19:48, 27 March 2006 (UTC)[reply]
I don't think the copyright question would come up, anymore than taking notes and passing them around to others in class would be a copyright violation. Assuming you got into court, it would certainly be "fair use", and even if not, the professor would have no claim to a loss of funds in such an instance, substantially reducing any incentive. --Fastfission 19:59, 27 March 2006 (UTC)[reply]
It wouldn't be hard to make up another use and claim that is what it is for. For example, rolling papers are for legal tobacco, not marijuana, right? Those 6-inch long vibrating things are muscle massagers, not sexual aids, right? Magazines showing kids in their underwear are product catalogs, not child porn, right? --Kainaw (talk) 17:59, 27 March 2006 (UTC)[reply]
Actually, if you took my test, you would be robbing me of my intellectual property. It's my metric for assessment in a class, and I not only cooked up the language, but I had a design, too. I may have cleverly asked about material from week 1, then week 6, then back to week 2, intentionally preventing list memorization from helping. There is absolutely copyright adherent to that. A list of answers, if it's just multiple choice, would be the answer key, and I'm not sure what applies there. However, any short answer or essay is another matter again. As for class notes, those are your property, as they represent your digest of what was said by me. If you tape my class, though, and try to pass that around, I can sue, again. (There are some companies and some profs who make small fortunes selling tapes of their lectures. ("Get an Ivy League education at home! Hear the Great Thinkers!")) As for a professor who uses only objective testing.... He or she is already a sitting duck. Geogre 21:11, 27 March 2006 (UTC)[reply]
  • If I took a set of professor's tests and tried to sell them as "My super-duper collection of practice tests," yes, I'd definitely be violating copyright. But simply passing around the answers in an ad hoc way -- whether by notes or magic pens -- is going to be well within the bounds of fair use, and in any case is not going to be looked at as a copyright problem for the developer of the magic pens. --Fastfission 21:48, 27 March 2006 (UTC)[reply]

Complicated music[edit]

What is the most complex piece of piano music ever composed? Despite Liszts "Hungarian Rhapsodies". I would appreciate any suggestions. -Funper 20:29, 27 March 2006 (UTC)[reply]

There is no way to answer this question. Many different people have different diffuculties. SOmebody may have trouble with rhythems, while other can't read a piece in the key of concert G. A beginner may feel Mary Had a Little Lamb is the most difficult piece ever. schyler 23:59, 27 March 2006 (UTC)[reply]
Specifically, complexity is difficult to measure. "Most instructions per phrase" might be a metric, but that's not necessarily accurate. For example, Philip Glass's music is "simple" melodically, but it is rhythmically very difficult, and yet the number of instructions is minimal. On the other hand, some of John Cage's Music for a Prepared Piano is devillishly complex in the number of instructions, but these instructions are really directions for the replication of a particular artifact. At a certain point, complexity becomes an impediment to aesthetics as well as performance, and therefore the most complex music ever written probably isn't performed. Thus, it seems like you're asking for "a list of highly regulated musical pieces that yet are pleasing enough to be performed." You're probably on the right track with Listz, as it would come from the virtuoso age of the late Romantic movement, probably, but it could go as late as Rachmaninoff or Prokofiev or Ives. (It ain't Ed Cobb and The Standells or The Shaggs, I promise.) Geogre 00:23, 28 March 2006 (UTC)[reply]
I've seen the sheet music for both Hungarian Rhapsodies and the piano duet arrangement of Holst's "Mars" from the Planets Suite. Liszt is easy. Grutness...wha? 00:33, 28 March 2006 (UTC)[reply]
  • Both? He wrote 19 Hungarian Rhapsodies. I assume you meant "relatively easy" compared with the Holst - Liszt is never easy.
    • wrong meaning of "both". Let me rephrase: "I've seen the sheet music for both the piano duet arrangement of Holst's "Mars" from the Planets Suite and the Hungarian Rhapsodies..." And yes, only relatively easy. Another toughie would be a piano arrangement of Falla's Ritual Fire Dance, but - as with the Holst - that wasn't primarily written for piano. Grutness...wha? 13:23, 28 March 2006 (UTC)[reply]
  • It used to be said that Scarbo from Ravel's "Gaspard de la Nuit" was the most difficult and complex piano piece ever written, but that's probably been superseded by Sorabji's Opus Clavicembalisticum.
  • The question was titled "complicated" music, but it actually asked about "complex" music. The two things are not the same. Some of Schumann's notation is needlessly complicated, but the inherent structure of his music is not particularly complex, compared with the likes of Scriabin or Godowsky. JackofOz 02:21, 28 March 2006 (UTC)[reply]
From what I've learned in school: Frédéric Chopin's Minute Waltz is supposed to be played within one minute, but practically nobody except for Chopin himself had managed to play it that fast. Have checked with the article, it does not say that in it; may very well not be entirely true, am just volunteering information. --Ouro 16:04, 31 March 2006 (UTC)[reply]
Chopin has been the subject of more romantic hyperbole than most composers. A reasonable performance of this piece - one that did not descend into mere vulgar display of technique (something Chopin never did) - would take closer to 2 minutes. The nickname is a misnomer; it leads one to assume the point of the music is how fast you can play it - but this has nothing to do with it. Chopin did not supply the nickname, and stories about him playing it in a minute are simply untrue.  :-) JackofOz 07:00, 1 April 2006 (UTC)[reply]

Names in Faust[edit]

Title added. —Keenan Pepper 22:00, 27 March 2006 (UTC)[reply]

What are the two (2) fools specific, exact names in Faust? There are several retellings of the story, but somewhere along the line I believe that they have names. They were very important ot the story line, and I would like their names for a novel I am working on. Thank you.

Full text of Goethe's version from Project Gutenberg, if that's of any help to you. --Bth 11:09, 28 March 2006 (UTC)[reply]
Whar specific scene are you thinking of? I would guess it's the one in Auerbach's cellar. There are several 'fools' there, though: Frosch, Brander, Siebel and Altmayer spring to mind. The inn is still there, so take a trip to Leipzig and do a bit of research there :) Best of luck! Asav 19:35, 29 March 2006 (UTC)[reply]

I was thinking of the two (2) fools who conjure up the devil just as well as the learned Dr. Faust. Thank you for your help.

What's it called? Not prohibitionism?[edit]

I was wondering if somebody remembered the political terms for the expression "anything not forbidden is allowed" and "anything not permitted is forbidden" in respect to law (the basic difference being that the executive can do whatever it pleases in the first case if there's no law against it and can't do anything unless it's explicitly told to in the second). I feel like there are terms for these two different approaches but I can't remember them at all. Any pointers? Thanks so much! sebmol 21:59, 27 March 2006 (UTC)[reply]

I don't know if you are in the USA but if you are that would probably fall under the ninth amendment. It protects the rights not speciffically enumerated in the BIll of RIghts. schyler 23:21, 27 March 2006 (UTC)[reply]
I don't know the answer but I know of an example of a law that has changed from the first case to the second case. The Australian road rules on U-turns were changed a few years ago. Previously, a U-turn was permitted unless there was a "No U-Turn" sign. Now, U-turns are forbidden except where there's a "U-Turn Permitted" sign. JackofOz 02:04, 28 March 2006 (UTC)[reply]
Googling around (and ignoring all the stuff about parsing input that just refers to the principles longhand), I see the first referred to as a Common Law (ie UK and related polities) concept and the second as a Napoleonic Code (Revolutionary France and related polities) concept, but, erm, not by people whose POV I trust. --Bth 11:04, 28 March 2006 (UTC)[reply]
Actually, the concept of "everything if allowed unless prohibited by law" is a French revolutionary concept. See Declaration of the Rights of Man and of the Citizen.
5. Law can only prohibit such actions as are hurtful to society. Nothing may be prevented which is not forbidden by law, and no one may be forced to do anything not provided for by law.
Note that the Napoleonic Code establishes that law cannot prohibit things retroactively. It may be validly argued that judge-made law, as in traditional Common Law countries, amounts to establishing laws after the case (thus, allowing people to be convicted for things that were not strictly establish as crimes before the case occured), though this is largely an academic and provocative consideration... David.Monniaux 20:28, 28 March 2006 (UTC)[reply]

"In my state you can turn right on red unless there is a NO TURN ON RED sign posted. So, it seems reasonable to assume you can also go straight thru a red light, unless there is a NO STRAIGHT ON RED sign." StuRat 03:25, 31 March 2006 (UTC)[reply]

Petrarch[edit]

Italian literature, originally a 1911 article, claims that Petrarch's political poems "are remarkable for their vigour of feeling, and also for showing that Petrarch had formed the idea of Italianita better even than Alighieri." My question - what is Italianita? (This might be better for the language desk, not sure.) zafiroblue05 | Talk 22:02, 27 March 2006 (UTC)[reply]

Italianità- Italianness. Don't ask me to define Italianness. ;) HenryFlower 22:17, 27 March 2006 (UTC)[reply]
Another example of why all 1911 articles need professional attention and why pasting them into Wikipedia isn't very helpful. (Petrarch knows what Italianness is better than Dante? Uhhh. Well, since Italy is, for both of them, a collection of states and not a single one, and for both of them peninsular nationality is still a dream, that dream's ability to reflect what nationalists would later say was true is a dubious achievement and not well measured. Isn't it enough to say that they're fine political poems?) Geogre 22:40, 27 March 2006 (UTC)[reply]
Petrarch & Dante Alighieri would think of the subject in a literary and cultural mood. The latter talked about the language of "Si" (now Italian), amongst other Latin-derived ones, "Oïl" (Northern France French) and "Oc" (Occitan), the three words meaning "yes" ; he was the first to publish in the vulgar (Italian). --DLL 20:19, 28 March 2006 (UTC)[reply]

Tarikah Es-Soudan[edit]

While reading the book, "The miseducation of the Negro, by Carter G. Woodson, it metions, Tarikah Es-Soudan on page 19. I tried to look up the term for clarity, but could find nothing on the topic. Is there anyone who can tell me what is meant by this saying?

Possibly a Tariqah of the Sudan? What is the context? Esquizombi 22:22, 27 March 2006 (UTC)[reply]
There's also a book titled Tarikh es-Soudan (probably a different transliteration for the same phrase) [37] Esquizombi 22:26, 27 March 2006 (UTC)[reply]
Tarikh means "history", as in History of the Prophets and Kings (book). AnonMoos 21:37, 28 March 2006 (UTC)[reply]

"From literature of the African was excluded altogether. He was not supposed to have expressed any thought worth knowing. The philosophy in the African Proverbs and in the rich folklore of that continent was ignored to give preference to that developed on distant shores of the Mediterranean. Most misionary teachers of the freedmen, like most men of our time, had never read the interesting books of travel in Africa, and had never heard of th tarikh Es-Soudan"

Is there an English version?

It seems there was a French translation by Octave Houdas & Edmond Benoist, not sure about English. There's a mention of the book here too The Negro, by W.E.B. Du Bois: VIII. African Culture where the title is translated as Epic of the Sudan. It's mentioned at African_Jew#Igbo_.28Ibo.29_Jews so maybe try posting on the talk page for that article for more info or look in the history and see who added it and post on their page. It's referred to there as a record, but I'm guessing that's in the broader sense of the word as a document. Esquizombi 23:06, 27 March 2006 (UTC)[reply]

It looks like you can find an English translation in the Timbuktu and the Songhay Empire: Al-Sa'di's Ta'rikh Al-Sudan Down to 1613 and Other Contemporary Documents, edited by John Hunwick. (Amazon link). --Cam 04:08, 29 March 2006 (UTC)[reply]

Divorced in Brazil[edit]

Is divorce in Brazil now legally allowed? When I lived in Brazil in the 1960s it was not. One might be able to get an annulment but other than that, legal divorce was not allowed.

Thank you in advance for your answer.

  • Looks like divorce following three years legal separation was legalized in 1977 or so. --jpgordon∇∆∇∆ 06:03, 28 March 2006 (UTC)[reply]

People in Pakistan[edit]

I have been looking everywhere for this information and I couldn't find it. Does anyone know what children in Pakistan call their parents?

I would appreciate it if someone could answer this question. —Preceding unsigned comment added by MeghanN2 (talkcontribs)

The two dominant languages there are Urdu and English. In English, they use "mother" and "father". In Urdu, they use "Ammi" and "Baap". As with English usage of "mom", "ma", "pop", "dad", "pa", etc... I'm sure there are many other ways of saying mother and father in Urdu. --Kainaw (talk) 00:35, 28 March 2006 (UTC)[reply]
Also Ummi and Abu I think. Esquizombi 03:12, 28 March 2006 (UTC)[reply]
My sources for the Urdu give abbaa for father and ammaa.N for mother (in ITRANS transliteration, I don't know the Urdu script). Baap is very informal and I have heard one wouldn't address one's father using that unless it was a particularly informal household. Also keep in mind there are many languages spoken in Pakistan besides Urdu, and what children would call their parents will sometimes vary based on the mother tongue spoken at home and the ethnic group. Panjabi's are likely to use the Panjabi word even if they speak Urdu in public. See our article on Pakistan and each of the articles for the languages spoken there. Then try looking for the words in each language, or asking Wikipedians that speak those languages. Unfortunately our Wiktionary articles on mother and father don't give the translations for any of the languages in Pakistan, though you could ask there for translations. - Taxman Talk 15:27, 29 March 2006 (UTC)[reply]

For father Abboo and for mother Ammi are the common usages. Other words as Baboo, Abbaa Jan, Abbaa Ji are also used people speaking Pashto may use the above mentioned terms or use DaDA(D as the Persian word Dal)and Mor or Mor-ay for mother also Da'ji for father and A'bii for mother. BABA is a common word used for grandfathers.(WAJID ALI)

March 28[edit]

Median Family Income[edit]

I am interested in the United States' epicenters of wealth. I would like to know if there is a subject on wikipedia where you can see a ranking of American cities' median family income, ranked highest and lowest.

I do know from doing other internet research that Scarsdale, New York has the highest median family income in the United States at over $200,000. I have found that you can go to the individual city pages and find median family income for a specific place, but I was wondering if there is a certain subject where these cities were ranked in order. Can you please help me?

"Per Capita income" is a subject on wikipedia, but you can only find these statistics listed on an international level, by country.

Thanks,

AUTiger789

Cant find a full list on here. Try searching google or the census site for "median household income". But bear in mind that comparisons can be misleading depending on how the local city/county boundaries are drawn. If a city is large in area rich and poor districts will be within the same city. In other cases the the wealthier outlying areas may be administratively separate from the inner city. It might be better to search by metropolitan areas, not cities, if you want to iron out those anomalies. Jameswilson 01:32, 28 March 2006 (UTC) yes[reply]
According to the U.S. Census Bureau, the wealthiest county of at least 250,000 people in 2004 was Fairfax County, Virginia, with a median household income (not per-capita income) of $88,133. The wealthiest city of at least 250,000 people was, by far, San Jose, California at $71,765. (The poorest? Miami, Florida.) PDF file here.
And have you seen this page? No date, unfortunately -- Mwalcoff 04:06, 28 March 2006 (UTC)[reply]

Total Rapes In Vietnam War[edit]

what is the total rapes in the vietnam war, can be an estiment.. I wasn't aware that the rape seed crop for oil was much used in Vietnam and anyway lots of it would have been destroyed during the fighting by napalm,willy-peter and Agent Orange?I assume you mean the police action by the USA in Vietnam but they've had a lot of wars there and kicked serious butt every time.hotclaws**==

Internalized Oppression?[edit]

Hi, I'm writing a psychoanalytical paper on a character from The Bluest Eye by Toni Morrison. I need help finding an article on anything having to do with social oppression, or anything that can explain how society or a community as a whole can be to blame for the upbringing of children in those communities. Basically, I need an article to support my thesis which is a little girl goes crazy because the town and everyone who believes she is crazy, therfore in the end she believes it to.

I would greatly appreciate anyone who can help me find an article about this.

--69.110.11.20 04:11, 28 March 2006 (UTC)[reply]

Interesting. Are the references in Social_psychology of any use?--inksT 05:09, 28 March 2006 (UTC)[reply]
Self fulfilling prophecy? Whether the link is blue or not, there has been a great deal of research on the power of self-fulfilling prophecies and the power of expectations of success or failure, and the social psychology research should be fairly plentiful. Geogre 12:31, 28 March 2006 (UTC)[reply]

many thanks for all the help.

Nazi Images[edit]

Would images taken by the Nazi government be subject copyright laws as far as Wikipedia is concerned? Captain Jackson 05:25, 28 March 2006 (UTC) Who would hold copy right?[reply]

Opinions differ. The government of Bavaria, for example, claims copyright on Mein Kampf (that, of course, started out as Hitler's copyright and not the Nazi government's). Realistically, Nazi government posters and documents have no copyrights that civilized people are obliged to honor, and not using them because of fear of copyright infringement is "copyright paranoia". - Nunh-huh 06:59, 28 March 2006 (UTC)[reply]


If you want to read a long discussion both ways on it, look at Template_talk:PD-Germany. There is nothing that makes the Nazi copyrights fall into public domain automatically, but just the same, as Nunh-huh has said, I'm not sure anybody has tried to enforce them. --Fastfission 21:39, 28 March 2006 (UTC)[reply]
Given that the current (reunified) Germany seems to be considered the legal successor to Nazi Germany (for the sake of restitution payments, etc), it would seem to me that they probably own the copyrights on any government material from that period which is still copyrighted. I doubt very much they'd enforce those copyrights though, unless the material was used in ways they didn't like (read: for antisemitic purposes). --BluePlatypus 22:58, 28 March 2006 (UTC)[reply]

many thanks to the responses.

Reichsmarks[edit]

My history professor said that at the end of World War II, the Soviet Union demanded that Germany pay them 100,000,000 reichsmarks in reparations. How could these be worth anything if the government which issued them no longer existed? Captain Jackson 05:27, 28 March 2006 (UTC)[reply]

According to the German Constitutional Court, Germany as an entity continued to exist following World War II (and has been in continued existence since the end of the Franco-Prussian War 1871). If the Soviet Union asked for reparations in Reichsmarks, it most likely also specified what exchange rate it was talking about. So, 100,000,000 Reichsmark could definitely be worth something even though there wasn't much in the way of a national currency between 1945 and 1948 (the year the Deutschmark was created). sebmol 05:42, 28 March 2006 (UTC)[reply]

Antique war time brooch from Scotland?[edit]

I have acquired a (pewter?) brooch which is mostly circular, the outine being the image of a belt, and within the belt is a fire, and around the border in the belt is inscribed "Stand Fast". On the back of the brooch is nothing but "made in gt.britain". I should think it is a wartime brooch from the UK, sounds very Scots to me. I am eager to learn where it is from and can find nothing on the net so far...

http://www.clangrant-us.org/symbols.htm. Clan badges are worn all the time, and up to date, so there is no particular reason (from what you describe) to assume any particular war. I don't think it would be worn in to war, there would be regimental badges for that. Notinasnaid 12:02, 28 March 2006 (UTC)[reply]
It does sound a lot like the clan badge of Clan Grant. They are quite common, and are still made today (often as souvenirs of Scotland). The value is unlikely to be much unless you can date it somehow - dating it might also help you find out more information about its history, of course. Grutness...wha? 13:30, 28 March 2006 (UTC)[reply]

The Pro Life Alliance[edit]

Does anybody have any specific information on this group? Their website is not helpful.

I would like to know:

- names of certain members, I already have Dominica Roberts as current Leader - (is this correct?) and Julia Millington as Political Director. If any more are known I would be grateful.

- are there any famous supporters of this group?

- Any information on their history. All I know is they were formed in October 1996.

- Their succes in past elections, I have some information on his but only for 2004.

All other information would be gratefully appreciated.

Thank you

Natasha

A Google search on "prolife alliance" turned up more than 32,000 references. That would be a good place to start. --Halcatalyst 23:40, 29 March 2006 (UTC)[reply]

That was the first place i tried, and whilst providing much of my information, it did not bring up answers to the above questions. hence the reason i am asking them.

Performer on the Ed Sullivan show[edit]

What is the name of the performer who had the act of spinning plates on the Ed Sullivan show?

  • It's pretty amazing; it seems to be an act everybody is familiar with -- tons of references and metaphors abound -- but I've not been able to actually identify the spinner (or spinners). See plate spinning, by the way. --jpgordon∇∆∇∆ 19:07, 28 March 2006 (UTC)[reply]

Penal Law in UK[edit]

Hello, in Switzerland an in Germany, you have a so-called "Strafgesetzbuch", a law/code that includes all crimes and their punishment. Being curious I googled (and searched Wikipedia) for a UK equivalent, but was unable to find one. I got the impression that their exist rather different laws, but I'm unsure which was is a) valid at the moment (in Wikipedia Articles you find a lot of historic information) and b) where I find the most "usual" crimes, e.g. libel/murder/assault/theft and so on). Further on, where can these laws be found? In CH and DE, there are all law texts on the internet at the webpages of the government, I failed in looking for something similar again. Thanks a lot for your answer! --Mbimmler 16:24, 28 March 2006 (UTC) (de:User:Mbimmler)[reply]

  • I believe that Switzerland and Germany are civil law systems, whereas the UK (generally) uses the common law system. As such, there is no single legal code. Shimgray | talk | 16:34, 28 March 2006 (UTC)[reply]
Ah, I see. So as far as I understand it, in UK a judge which has to deal with assault would rather look at precedent cases than at (as in CH) a code which tells him "For assault the punishment is no less then 2 years of prison and not more then 10 years of prison"? --Mbimmler 16:49, 28 March 2006 (UTC)[reply]
Well, yes and no. For most crimes, there's some relevant legislation; in the case of assault, it's the Criminal Justice Act 1988, where s.39 tells us that "Common assault and battery shall be summary offences and a person guilty of either of them shall be liable to a fine not exceeding level 5 on the standard scale, to imprisonment for a term not exceeding six months, or to both." [42] But what actually constitutes "common assault and battery" is likely to be controlled by precedent, rather than by a law; see, say, [43]. Shimgray | talk | 18:52, 28 March 2006 (UTC)[reply]
Actually, come to think of it, the CPS guidance site is pretty much what you're looking for anyway... Shimgray | talk | 18:53, 28 March 2006 (UTC)[reply]
  • (I'm not a lawyer but I think the difference is this)
  1. Precedents are to determine what is or isnt legal (when the law is unclear). In Shimgray's example, does "hitting someone with a lettuce" constitute common assault or not? Also precedents are binding for procedure (what evidence is admissible, etc). The judge has to follow what was decided when the same doubt occurred before.
I think we need a wikipedia article on Assult with Vegetables :-) --WhiteDragon 20:29, 4 April 2006 (UTC)[reply]
Perhaps User:ProhibitOnions can become a leader in the vegetable control field... Joe 20:35, 4 April 2006 (UTC)[reply]
  1. But sentencing is a different thing. There the judge or magistrate has much more freedom. He doesnt need to follow previous cases. As Shimgray says, the law about common assault was last changed in 1988 and that text includes the maximum punishment - 6 months. Below that maximum, he would look at the sentencing guidelines, which are updated more frequently. But they are only guidelines, not rules. So the punishment given can vary widely, especially in Magistrates' Courts, which handle minor offences (max sentence one year). Criminal A might get a small fine, whilst identical criminal B gets sent to gaol. Jameswilson 23:28, 28 March 2006 (UTC)[reply]
I am not a lawyer either but I worked for many years as a court administrator in Scotland, which until 1707 was a separate country in every respect from England, Ireland, and Wales, despite having a common Monarch, the throne of England and Scotland being occupied by James VI of Scotland on the death in 1603 of Queen Elizabeth 1 of England (Union of the Crowns) - note, there never was a Queen Elizabeth the First of Scotland so the present Queen Elizabeth is the second of Great Britain but only the first of Scotland. Confused? Hardly surprising. When in 1707 in the reign of Queen Anne, the respective parliaments of England and Scotland united to become the Parliament of Great Britain (Act of Union), based in Westminster with Members of Parliament from both countries, the Scottish Parliament ceased to exist, their Parliament Building becoming the permanent home of the Scottish Supreme Court (The civil high court is called The Court of Session and the criminal high court is called The High Court of Justiciary). But enshrined in The Act of Union was Scotland's continuing independence from England in its Religion, Education, and Law. Hence, The Queen is not the head of the Scottish Church whereas she is the head and supreme governor of the Church of England (Scotland believed that for the Scottish Church, God fulfilled that role quite effectively). So from 1707 onwards Scottish Law remained much the same as before with most of its criminal jurisdiction being performed by Magistrates in the police and district courts at the lower tariff level; in the Sheriff Courts in the middle tariff level, and in The High Court of Justiciary for retained crimes of Murder, Rape, Treason etc. Scotland also retained its 3 verdicts of Guilty, Not Guilty and Not Proven (still in use today). And there are still 15 people on a jury with a simple majority of 8 casting the same vote determining the outcome of a criminal trial. But most of British Law then and now was and is based on Common Law, which essentially holds that any reasonable and God-fearing person would know that it is against God's Law to commit murder, or rape a (specifically) woman, without the need for Parliament to write it into a Statute Book. I always found it strange that there is no statutory crime of Murder, but the Common Law charge of murder is sufficient to bring a suspected perpetrator to trial. It is interesting perhaps to note that even today, the Supreme Criminal Court in Scotland (the Court of Appeal) resides in Edinburgh - there is no further appeal to either the English Courts or to the UK Parliament. However, since 1707, much of the civil law of both jurisdictions (England and Scotland) has been brought ever closer and as such, a decision in The Court of Session Inner House (Scotland's Highest Civil Appeal Court) can be, and often is, appealed even further to the House of Lords in its judicial capacity. So in summary, the previous respondent was correct in stating that a mix of Common and Statute Law is used by UK Courts but it is important to make the distinction that there is no single UK statute book (list of crimes) applicable to both countries (jurisdictions). Another confusing issue is the way in which statutory crimes and prescribed punishments are developed according to, say, new technology and experience. In most such circumstances, the new law will be written into, or as an extension of an already extended law. There are many practitioners and observers of UK law who would prefer to see a constantly updated and current codified statute book, but the counter arguments suggest it would be too mammoth a task to create and maintain and anyway, any person who needs advice on the law can always consult his friendly, expert and expensive lawyer. As one famous Judge said, the only person who really needs to be an expert in the law is the man in the street, as only then can he be absolutely certain in his actions that he is not breaking some law or other.

Astronomy[edit]

What sort of fields were most popular for amateur/professional astronomers to research in the 18th century?

Transit of Venus was huge. They were also very interested for the longest time in the longitude, which they attempted to divine by a lunar method. Amateurs and professionals alike worked at this. Then there was the continuing work on sun spots. They were also extremely interested in figuring out comets, especially after Halley. Geogre 21:04, 28 March 2006 (UTC)[reply]
Though not entirely the same field, ISTR several astronomers were also involved in the attempts to create accurate clocks for use on board ship (which culminated in John Harrison's chronometer. Grutness...wha? 02:01, 29 March 2006 (UTC)[reply]
I don't mean to nitpick, Grutness, if they were astronomers, they were probably trying more to determine longitude by astronomical methods (calculating the orbit of the Moon was one popular way, another was by timing the Galilean satellites of Jupiter) than by actually building something as mundane and earthly as a clock... — QuantumEleven | (talk) 14:16, 30 March 2006 (UTC)[reply]
Well, that's my point exactly. They weren't making clocks per se, but they were involved in the testing of them, by using comparison with astronomical methods, and were also probably attempting to come up with astronomical methods that would be easier for seafarers to use so as to negate the necessity of clocks. Timekeping at sea was an enormous problem in the 18th century, and scientists from many different disciplines were involved in it. Grutness...wha? 00:35, 31 March 2006 (UTC)[reply]
The Titus-Bode law was of interest after the discovery of Uranus in 1781. Finding the next planet in line was one area of activity. Marskell 14:17, 3 April 2006 (UTC)[reply]
(It's the Titius-Bode law Ronstew 20:22, 14 October 2007 (UTC))[reply]

What is the difference between a U.S. citizen and a national?[edit]

Dear Wikipedia,

Under the definition or meaning of the "REAL ID Act" it says under the section of "Documentation required before issuing a license or ID card" that the applicant has to provide "Documentation showing that the applicant is legally present in the US (in other words, is a US citizen or national, is an alien with permanent or temporary residence status or a valid visa, has applied for or been granted asylum, is a refugee, etc.)." It states, "is a US citizen or national", could you please answer what is the difference between a US citizen and a national? I have copied the info exactly from your definition of REAL ID Act below. I am new to Wikipedia and not sure how to get my answer back from you guys but would certainly appreciate an answer as soon as possible. Thank you in advance.

Sincerely, Richard Alan

REAL ID Act From Wikipedia, the free encyclopedia (Redirected from REAL ID Act of 2005) Jump to: navigation, search The REAL ID Act of 2005 is Division B of United States Public Law 109-13 (H.R. 1268).

Documentation required before issuing a license or ID card

Documentation showing that the applicant is legally present in the US (in other words, is a US citizen or national, is an alien with permanent or temporary residence status or a valid visa, has applied for or been granted asylum, is a refugee, etc.).

  • Good questions! http://uscis.gov defines a "national" as "A person owing permanent allegiance to a state." What the heck does that mean? Ah, well, I did find another site explaining, "The term “national of the United States” designates a citizen of the United States or a native resident of a possession of the United States." But which possessions are these? --jpgordon∇∆∇∆ 18:55, 28 March 2006 (UTC)[reply]
    That would be anything in our {{United States}} (political divisions of the US) template.

Number of Canadians Living in the US?[edit]

Most recently a column in the globe&mail mentioned there were 2.9 Canadians living abroad. With approximately 1million Canadians living in California, does anyone know where to find data on how many Canadians are living in the USA? OR, simply the number of how many are living there in total? Dozens 19:36, 28 March 2006 (UTC)[reply]

According to the 2000 U.S. census, there were 820,770 Canadian-born people living in the U.S., of whom 378,060 had become nationalized U.S. citizens. Only 141,181 Canadian-born people lived in California. Other states: Florida, 99,139; Michigan, 49,515; Mass., 40,247; N.Y., 54,876; Washington state, 47,568. These numbers are not exact counts but estimates based on sample data. -- Mwalcoff 23:47, 28 March 2006 (UTC)[reply]

iconic or classic lines in popular music[edit]

I'm working on a presentation about popular music, focusing on Bob Dylan. Part of it is that I want to give people a list of classic quotes from songs, and get people to try and guess which ones are by Dylan and which aren't. I can come up with enough iconic lines from Dylan (Desolation Row alone is fertile enough). But I can't possibly find enough gems from other people that are even half clever enough to be confused with Dylan at his best. Without a good supply of classics from other artists, the challenge of sorting Dylan from non-Dylan would be painfully simple. Can anyone help with lines from their favourite songwriters? I'm looking for lyrics that won't look second-rate compared to, say, "Dr Filth, he keeps his world/ Inside of a leather cup/ But all his sexless patients/They are trying to blow it up." (Desolation Row) or "I've been in jail, when all the mail showed./ A man can't give his address out to bad company." (Absolutely Sweet Marie). Sorry for going on about Dylan, but I'm levelling it as a challenge for all followers of other religions. The Mad Echidna 20:23, 28 March 2006 (UTC)[reply]

Here's a few from two bands I've seen in the last year and one that I didn't: "Words that are spoke alone/Phrases you will never hear/Empty rooms and a telephone/That I will never use, never fear"; "You said the things you did in the past/Were all because you're living too fast/But now I'm here you're taking it slow/You know what you know you know you know now"; "Avalanche is sullen and too thin/She starves herself to rid herself of sin/And the kick is so divine when she sees bones beneath her skin". Notinasnaid 20:45, 28 March 2006 (UTC)[reply]
Mouse and the Trapps and Leonard Cohen were both accused of being the "next Dylan." Also, for that matter, Steve Forbert, with his "Jackrabbit Slim" record, was Dylanesque. If you want to really trip them, though, grab lines from Bruce Springsteen's "Greetings from Asbury Park." He was in major Dylan mode then, so, if the audience isn't familiar with Springsteen, they'll miss every time ("Blinded by the Light" is pure "Desolation Row"). Geogre 21:07, 28 March 2006 (UTC)[reply]
I'm a huge fan of Robyn Hitchcock, who himself is a dedicated Dylan fan. Some of his less whimsical and more poetic lines are very Dylanesque (an example: I'll reach your lungs like smoke in the orchard, scattered in bushes, the firemen laughing, from "Cynthia Mask"). Also the 1980s Australian band The Church (best known for singles"Under the milky way" and "Unguarded moment") had a lot of Dylanesque lyrics (an example again: Anna comes and goes in shadows/paintings of the open windows/her photograph is always faded/her sister's eyes are blank and shaded/Don't you understand her science/Merging in a strange alliance to her? from "Sisters"). Grutness...wha? 02:18, 29 March 2006 (UTC)[reply]
For cleverness, Richard Thompson trumps Dylan. Indeed, his best stuff is too good to be mistaken for Dylan. <ducks> But pick the right lyric, and the two might be confused, e.g. You can be a gambler/Who never drew a hand/You can be a sailor/Who never left dry land/You can be Lord Jesus/All the world will understand ("Down Where the Drunkards Roll"); or I gave my youth to king and country/But what's my country done for me/but sentenced me to misery ("Al Bowlly's in Heaven"). --Kevin Myers | (complaint dept.) 06:50, 29 March 2006 (UTC)[reply]
Joni Mitchell , Patti Smith or make it up yourself!hotclaws**==
Even as Hotclaws's comments tend toward the incomprehensible, this one is rather interesting. Your prospective competition might be interesting if you included lyrics you wrote yourself in the style of Dylan and then queried participants as to whether the lines were Dylan's or yours. Of course, for many readily recognized lyrics, this wouldn't work, but surely you could essay the finding of several esoteric lyrics, with which then to compare your own (à la, for instance, Balderdash). Joe 21:20, 30 March 2006 (UTC)[reply]
Of course there are certain Simon and Garfunkel (or even Paul Simon) lyrics that approach Dylan's as poetry and icon, but those lines are surely too readily known to be confused with Dylan's. From amongst modern artists, I think certain phrases of the Counting Crows are Dylanesque, especially in the song Mrs. Potter's Lullaby, for which the lyrics are available here. Joe 06:10, 30 March 2006 (UTC)[reply]
Not all of Paul Simon's stuff is so well known that it can't be used - slip in a few lines from the beginning of "Papa Hobo" or "Overs" and you'd probably fool some of the people all of the time (not to be confused, of course, with "Most of the time", which is Bawb himself) (At the risk of blowing my own trumpet, I've written songs in deliberately Dylanesque style myself. If anyone wants to see the resulting lyrics, contact me on my user talk page) Grutness...wha? 00:51, 31 March 2006 (UTC)[reply]


Thanks for the advice: all very helpful, though if people are still reading this, I need a few more direct quotes, because then I've got the leads for checking out the best ones (checking all is painfully slow). I think I will try the one about making it up myself. I came up with one a while back, when I was first getting into Dylan, and in fact I was listening to Desolation Row, if memory serves me rightly. Part of it went: "Then I showed them all my painting, and some of them started to cough/ They said it wasn't artistic cause - it didn't piss them off." Kind of a lesser imitation I feel, but some people may be thrown. Feel free to add your own in snippets, and I'll contact for the full version if it's up my alley. Tally-ho, signed The Mad Echidna (not logged in coz no time to do so and must get to bed straight away).

Fred Small isn't particularly Dylanesque, but I always thought his song "No Limit" showed more Dylan influence than most of his work:
I asked my brothers, they said we have wanted your embrace
Your heart has been a fortress, your words a state of siege
We will gather in the harvest with the strength of honest men
We will speak in awkward wonder, we will weep, we will begin.
JamesMLane t c 21:05, 2 April 2006 (UTC)[reply]

Protests[edit]

Today and yesterday there were protests outside my school and other schools in the metroplex (I live in Bedford, Texas). The kids seem to be protesting some new immigration law. What exactly does this law include and why would the kids (mostly of latin american ancestory) be protesting it. schyler 00:15, 29 March 2006 (UTC)[reply]

It's a bill sponsored by Senator Kyle of Arizona. It would, if passed and signed, make all presently illegal aliens into felons and make a felony any act of aiding the arrival of illegal aliens. The latter would, for example, make priest and ministers felons who leave water and food out in the deserts to try to prevent illegals from dying of exposure. Another bill, though not receiving the same chances as Kyle's, would construct a 700 mile long fence across the Mexican border. These two bills are causing significant protest by those who have descended from recent Latino immigrants, including a recreation, specifically, of the Los Angeles school walkouts. The Senate is debating immigration reform. There are less punitive bills as well -- one creating a guest worker status where aliens would need to go back to their nation of origin and register and then return and one creating a guest worker status for aliens presently here without their going home. Both of these are given less chance of passage than the harsh and punitive measures. The current President favors a guest worker status, but he has been very vague otherwise, and so the Republican Party is not getting led very powerfully away from the punitive measures, and this leads those representatives of border areas to answer to hyped up fears of "terrorists" and "illegals taking our jobs" by pushing solely for harsh laws. The protestors have every reason to try to remind senators that there are people involved, although it's likely that the areas seeing significant protests are already represented by a pro-guest worker representative, so the political calculus may not help. Geogre 02:47, 29 March 2006 (UTC)[reply]
The most controversial part is, as Geogre noted, the part which would make those who aid illegal aliens into felons, which apparently also extends to giving medical aid. --Fastfission 03:00, 29 March 2006 (UTC)[reply]

removed due to parents seeing schyler 03:18, 29 March 2006 (UTC)[reply]

Immigrant helpers encounter the same problem in France since law is getting harder. Near Calais (facing UK's coast), first a helping center was legally destroyed and people were sleeping in the street ; then helpers were arrested for giving clothes or shelter. Countries going towards barbary and giving lessons. --DLL 17:30, 29 March 2006 (UTC)[reply]
DLL, you are referring to Sangatte, I take it. This was finally closed by the French authorities after long British protests at the French helping illegal immigrants to cross into Britain by offering them food and shelter in Calais. There was a massive tabloid campaign here in Britain accusing the French of getting rid of their unwanted asylum seekers and illegal immigrants by encouraging them to leave Paris and move on to Britain. Jameswilson 00:53, 30 March 2006 (UTC)[reply]
To be fair, the French accused the British of being hypocritical: they said that the British pretend to fight illegal immigration, but in practice let entire industries run with illegal migrant labor (which is cheap because illegal immigrants don't have rights). In a nutshell, this meant that the British government catered to the business class by giving them cheap labour, while catering to the populist tabloids by shifting the blame to foreign countries such as France. David.Monniaux 21:52, 30 March 2006 (UTC)[reply]
Yes, there was a shift in policy in the 1990's. Before, the preferred option was to have high interest rates and therefore high unemployment (Norman Lamont's famous "price worth paying") to avoid an "inflationary" labour shortage. Now we have low interest rates and low unemployment but lots of immigration to achieve the same result. In other words we (and Spain and Ireland) have adopted the USA/Australia model. Jameswilson 22:46, 30 March 2006 (UTC)[reply]

social Science[edit]

I just want to know whether Nikko(Japan) is part of the four main Islands if Japan.So can you please tell me where in the four main islands is Nikko located.Thank you for your help

We have an article on Nikko, Tochigi, which is located on Honshū, the mainland. Is this what you mean? See the article on Japan, and particularly that on Geography of Japan. --Halcatalyst 03:36, 29 March 2006 (UTC)[reply]

"Die yuppie scum"[edit]

From where does the phrase "die yuppie scum" originate? --CrypticBacon 05:11, 29 March 2006 (UTC)[reply]

I've seen it in the American Psycho movie (it's written (in blood, I think) by Patrick Bateman on a wall in Paul Allen's house), I haven't read the book, so I don't know it's in there too. Of course, there may be an earlier usage of the phrase, though the term yuppie existed for only 10 years before the book was written. --Aramգուտանգ 07:46, 29 March 2006 (UTC)[reply]

Thanks for the info :-) --CrypticBacon 08:47, 29 March 2006 (UTC)[reply]

  • The oldest cite on Google Groups predates the novel (which was published in 1991). A 1990 post on rec.music.gdead (a Grateful Dead fan group) states (dating the expression back to 1988):
i remember at alpine valley a few years ago(88) someone was selling "die yuppie scum" stickers. a few of us started hanging around her booth chanting "love yuppie scum" [44]

--Metropolitan90 02:21, 30 March 2006 (UTC)[reply]

I saw it scrawled on a wall in Berkeley in the early '80s. Around the same time as "Eat the rich" and "Keep Bob Avakian On the Streets" (ok, you had to be there.). --jpgordon∇∆∇∆ 03:20, 30 March 2006 (UTC)[reply]

Pumpkin to a carriage ??[edit]

I daresay if it is possible to turn rags into a gown and a pumpkin into a carriage, then glass slippers that are comfortable are possible. Looking to science is not right for a fairy tale. Bippity Boppity Boo! Esquizombi 23:45, 27 March 2006 (UTC)


may i please know how a pumkin may be turned into a carriage ?

--203.124.2.16 11:47, 29 March 2006 (UTC)[reply]

You have to find a fairy who has become your godmother. (People forget that "fairy godmother" is from the land of fairy, which means that she can only be in the human world at certain times of the year and her magic would all be vegetative and that she has to have a trick in her magic.) The fairy will be good at changing vegetables into things and changing animals into other animals, but she'll have a catch to it, like only lasting until midnight (when she has to return to the land of fairy) or making you smell of pumpkin pie or something. Geogre 11:54, 29 March 2006 (UTC)[reply]

oh . k. thanks. so... on the whole i understand it is not possible.( i knew it in the beginning, just wanted to confirm) thank you :)

Read "Withches Abroad" by Terry Pratchett for further help.hotclaws**== Cat helped with typing-It's "Witches" hotclaws**==

It depends on the assumptions you make about the universe. The ones in fiction are not always the same as the ones in our world. Magic is sometimes just fun, but sometimes works as a metaphor. I think that the way that turning a pumpkin into a carriage in a fairy tale works is quite different to other contexts. For great justice. 01:39, 31 March 2006 (UTC)[reply]
The reason I emphasized "a godmother who is a fairy" is that the Charles Perrault Cinderella, which is the basis of the Disney one, brings in the dead mother's friend, a fairy. In French literature, through Brittainy, there was a great tradition of fairies. Fairies were vegetable magic and fertility spirits. Usually, their world would intersect with the human world only in certain places or at certain times, when a gateway would be open between the two worlds. If a human fell for a fairy, he would often be enslaved forever, etc. (Lots of ballads tell that tale.) So, here is a fairy who has promised to look after the child if her parents die. The child is being absued by a wicked step-mother, and the godmother acts, but only for the brief time when the two worlds are open (hence the "all my magic comes undone at midnight" catch). In folk lore, the fairy magician was a well worked out trope by the 19th century, when a renewed interest in "quaint" tales was underway. In other versions of the story, magical birds or the dead mother's spirit do the avenging work. Perrault, however, works with a very strict sense of poetic justice, and so he can't do things like have the step sisters pecked to death by birds. The fat old lady with wings from Disney has nothing to do with Perrault, and Perrault has little to do with the Grimms, and the Grimms may or may not be accurate in their transcription of the elderly housekeeper's memory of the story she heard from her grandparents. Geogre 02:38, 31 March 2006 (UTC)[reply]

population of Cape Breton—1910[edit]

What was the population of Cape Breton Island in 1910?

Thankyou

---74.12.2.246

The Encyclopedia Britannica 1910-11 edition says it was "over 100,000" in 1906, but doesn't give a more precise figure. From that I'd guess between 100,000 and 120,000 in 1910. Grutness...wha? 06:32, 30 March 2006 (UTC)[reply]

Religous roots of Chinese occupation in Tibet[edit]

Seeing as Tibet is a strongly budhist state, and China has strong influences of philosophys such as Confucianism and Toaism, I would like to know what few events are most important in causeing the invasion of Tibet in 1949, in a religious context - Particularly religous causes for the conflisct. What caused the level of imperialism displayed by the Chinese? Why Tibet- it is not the only state considered to be a part of China by the Chinese government. Did religion play a significant, or even noticable role?

China's Ideolog then and now is communism. Their goal for invading tibet was to created a barrier between them and india, and to further advance communism. Since communism will not tolerate any opposing ideologies, once it invaded and occupied Tibet china began to suppress its religion of Buddism. Because Buddism is strong their, Buddism and its leaders have become a rallying point the tibet soverignty campaign.

Read Tibet. It isn't just that China said "Hey, let's beat up on those guys in the funny orange robes." --Kainaw (talk) 21:22, 29 March 2006 (UTC)[reply]
Quoting Mao Zedong from the movie Kundun: "religion is poison. It makes you [Tibetans] poisoned and inferior." deeptrivia (talk) 04:02, 30 March 2006 (UTC)[reply]

Althusser's theory of ideology[edit]

Hey, I've a problem understanding the difference between the early Marxist/Feuerbachian theory of ideology and Althusser's theory as proposed in Ideology and Idealogical State Apparatuses. ie. this sentence:

"it is not their real conditions of existence, their real world, that 'men' 'represent to themselves' in ideology, but above all it is their relation to those conditions of existence which is represented to them there."

Well I kinda get it, in that Marx's ideology is once removed, while Althusser's ideology is twice removed... but I can't think of an example to illustrate the difference. Essentially that's what I'm asking for.

The portion of the text where Althusser makes the distinction can be found here. (It's on page 164)159.134.255.224 21:09, 29 March 2006 (UTC)[reply]

Well, I'm not positive, but what I think you're up against is Althusser arguing, as many other neo-Marxists (let's call them what they are: Neo-Hegelians) do, is that ideology is not the material placement of a worker, but the belief and identification of the worker. Thus, a poor person's ideology may be poor, or it may be bourgeoise, because it is not the actual relationship of work to life that is ideology, but the beliefs about the work position that is ideology. Marx's ideology is more straightforward: if you do a type of work, you are part of X or Y ideology. For Althusser, if you do the work, your ideology is the relationship of the relationship between work and living.
This development has the advantage of allowing for a Marxist discussion of fine arts and to understand why some people have ideological projections that are empirically wholly unsupportable. At least that's what I remember getting from reading that stuff. Geogre 01:36, 30 March 2006 (UTC)[reply]
I don't think that is exactly right: in Marx too there is a difference between the material placement of a worker and the misrepresentation of that placement in ideology. But if I understand him correctly, Althusser goes one step further, like the questioner suggested. From page 165: "all ideology represents in its necessarily imaginary distortion not the existing relations of production (and the other relations that derive from them), but above all the (imaginary) relationship of individuals to the relations of production and the relations that derive from them." In other words, for Althusser ideology is a distorted version of an already imaginary relationship. David Sneek 07:42, 30 March 2006 (UTC)[reply]
Hey, it's user 159.134.255.224, ie. the person who asked the question, just saying thanks for the responses, the latter one was kind of what I was siding towards already. 134.226.1.136 09:07, 30 March 2006 (UTC)[reply]
I misspoke, for I agreed with David Sneek and didn't explain myself. What's tricky is that the real productive position of the person still matters. I wondered whether Althusser would suggest that ideology is greater or lesser in its activity based on how exaggerated its distortion is, but he doesn't really say that, and it appears that the imaginary projection is never really more or less acute. Geogre 11:39, 30 March 2006 (UTC)[reply]

yes or no?[edit]

+that is the question--205.188.117.12 18:18, 29 March 2006 (UTC)[reply]

sí o no. no sé. schyler 23:24, 29 March 2006 (UTC)[reply]
The correct answer is "or." --Cam 03:30, 30 March 2006 (UTC)[reply]
I say xor. —Keenan Pepper 06:01, 30 March 2006 (UTC)[reply]

42.........hotclaws**==

mu --WhiteDragon 20:36, 4 April 2006 (UTC)[reply]

When asked to be that specific by 'her indoors', and I am not certain, I answer, "YO". She doesn't ask again.

cervantes, 'witz', and freud[edit]

There is a great deal of speculation about the causes and effects of laughter in 'don quixote'. Is thier any evidence that Freud's joke theory was influenced (oedipus sytle) by his experiences of reading don quixote?

  • According to this book review, there is a book in Spanish which claims that Cervantes was an influence on Freud. It wouldn't surprise me if Freud had read him while young; Spanish was one of his early languages he picked up and he was always a fan of classics. --Fastfission 17:24, 30 March 2006 (UTC)[reply]

very useful, thanks very much

March 30[edit]

Music/Italian words[edit]

Are dolcissimo and largissimo even words in Italian? I'm not sure, even though I've seen dolcissimo in Chopin's Nocturne. But largissimo would mean "very slow" if it is an Italian word, right? :\ —CliffHarris (-T|C-) 23:35, 5 April 2006 (UTC)[reply]

Both dolcissimo and largissimo are technically genuine Italian words. Their meanings, however, are not what a musician may expect them to be. The suffix issimo can be added to pretty much any adjective to mean very or very much, e.g. Buono = good, buonissimo = very good. Largo in Italian, means wide / broad, but in musical terms it means slowly. Dolce means sweet so both of the words you mention could easily be used by an Italian, but don’t assume that their meaning to a musician is the same as their meaning to an Italian. In practice, I find this is very rarely true. Anonymous Bob 10:21, 6 April 2006 (UTC)[reply]

Copyright in Canadian schools[edit]

When a Canadian public school teacher plays music or a video for the class, and the license prohibits public performance, is this entirely legal? If so, is it fair use or part of the Access Copyright license? Seahen 00:06, 30 March 2006 (UTC)[reply]

Prime Ministers[edit]

Hi how are you today? Could you please answer the following question which is required for a child's homework.

Who was the prime minister of Cuba in 1927?

Thanks for your time MICHELLE

The president of Cuba in 1927 was Gerardo Machado y Morales. Cuba did not have its first prime minister (primer ministro) until 1940. --Cam 03:26, 30 March 2006 (UTC)[reply]

Italic textThanks for that it was really appreciated. No wonder I couldn't find the answer anywhere on the net.

Comrade[edit]

In Soviet Russia (especially during the time Stalin was in power), did people really refer to eachother as comrade, or is this just a sterotype? schyler 03:40, 30 March 2006 (UTC)[reply]

  • It was (is?) common for dedicated communists to refer to each other as such. Though the Russian word is tovarishch. See Comrade. Eivindt@c 03:49, 30 March 2006 (UTC)[reply]
In Soviet Russia, comrades refer to YOU! Brian Schlosser42 17:59, 4 April 2006 (UTC)[reply]

Bad Credit[edit]

I have heard it said that when a man and a woman get married in the U.S., the woman takes on whatever credit rating the man has, good or bad. Is this true? If it is, is there a way to prevent a bad rating (on the part of the man) from affecting the womans good standing? Any information will be helpful. thank you.Djfloc 04:33, 30 March 2006 (UTC)[reply]

I seriously doubt if this is the case, though of course if they apply for a joint bank account or mortgage the credit rating of both would be relevant & considered by the bank. AllanHainey 13:47, 30 March 2006 (UTC)[reply]
That is not correct. In the US, the ratings stay separate, but if they try to apply for joint credit, the worse one will be considered, and may lead to not qualifying. So that may have led to your thinking, but the woman's rating itself stays good unless late payments or other problems come up on joint accounts. Joint accounts show up on both people's ratings. Good news is credit ratings can be improved pretty quickly if no late payments are made and a few other things. We have a few articles on the topic. See credit score (United States) and credit agency. - Taxman Talk 17:24, 30 March 2006 (UTC)[reply]

what is life?[edit]

It's the fifth track on All Things Must Pass. Grutness...wha? 06:35, 30 March 2006 (UTC)[reply]

Here, read all about it. JackofOz 11:58, 30 March 2006 (UTC)[reply]
We actually have a number of articles on What is life? --Fastfission 17:36, 30 March 2006 (UTC)[reply]

What passes right in front of your eyes before you're dead..hotclaws**==

I like to put it as "just a peculiar behavior of matter" ☢ Ҡiff 09:55, 31 March 2006 (UTC)[reply]

can anyone be the only person in the whole world to have a shoe size ? --203.124.2.18 10:35, 30 March 2006 (UTC)[reply]

I suspect that foot sizes fall into a nice bell curve, so most people will have shoe sizes in the middle of the range. People with very big feet (no sniggering please) or very small feet may have very few people with the same shoe size. There are also some medical conditions (such as elephantiasis) which can cause people to have distended feet. So it is theoretically possible for people with feet that are significantly different from the average, but it would still probably be unlikely for the size to be truly unique. Grutness...wha? 10:54, 30 March 2006 (UTC) (UK size 12 male)[reply]


what is distended feet ?--203.124.2.17 11:32, 30 March 2006 (UTC) what if the shoe is made of glass? will more than one person be able to fit their feet into the same shoe ?--203.124.2.21 11:40, 30 March 2006 (UTC)[reply]

If you're talking about absolute foot size, then there would be much fewer people with that same foot size. Shoe sizes allow for a bit of "wiggle room", so people whose feet are slightly different sizes can still fit in them. If you measure your foot to a high enough degree of precision, you could find that you are the only person in the world whose foot is, say 8.07620728180123 inches long, or something. However, note that everyone in the world whose feet are larger than yours must have had feet your size at some point.
As for glass slippers, they would certainly make it difficult for anyone with a size too large to fit into them. However, my guess is that even a prince wouldn't be able to tell if someone's foot was .001 inches too 'short. — Asbestos | Talk (RFC) 16:35, 30 March 2006 (UTC)[reply]
Furthermore, the prince wouldn't have to check everyone "in the world" just the female people within the kingdom. Or you could just get to the point where you say "hey, it's just a story . . . " --LarryMac 16:46, 30 March 2006 (UTC)[reply]
Besides most of the original versions of the story included a fur slipper not a glass one, making it even more unlikely. So yeah, it's just a story. Qeee1 09:16, 31 March 2006 (UTC)[reply]

Penguins - blessed by God[edit]

No, the subject line isn't a joke. A friend of mine has heard somewhere that it was once thought that penguins were blessed by god because they walk on two legs, as does god's chosen chordate. It sounds feasible, but I'm not going to add something so apocryphal to the penguin article - if anyone can find a source for this I'd be fascinated to read it. I tried searching google, but if the information is out there anywhere then Penguin Books skews the search beyond usefulness. Hughcharlesparker 11:30, 30 March 2006 (UTC)[reply]

I'm not sure where the concept originated, but your question instantly reminded me of Anatole France's novel Penguin Island which makes fun of the concept of penguins being blessed by god. Read it if you haven't already - highly enjoyable :P -- Ferkelparade π 13:42, 30 March 2006 (UTC)[reply]
What I'm wondering, is what bird doesn't walk on two legs? --BluePlatypus 16:30, 30 March 2006 (UTC)[reply]
Well, most don't do much walking, I suppose. But certainly a pigeon would qualify, and they don't seem very blessed to me. Much less an ostrich, which are filthy beasts. --Fastfission 17:38, 30 March 2006 (UTC)[reply]
I believe all birds walk on two legs, as it is necessary for winged flight. Superm401 - Talk 01:00, 31 March 2006 (UTC)[reply]
While all birds walk on two legs, most don't have an erect spinal column when they do. That is, other bird's heads protrude forward and their tails backwards. Penguins and people however, have a vertical spinal alignment when walking. StuRat 02:18, 31 March 2006 (UTC)[reply]
And the gannet soils its nest, so we'd want the expurgated version. ISTR some of the bigger seabirds such as albatrosses only return to land once a year to mate, so they do virtually no walking. Penguins are definitely blessed by God, though - a lot of them live in New Zealand ;) Grutness...wha? 00:44, 31 March 2006 (UTC)[reply]

Thanks for your help everyone. Lots of interesting stuff, even if we didn't find what my friend was remembering. A friend of mine works at a college and has a tame theologist, so if he can turn up anything useful I'll add a section to the penguin article. --Hughcharlesparker 13:22, 5 April 2006 (UTC)[reply]

Fanya Baron was executed for what specific offences? Vess

According to the article, she wasn't charged with an offence. --Hughcharlesparker 14:31, 30 March 2006 (UTC)[reply]
Maybe she was not executed either. And also : are we sure that she existed ? --DLL 21:32, 30 March 2006 (UTC)[reply]

Histery[edit]

do you know who is the founder of Kansas City Mo?

Frenchman Pierre Laclède founded KS city!

United States Demography[edit]

I'm from belgium and I've got to make a study about something of the united states. Therefore I need to find how many people are over 65 in the united states. I have all years except 2003 and 2004, can somebody help me?

How many 65+ in 2003? How many 65+ in 2004?

Thank you

According to this page at factfinder.census.gov (if the link dies, go to Factfinder, People -> Age & Sex -> 2004 -> Age groups), 12.0% of the population was 65+ in 2004, and the total population size was 285,691,501. You can than work out the total number of 65+.
I don't know about 2003 data, but I know that they don't run a census every year, so it might be difficult to find information on 2003. — Asbestos | Talk (RFC) 17:07, 30 March 2006 (UTC)[reply]

Mythology[edit]

Which Titan (or other mythological character) took or regained their strength from the earth? When they were knocked down in battle, they rose stronger than ever after absorbing more strength from the earth?? Thank you in advance for ponsering this with me? It's been a long time since I studied mythology...

Antaeus alteripse 17:15, 30 March 2006 (UTC)[reply]

Yes, "Antaeus alteripse". Fought with Herakles. Some just called him colloquially "Antaeus". --DLL 21:13, 30 March 2006 (UTC)[reply]
alteripse was part of the user's signature :) — QuantumEleven | (talk) 14:18, 31 March 2006 (UTC)[reply]
Couldn't resist, they quite match ;-). And did not Herakles fight with another himself (→alter ipse), after all every victory is upon oneself. --DLL 19:14, 31 March 2006 (UTC)[reply]

Penal Colony Transportees Returning to Britain.[edit]

I know that many of the convicts transported to Botany Bay and other Australian penal colonies were often sent there for life, with the alternative sentence of death by hanging being imposed should they ever return to Britain. What I don't understand is how the British authorities controlled this. I don't believe passports as we know them were in use then. Any information would be gratefully appreciated.

I don't know about Botany Bay, but people who were transported to America did return, but they had to assume a new identity. If they were recognized (not unlikely, if they went back to the same old cities, given that these cities at the time had populations a tenth of the present day) and apprehended, they could be hung. The trip back from Australia would have been a lot harder and a lot more expensive, but those who got rich from planting in America could/would return and buy their way out of whatever had them transported. Geogre 20:35, 30 March 2006 (UTC)[reply]
Agreed, it was just too expensive to return from Australia in those days. Jameswilson 22:53, 30 March 2006 (UTC)[reply]
Some of those transported to Australia returned too, but the expense was enormous, and those sent away were generally very poor. Cost of travelling to the UK from Britain was probably on par in contemporary terms with seats on Concorde. A friend of mine is descended from one of the most well-known returnees from transportation - Thomas Standfield. Grutness...wha? 12:24, 31 March 2006 (UTC)[reply]

African-American Soldiers in Vietnam?[edit]

Can anyone give me any information for African American soldiers who fought in Vietnam, for example the political fallout which occurred as a result seeing as so many did fight in it (In proportion to their population)

I believe many black leaders objected to the war, particularly Malcolm X. Mohammed Ali, who was convicted of draft evasion, had this to say:
"They want me to go to Vietnam to shoot some 
 black folks (Vietnamese) that never lynched 
 me, never called me nigger, never assassinated 
 my leaders." 
Here's a good article: [45]StuRat 01:44, 31 March 2006 (UTC)[reply]

Religious image source[edit]

Anyone have any clue where this painting image was found. A documentarian has contacted us, wishing for a higher-resolution copy. -- Zanimum 21:00, 30 March 2006 (UTC)[reply]

Try User talk:Dante Alighieri; he uploaded it to Commons last year. Shimgray | talk | 21:22, 30 March 2006 (UTC)[reply]
The origin of the photo is possibly this page which has an image of identical size. (The Internet Archive shows that the photo has been there since at least February 2001.) Someone at that church may know where the original icon is. --Cam 00:56, 31 March 2006 (UTC)[reply]

China DVD ? VCD English Translation[edit]

File:DVDVCDquestion.jpg

I was wondering what this DVD/VCD title translates to in English? Also if they make it with English Subtitles? Where to go for this? I've tried looking for it, even going to Chinatown, not found. Thanks. You can delete pics after translation, just added for temp. --Jacob

It's a TV series from 2001 called "Xiao Ao Jiang Hu" based on the 1967 novel The Smiling, Proud Wanderer. Here's a review of the series. There's discussion of a subtitled DVD release on this forum. --Cam 00:31, 31 March 2006 (UTC)[reply]

council of Nicaea[edit]

I am trying to obtain a good copy of the icon featuring the council of Nicaea. I would like to use it in a documentary so I need the best copy available.

Thanks Roger C.

Assuming you mean the First Council of Nicea, Wikipedia uses Image:Nicaea icon.jpg. The direct link to the image is Media:Nicaea icon.jpg. You can use it freely in your documentary, as it is public domain. Superm401 - Talk 01:05, 31 March 2006 (UTC)[reply]
I assume that you have seen that someone asked the question on your behalf a little earlier, a couple sections up. Hopefully the link shown there will lead you further. — Asbestos | Talk (RFC) 01:55, 31 March 2006 (UTC)[reply]

Hairy Coo names[edit]

Quick! Anyone! What would be a good name for a small stuffed hairy coo (female)? Feel free to use the word "coo", or a Scottish rolled 'r' or something in the name. Thanks! — Asbestos | Talk (RFC) 23:54, 30 March 2006 (UTC)[reply]

Is this some kind of delicacy? For great justice. 01:30, 31 March 2006 (UTC)[reply]
What about Coo Starrrrrrrk (with a Scottish rolled 'r')? Phileas 01:36, 31 March 2006 (UTC)[reply]

J-Low..geddit???hotclaws**==

Arrr-Coo-Dee-Coo -- Ferkelparade π 12:50, 31 March 2006 (UTC)[reply]

Thanks for the suggestions, all. For Great, I don't think my girlfriend would be very please if we stuffed and ate her new stuffed toy. — Asbestos | Talk (RFC) 15:07, 31 March 2006 (UTC)[reply]

March 31[edit]

lucretius[edit]

what did he care most about? what he said, or how he said it?

Lucretius was an Ancient Roman Epicurean philosopher and author who wrote De Rerum Natura. He claimed that all matter was made up of indivisible particles, or atoms, and that there is no afterlife, so no one should fear death. Brian G. Crawford 00:27, 31 March 2006 (UTC)[reply]

"no afterlife, so no one should fear death." Eh? Superm401 - Talk 01:08, 31 March 2006 (UTC)[reply]
The question also asked how Lucretius said it. Lucretius wrote his philosophical poem in dactylic hexameters. Bucketsofg 05:59, 31 March 2006 (UTC)[reply]

Origin of music for "Chinese motif"[edit]

I am desperately looking for the history of a particular short musical phrase that is used to represent "Chinese"-ness, or Asian culture in general. I've seen it referenced in cartoons, stand-up comedy, and in the popular song Kung Fu Fighting.

I've uploaded a short audio file of the musical phrase in question: Image:ChineseMotif1.ogg

What is the origin of this phrase? Who first composed it? How did it come to be the quintessential chinese-stereotypical music. Does it have any actual connection whatsoever to genuine Chinese music, or is it just a western invention? -Alecmconroy 01:13, 31 March 2006 (UTC)[reply]

Also in "Japanese" music.. The 80's hits "Turning Japanese", "Japanese Boy" and Bowie's "China Girl". If memory serves me (not really my favorite kind of music :)). It's a pretty simple motif, just three tones. I'm not sure if it's actually Chinese or not (could very well be coincidental), but the origin of the "asianness" is the transition from a sharp note down to the next sharp note. (three semitones) --BluePlatypus 02:08, 31 March 2006 (UTC)[reply]
The three tones are the "thirds" used when practicing 8-note scales. The plucky sound is a sanxian (though the sound clip included mimics appears to use flutes to mimic the sound). The only odd thing about it is that the Chinese the top note, then the middle, then the bottom, then the middle. Europeaners play it bottom, middle, top, one octave up, top, middle, bottom. I seriously doubt that it was ever unique to one song. Since the sanxian has no frets, all songs probably began with the sanxian player plucking all three notes to ensure his chords were correct and, as a side benefit, setting the tempo for the rest of the musicians. --Kainaw (talk) 13:12, 31 March 2006 (UTC)[reply]
I found a whole discussion on the exact topic at the Straight Dope message boards: here. The most qualified answer (from a music researcher) seems to say "noone knows". But his theory is that it's a simple thing to come up with by playing on the black keys of a piano, and thus emulate the chinese pentatonic scale. --BluePlatypus 14:46, 1 April 2006 (UTC)[reply]

Well, if this many people have investigated it, it deserves it's own article, so i wrote one: Asian Riff . Perhaps not the best title, but what would be better. --Alecmconroy

Ehud Olmert[edit]

How do most Isrealies view Ehud Olmert?

From a distance. --Kainaw (talk) 02:30, 1 April 2006 (UTC)[reply]
DO you mean "Israelis"? AnonMoos 13:48, 1 April 2006 (UTC)[reply]

abortion[edit]

Outside of the U.S. where is Abortion a controversial issue?

See Category:Abortion by country. Dismas|(talk) 02:40, 31 March 2006 (UTC)[reply]

Everywhere! It's controversial all around the world. —This unsigned comment was added by 139.168.254.13 (talkcontribs) 08:56, March 31, 2006 (UTC).

False. It's largely uncontroversial and uncontested in most of Europe, for instance. In Sweden for instance: "The issue is largely settled, the question of the legality of abortion is not a political issue, and no relevant debates exist on the matter.". Which is quite true, not one political party in their parliament is against abortion, including the Christian Democratic Party. --BluePlatypus 13:50, 31 March 2006 (UTC)[reply]
I would say it depends largely on what you mean by 'controversial'. I would not limit this to political parties, but take into account that there are a good few hundred million people living in Europe, and it is possible that they have different views about this, even in Sweden. And around the world. So I'd say, yes, everywhere. Definitely in Poland! --Ouro 16:17, 31 March 2006 (UTC)[reply]
Certainly I didn't mean to say that not one single person has a dissenting view on abortion. But if, say, 95% of the population hold one view and the opposing view doesn't have any political representation (assuming we're talking about a democracy), then yes, I would say that's uncontroversial. --BluePlatypus 13:14, 1 April 2006 (UTC)[reply]

If their views differ significantly there would be political parties to pick up on it. I live in Scandinavia as well and I have not heard a single politician ever voice any intention what so ever to limit abortion from its current status. It would be tantamount to political suicide. Celcius 10:40, 1 April 2006 (UTC)[reply]

ghosts[edit]

If their are ghosts, why are they in that state?

See the article Ghost. "If there are ghosts" though? If there aren't ghosts then they aren't in that state! --Canley 02:46, 31 March 2006 (UTC)[reply]

Are they in any particular state? I thought they weren't limited by pesky physical objects like country bordes etc ;P Celcius 10:42, 1 April 2006 (UTC) They have not yet been sucked up by the Ghostbusters---hotclaws**==[reply]

Cheating[edit]

In many case would it not be kind of like cheating to ask homework question in this section?

Yes, that's why it's not allowed. StuRat 02:39, 31 March 2006 (UTC)[reply]
Theoretically.. Superm401 - Talk 04:53, 31 March 2006 (UTC)[reply]
Actually, most teachers would count it in your favor that you are actively seeking help with homework: what they won't appreciate is if you're having someone else do it. - Nunh-huh 05:23, 31 March 2006 (UTC)[reply]
There's a distinction to be made between asking for help eg. explaining a concept or looking for resources, and asking someone to do your homework. The former is generally ok, and if you ask the latter most people here will tell you off anyway. Qeee1 09:21, 31 March 2006 (UTC)[reply]
Another distinction is that it is always OK to ask for help (I'm talking about life in general, not the Wikipedia reference desk), but that doesn't mean you will always get what you ask for. JackofOz 09:31, 1 April 2006 (UTC)[reply]

Saddam education[edit]

I read that Saddam Hussien attended law school in Cairo; how close did he to getting a degree?

The US News bio suggests that he received a law degree from the University of Cairo in 1962. One wonders whether, given his indecorous (though humorous and sometimes accurate/fair) outbursts at his (sham) trial, he might have missed the days when courtroom behavior and style were discussed. Joe 07:14, 1 April 2006 (UTC)[reply]

The Mob in Springfield Mass[edit]

Does anyone have any information on organized crime, in springfield mass?

Just send a self addressed, stamped envelope to:

The Mafia
100 La Cosa Nostra Parkway
Palermo, ND 56769

Brian Schlosser42 18:19, 4 April 2006 (UTC)[reply]

Kim Jong ill[edit]

where is his offical residence, in North Korea?

Correct.
Giuseppe Verdi was once asked his address, and replied "I think Italy will be sufficient". JackofOz 08:00, 31 March 2006 (UTC)[reply]
Perhaps he said that because "Verdi" was sometimes used as an acronym for "Vittorio Emanuele Re D'Italia". David Sneek 16:04, 31 March 2006 (UTC)[reply]
Kim Jong-il most likely has numerous residences around the country. This article says there are at least eight outside of Pyongyang, North Korea's capital, and refers to them as "palaces" which probably isn't far off the mark. --Fastfission 01:19, 1 April 2006 (UTC)[reply]
Actually, wow, check out our article Official_residence#North_Korea. I have to say, I'm impressed we already have an entry for that! --Fastfission 01:21, 1 April 2006 (UTC)[reply]

UFO's[edit]

What are the prevailing scientific theories on the them?

I don't think many scientists spend much time studying UFOs. They don't like vague, unrepeatable evidence, they want precise reproducible experiments. If you had an alien body or spaceship to study, then they would get involved. However, others have been interested in studying them, such as the US Air Force during the Cold War (probably suspecting some were Soviet reconnaissance aircraft). Their conclusion was that they are a wide range of things, from hoaxes, to natural phenomena (ball lightning, sprites, moon reflecting off water and clouds, mirages, etc.), to man-made phenomena (experimental aircraft, weather balloons, etc.). See Project Bluebook for more specifics. StuRat 02:27, 31 March 2006 (UTC)[reply]

Read The Demon-Haunted World by Carl Sagan for a potentially life-changing book that is sure to have an influence on your life unless you are blind to science, and drowning in psuedoscience. He talks about UFOs, aliens, and extraterrestrials for several chapters. -- Mac Davis] ⌇☢ ญƛ. 08:47, 31 March 2006 (UTC)[reply]

Scientists tend to focus on things that are easy to study, and governments will not fund things that they want to cover up. The scientific method is not good at dealing with things that are not easy to control. For great justice. 16:14, 31 March 2006 (UTC)[reply]
What makes you assume the government wants to cover up UFOs. Perhaps you've been watching too much X files? - Mgm|(talk) 09:56, 7 April 2006 (UTC)[reply]
Why do you suppose the scientific method is limited in this way? The function of science is simply to investigate phenomena, acquire knowledge, and make predictions about the world. A phenomena need not be docile in order to be studied scientifically. Hurricanes, muons, and black holes aren't easy to control, but we've managed to learn some things about them anyway.
The existence of UFOs is neither rejected nor confirmed by science. In the words of Carl Sagan, "extradordinary claims require extraordinary evidence." Most (perhaps all) scientists would love to discover extraterrestrial life, but there simply isn't enough concrete evidence to assume that it exists. I agree with Mac Davis that anyone interested in UFOs or extraterrestrial life ought to check out The Demon-Haunted World, a brilliantly concise and accessible explanation of scientific skepticism. Bhumiya/Talk 01:16, 1 April 2006 (UTC)[reply]
If they are truly "UFOs" then they are just "unexplained". To assume that something unexplained is supernatural, extraterrestrial, or otherwise outside the realm of traditional explanation would require some very hard evidence indeed. There seems to be no reliable evidence for extraterrestrials existing at all (much less having visited the earth) and none for supernaturalism either. --Fastfission 01:33, 1 April 2006 (UTC)[reply]
I would go further than both Bhumiya and Fastfission. There is no evidence whatsoever of any extraterrestrial sentient life, and a lot of looking has been done. The looking will go on because a lot of scientists take the position that extraterrestrials do exist, not as a proven scientific fact, but because it is close to mathematically impossible for them not to. And if they exist, it's worth looking for them. A bit like neutrinos, really. A lot of other scientists take the doubting Thomas approach, as is their right (unless I see ... I will not believe). Could both groups be said to be speaking for science? JackofOz 09:22, 1 April 2006 (UTC)[reply]
There are a number of scientists who don't think it is even worth looking though. Depending on how you juggle the Drake equation, the odds of us ever detecting, much less being in contact with, sentient extraterrestrials could be vanishingly small, a completely independent affair from whether or not they did/do/will exist or not. --Fastfission 17:21, 1 April 2006 (UTC)[reply]
That's right, the number of scientists who do look for extraterrestrial life is small. But they are scientists, who do use scientific principles to underpin their work. And if one of these unheralded weekend hobbyists produced verifiable evidence of extraterrestrial life, it would be hailed as great day for science.
My uni stats is/are rusty, Fastfission, but the existence of aliens seems to be an assumption when working out the odds of us ever detecting any, otherwise the calculation would be meaningless. The two questions cannot be as independent as you claim. :--) JackofOz 09:34, 2 April 2006 (UTC)[reply]

Pope[edit]

Who are likely canidate succeed the ageing pope benedict, after his death? Wayne Gretzky? hotclaws**=

It's anyones guess really, but it usally a cardinal, so you might want to have a look at List of cardinals. Eivindt@c 06:31, 31 March 2006 (UTC)[reply]
FWIW (not much; I'm an atheist who doesn't follow the politics of the Catholic Church too closely), I'm not as confident as others that the next Pope will come from the Southern Hemisphere; I think Cardinal Schoenborn is a very likely successor. Our article on the subject of those whom most viewed as candidates to replace JPII might be of some relevance; of course, whether the probable candidates for the 2005 conclave are probable candidates at the next conclave depends largely on how much longer Benedict serves. The odds currently given by Irish bookmaker Paddy Power are here; Venice's Cardinal Scola is the favorite (at 6-1), with Schoenborn and Cardinal Maradiaga following at 7-1.
Hey! They're offering 1000-1 odds on Bono for Pope, and also for Father Dougal from Craggy Island (Father Ted). -- Arwel (talk) 02:02, 3 April 2006 (UTC)[reply]

Bernard Kerik[edit]

Does former NYPD commisioner Bernard Kerik hold a BA; his biography is silent on that issue?

Maybe not. This press release mentioned that the First Deputy Police Commissioner, Joseph Dunne does have a BA, but there is no mention of such a degree for Kerik. --Canley 02:44, 31 March 2006 (UTC)[reply]
Not even close. He's a high school dropout. Whatever education he had came from the Army and then as a security expert and then a cop. --jpgordon∇∆∇∆ 03:41, 31 March 2006 (UTC)[reply]

Meaning of Major & Minor[edit]

In music, the terms 'Major' and 'Minor' have specific meanings and while 'Piano Concerto No 1 in C Major' means a lot to students of music, to the rest of us it is mostly jargon. So to my question: what single words could you use to replace terms 'Major' and 'Minor' the would give a better descriptive feel for the music being played? Thankyou

I'm definitely not an expert in music, but I know this: there's not single word. You could greatly simplify things and say major is perky and minor is depressing. However, please see major scale and minor scale. Superm401 - Talk 04:58, 31 March 2006 (UTC)[reply]
That's right. See also major and minor. It's certainly not a black and white thing. Other simplistic characterisations of major/minor are: happy/sad, cheerful/gloomy, up/down, and optimistic/pessimistic. JackofOz 05:08, 31 March 2006 (UTC)[reply]
For those kind of characteristics, I'd look at Musical_mode#The_major_and_minor_modes, though we could use some elaboration there. The problem is that Wikipedia can't really write well about this sort of thing, as someone will delete it as a POV. - Nunh-huh 05:21, 31 March 2006 (UTC)[reply]
Some examples might help. The opening of Beethoven's Symphony No 5, the famous da-da-da-DAA, da-da-da-DAA is clearly in a minor key (C minor). It is dramatic, dark, even frightening - more suited to a funeral than a wedding. That's minor. On the other hand, Chopsticks is irretrievably humorous and jocular and puts smiles on people's faces. It's a lot more towards the wedding end of the spectrum. That's major. Is that in any way helpful? JackofOz 07:34, 31 March 2006 (UTC)[reply]
Minor chords are kind of saddening, delicate, and tearful, while Major chords are lively, joyous and robust -- Mac Davis] ⌇☢ ญƛ. 08:43, 31 March 2006 (UTC)[reply]
A very good piece to listen to to see the difference is the opening bars of Richard Strauss's Also Sprach Zarathustra (the main theme to 2001: A Space Odyssey). you hear dum...dum...dum...DA-DAAAH twice in a row at the beginning, but in the first one those last two chords are a major followed by a minor, and the minor ending giving it an edgy, slightly unnerving sound. The second time through is exactly the same except that the order of the two chords is reversed (minor then major), and the major ending gives a very "up" feeling - hopeful and optimistic (the major and minor chords are both of the same root note, IIRC C# major and C# minor). Grutness...wha? 12:33, 31 March 2006 (UTC)[reply]
Or any piece with a Picardy third can be used to illustrate major/minor, though there are still psychological and cultural associations with pieces in major and minor modes that aren't fully dealt with in Wikipedia. And (mostly prior to the establishment of equal temperament tunings, there were specific emotions and connotations associated with specific keys (e.g. A# minor vs F minor). - Nunh-huh 23:58, 31 March 2006 (UTC)[reply]

Aussie myths[edit]

I am currently studying Australian myth as part of a Communications and Culture degree and was wondering where the term "sheila",a colloquial term for woman, originated from?

According to Wikipedia's sister project, Wiktionary, sheila is just a generalization of Sheila, a female given name. --dcabrilo 08:06, 31 March 2006 (UTC)[reply]
This paper from the Australian National University puts forward a different interpretation, claiming that as Sheila wasn't a popular Irish name at the time, the Australian use of the term is more likely to have derived from the Irish word SÌle (pron. sheela), meaning an effeminate or homosexual man, or a man disposed to do the work of women. Natgoo 09:53, 31 March 2006 (UTC)[reply]

You mean sheilas are mythical?? Tell us it isn't so!! alteripse 11:36, 31 March 2006 (UTC)[reply]

Test of acceptance[edit]

Dear Sir, this text appears on your site can you point me to the source

[long quote from Offer and acceptance#Test of acceptance redacted]

G Wilmore

[duplication of entire question redacted]

Please clarify your query: do you want the source used for the information, which is presumably Tretel's Law of Contract as stated in the first sentence or are you looking for the author of the article, in which case you should read Wikipedia:Citing Wikipedia?
You could also try asking on the article's talk page. --Bth 10:56, 31 March 2006 (UTC)[reply]

The Misfortunes of Virtue[edit]

"Nonetheless, Sade invites us to live virtuously in hope of heavenly reward." But marquis de Sade was an atheist! Vess

Are you the anon who said the same on the article's talk page nine months or so ago? The wording's almost identical. I think Jmabel's answer there is the main point, but it's also worth pointing out that at that time the social acceptability of a book containing all manner of vices would be somewhat enhanced by its claiming however tenuously to have a conventional "moral". (qv Moll Flanders which is an interesting counterpoint, since Defoe was a committed Christian who really believed it was a redemption narrative, but it's similarly a large amount of vice described with relish, with a relatively small pious ending). --Bth 13:53, 31 March 2006 (UTC)[reply]
I'd be careful with citing Moll Flanders in that regard. We've all asked ourselves how aware Defoe was of the fact that he spent more time with happy sinning than tearful repentance, but it's extremely important to remember that he was trying to sell books and trying to emulate the criminal biography (which he had helped to invent with his work at Applebee's Original Weekly Journal). Also, it's part of the Puritan theology that the depth of the sin is irrelevant, that the vilest person may be redeemed. Therefore, a Puritan would want an exceptionally vile person to star in a conversion narrative. In American folk ballads dating from the end of the 18th century, we have lots of Pretty Polly-like songs with murderers repenting. Like Moll Flanders, those songs spend most of their time talking about the crimes. Again, this is not to say that a modern reader doesn't read Moll Flanders with some sense of titilation, nor that Defoe didn't know his own readers would do the same, nor that Defoe didn't capitalize on it, but I think questions of sincerity are safely put away by his theology (and the moralizing in Robinson Crusoe ought to convince anyone that Defoe was sincere). Geogre 23:50, 31 March 2006 (UTC)[reply]

Bush-Blair Iraq Memo from Jan 2003[edit]

Last night, on The Daily Show of all places, there was a story about a memo containing details of a meeting 2 months before the start of the 2003 Invasion of Iraq, where Bush asserted he was committed to invading regardless of UN resolutions or actions of Saddam, and even provided a date when the attack would begin. Why is this not covered by major news outlets? Not enough sources are yet covering it for it to become a stand-out story on Google News, and I didn't see any mention of it at the Wikinews site. Is it too controversial? Too unverifiable? It sounds like something everyone deserves to know about. --Jmeden2000 14:37, 31 March 2006 (UTC)[reply]

Well, partly this isn't new news. Channel 4 News had it at the beginning of February. Judging by this BBC article, the current fuss is just 'cos the NYT has gotten hold of it, though given that it's surprising it's not creating more waves. (And that other more famous memo was the subject of a File on Four (no relation) literally years before anyone started making a noise about it. I'm not often given to patriotism, but British journalism rules.)
My answer to the more general question of why this sort of thing is so slow to percolate through the US media is probably too POV and inflammatory for the Ref Desk. --Bth 15:09, 31 March 2006 (UTC)[reply]
One reason it isn't all over the press is that what the memo says and what people think it says are two different things. In this question here, it uses the phrase "regardless of UN resolutions or actions of Saddam." The article about the memo uses the phrase "irrespective of whether the United Nations had passed a new resolution authorising the use of force." We might as well claim it say Bush decided he was going to go to war "regardless of whether the UN sent every military person in the world to Iraq to defend Saddam." This story is not about relaying facts. It is about getting an "I told you so" for all those who didn't vote for Bush. --Kainaw (talk) 21:25, 31 March 2006 (UTC)[reply]
Kainaw, I think most people accept that the invasion was going to happen whatever the UN said or did by that stage. Or dont you? Planning a war takes a long time and costs a lot. As in 1914, things reach a point where governments feel they cant change their mind even if they wanted to. The thing has its own momentum. So in a sense I would say it was just window-dressing as happens before every war where everyone tries to look good by making one more try for peace. France and Russia werent having it and blocked any new resolution, especially if their oil deals with Iraq werent going to be recognised by the US, and politically, they calculated war was inevitable and they were making sure they ended up on the right side of world opinion afterwards and leave Bush and Blair to get the blame. Jameswilson 23:05, 31 March 2006 (UTC)[reply]

us population in 1861[edit]

hello I can't seem to find the population for the United states of America for the years 1861-1865, if you could be so kind to answer that, that would be great.

thank you

Try this page. --Ouro 22:05, 31 March 2006 (UTC)[reply]

Repeal of the U.S. Constitution[edit]

Leaving aside such concepts as the right of revolution, my question concerns the legal possibilities surrounding a hypothetical repeal of the United States Constitution by the states. Presumably an amendment could be proposed without the consent of Congress, according to the never-used two-thirds-of-states-propose-a-convention method, including words to the effect of: "the whole of the above is henceforth invalid, the union is dissolved, and the individual states are independent, sovereign countries with no obligation to, nor authority over, one another." Would such an amendment be valid under the Constitution as it exists at present? Presumably ratification would have to be unanimous as every state would lose its representation in the Senate, and would therefore all have to give their consent. Could three quarters of the states legally repeal everything in the Constitution except those provisions relating to states' representation in the Senate, including removing all of the Senate's powers, thus making their inalienable representation irrelevant? Legally speaking, would a repeal of the Constitution cause the United States to revert to the Articles of Confederation as its basis of government, or were they rendered completely null and void by the adoption of the Constitution? I suppose what I'm asking here is whether the creation of the union under the Constitution was an irrevocable act that deprived the states of their sovereignty once and for all, or whether the states could conceivably pull the rug out from under the federal government's feet, depriving it of its legal foundations without its consent? On a related matter, if a consensus emerged that the present Constitution needed replacing by another set of rules, leaving the union intact this time, what are the legal avenues that would allow this, or is the USA stuck with what it has forever, with only the possibility to tack amendments onto the bottom? On another related matter, do we have an article on the legality and possibility of individual states seceding from the union under the present arrangements? Thanks guys! 86.136.1.31 23:59, 31 March 2006 (UTC)[reply]

"hypothetical repeal of the United States Constitution" hypothetical? you don't follow US politics I take it?--205.188.117.12 00:00, 1 April 2006 (UTC)[reply]
Not in detail, I'm afraid. Perhaps you could point me to some discussion of the more-than-hypothetical possibility of repeal you seem to be referring to? 86.136.1.31 00:05, 1 April 2006 (UTC)[reply]
(I think 205.188.117.12 was making a joke. —Keenan Pepper 00:35, 1 April 2006 (UTC))[reply]
I did wonder, especially since it had just ticked over into April Fools' Day UTC by the time he made his comment. 86.136.1.31 01:13, 1 April 2006 (UTC)[reply]

Also, if anyone could help with the questions on Talk:Twenty-seventh Amendment to the United States Constitution, Talk:Gregory Watson or Talk:Entrenched clause I'd be most grateful! Yes, I'm full of questions, I know! 86.136.1.31 00:05, 1 April 2006 (UTC)[reply]

I'm pretty sure you could vote, legally, to dissolve the constitution. It has happened before, anyway: see Philadelphia Convention. --Fastfission 01:37, 1 April 2006 (UTC)[reply]
It depends. The northerners can do what they want. When the southerners vote to drop the Constitution and secede, the northernerns go to war. --Kainaw (talk) 02:34, 1 April 2006 (UTC)[reply]
Not anymore we don't, cede the union, see if i care
Are you crazy - if the south secedes again, you'll lose Disney World. What would the United States be without Disney World!? --Kainaw (talk) 02:46, 1 April 2006 (UTC)[reply]
[Shudders to think of it] :P —OneofThem(talk)(contribs) 02:49, 1 April 2006 (UTC)[reply]

In fact, so far as I know (and I hope I'm wrong), the constitutional case behind the South's secession was never settled in court. This is sadly important, as the lack of ruling has been used by every disgruntled group since to suggest that not paying taxes is a valid option, that renouncing citizenship and staying in the country is fine, that declaring one's house a nation is legally valid, etc. The US Constitution was set up with the idea that the states were willingling confederating, and the Constitution's language certainly does suggest that the states can walk away from it. In the case of the US Civil War, the matter was solved by force of arms rather than legal decision. In fact, in 1861, Jefferson Davis staid in Washington DC after Sumpter hoping to be arrested so that a legal case could be forced. He wanted a legal ruling. Instead, no one arrested him, and, instead, the US announced a military pacification of the rebelling states. That forced him south to assume the position of President of the CSA. I keep hoping that there will be a ruling that disallows dissolution so that all these other idiocies can be silenced, but I don't think there has been an explicit ruling still. (I shall be delighted to hear that I'm wrong.) Geogre 14:03, 1 April 2006 (UTC)[reply]

If there ever is a final ruling about secession, it will likely be with Hawaii, not the south. There are many who feel Hawaii was stolen into the Union, not voluntarily drafted. The reasoning is that the voters in Hawaii who chose to join the union were not Hawaiians. They were American military members stationed in Hawaii. So, there is a claim the vote was invalid and Hawaii should be allowed to secede. --Kainaw (talk) 20:47, 1 April 2006 (UTC)[reply]
It's an interesting thought. But there's nothing in the Constitution that says the populace of a territory has to consent to be brought into the Union. And Geogre is wrong about about the consitutional case never being settled, Texas v. White being rather explicit -- "the Constitution, in all its provisions, looks to an indestructible Union, composed of indestructible States". (Some people certainly disagree with that decision. But that is, at least so far, the final constitutional authority on the issue.) --jpgordon∇∆∇∆ 15:44, 2 April 2006 (UTC)[reply]
Even with that ruling (thanks for the link), though, I think the original questioner's theory of the states collectively submitting and passing an amendment saying "we scrap the whole thing" would be viable. It falls within the established text of the Constitution, and amendments are free to wipe preceeding portions of the Constitution as they see fit. There would not, however, be an automatic reversion to the Articles of Confederation unless the amendment also spelled that out. — Lomn Talk 14:01, 3 April 2006 (UTC)[reply]
I'm glad there is a case, although I must say that it's perfectly worthless as a case. If it has been taken as precedent, then that's all that matters, but what a weird opinion. "Without the consent of the states" is.... I mean, if that's so, then the states that left had certainly consented, and what the court meant ("all of the states") isn't stated. Blug. Terrible writing. (Chase was not a very bright light, IMO, anyway.) They couldn't shake the Lockean basis that they relied upon for the original Revolution. I suppose that's all that matters -- that the case has been used as precedent to establish that there is no right to seceed. Geogre 17:20, 3 April 2006 (UTC)[reply]
  • A couple points to add to the above. The Confederate states never attempted to amend the constitution. Instead, they declared themselves as sovereign states, and together entered a new union under the Confederate constitution. There is nothing in the US constitution that would have kept the southern delegation in the Senate from introducing an amendment, or series of amendments, that would have peacefully seperated the two nations... of course, the full senate and the state legislatures would have had to vote for it, so it would likely have never happened, even if it had been tried. Which goes back to the original question: yes, TECHNICALLY the Constitution could be voided by a new amendment proposed by the States, but how would the States manage to do so without the Federal government getting wind of it? The fact is, the state and Federal governments are so intertwined, and the power structure so entangled, it could never happen, not without some extremely extenuating circumstances. Brian Schlosser42 18:42, 4 April 2006 (UTC)[reply]
Why would it matter if the federal government got wind of it? There is nothing they can do to stop it. Not legally, anyway.